/* */

PDA

View Full Version : The Medical student Review



جوري
08-31-2008, 01:26 AM
I don't know how many med students there are on board, or how useful this review will be for you.. I am a firm believer in slow and consistent than an intellectual enema two days before the exam where once released never again to be regained.. So what I plan to do here, is share some things I feel are important..

In my pre-clinical years, I was president of the pharmacology club, and I enjoyed teaching it and exchanging ideas with others.. so if you have your own forte, quirks mnemonics that you'd like to share we can make a useful compendium..

I am only going to focus here on pharmacology and diagnostic testings.. so every day I'll give you five of each..

I am not going to start in any particular order but once in a section, I plan to complete it.. this will be just the high yield..


__________________________________________________ ___________

Oncology diagnostic testing
_____________
AFP (what disease is associated with it, and when do you answer this for a question?)
AFP is associated with the development of
1-hepatocellular ca.
2-ovarian cancer
3-non-seminomatous germ cell tumors
Answer AFP when you see a patient with alcoholic cirrhosis or chronic hepatitis B or V. AFP together with radiological imaging is used to screen for hepatocellular ca

____________________________________________
CEA
1-A protein elevated in a variety of cancers including colorectal cancer
2-serum CEA have a prognostic value in pts with newely diagnosed CRC. Those with higher levels have worse prognosis.
3-CEA level to monitor in colon cancer in pts after a surgical resection, it determines the presence of persistent, recurrent or metastatic disease

__________________________________________________ ___________
Colposcopy
Colposcopy is the direct visualization of the cervix, by use of a magnifying scope with a lamp
2-The transition zone must be visualized to ensure an adequate colposcopy. The border between squamous and columnar epithelium
3-a colposcopy is the answer for a pt with an abnromal pap
Atypical squams can't exclude high grade lesion ASC-II
low grade squamous intraepithelial lesions LSIL
high grade intraepithelial lesions HSIL
Atypical sqams of undetermined significance ASCUS if HPV DNA testing is positive
__________________________________________________ ___________
Estrogen and progesterone receptors
Should be done on all pts with breast cancer, in order to determine who should receive hormone therapy
2-Therapy with either tamoxifen or raloxifene should be added to any pt. with positive receptors. This is either for estrogen or progesterone positivity alone or in combination. The response to tamoxifen is better if both receptors are positive
__________________________________________________ ____
Mammogram
screening should begin at age 40 and should be performed every 1~2 years, screening at age 50 should be yearly
2-when mammogram shows abnormalities, a core biopsy including sentinel lymph node biopsy is the next best step. carcinomas of the breast are associated with clustered polymorphic microcalcifications.
3-screening lowers mortality most after age 50 and the dec is greater than that of a colonscopy or a pap smear
__________________________________________________ ________________

Now pharm
_____________

will start with infectious disease because it is the longest chunk
Acyclovir/valcyclovir/famiciclovir

all the above are the correct answer for
Herpes simplex including, genital, cutaneous, orolabial
for Herpes Encephalitis (acyclovir) IV form only in a hospital setting
Varicella zoster
shingles: Herpes zoster or reactivation
Bell's palsy

the above meds work by inhibition of thymidine kinase
-most common adverse effects are nephrotoxicity presumably from precipitation of the meds in the kidney tubule, sx of neurological toxicity in the kidney tubule, sx of neurological tox, such as confusion, tremors and hallucination occur rarely
__________________________________________________ __
Rifaximin
used to treat travelers' diarrhea such as that from E.coli, it isn't used for invasive diarrhea. an associated fever and bloody diarrhea indicate and invasive pathogen, such as campylobacter. When fever and bloody diarrhea are described, ciprofloxacin is the best answer.

rifa is a nonabsorbed version of the rifamycin antibiotic, it inhibits ribosomal RNA production of essential proteins
There are no major side effects since it isn't absorbed from the GI tract. it doesn't cause C.Diff, and may in fact treat it.
_________________________________________________
Daptomycin and Linezolid

they are both used for gram +ve organisms such as MRSA, streptococci and vanc resistant enterococci. Linezolid is the only oral antibiotic for MRSA. They can both be used for Vanc resistant organisms .

Liezolid is an oxazolidinone and inhibits protein synthesis at the ribosome. Daptomycin is a cyclic lipopeptide and disrupts cell membrane they are both unique classes of drugs

Linezolid commonly causes thrombocytopenia, and is a MAO inhibitor, avoid tyramine foods. Dapto caused CPK on liver functiion tests to be elevated!!
__________________________________________________ _____________
Tigecycline
an extremely broad spectrum anti-biotic that covrers MRSA, staph aureus, and well as gram negative bacilli. Tigecycline is the answer for complicated hospital of ICU acquired infections, tigecycline alone is equivalent to vanc and aztreonam in combination, it is also active against resistant enterococci and PCN resistant penumococcus

tigecycline is a glyclycyline antibiotic that binds to the ribosome and inhibits protein synthesis, it is unique that it covers staph, strep, gram negatives, anerobes and organisms resistant to vanc
tigecycline is hepatotoxic , caused nausea and diarrhea
__________________________________________________ ________

Polymyxin B (PMB) and colistin
useful for conjunctivitis, infections of the skin, and otitis externa, also correct for multi drug resistant gram negative bacilli, that cause ventilator associated pneumonia and sepsis from pseudomonas or acinetobacter.

works by disrupting phospholipids in the cell wall membrane
Polymyxin B (PMB) and colistin are very toxic to the kidney and nerves and is limited to topical applications of the skin and ear, they are also used for multi drug resistant gram negative bacilli when there are no other therapeutic options..

__________________________________________________ ______
That is it for today..
:w:
Reply

Login/Register to hide ads. Scroll down for more posts
جوري
09-01-2008, 01:46 AM
ok today's 5 and 5

in diagnostic testing, I am finishing the last two in oncology and starting three in cardiology, and we'll continue cardio tomorrow insha'Allah

Diagnostic onco/cardio

PET scanning
________________
Positron emission tomograpy
this measures the metabolic activity of a lesion seen on a CT or an MRI scan. It is based on the ability of most cancers and some infections to have an increased utake of 18-flurodeoxyglucose. it is a noninvasive test of the actual function of a mass lesion, not just its size and location.

most cancers and some infections have an increaed iptake of tagged glucose

A pet sca can tell if a lesion, such as a lung mass, that might otherwise look malignant, really is maliganant, for instance, if you are show a case of a localized lung ca. diagnosed with a biopsy, you can use the PET scan to see if there are metastases. if there s a mass in the contalteral chest that is malignant, it makes the pts. ineligible for surgery with curative intent. If the PET scan shows a low uptake of the tagged glucose, then the lesion is likely benign, and you should proceed with the surgery to remove the promary tumor site.
_______________________________________
Sentinel lymph node eval
it is used in the eval of breast cancer
the sentinel node biopsy is when dye is introduced into the operative field, the first node it goes into is the sentintel node. if it has cancer, you dissect the axilla. if it is negative, you don't do an axillary lymph nde dissection.

look for a pt with abnormal mammogram in whom the biopsy shows cancer, then the answer is sentintel node biopsy. Sentinel node biopdy is the best test to do after the initial diagnsis of breast cancer has been made with either excisional or needle biopsy. sentinel node biopsy can eliminate the need for axillary lymph node dissection.
__________________________________________________
PSA
increases can occur in prostatitis, BPH and prostate cancer, levels are undectectable after a total resection of the prostate.

PSA testing is extremely conttoversial. There is no evidence that PSA testing lowers mortality when used as a routine screening test.
Biopsy of the prostate is most accurate test.
If the PSA is elevated, a DRE is performed to palpate a lesion. if a Lesion is found, it should be biopsied, if no lesion is found, a transrectal ultrasound should be performed to find a lesion to biopsy. If no lesion can be found, then 'blind biopsies' implemented

Ok on to cardio diagnostics



easy one EKG shows asystole,
there is no activity of any kind, astystole will not however give a perfect flat line. there will still be modest baseline undulation.

Asystome occurs in a pt who suddenly loses pulse and BP. but other causes of pulselessness have been excluded. It is not possible to distinguish between asystole, ventricular fibrillation, pulseless electrical activity from tamponade, or tension pneumothorax without an EKG.



this EKG shows atrial fibrillation (Afib)
it is charcterized by a normal QRS duration of <120 miliseconds and the presence of fibrillatory waves. Atrial fibrillatory waves eliminate any organized atrial activity such as a P-wave or even a flutter wave. The atrium is having a 'seizure' the rhythm is irregulary irregular. There is no fixed pattern between the R waves.

Afib pts presents with palpitations and has an irregularly irregular heart rhythm. the HX will often contain CHF, HTN, or hyperthyroidism. although simple caffeine excess is also a possibility.
__________________________________________________ _________
ok now
Pharm
still on with infectious disease

what do you fo for someone who has been bitten by a dog, and a dog that is has been bitten by a human being?

bites are treated with local wound care and tetanus prophylaxix, antibiotics aren't routinely indicated.
do answer antibiotics in the following
hand bite wounds
deep puncture wound, wounds requiring surgical debridement, older pts, bites near a prosthetic joint or any infected wound, if antibiotics are indicated.
Amoxicillin/clavulanic acid is the best antibiotic to use and PCN allergic pts are given clindamycin and trimethoprim/sulfamethoxazole.
Human bites transmit Eikenella corrodens, DOgs and cats transmit pasturella multocida

two cases, a 28 yr old from massachusetts landing from New Zealand, the evning after she arrives she finds a tick attached to her ankle, which treatment should she use?
simple reassurance, this is an asymptomatic tick bite from a country that is not endemic. in addition the tick has been attached for <24 hrs. Ticks, even if infected need more than 24, 36 hrs to transmit the organism for lyme dz.

2nd scenario
7 year old goes camping in the woods in connecticut, a weej after, he devoped a circular erythematous rash with central clering that is >5 cm what is the best RX
amoxil
there is no need for serology since the rash is characteristic. Lyme rash is typically >5cm. doxycycline and amoxil are equally efficatious, but there is no need to use doxy on a child less than 8 yrs old.
__________________________________________
23 yer old comes in with myalgia, fever, cough, headache, arthralgia, and sore throat for 24-36 hrs

best therapy is oseltaivir and zanamivir, both neuraminidase inhibitors that can be used to RX influenza A and B, within 48 hrs of the development of sx. They shorten the duration of illness 1-2 days.
the inhibition of neuraminidase prevents the realease of virus from cell surfae
zanamivir may cause wheezing because it is administered by inhalation
______________________________
Man with AIDS comes into ER with blurry vision, he is found to have a retinal lesion and his CD4 count is <50/MM3

CMV retinitis is the most common cause of retinal lesion in pts with AIDS and low CD4 counts, the best therapies are Valganciclovir, ganciclovir, foscarnet or cidofovir, because it can be sed orally valganciclovir is the drug of choice.
all these drugs interefere with the viral replication of CMV
the adverse effects are
Ganciclovir:neurtopenia
Foscarnet: nephrotoxicity, urethral ulcers, hypocalcemia, hypomagnesemia.
cidofovir: nephrotoxicity
valganciclovir: seizures, neutropenia
________________________________________________

drotrecogin the famous XIGRIS
used for severe sepsis with multiorgan dysfunction and a high APACHE II score,
it reduces mortality and sepsis and is used in addition to antibiotics, fluids, possible pressorts such as norepi. if a pt with hypotension and tachycardia, hyperventillarion and hypoxia from sepsis leading to ventillator dependence, then drotrecogin is indicated.
drotrcogin is an activated protein C analog. it has fibrinolytic and anti-inflammatory properties
the most common adverse effect is bleeding..
Reply

جوري
09-01-2008, 06:55 PM
Today's 5/5
diagnostics still cardio


This is an atrial flutter, the QRS is of normal duratio <120 MSEC and there is a saw tooth pattern to the atrial waves. The R-R intervals are regular. The flutter waves are identical to each other. This regularity as well as the presence of atrial activity is what distinguishes this from narrow complex tachycardia and Afib

the patient will have palpitations and maybe lightheaded. the pulse will be regular. The rate is often slower that that of SVT. You can't distinguish atral flutter from SVT without an EKG they both have the same heart rate.
____________________________________________



This is supraventricular tachycardia (SVT). It is a regular tachycardia with narrow QRS complexes. and no apparent P waves, fibrillatory waves or flutter waves. A normal QRS duration <120 MSec

the pt will have rapid palpitations and lightheadedness, triggers include caffeine or other sympathetic system stimulating drugs.. psychological stress and hyperthyroidism .
RX includes a carotid message, followed by IV adenosine. If adenosine doesn't work, then a calcium channel blocker, e.x verapamil or diltiazem, digoxin or beta blockers can also be tried

__________________________________________________ _


multifocal atrial tachycardia
Mat is characterized by irregularly irregular rhythm as demonstrated by variability in the P-P intervals. There are multiple p-R intervals and at least three different P wave morphologies. The atrial rate is over 100 per minute, when the rate is lower it is no longer tachycardia it is now called 'wondering pacemaker'

MAT is caused by right heart strain and the effect of chronic pulmonary disease on the right side such as in COPD
-__________________________________________________



Torsades de Pointes

The most distinctive feature is the undulating amplitude that gives the impression the EKG is twisting around a point. Torsdes is a form of wide complex tachycardia. The QRS width or duration is >120 MSEC. essentially, Torsades is ventricular tachycardia with an undulating amplitude.

Just like ventricular tachy, Torsades may present with anything from simple palpitations to syncope to sudden death. Can't tell without the EKG. Torsades can also be caused by toxicity to meds such as tricyclic antidepressants, amiodarone, dofetilide, ibutilide, macrolides and rarely quinolone antibiotics. . Look for hypomagnesemia.
________________________________________________



this is Ventricular fibrillation
The EKG has no recognized activity at all, there maybe either high or low amplitude to the EKG. if you see organized P waves, QRA, or regular rhythm then it isn't VFIB. But there is electrical activity so it isn't a flat line like asystole
Pts with Vfib, have no pulse, no respirations. They have no blood pressure, and have suffered a cardiac arrest. you can't distinguish between vfib, pulseless electrical activity and certain forms of pulseless ventricular tachycardia without an EKG

__________________________________________________ ______

Now Pharm, continuing on with infectious DZ.
An intern gets stuck by a needle from a known AIDS pt.
HIV infected injuries should be treated with the following prophylactic regimens
Zidovudine, lamivudine and a protease inhibitor (or efavirenz)
or
Tenofovir, emtricitabine and a protease inhibitor or (efavirenz)
the recommended protease inhibitor is lopinavir/ritonavir.
the alternatives are atazanavir, fosamprenavir or nelfinavir.
The risk of transmission is 0.2% or 1 in 300 without therapy
Nausea and vomiting are the most common adverse effects of therapy.
__________________________________________________ __

a middle aged man going for a visit to india

should be given mefloquine or atovoquone/proguanil for malaria prophylaxix. Doxycycline is used when there is mefloquine resistance. chloroquine can be used in those areas in which the malaria is still sensitive to it. chloroquine sensitivity is retained in central america and some parts of the middle east.
Mefloquine is associated with neuropsychiatric abnormalities, such as psychosis, hallucinations and seizures. It may cause cardia rhythm effects. such as bradycardia, and QT interval prolongation. if the person is on beta blocker or having an arrhythmia, use atovoquone/proguanil instead of mefloquine.
Doxycycline is asscoaited ith photo sensitivity.
_______________________________________________

nurse struck by a needle from a hep B surface antigen Positive pt.
and a middle aged trauma surgeon has been lacerated by a scalpel with blood from a pt with hep C
approach to both.
case one, check her Hep B status. if she has protective levels of antibody, then no further post-exposure prophylaxis is needed. The presence of surface antibodies indicated that the person is protected. if the injured has no protective antibody, then she should receive hep B immune globulin and a full course of hep B vaccine

case 2. there is nothing you can do. there is no prophylaxis for hep C
__________________________________________________
child wakes up in the middle of the night to find a bat in his room, which flies out, no injury is noted..
case 2. Man finds his neighbor's dog in his garden, when he chases him the dog bites him..

case one, the most common vector for transmission of rabies in the U.S are bats. prophylaxis is indicated with rabie immune globulin and rabies vaccine for any level of injury, including bites, scratches and mucus membrane exposure. prophylaxis is also indicated if the bat is found in a room while someone is sleeping, and contact is undetermined.

case 2, rabis prophylaxis doesn't need to be given for dog bites, if the dog is known as healthy. if the dog is wild or unknown, then the dog should be observed for ten days. if abnormal behavior develops, then prophylaxis should be offered. Pt with bites from a dog with abnormal behavior at any time should receive rabies immune globulin and vaccine.
____________________________________________

Woman working in a construction site and is accidentally hit by a nail, that was lying in the dirt, she received no vaccines since childhood

determine RX as follows.
prophylaxis for tetanus, should be given with any wound if more than ten yrs have passed since the last vaccination. if a vaccination has been given previously, prophylaxis consists of only tetanus toxoid.
If a person has never been vaccinated, then tetanus toxoid and tetanus immune globulin should be given. only people who have never been vaccinated should be given immunoglobulin.
If the person has received a dirty wound , then a booster of tetanus toxoid should be given if more than five years have passed since the last booster. clean .
clean wounds are protected for twn yrs.. dirty wounds for five years.
Reply

جوري
09-02-2008, 09:52 PM
Today's 5 and 5 still in cardio and infectious respectively



This is wide complex tachycardia (ventricular tachycardia)
the pt. presents with syncope, lightheadedness, sx of CHF or sudden death, the range is broad and there maybe palpitations

The rx depends on the pts stability
for unstable pts we use cardioversion
stable pts amiodarone, lidocaine, or procainamide
______________________________________________


Wolff-Parkinson-White Syndrome
this is a ventricular pre-excitation syndrome
The EKG shows a short PR and a slurring of upstroke of the QRS known as delta waves
the pt ca have supraventricular tachycardia alternating with ventricular tachycardia. There will be pre-excitation on EKG and paroxysmal tachycardia. The SVT maybe worsened after ca2+ blocker or digoxin
this is an accessory pathway for cardiac conduction from the atria to the ventricles that bypasses the AV node and causes earlier activation (pre-excitation) of the ventricles.
__________________________________________________ _____

Coronary angiography or cardiac cath
placement of a catheter from the brachial or femoral artery to the coronary artery with injection of iodinated contrast
angiography is the answer when
the cardiac stress test is abnormal
angiography provides an exact measure of degree of stenosis of the coronary artery and the number of vessels involved, The degree of stenosis needs to be >70% to be considered significant
__________________________________________________ ____
BNP
a hormone released from the heart and the brain in response to stretch in the atria and ventricles. BNP has diuretic, natriuretic and hypotensive effects and inhibits the renin angiotensin system in response to fluid overload states.
BNP goes up with CHF and other causes of hypervolemia

and elevated BNP is confirmed by an echo
__________________________________________________ ____

Pharm/ ID

Flu Vaccine
Everyone over 50
health care workers
anyone with COPD, liver cirrhosis, or kidney dialysis, diabetes, HIV, or who uses steriods, has no spleen, ir is a resident of a chronic health care facility
pregnant women
Flu vaccine is constructed with surface viral antigens, neuroaminidases, hemagglutinins from the previous year's most important strains
flu vaccines are given yearely
the vaccination can induce a mild fever, muscle soreness, very rarely associated with Guillain-Barre syndrome.
Influenza vaccine is contraindicated when the pt is allergic to eggs. the live attenuated intranasal vaccne should not be given to pregnant or immunocompromised pts.
__________________________________________________ ____
pneumococcal vaccination
should be administered to
everyone over65
anyone with COPD, liver Cirrhosis, kidney dialysis, diabetes, or HIV or who has no spleen, uses steriods, or resides in chronic care facility
alcoholics
people with cochlear implants
this vaccine is derived from the capsular polysaccharide surface of the pnumococcus
most ts over 65 receive a single injection, a second injection is give after five years only in immunocompromised pts and those in whom the first injection was before age 65
penmococcal vaccine is safe
_______________________________________________

clindamycin
best therapy for predmonantely anerobic
such as lung absess
aspiration penumonia from oral or abdominal anerobes
it is also useful in treating gram positive organisms, such as staphlococci and step. although not as effective as a beta lactam. like PCN or cephalosporin. It is used when pts has a life threatening anaphylactic reation to PCN
ex of such infections
include
cellulitis, erysipelas and impetigo
oral infections
pelvic infections
other second line uses include rx for the following
Penumocystis pneumonia
bacterial vaginosis
toxoplasmosis
MRSA
clindamycin is licosamide atntibiotic it inhibits ribosome, it often leads to dirrhea and c.diff
__________________________________________________ _
Metronidazole

effective against anerobic gram negative bacilli found in the abdomen, example of treatable conditions include
C.diff colitis
diverticulitis in combo with quinolone
perforated abdminal absess
pelvic infections, PID, Vaginitis
it is also effective agsinst potozoal infections
such as Giardia
enamoeba histolytica
trichomonas
Metronidazole inhibits bacterial mitochondia, interferes with numerous cystosolic products, also blocks
____________________________________________

1- pt comes in for follow up care for TB. complains of blurry vision, he is on isoniazid, rifampin, pyriziniamide, and ethambutol
shows no sx that he has developed hypruricemia after six weeks of TB meds

stop the ethambutal, it causes optic neuritis and can cause bluryr vision, color vision abnormalities can be the first sx.
hyperuricemia is most likely from pyrizinamide, however because the pt is asymptomatic, you should not stop meds. pyrizinamide is essntial to shorten the duration of therapy from 9 months to six months

isonizid causes peripheral neuropathy, and its use should be accompanices by pyridoxine replacement. Rufampin causes red orange discolaration of body fluids but does not need to be stopped.
All TB therapy can result in hepatotoxicity..
Reply

Welcome, Guest!
Hey there! Looks like you're enjoying the discussion, but you're not signed up for an account.

When you create an account, you can participate in the discussions and share your thoughts. You also get notifications, here and via email, whenever new posts are made. And you can like posts and make new friends.
Sign Up
جوري
09-04-2008, 12:46 AM
Today's 5/5

starting wtih pharm still on infectious dz
_______________________
42 year old homeless, immigrant, health care worker, alcoholic presents with fever, cough, sputum, and weight los over tthree months. chest x-ray shows apical infiltrate, and sputum is + for acid fast bacilli

TB cases should be placed on respiratory isolation and started on foru TB meds. a positive smear is sufficient to initiate therapy. INH, rifampin, pyrizinamide and ethambutal are medications for initial therapy.

All four meds should be used for the first two months, after sensitivities are known, ethambutal an pyrizinamide can be stopped. INH and rifampin should contiunue for four more onths for a total of 6 months therapy

standrd rx is 6 months therapy. RX is prolonged is the pt has the following osteomyelitis, meningitis, pregnancy, or milliary TB
__________________________________________________ ___
25 year old medical student is entering his residency. his PPD has a 12MM of induration. He had Bacille Calmette-Guerin (BCG) as a child, he has no sx, and his chest x-ray is normal

the student has a PP with a >10MM of induration and a normal x ray, he should receive 9 months of isonizd. the hx of BCG is not important, you can't distinguish a true + from TB exposure in the past from a false + from BCG vaccination
a + PPD confers a 10% lifetime risk of TB. Most reactivation occurs within the first two yrs of exposure
Isonizid reduces this 10% risk by 80-90%
isonizid can cause hepatotoxicity. routine monitoring of LFT's is not necessary unless underlying liver dz or alcoholism is present.
__________________________________________________ __

Posaconazole

is used as an empric theray with fever and neutropenia in pts who have no responded to empiric RX antibacterial coverage. Accepted empiric coverage is with imipienem or meropenem, cefepime or piperacillin/tazobactam. It isn't known whether posaconazole is superior or the echiocandins or voriconazole for febrile neutropenia. Posacaonazole is effective in the following conditions
candida
aspergillus
mucormycosis
cryptocococcus, histoplasmosis, coccidiodiomycosis

azole antifungals block ergosterol synthesis
adverse is liver tox. or QT prolongation may also occur.
__________________________________________________ _______________
echinocandins
Caspofungin
micafungin
anidulafungin

echinocandins such as caspofungin, are the best initial therapy in the rx of pts with fever and neutropenia who are still febrile after using bacterial antibiotics. echinocandins are at least as efficacious as amphotericin and much less toxic. Echinocandins are used to rx the following conditions
candida infxns
esophageal candidiasis
aspergillus especially if other therapy fails
echinocandins don't affect cryptococcus
echinocandins blocks 1,3-D-Glucan. this is a polysaccaride in the fungal wall that doesn't exist in humans.
these meds have few adverse rxns
minor histamine release, flushing, headache, utracaria, and pruritus.
__________________________________________________ ___________

a pt with acute leukemia has recently undergone allogenic bone marrow transplantation, after two weeks of fever and neutropenia, she develops a lung infection diagnosed as aspergillus
RX with voriconazole which is superior to amphotericin in the management of aspergillus. voriconazole results in greater survval with less adverse effects than amphotericin. itraconazole is not powerful enough in severe cases of aspergillus. Echinocandins (caspofungin) are used as a salavage if the pt. fails to respond to amphotercin or voriconazole. voriconazole will not treat mucormycosis.
imidazoles and triazoles inhibit microsomal cytochrome P450 (CYP) enzymes. this impairs biosynthesis of ergosterol for the cytoplasmic membrane. amphotericin is a polyene antifungal that interfers with sterol production in the cell membrane .
amphotericin will raise the creatining level and cause distal renal tubular acidosis with hypokalemia. voriconazole can be hepatotoxic like all azoles, it can prolong the QT on an EKG. the most unique adverse effect of voriconazole is a transient visual disturbance.
__________________________________________________ ____
6 diagnostics still in cardio I added an extra one because I know how excited you are to read these?
CK-MB

this is elevated serum marr for Myocardial injury. CK-MB is useful in detecting infarction and therefore the right answer in assessing any pt with chest pain especially with cardiac factors. CK-MB normally increases within 4-6 hrs fom the start of chest pain, it peaks about 12-24 hrs. C-MB is particularly useful in detecting re-infarction.

The EKG doesn't have to have St segment elevation in order for Ck-MB to be abnromal. also the answer whrn chest pain re-occurs after an infarction within the last few days
______________________________________
Dobutamine stress

stress testing is a form of exercise intolerance or stress testing that does not use physical exercise. it is a noninvasive method to determine myocardial perfusion
Dobutamine is a direct acting agonist at beta-1-adrenergic receptors, causing inotropic stress. dobutamine is injected and the echo is observed for a decrease in the wall motion. ischemic myocardium does not move as well as normal myocardium.

dobutamine is used in pts who can't exercise look for a person who has a hx of chest pain but in whom the story is equivcal and and the EKG is nondiagnostic. The indications are the same as those for persantine stress testing. it is also used for pts with reactive airways diseases who can't undergo a persantine thallium test

the coronoary angiogram is the most accurate test of myocardial perfusion.
__________________________________________________ ________
Cardiac electrophysiological EP studies
and EP study is an introduction of a catheter with an electrical sensing and stimulating electrode into the heart. This allows both the detection of abnromal cardiac rhythm disturbances as well the stimulation of the heart to determine the sire of an abnromal rhythm.

EP studies are used for
defintive diagnosis of abnromal cardiac rhythm, particularly the etiology of unexplained syncope.
mapping of the cardiac conduction system
ablation of abberant conduction tract abnromalities such as SVT or wolff-parkinoson-white syndrome
placement of automatic implanted cardioverter/defbrillator
searching for the site of origin of sustained ventricular tachycardia.
_________________________________________
Ergonovine test
involves the injection of ergonovine to provoke coronary vasospasm. this is diagnostic to detect prinzmetal's angina which is coronary vasospasm.
it is the most accurate test when they give a younger pt <45 complaining of atypical anginal type chest pain who has clean cornories on cardiac cath. the pain will not bear a fixed reltionship to exercise
prinzemetal variant angina is treated with a ca2+ chanel blocker.

__________________________________________________ _
Holter monitoring or 24 hr continuous ambulatory cardiac monitoring

is a way of recording an EKG for 24 hrs of continuous reading from a single lead
the pt wears leads on his chest and carried a recording device around his neck like a portable CD while at home. the strop can then be analyzed at high speed by the physician to detect a brief rhythm disturbance that may not have been detected on the original EKG
abnromalities on the holter can be further analyzed by electrophysiological studies in a labortatory by echocardiogram.

answer holter monitor when a pts has palpitation or syncope as the ekg does not detect an abnormality.
_______________________________________________
MUGA scans (Multiple Gated Acquisition scan)

this is the single most accurate method for assessing ejection fraction. MUGA also decreases left ventricular wall motion and cardiac muscle damage

MUGA is performed by injecting RBC radiolabeled with technetium 99 into the ps' blood stream and recording the emissions with a gamma camera.
MUGA is very useful in assessing and following cardiac function in pts during the delivery of potentially cardiotoxic cemotherapy such as adrimycin. the accuracy and reproducability of the study make it suitable to detect subtle, early changes in cardiac function that might easily be missed by other techniques such as an echo....
Reply

جوري
09-05-2008, 01:24 AM
Today's 5/5
still on with infectious disease in phatm

50 year old man with AIDS comes to the ER dept with S.O.B, dry cough, increased LDH, a pO2 of 64 and CXRAY with bilateral interstitial infiltrate. He has a CD4 count 105 and is on no medications:

Pneumocystis pneumonia PCP is initially treated with IV TMP/SMX, milder cases of PCP without severe hypoxia can be treated with atovoquone
adverse effect of TMP/SMX is a rash, bone marrow suppression, secondary to its ability to inhibit folate metabolism and renal insufficiency can also occur.

if the pt is allergic to sulfa meds, pentamidine is the alternative for severe PCP

sterioids are added for severe PCP, which is defines as pO2 <70 or an Aa gradient >35 (know how to calculate Aa gradient )PAO2 = ( FiO2 * (760 - 47)) - (PaCO2 / 0.8)
therefore A-a gradient = PAO2 - PaO2

that is arterial - alveolar but I digress
you should know also that Normal A-a gradient = (Age+10) / 4

__________________________________________________ ______

24 year old man comes to clinic with urethral discharge/ pregnant woman with cervicitis

urethritis is treated for both chlamydia as well as gonorrhea.. for chamydia we use a single dose azithromycin or doxycycline for a week. for gonorrhea use a single dose cefixime or ceftriaxone. Quinolones such as a single dose ciproflixicin, levofloxacin or oflaxicin can be used for gonorrhea if the rate of resistance is the community is low.

cervicitis is managed the same way as urethritis, pregnany women however, should not use doxycycline or quinlones in pregnancy, use azithromyin and ceftriaxone.
__________________________________________________ ______

woman comes to the ER with lower abdominal pain and tenderness, fever, and cervical motion tenderness. pregnancy test is negative

outpatient therapy for PID consists of levofloxacin alone for two weeks or ofloxacin and metronidazole. an alternate outpatient therapy is ceftriaxone or cefoxitin with doxycycline for two weeks.
inpatient therapy for PID consists of cefoxitin or cefotetan with doxycycline. PCN allergic pt can receive clindamycin with gentamycin
admission criteria include failure or intolerance of oral theray or fever or severe pain which may indicate the presence of an abscess.
__________________________________________________ _
protease inhibitors
Nefinavir, Ritonavir, Lopinavir, indianavir, amprenavir, saquanivir, atazanavir, fosamprenavir, daruanavir, tipranivir

protease inhibitors are highly effective anti-retorviral therapy
they work by inhibiting the protease enzyme which is essential for the virus to be packaged as it buds off of a CD4 cell and leaves to infect other T cells

All protease inhibitors cause hyperlipidemia and hyperglycemia in addition to being hepatotoxic, indianavir has a unique adverse effect of causing kidney stones nephrolithiasis

protease inhibitors are a part of the best initial therapy for HIV when the CD4 count goes below 350 cells of the patient has has an opportunistic infection. the other most common question for protease inhibitors describes a pt who develops either an increased lipid level or kidney stones and asks which medication should be stopped
__________________________________________________-
Dapsone
this is a sulfonamide antibiotic which is very selective against leprosy and is an alternative med for pneumocytis pneumonia
it is a folic acid inhibitor
the most common dverse side effect is a rash, it can also cause hemolysis with glucose 6 phosphate dehydrogenase deficiency. Methemogloblinemia can also occur, aplastic anemia can happen with all folic acid inhibitors.
Dapsone is used to rx the following conditions
PCP and toxoplasmosis prophylaxis where the pt is intolerant of TMP/SMX
Leprosy
Autoimmune skin disorders like Bullous pemphigoid and lichen planus and dermatitis herpetiforms.. which is a bit of shock to me as I often thought those were treated with hig dose steriod.. eh Dapsone it is!
__________________________________________________ ___________

all right not for diagnostics still cardio
Persantine Thallium

is a form of exercise intolerance stress testing. It is non-invasive method of determining mycoardial perfusion that uses med in lieu of physical exercise
It is also called dipyridamole is a phosphodiesterase inhibitor that dilates the coronary arteries and increases O2 flow to the myocardium. Thalium is a radioisotope that shold be picked up by normal myocytes. Persantine increases thalium uptake in the normal areas of the heart and less in the disesed parts of the heart.
Answer persantine thalium when pts cannpt exercise sufficiently to do a standard exercise tolerance test. the pt has a history of chest pain but you are not certain if it is ischemic in nature. It can also be used for preoperative screening.
Persantine can't be used for pts with asthma, COPD or emphysema, because it provokes bronchospasm. stress testing in general should not be done with an acute infarction or unstable angina.

use a dobutamine echo stress test when you can't use persantine
the single most accurate tst for myocardial perfusion is the coronary angiogram.
__________________________________________________ _
Sestamibi testing

this is a nuclea stress test also named Tc99m
TC 99m is a labeled compounds require intact perfusion and viable myocardial cells. essentially a myocardial perfusion agent is indicated for detecting coronary artery diseease. Diseased or infarcted myocardium picks up less of the nuclear isotope. Ischemic myocardium re-perfuses at rest. The defect is reversible infarcted myocardium does not change with rest.
the bottom line is sestamibi is a nuclea stress test that is used on obese pts and female pts with very large breasts.
__________________________________________________ ____

stress exercise tolerance
this is an attempt to detect myocardial ischemia. pts without the need for cornoary angiography. it is used when the diagnosis of ischemia is not clear in a person with chest pain

the pt exercises to >80% max heart rate bases on a maximum rate of 220-age. The EKG is observed for signs of ischemia. such as St segment depression. Hypotension, light headedness, and chest pain are taken as positive tests as well.

Stress test is answer
when pain is atypical and the story is equivocal
post MI to determine the need for angiography
with pt on meds to determine the amounts of meds are sufficient to prevent ischemia.
Abrnomal stress test is confined with angiography
__________________________________________________ _______
stress echo

is used to confirm the suspicion of coronary artery dz and estimate its severity
ut uses both 2D echo and doppler echo to detect ishcmia. MI gives decrease systolic contration of the ischemic area, called regional wall motion abnormality. the pt is asked to exercise using a tradmil or bicyle and if can't exercise dobutamine can be ised to increase myocardial demand.
a positive test consits of new regional wall motion abrnomalities a decline in ejection fraction and an increase in end systolic volume with sress on stress echo.

answer stress when case is equivocal for ishcemic heart dz and the EKG is so abnormal that you can't read it for ischemia.
__________________________________________________
Thalilium stress testing
this is the most accurate method of assessing myocardial perfusion without using angiograpy.
Thallium-labeles red cells are injected into the pt's blood stream. a Thallium scan provides a view of the blood flow into the heart muscle. stress thallium is an alternative to stress echo. the indication for stress thallium are
When resting EKG changes make exercise EKG difficult to interpret. such as the presence of LBBB, baseline St changes, left ventricular hypertrophy, pacemaker, or the effect of digoxin on the EKG,
to localize the region of ischemia
to assess revascularization following bypass of angioplasty

the most accurate test for myocardial perfusion is a coronary angiogram!
Reply

جوري
09-06-2008, 01:17 AM
today's 5/5 we are finishing cardio and moving on with endocrine with the last of the first 5, still on ID in pharmacology

Diagnostics/ cardio

Title table testing
is used to evaluate unexplained syncope. It is the measurement of blood pressure and pulse at various angles while the patient is tilted up and down while lying on a table

blood pressure and pulse are measured before and after moving the patient into different positions. The development of syncope, dizziness, abnromal drops in blood pressure constitute an abnormal test . Isoproterenol and nitroglycerin can be used as provocative testing.

table tilt is done when there is unexplained syncope, particularly when there are signs of orthostasis such as an inappropriate bradycardia when standing up.
_____________________________________________
Transesophageal echocardiography TEE

this is an extremely sensitive method os assessing cardiac structures. TEE is particularly sensitive at assessing posterior structures of the heart such as diseases of the aorta, example, dissection or aneurysm. Atrial thrombi, patent foramen ovale, or vegetations on the valves for the diagnosis of endocarditis . TEE is used when limited Transthoracic echocardiogram is inadequate.

TEE is performed by placing a sonographic transducer through the mouth of the esophagus
TEE is used to assess valvular heart disease as well as before the cardioversion of atrial fibrillation to exclude thrombi in the left atrium

The most accurate diagnostic test is a cardiac cath of the left heart.
_______________________________________
Transthoracic or 2D Echocardiogram TT
E

TEE is used to detect chamber size and function, valve abnormalities, intracardiac masses, pericardial effusions and aortic disease

Images of the heart are obtained from a sonographic transducer placed over the anterior chest wall
TTE is done to assess every patient with an ausculatory abnormality for the severity of valve dysfunction. TTE has become the second most frequently done test to evaluate the heart after EKG.
TEE is done to assess every pt with CHF to obtain ejection fraction. Therapy for CHF differs markedly based on whether there is systolic or diastolic dysfunction.

Both transesophageal echo and coronary angiograms are more sensitive than the TTE
__________________________________________________ _

Troponin levels
troponin is an enzyme that is released only from injured myocardium. Troponin is extremely specific for myocardium and is rarely derived from other sites in the body.

Troponin levels are the answer in most acute coronary syndromes as the most accurate means of assessing myocardial ischmia or infarction. Answer troponins in all patients in the emergency dept with acute severe pain. this is true even if the EKG is normal.
Myocardial ischemia as found by elevated levels of troponins is confirmed with EKG, echocardiography and angiography.
___________________________________________

last in diagnostics today is first in Endocrinology
we start off with
antimicrosomal Antibodies
Antithyroid peroxidase Anti-TPO
Antibodies
Antithyroglobulin antibodies

These antibodies are associated with Hashimoto's thyroiditis and graves' disease. If hypothyroid, if they are positive, you will move straight to treatment with synthroid because etiology of the hypothyroidism has been confirmed.
They are the answer once you have proven hypo- or hyperthyroidism. They are not to diagnose the thyroid function state. they are to confirm the autoimmune etiology of thyroid state,.
You expect the levels to be high with autoimmune thyroiditis. this is whether the person is hypo or hyperfunctioning. nutritional problems and synthroid abuse will give low levels of the antibodies.
_______________________________________________
pharm still on infectious

43 year old female comes to the ER having fever, headache, photophobia for the last six hours. Her lumbar puncture shows
1,400 white cells that are predominately neutrophils

Best initial therapy for bacterial meningitis is ceftriaxone, vancomycin and steroids.
ceftriaxone and vanc both inhibit cell wall of bacteria and are bactericidal. Ceftriaxone is a beta lactam antibiotic that inhibits peptidoglycan cross-linkng. vancomycin is glycopeptide antibiotic.
Glycopeptides inhibit the disaccharide precursors to peptidoglycans. Steriods presumably decrease inflammation of the central nervous system and decrease mortality in acute bacterial meningitis.

Ampicilin is added to the other meds when the pt may have Listeria infection. Listeria monocytogenes occurs more frequently in the elderly, neonates, and those who are immunocompromised, such as pts on steriods, chemotherapy, asplenic pts and those with HIV.
__________________________________________________ __________

Man has been in the hospital for the last ten days on a ventillator f pneumonia. he develops diarrhea, and the stool toxin is positive for c. diff

2- man is readmitted for recurrence of colitis from C.diff

Metronidazole is the best initial therapy for c.diff, should be used orally, IV metro should only be used if oral rx isnt possible
Metronidazole gives a disulfiram like rxn with alcohol, resulting in vomiting. It can also cause metallic taste in the mouth and peripheral neuropathy.

The small percentage of pts who do not respond to metro should be switched to oral vancomycin. vanco can't be used IV for colitis.

recurrent episodes can be treated with oral metro. is the pt responded to it the first time.
_________________________________

48 year old injection user comes to the ER with fever, rash, on P.E he has a pan-systolic murmur heard best over the left lower border. The murmur increases on inspiration

tricuspid valve endocarditis occurs more often in the injection drug users because of the constant bathing of the right side of the heart with staphylococus. Tricuspid regurg is best heart at the lower left sternal border and all the right sided lesions will tend to increase in loudness with inspiration, which increases venous return to the right side of the heart.
The best intial therapy with presumed endocarditis is to draw blood cultures and start empiric therapy for endocarditis is vancomycin and gentamycin
vanc. inhibits the cell wall and aminoglycosides and gentamycin inhibits the ribosome.
vanc can result in 'red man syndrome' froom a rapid infusion. this is flushing from histamine release by the vanco. aminoglycosides are nephrotoxic.
_________________________________________________

vomiting and severe diarrhea ensue after a sushi meal, in addition to rash and angioedema, wheezing and flushing

this is scombroid fish poisoning charcterized by suden onset of diarrhea, wheezing and flushing, scombroid fish poisoning occurs from certain types of fish such as tuna, mackerel, bonita, and mahi-mahi. bacteria within the fish result in the production of histamine. scombroid fish poisoning has the fastest onset of all forms of food poisoning with symptoms occuring within mins of ingestion.

RX for scombroid fish poisoning is with antihistamines such as diphynhydramine. Epinephrine is used in sevre cases.
__________________________________________________ ______
a woman with oncyomycosis of her toenail, the KOH of a scraping of the nail shows fungal hyphae
Terbinafine is the best theray for onychomycosis. it has greater efficacy than griseoflulvan. Terbinafine is used for 6 weeks for fingernails and 12 weeks on toenails. itraconazole is also superior to griseofulvan but not as efficatious as terbinafine

terbinafine works by inhibiting squalene epoxidase, this interfers with the production of ergesterol, whch is necessary to make the fungal cell wall. Squalebe accumulates within the cell wall which may account for terbinifines fungacifal activity.

The most common adverse effects of terbinafine are hepatotoxicity and taste disturbance..
Reply

جوري
09-06-2008, 11:17 PM
today we continue on with endocrinology in diagnostics and still on infectious disease in pharm

24 hour urine cortisol level
when collected for 24 hrs, urinary cortisol excretion is the most sensitive indicator of endogenous hypercortisolism and is the most sensitive test for Cushing's syndrome.
Cortisol is a glucocorticoid hormone produced in the adrenal cortex
look for a question with a case with truncal obesity buffalo hump striae, and easy bruising, answer 24 hour urine cortisol or overnight dexamethasone suppression test as best initial
_____________________________________________
ACTH (cosyntropin) stimulation test

this test evaluates adrenal function, and is the best initial test for adrenal insufficiency. baseline cortisol levels are taken, then ACTH is administered and follow up cortisol levels monitored. If cortisol levels rise, he adrenal gland is functioning. If cortisol is low think primary adrenal insufficiency.

look for a fatigued patient with skin hyperpigmentation, hyperkalemia, slight metabolic acidosis, and taste for pickle juice (high salt content) also look for ICU patient with unexplained hypotension.
_______________________________________
C-Peptide
serum measurement of C-peptide is the best test to distinguish endogenous from exogenous hyperinsulin in pts with hypoglycemia.
low values of C petide in pts with hypoglycemia represent factitious insulin injection. The pt is abusing insulin.
High C peptide values represent endogenous insulin production, such as that indiced by sulfonureas or from an insulinnoma.

look for pts with unexplained hypoglycemia, high insulin levels, and multiple hospital visits. Answer C-peptide as the next best step to determine etiology.
_______________________________________________
Fine needle aspiration of the thyroid
a sample of the thyroid cells collected by a needle inserted into the thyroid is the initial test for all solitary nodules unless the patient is thyrotixic
look for a case of palpable nodule and normal thyroid function tests
this however isn't what to do for a patient with a solitary nodule who are thyrotoxic. in such pts a SCINTISCAN (I121 scan) should be done first to see if the nodule is hyperfunctioning (hot) or hypofunctioning (cold). hyperfunctioning nodules are almost never malignant.
________________________________________________
oral glucose tolerance
this is a measurement of serum glucose levels at baseline and two hours after ingestion of an oral load of 75 grams of glucose
a positive test means that the glucose level 2 hours after the ingestion of the oral glucose load is >200mg %. this is one of the methods of diagnosing diabetes. The ordinary method is to find two fasting glucose levels to be >126mg/dl
the strongest indication for an oral glucose tolerance test is in screening for diabetes during pregnancy. because of the short duration of pregnancy, the potential for marked worsening over a short period of time and harmful side effects of diabetes on the fetus. this is the single most urgent time tim make a prompt diagnosis of diabetes. Oral glucose tolerance testing is like putting your pancreas on a treadmill to evoke subtle amounts of disease not detected by ordinary test.
__________________________________________________ _____
now pharm
60 year old man comes to the ER with S.O.B, cough, purulent sputum, temp 103 F. his WBC is 18,000. His pO2 is 65 on room air, CXRAY shows infiltrate in the right upper lobes

the most important factor in determining the need for admission in pneumonia is the severity of hypoxia. The white cell elevation, the purulent sputum don't denote then pt needs admittance. CXRAY abnormality by itself doesn't mean the pt needs admission. It is not the number of lobes involved rather the level of hypoxia, hypotension, tachycardia, or confusion tat idnciates a need for IV therapy.
outpatient pneumonia is best treated with oral macrolide, such as azithromycin, or clarithromycin. if there is serious underlying lung disease, respiratory fluroquinolones such as levofloxacin, gatifloxacin or moxifloxacin are the best first choice.
in patient pneumonia is treated with the combination of ceftriaxone and azithromycin or with a fluroquinolone.
__________________________________________________ ____

a man comes to the clinic with genital ulcer and surrounding lymphadenopathy. the ulcer i painless and heaped up. indurated edges, his darkfield examination and rapid plasma reagin is negative

a single intramuscular injection of benzathine PCN is the best initial therapy for primary or secondary syphillis. sx include a chancre, rash, lymphadenopathy, or condylomata laa. RPR can be negative in primary syphillis
PCN allergic pts with primary and secondary syphillis are best treated with doxycycline or a macrolide, doxycycline is superior in efficacy.
besides allergic rxn, the most common adverse effect of RX is Jarisch-Herxheimer reaction. this consists of fever, rash, mylagias, and a headache . this is thought to result from the release of lipopolysaccharides from dying spirochetes. and is treated with ASA.
__________________________________________
24 year old pregnant woman is in the clinic fo routine pre-natal care. her VDRL or RPR is reactive at a titer of 1:164, and the FTA-ABS is reactive, she has no sx

secondary syphilis of indeterminate duration is referred to as 'latent' syphillis, when the serology for syphilis is positive but the duration of the infection is unknown, the syphilis is latent and treated with three intramuscular injections of benzathine pcn give at weekly intervals. the same rx in pregnancy

pregnant pt who is allergic to PCN should undergo desensitization. true for any stage of syphillis
_________________________________________________
24 year old female with vaginal itching, and dyspareunia. her wet mount shows clue cells and ph 6.5

34 year old woman has discharge and wet mount shows motile forms

the sx of bacterial vaginits are clue cells and high ph. Normal vaginal PH is <4.5. Metronidazole or clindamycin are the best initial therapy for BV and can be used orally or locally

Bacterial vaginits in pregnancy is best treated with oral mes to prevent pre-term delivery. Oral metronidazole is not dangerous in pregnancy. untreated vaginitis is.

case two trichomoniasis is treated with only oral meds, oral metronidazole or oral tinidazole are best.
pregnant women with trichmoniasis should receive oral metronidazole.. there is no proven tetragenocity

will finish the lat case after I break fast insha'Allah
__________________________________________________ _____
ok post breakfast last caveat for the day
25 year old woman who is HIV positive finds out she is pregnant, she is on zidovudine, lamivudine and neflanavir

a woman who tests positive for HIV as a part of routine prenatal care, her viral load is 300,000, and her CD4 is 85

a patient who is already on antiretroviral medication for her own health with a low CD4 count in the past should be continued on the same therapy, don't stop any antivirals evern during the first trimester if the mother's life is dependent on the medication.
for case two a woman with low CD4 count (<350) or high viral load (>50,000) should be started immediately on antiretroviral therapy. even if she is pregnant. Antiretrovirals should be started even in the first trimester of pregnany if the woman needs medication for her own health. Triple therapy with two nucleoside reverse transcriptase inhibitors and a protease inhibitor should be used and continued life long, even aftr delivery is completed.

A ceasarian section should be performed if the patient's viral load is not fully controlled to a lavel <1,000 copies. the majority of perinatal transmission occurs at the time of delivery
Efavirenz is a non-nucleoside reverse transcriptase inhibitor. it is specifically contraindicated in pregnancy because of the risk of neural tube defects,.

the baby should receive zidovudine (AZT) syrup for at least six weeks after delivery.
Reply

جوري
09-09-2008, 01:06 AM
Today it will be 10 and 10 insha'Allah to make up for tardiness of yesterday ( we had family for iftar and I did nothing)

Still on endocrine for diagnostics

Beta-Human Chorionic Gonadotropin (B-HCG)

this is pregnancy test
urine and/or serum measurements of B-hCG is the best initial test for any woman of reporiductive age complaining of a missed period (amenorrhea), delayed period, vaginal bleeding or abdominal pain, any unexplained abdominal pain in a woman is an ectopic pregnancy until you have the B-hCG
_________________________________________________

5-HIAA

24 hour urine level of 5-HIAA is the best initial diagnostic test for carcinoid syndrome. 5-HIAA is the end product of sertonin metabolism

pt might present with episodic diarrhea, and or hypotension associated with unexplained flushing or wheezing.
the most accurate test for carcinoi syndrome is a biopsy. Although a CT and MRI scaning can localize the disease, they are not as accurate as the tissue diagnosis you obtain from a biopsy
__________________________________________________ ____

17-Hydroxyprogesterone level
this is the best initial test to diagnose 21-hydroxylase deficiency or congenital adrenal hyperplasia.
17-hydroxyprogesterone in a patient presenting with one of the following syndromes

female infant with ambiguous genitalia (simple virillizing)
infant with hyponatremia, hyperkalemia and hypotension (salt-wasting syndrome)
young woman with hirsuitism, acne, and irregular menses
child wih precocious puberty.
__________________________________________________
IGF-1 (Insulin- like growth factor-1)

this is the best initial test for acromegaly
IGF-1 is a somatomedin secreted from the liver. Growth hormone when combined with thyroid hormone stimulates linear skeletal growth in children through IGF-1. it stimulates protein synthesis in muscle (anabolic effect) in children and the release of fatty acids from adipose tissue (catabolic effect) in adults. Growth hormone secretion is pulsatile, has a short half life and reaches maximum life in the middle of the night. a single test for GH level is not accurate. IGF-1 levels are more stable an reliable.
The best confirmatory test for acromegaly is the oral glucose tolerance test. Glucose normally suppresses GH. in acromegaly glucose doesn't suppress GH
__________________________________________________ ___

Pro-Insulin assay
is used to distinguish autoimmne causes of hypoglycemia from insulinoma
high levels of pro-insulin represents insulinomas
answer a pro-insulin level when you see a case of recurrent fasting hypoglycemia, symptoms of weakness and a high c-peptide levels
__________________________________________________ ______
progesterone challenge
oral progesterone of 10mg dmily for 5 days is used in amenorrheic women after pregnancy is excluded to determine is the cause of the amenorrhea is due to outflow tract obstruction or secondary to the hypothalmus and pituitary. Look for a woman who has no periods and a negative hCG.
if bleeding occurs following the challenge, it indicated that there is adequate estrogen and that the outflow tract exists.
the next best test to do is to check the FSH, LH, prolactin and TSH to rule out a hypothalmaic or pirutitary etiologu
if no bleeding occurs, the next best test is to evaluate for an outflow obstruction with ultrasound, and with MRI if ultrasound isn't definitive.
__________________________________________________ _______

Metaiodobenzylguanidine (MIBG) scan
this is used to detect for pheochromocytoma if the blood urine testing (VMA, catecholamines, metanephrine) are positive and the scans are nondiagnostic.
MIBG is injected into the body. MIBG is an agent similar to norepinephrine and should be taken up by adrenergic tissue. the MIBG scan allows visualization of potential adrenal masses.
Answer MIBG scan when you see a patient with palpitations, intermittent tachycardia, and biochemical evidence of pehocrhromocytoma (e.g., catecholamines, metanephrines) when the CT and MRI can't visualize the lesion. you can't remove a pheochromocytoma and cure it, if you can't find it!
__________________________________________________ _______

RAIU (radioiodine uptake)
the measurement of orally ingested radioactive iodine (I121) that accumulates in the thyroid gland is used to differentiate Graves toxic multinodular goiter from thyroiditis/factitious.
RAIU is increased in Graves disease and wth hot nodules (toxic multinodular goiter, toxic solitary nodule)
RAIU is decreased when thyroiditis induces a thyrotoxic state or when a person is abusing thyroid hormone (thyroiditis factitia)
Answer RAIU scan when there is a case of hyperthyroidism. The Question will include a high T4 level, for instance and will ask what is the next step to determine the etiology.
__________________________________________________ __

TSH
Thyroxine (t4)
Thyroglobulin

serum levels of thyroid stimulating hormone (TSH) and throxine (t4) are measured to assess thyroid function (both hypofunction and hyperfunction). Thyroglobulin is used to monitor recurrence in pts who have been treated for papillary or follicular cancer of the thyroid.
TSH is produced by the pituitary to stimulate the thyroid. T4 is once of the principal metabolism-inducing hormones produced by the thyroid gland, and thyroglobulin is an iodine-containing glycoprotein synthesized by the thyroid gland from which thyroxine T4 is derived. Thyroglobulin is the storage form of thyroid hormone. Thyroid-binding globulin is the transport protein.
TSH and T4 are best initial tests
thyroglobulin is to follow the response to the treatment of thyroid cancer.
__________________________________________________ ______
Venous blood gas

use VBG for follow up of diabetic ketoacidosis (DKA), a VBG could replace the arterial samle for assessing the degree of acidosis and hypercarbia when all you need is ph and not the pO2 while avoiding an additional painful procedure
________________________________________________

PHARM
for pharm I'll be finishing infectious disease and moving on to neurology
________

an HIV + male comes with several weeks of worsening difficulty and pain (odynophagia) with swallowing. His CD4 is 85

start treatment with fluconazole. Dysphagia and odynophagia in a person with AIDS and CD4 count <100 is often from esophageal candidiasis that there is no need for endoscopy, therefore treatment should be started right away.

imidazoles and triazoles such as fluconazole, impair the biosynthesis of ergesterol for the cytoplasmi membrane.
Azole antifungals are extremely benign. After prolonges use at high dose they can be hepatotoxic.
if the condition doesn't improve with fluconazole, endoscopy should be performed to diagnose rarer causes of esophageal lesions in HIV such as Herpes or CMV.
__________________________________________________ ______

34 year old man has been newly diagnosed with AIDS, His CD4 is 20 and his viral load is 450,000. He is currently asymptomatic

antiretroviral therapy should be started in most pts with CD4 cells <350 and in all pts with CD4 cells <200 two nucleoside reverse transcriptase inhibitors (NRTIs) with either efavirenz or a protease inhibitor (PI) should be started. Pneumocystis carinii penumonia (PCP) prophylaxis with TMP/SMX should be given when the CD4 drops below 200. Azithromycin for mycobacterium avium intracelulare (MAI) prophylaxis should be given to pts with CD4 cells <50

NRTIs inibit the transcription of HIV RNA into DNA, they terminate the synthesis of DNA chains. Normally, the single-stranded RNA virus is transformed into DNA, which is integrated into the host cell genome NRTIs do not affect host cell DNA plymerase. PI's inhibit the viral protease enzyme that is crucial to maturation of HIV, allowing budding from host cells.
All NRTIs can cause lactic acidosis. All PI's can cause hyperlipidemia and hyperglycemia. ZIdovudine can cause aneia. Didanosine and stavudine can cause pancreatitis and neuropathy. Tenofovir can cayse renal insufficiency and fanconi's syndrome.
_______________________________________
a man with peptic ulcer disease is admitted for a perforation, he is febrile, hypotensive and tachycardic. He is started on imipenem and gentamycin, three days later his creatinine rises to 3 and he has a seizure

Gentamycin, tobramycin and amikacin are aminoglycoside antibiotics. they are primarily effective against gram negative bacilli. Imipenem, meropenem and ertapenem are carbapenem antibiotics. they cover the aerobic gram negative bacilli covered by aminoglycosides as well as anerobes. The anerobic coverage of carbapenems is equal to that os metronidazole. in addition, carbapenems cover all methicillin or oxacillin sensitive staphylococci and streptococci.

carbapenems inhibit cell wall synthesis in the same way PCNS by inhibiting peptidoglycan synthesis. They binf to PCN-binding proteins. Aminoglycosides inhibit protein synthesis by binding to the 30s ribosomal subunit.
Aminglycosides lead to nephrotoxicity, and ototoxicity. rarely they cayse neuromuscular blockade and produce mysthenia like weakness. Carbapenems can cause neurotoxicity.
the pts seized as an adverse effect of the imipenem. the level of imipenem rose due to his renal impaired function and a toxic level caused a seizure.
__________________________________________________ __
23 year old woman with dysuria and WBC in her urine started on TMP/SMX, she develops a rash and is switched to cipro

43 year old woman with pyelonephritis and E.coli bactermia develops anaphylaxis to ampicillin, she is switched to aztreonam

Cirpo is broadly active against the majority of gram negative bacilli. in addition all fluoroquinolones are active against mycoplasma, legionella and chlamydia pneumonia. Aztreonam, a mono bactam the only drug in its class active against gram negative bacilli. There is no cross reactivity between aztreonam and PCN. Hence, aztronam can be used even if there has been anaphylaxis with PCN.

Quinolones, such as cipro, exhibit concentration dependent bactericida activity by inhibiting the activity of DNA gyrase and topoisomerase, enzymes essential for bacterial DNA gyrase and topoisomerase enzymes essential for bacterial DNA replication, aztreonam is a beta lactam antibiotic and inhibits cell wall synthesis.

quinolones can rarely be associated with central nervous sysem CNS toxicity. inhibition of chondroblasts can lead to achilles tendon rupture. this is why they are contraindicated in pregnancy and in children. Gatafloxicin can cause hyperlycemia, occasionally prolongs the QT interval, Aztreonam is generally safe wit few side effects.
__________________________________________________ __________________________________

64 year old man with diabetes develops an ulcer over his tibia. The X-Ray is normal, but the MRI shows osteomyelitis. a bone biopsy is performed, which grows an organism

osteomyelitis can't be effectively treated without a biopsy of the bone. Staphylococci accounts for only 50-70% of isolates. There is no way to determine the sensitivity of the organism without a biopsy. treatment is as follows
Staphyloccocus aureus: oxacillin sensitive organismss are best treated with nafcillin or oxacillin. ceftraiaxone or cefazolin can be used as well. if the pt suffers a rash due to PCN, cephalosporins can be safely used. Oxacillin organisms are treated with vancomycin, liezoli, or daptomycin.
Gram negative bacilli. is the organism is sensitive the drug of choice is a fluroquinolone, such as sipro, quinolones can only be used orally
The ESR can be used to follow the response to therapy. Most pts are afebrile at the beginning oftherapy and have a normal WBC count, so those measures can't be used to monitor therapy.
__________________________________________________ ____________________________

in a congressional room 10 envelopes with white powder arrives marked 'pls inhale upon opening'
inhalational anthrax spores, best prophylactic meds are cipro, or doxycycline, Amoxixillin is a second line agent when quinolones of doxycycline are contraindicated.
__________________________________________________ ________________________________

53 year old alcholic with cirrhosis an ascites is admitted for fever and abdominal pain, a paracentesis reveals 1.200 WBC that are 80% neutrophils

spontaneous bacterial peritonitis (SBP) is treated when the ascitic fluid reveals either total blood cells >500 or neutrophils >25-
cefotaxime is the treatment of choice for SBP
prophylaxis is indicated for the following conditions
pt has experienced previous SBP
Ascites occurs with variceal hemorrhage
Ascitic protein content <1.0 grams/dl
the best prophylactic medication is either norfloxacin or TMP/SMX
__________________________________________________ ________________________________
child comes to the ER with sudden onset of severe sore throat and stridor. voice is muffles, she is drooling, P.E in the operating room with direct fibroptic laryngoscopy reveals epiglottitis

in cases of epiglottitis, securing the airway with naotracheal intubation is the most urgent step. The airway may close off abruptly and the examination should be done in the OR

antibiotic treatment for epiglottitis is with ceftriaxone and clindamycin.

woman comes in for genital warts on her labia majora. They are not uncomfortable but she would like them removed

condylomata accuminata, can be treated with a large number of methods, equal in efficacy. these are cryotherapy, podophyllin, laser removal, trichloacetic acid and imiquimod. Imiquimod is a local immunostimulant that acts by local cytokine induction and leads to sloughing off of the lesion very slowely over a period of months.
all therapies with exception of imiquimod can damage surrounding healthy tissues.. podophyllin is teratogenic.
__________________________________________________ ___________________________________

ok we are officially done with infectious and now have moved on to Neurology we start by Dopamine agonists

carbergoline
Bromocriptine
Ropinerole
Pramiperxole
Pergolide

Dopamine agonists are the correct medications for the following conditions

Parkinson's disease, these medications are less potent but have fewer side effects when compared with levodopa/carbidopa.
Restless leg syndrome
prolactinoma, best initial over surgery

adverse effects
Nausea, vomiting, sleepiness, orthostatic hypotension, confusion and hallucination. Pergolide was taken off the market for adverse cardiac effects (valve regurgitation)
__________________________________________________ _____

Amantadine

best initial therapy for
Acute influenza A, within 48 hrs of onset of sx, Amantadine is inferior to ossltamivir
mild parkinsons disease (PD) is characterized by predominately a tremor, and the pt retains the ability to perform activities of daily living

Amantadine and rimantadine both inhibit the replication of the influenza A virus, Amantadine enhances dopamine release from the substantia nigra and prevents re-uptake.

adverse
it may cause confusion, hallucination, nightmares, depression and restlessness, also livedo reticularis can occur.


___
Reply

جوري
09-11-2008, 02:41 AM
I was struck and aggrieved by Snakelegs passing yesterday--

I'll be doing 10 of each today to make up for yesterday.

we are on pharm (Neurology) and Endocrine diagnostics
let's start with Pharm
___
45 year old woman evaluated because of painful contracted muscle. There is marked hypertonicity and hyperreflexia.

Baclofen and Tizanadine can be used for spasticity. Diazepam and dantrolene can also be used but are less effective
Vaclofen is an analog of GABA. the precise mechanism of relieving spasticity isn't known. Tizanadine is centrally acting alpha adrenergic agonist but does not lower pressure. Tinazadine inhibits pre-synaptic motor neurons.
The adverse effects of baclofen are dizziness, lighteheadedness, drowsiness, and weakness. Tinazidine causes dizziness, lighthededness and dry mouth
the rate of progression of MS can be slowed with beta interferon and copolymer I (glatirimer)
__________________
A man is evaluated for a motor neuron dz in which he has fasciculations, muscle wasting, hyperreflexia, weakness, and upgoing toes, but cognition is intact

ALS is a combination of upper and lower motor neuron dysfunction. There is weakness, wasting, hyperreflexia, and fasciculations, but intact cognition, bowel, bladder, and sexual function. Als progression can be slowed down by riluole.
Riluzole inhibits glutamate release presynaptically b blocking sodium channels.
Riuluzole can case nausea, vomiting, weakness, and spasticity.
________________
Lamotrigine

used to treat the following conditions
Seizure disorder, generalized or partial
Peripheral neuropathy
mood disorders, such as bipolar disorder
Lamotrigine blocks the repetitive firing neurons by inactivating voltage dependent sodium channels. It may selectively influence neurons that synthesize glutmate and aspartate.
Lamotrgine can cayse very severe skin reactions, such as steven's Johnsons syndrome.
______

A man comes to the ER because of sudden, severe, sharp, pain in his face while he was eating. It started near the side of the mouth and progressed to the ear. It resolved after a few seconds and then recurred. Currently his sensory exam is normal

caebamazepine is the best initial therapy for trigeminal neuralgia. it should be effective in more than 90% of cases. for those not responding. Phenytoin, baclofen or gabapentun maybe effective, surgery is performed if all meds fail

carbamazepine works in a similar fashion to phenytoin. It blocks the sodium channels at therapeutic concentrations. and inhibits high frequency repetitive firing in the neurons in culture. It also acts presynaptically to decrease synaptic transmission.

carbamazepine is also effective as first line medication for seizures and for peripheral neuropathy.
carbamazepine can cause diplopia, ataxia and hyponatremia, rarely it brings on aplastic anemia.
____
Levetricetam

Levetricetam is effective in the treatment of seizures.
it modifies the synaptic release of glutmate and GABA throigh action on vesicular function
levitricetam has virtually no adverse effects and does not require drug monitoring. Some pts feel weak, sleepy and dizzy.
____

pt with generalized recurrent seizure. there have been several short seizures over a few months that resolved spontaneously. There are no focal deficits, and all lab testing is normal.

these meds are acceptable as initial treatment for generalizes seizures.
it isn't clear which is superior

Valproate oxcarbzepine
Carbamazepine Phenobaribital
Phenytoin Topiramate
Lamotrigine

the adverse effects are


Valproate : weight gain, tremor, hepatotoxicity
oxcarbzepine: vertigo, Ataxia, hyponatremia
Carbamazepine : Hyponatremia, lethargy, neutropenia
Phenobaribital: sedation
Phenytoin : Gingival hyperplasia
Topiramate : Glaucoma
Lamotrigine : Stevens-Johnsons Syndrome

_________________________
62 year old man with diabetes comes to the office, he has been experiencing numbness and tingling in his feet for the last few months

either gabapentin or pregabalin is the best initial rx for peripheral neuropathy, especially from diabetes or HIV. Both of these meds can be used for seizure as well.
Both meds are analogs of GABA
the most common adverse effects are somnolence, dizziness, ataxia, headache and tremor.
peripheral neuropathy. Tricyclic antidepressants are also effective against neuropathy but can cause dry mouth, urinary retention and constipation because of their anti-cholinergic effect
________________

tricyclic antidepressants
Amitriptyline
Imipramine
Nortriptyline
Desipramine
Protriptyline

these are indicated for
Major depressive disorder
obsessive-compulsive disorder, phobia, anxiety
Attention deficit disorder
chronic pain syndrome, and peripheral neuropathies
nocturnal enuresis
migraine headache prophylaxis
TCA block the reuptake of amine neurotransmitters norepinephrine, and sertonin, leaving more of the neurotransmitters in the synapses.
TCA can cause many anticholinergic adverse effects, even at therapeutic doses such as dry mouth, urinary retention,, constipation, and orthostasis. At toxic level they can cause QT prolongation, arrhythmias and seizures.
______________________________________________

Catechol-Omethyltransferase (COMT)inhibitors:
Tolcapone
Entacapone
Al capone (kidddddddddddding) just checking to see if you are awake hehe

Tolcapone and entacapone are COMT inhibitors that extend the duration of levodopa replacement treatment for parkinsons' dz.
COMT inhibitors prevent the degradation of levodopa. Levodopa is normally inactivated by COMT. COMT inhibitors keep levodopa levels higher for longer. Entcapone andtolcapone can't be used alone, they must accompany Levodopa replacement.
this is for use in pts whose sx are progressing in spite increasing levodopa/carbidopa use.
Adverse: Orthostatic hypotension, dyskinesias, confusion and fatal hepatic failure.
__________________________________________________ _
72 year old female comes to office with tremor difficult walking, she is able to take care of herself at home but foesn't like her increasing slowness. She has mild cogwheel rigidity ...
Mild parkinson's disease is best treated with monoamine oxidase inhibitors MAOis selegeline, rasagilene tremor only
<70 year olds, the anticholinergics trihexphenidyl and benztropine
>70 yrs of age Amantadine
Moderate disease
Dopamine agonist: prmipexole and ropinerole: these have less postent, less adverse effects than levodopa replacement.
Severe parkinsons
Kevodopa/carbidopa (sinemet); if levodopa replacement doesn't control SX, a catechol-o-methyltransferase (COMT) inhibitor is added. Severe is defines as the inability to take care of oneself or to be employed.

levodopa: carbidopa is added to bloc peripheral metabolism so more levdopa enters the brain
selegeline and rasagiline. These MAO inhibitors block dopamine metabolism.
Amantadine, increases dopamine release from the substantia nigra
the adverse effects of these treatments is as follows
levodopa: dyskinesia, the on'off phenomenon
selegiline/rasagiline confusion, also, these medications interact with tricyclic antidepressants and SSRI's
anticholinergics dry mouth, constipation and urinary retention.
________________________________
Diagnostics we'll finish endocrine and move on to GI

24 hour urine for metanephrine, vanillylmandelic acid (VMA) and catecholamines

24 hour urine collection for metanephrine, VMA, and the catecholamines is the best initial test for pheochromocytoma.
if VMA is elevated, then do a CT or an MRI of the adrenal glands as the next best thing for pheo
if ct/MRI find nothing, a metaiodobenzylguaniidine (MIBG) scintigrapy would be the next best option. This is a norepinephrine analog that concentrates in the adrenals an pheochromocytomas.
Pheochromocytomas is associated with MEN II and MEN III.
look for a case of episodic HTN, particularly with flushing and diarrhea.
_____________
diagnostics GI



this is an abdominal x-ray
the abnormality is a small bowel obstruction, there are multiple air-fluid levels seen throughout the bowel
the most accurate test for small bowel obstruction is a combo od abdominal x-ray and CT scan. there is no blood test to diagnose obstruction, an there are no findings on endoscopy
pt usually has abdominal pain and distension, hypoactive or absent bowel sounds, possible electrolyte abnormalities such as potassium, ca2+, mg 2+ disorders, and failure to pass stoll or flatus. this is normal after abdominal surgery.
_____________________________


colon cancer on barium enema


diverticulosis on barium enema

Barium Enema creates contrast picture to outline the lining of the rectum and colon. BE can be used in detecting colorectal cancer of the rectum and colon. BE can be used in detecting colorectal cancers, polyps, diverticulosis, and bowel obstruction and to evaluate the extent of inflammatory bowel disease.
BE is inferior to colonoscopy if you are suspecting a lesion in the colon. Colonoscopy is also superior to enema in that it can directly biopsy lesions or remove polyps.
Pts who may have peritonitis or bowel perforation must NOT undergo this test because the contrast material can leak into the peritoneum through the perforation. BE is also contraindicated in diverticulitis, in which it in creases the risk of perforation.
__________________________________



this is an upper endoscopy showing a Barrett's esophagus
this occurs when the distal stratified squamous mucosa is replaced by metaplastic columnar epithelium containing goblet cells. The metaplasia results as a form of defense from the persistent acid refluc that cells get exposed to. Barrett's esophagus is not reversible but is preventable with PPIs
complications are ulcers, strictures and adenocarcinoma.
__________________________________



this is a capsule with a camera and a small transmitter. once swallowed it begins transmitting images of the inside GI to a receiver worn by the patient. the doctor can review 6 hrs with of pictures for any abnormality.

this is the procedure of choice for suspected small bowel bleeding that was not detected on upper endoscopy.
___________________________________
Amylase/Lipase levels
those are initial tests in the diagnosis of acute pancretitis. Lipase is more specific for the pancreas. Amylase can be elevated from damage to the salivary glands, esophagus or small bowel.
the patient presenting with the acute onset of severe epigastric pain radiating to the back. such pts may have a history of alchol abuse or gallstones.
most accurate test for acute pancreatitis is an abdominal CT scan is more accurate than Ranson's criteria. Pancreatic necrosis on a CT scan is extremely specific for severe pancreatitis and is the main indicator of the need for pancreatic biopsy.
____________________________________
Antiendomysial
antigliadin antibodies

serum assay for endomysial and antigliadin antibodies are made against wheat or gluten (gliadin) antigens and the villous (endomysial) linning of the small intestine. They are used to confirm dx of celiac disease
celiac dz, presents with oily, greasy diarrhea, foul smelling stool witout evidence of cheonic pancreatitis.
they are the tests to answer to confirm gluten sensitive enteropathy after sudan black stain has confirmed a fat malabsorpption
small bowel biopsy is the most accurate test for celiac dz. it must be done to excluse lymphoma as well to dx the disease.
_______________________________________

Anti-mitochondrial antibodies (AMA)
serum antimitochondrial antibodies is the best test to make a specfic diagnosis of primary biliary cirrhosis (PBC)
look for a middle aged female presenting with itching (pruritus) and elevated alkaline phosphatase with a normal bilirubin.
the most accurate test however is PBC is a liver biopsy.
__________________________________________________ ________
Anti-smooth muscle antibodies
serum assay detection of anti-smooth muscle antibodies (ASMA) is the best initial test for autoimmune hepatitis.. you know I personally always forget this one too so many anti this and anti that, so remember liver is smooth and you'll remember anti smooth lol
this is the test for a young woman with lover disease who doesn't drink or have inflammatory bowel disease or infectious hepatitis.
Auto-immune hepatitis is associated with antinuclear antibodies (ANA) and antibodies to liver/kidney microsome type 1 (anti-LKM1)
again liver biopsy is the most diagnostic .
____________________________
Barium esophagus
this is the best initial test for dysphagia. this is not in scenarios where there are clear signs of obstruction, for which upper endoscopy would be best initial. For achalasia barium esophagram shows a bird beak

at the esophageal junction. in diffuse esophageal spasm, barium esophagram shows a corkscrew

at the time of spasm.

Barium esophagram is the most accurate test for esophageal rings, webs, and diverticuli.
in achalasia, esophageal spasm, and nutcracker esophagus, the most accurate test is manometry. with cancer the most accurate is endoscoy for a biopsy.
Reply

جوري
09-12-2008, 11:04 PM
Salaamz..
sorry about yesterday, as usual with days missed will make up for it by doubling the number today, we start with pharm, finish Neurology and go on to oncology
___________

Ethosuxamide

is an anti-seizure med that is most effective in absent seizure
it reduced low threshhold calcium currents in the thalamic neurons. The thalamus plays an important role in the generation of 3-HZ spike-and-wave rhythms typical of absent seizures

most common side effect is drowsiness ..
this is best used for a child who has a 10 to 30 second period of loss of consciousness. formerly called petit mal. there can be eyelid fluttering, but the child does not fall or convulse. the child abruptly stops activity. then just as abruptly resumes it with no postictal sx or knowledge that a seizure has occured. Absence seizure are genetic and occur predominantely in children.
_____________________________
Capsaicin

is a cream that is used as an analgesic
derived from red peppers and inhibits the release of substance P from sensory fibers. Substance P causes pain if you recall
capsaicin causes burning and erythema at the site of application.
capsaicin is a second line medication not the most effective for the following conditions
postherpetic neuralgia
osteoarthritis
rheumatoid arthritis
diabetic neuropathy.
_________________________________

26 year old woman comes to the ER with unilateral throbbing headache, she is nauseated and notices some bright flashes of light. she had several of these before. she takes OCP and is mensturating

migraine headaches are aborted by dihydroergotamine and ergotamine combined with caffeine or one of the triptans. such as suatriptan, zolmitriptan, naratriptan or elitriptan.
triptans and ergotamine are sertonin agonists that constrict large vessles, they are therefore contraindicated in coronary artery disese
migraine headaches are best prevented by chronic use of beta blockers, such as propanaolol, the other prophylactic meds for migraine are valproic acid, gabapentin, calcium bloackers and tricyclic anti-depressants.

__________________________________________________ ____

60 year old man comes in for several severe unilateral, sharp headaches occuring over several days, they are associated with tearing on the eye, redness of the eye, ptosis, constriction of the pupil and rhinorrhea

cluster headaches are aborted by inahling 100% oxygen. migraine medications such as sumatriptan or ergotamine may be effective as well

cluster headaches are best prophylaxed with the use of lithium or prednison for sveral weeks, beta blockers are not as good a choice. the therapeutic benefit of propanolol to prevent headaches takes several weeks to become evident, and the cluster will be over by the time the beta blocker becomes effective.

_______________________________________
a 79 year old man is evaluated for increased forgetfullness and short term memory loss, he has no focal neurological defecits, the head CT shows bilateral symmetrical atrophy. the B12 level, rapid plasma reagin RPR and thyroid function tests are normal
the best initial therapy for alzheimer's dementa is wih donepezil, galantamine or rivastigmine.
the medications are ani-cholinesterase inhibitors. they act i the central nervous system to increase the level of ach.
cholinsterase inhibitors can cause diarrhea and mild nausea and vomitting, this is because ACH acts as a promotility agent got the GI tract.
_________________________________________

34 year old man is admitted because of severe lower extremeity weakness which has developed over the past several week. it started in his feet and has ascended his upper legs, he recently had an episode of gastrointeritis. the deep tendon reflecex of his lefs are absent, Electromyograhy and nerve condition studies are normal

Guillain Barre syndrome is treated with plasmapharesis or infusions or IV immunoglobulins. There is no benefit to combined therapy.
Death from Guillain Barre occurs from progression to involvement of the respiratory muscles. monitoring of the vital capacity is needed to determine if the pt needs intubation.
_____________________________________________

A man comes to the ER after a continuous seizure for the last 30mins. There has been no response to repeated injections of benzodiazepines (lorazapam, diazapam)
phenytoin or fosphenytoin us used to treat status epilepticus tat has not responded to benzodiazepines. Fosphenytoin is equally efficacious compared to phynytoin but has fewer side effects, such as hypotension and heart block.
if fosphenytoin does not work then treatment with phenobarbital for the rare patient that does not respond to benzodiazepine, fosphnytoin or phenobarbital, then general anesthesia with midazolam, propofol or pentobarbital should be used. Neuromuscular blocking agents such as vecuronium or succinylcholine, will stop the mucle contractions but will not stop the seizure in the central nervous system.
__________________________________________________

42 year old man comes to ER dept with sudden onset of very severe headache. There is neck stifness as well, his BP is 170/120. The CT scan shows a subarachnoid hemorrhage.

SAH is initially managed by controlling the blood pressure to systolic <160 to prevent further bleeding.
Nimodipine is a calcium channel blocker that is used to prevent subsequent vasospasm, Nimodipine is the standard of care after SAH to prevent stroke.

A cereberal angiogram is performed to guide operative therapy. Surgical clipping is performed to prevent rebleeding. Embolization of the site of aneurysm is useful and can be done by a catheter, a platinum wire is placed into the site of bleeding. embolization can be as effective as surigcally placed clip and doesn't require a craniotomy.
__________________________________________________ _

a man is evaluated for dilopia, and weekness. His anti-cholinesterase antibodies are positive. single fiber electromyography shows a diminishing amplitutude with repetitive stimulation

oral anticholinstrase meds such as pyridostigmine or neostigmine are the best initial therapy for myasthenia gravis.
These meds increase the amount of ACH at the neuromuscular junction b inhibiting its metabolism.
overdosage of pyridostigmine can lead to diarrhea, cramping, salivation and bronchorrhea.
if pyridostigmine is ineffective, a patient should have the thymus removed, Thymetcomy is generally only effectve for those under 60 years old. Older pts or those in whom thymetcomy is ineffective, can be treated with prednison, then switched to azathioprine, mycophenolate or cyclosporine. these medications are steriod sparing, acute, severe mysthenic crisis is treated with plasmapheresis or IV immunoglobulin.
___________________________

still pharm but now moved on to oncology meds.
________________________

Imatinib (Gleevec)
is used for CML chronic myelogenous Leukemia. when the Q is of best initial therapy. Pts will have high white cell count consisteing predominantly of neutrophils of normal appearance. the leuckocyte alakaline phosphatase score should be low and the philadelphia chromosome present.

Imatinib inhibits production of the philadelphia chromsome. which codes for Bcr?Abl gene, which produces tyrosine kinase.

Imatinib has very few adverse side effects. occasionally muscle cramps, gynecomastia.
________________________________________

Aromatase inhibitors
Letrozole
Anastrazole

these are the newest adjuvant hormonal therapy after resection of breast cancer. The benefit is best for post menopausal women with estrogen receptor positive breast cancer. It is not clear if aromatase inhibitors anastrazole, letrozole are superior to tamoxifen in terms of efficacy. Adverse effects are certainly less than those of tamoxifen.

Aromatase inhibitors block the production of estrogen in all tissues, especially in the ovary and adipose tissue, they decrease estrogen levels 97-99%
the most common side effects are bone loss secondary to inhibition of estrogen, stroke, endometrial cancer, venous thromembolic events, hot flashes and vaginal bleeding, are all less common with anastrazole than tamoxifen. Bone fractures and musculoskeletal pain is more frequent..
___________________________________________

will finish diagnostics after break fast insha'Allah

__________________________________________________ _
posst iftar.. ahhhhhhhhhh

alright diagnostics continuing on with GI

Bernstein test

done to diagnose GERD
it involves the placement of a nasogastric tube into the esophagus with the tip at the end of the esophagus near the gastroesophageal junction. Dilute HCL acid is instilled or dripped into the esophagus in an attempt to reprodice the symptoms of the reflux disease. This is an older test and has been made obsolete by the use of empiric trials of PPIs or the 24 Hr PH monitor.
just be aware of it but it is for practical purposes replaced.
____________________________
Bleeding scan
or technetium labeled RBC scanning is performed in pts with small amount of active lower GI bleeding in which the bleeding site could not be identified with endoscopy. Bleeding scan is sensitive but not specific, if the bleeding scan is positive you can't identify the precise etiology of the bleeding. Angiography is another diagnostic option for the patient with persistent, severe GI bleeding without an identified source.

done on a pt with severe GI bleeding in whom the lower endoscoy is unable to find a source, the bleeding continues and the pt may still require transfusions.
________________________________________
Ceruloplasmin

serum ceruloplasmin levels and the presence of kayser Fleischer rings on exams are the best initial test for Wislons's dz.
look for pt with the combination of unexplained hepatic and psychiatric abnormalities with movement disorder.
An extremely low serum ceruloplasmin levels should be taken as strong evidence for diagnosis of wilson's disease.
twenty four hour urine copper excretion will be abnormally high. The most accurate test is the lover biopsy. Wislon's disease is a disease of copper overload. This is because of the inability of the lover to incorporate copper into the protein ceruliplasmin for excretion into the bile. This leads to hepatic copper accumulation and injury. This eventually spills into the blood stream and deposits into the brain (behavior and movement changes) kidney (renal insufficiency and corenea (kayser_Fleischer rings)
________________________________________________

Colonoscopy
recommended as screening for colon cancer.
General population
every 10 yrs for pts over 50, colonoscopy is superior to occult blood testing or sigmoidscopy
single family member with colon cancer
start at age 40 or 10 years earlier than the family member's diagnosis which ever is earlier.
Heriditary non-polyposis cancer syndrome (HNPCC or Lynch syndrome)
Three family members, two generations, one premature. Premature means the cancer was before age 50. Start at age 25 and repeat every 1-2 yrs.
__________________________________________________ _
D-Xylose test
used in determining the etiology of different malabsorption syndromes. D-Xylose is used to differentiate between intestinal malabsortpion and chronic pancreatitis.
Celiac disease is the most common cause of intestinal mal-absorption. D-Xylose is a sugar that should be absorbed if the small bowel mucosa is intact. It doesn't requie digestive pancreatic enzymes. A low D-Xylose level in the urine indicated that the intestinal mucosa is defective and that D-Xylose was not absorbed
Lok for a patient with foul smelling stool without blood, who has weight loss, malnutrition, fat in still and positive sudan black stain.
The most accurate test of course if a biopsy for histology and architecture.
D-Xylose is abnromal with celiac, tropical sprue and whipple's disease.
__________________________________________________ _

Endoscopic retrograde Cholangiopancreatography (ERCP)

is a direct visualization of the billiary tract.
ERCP is the most accurate diagnostic test for stones in the billiary system, ERCP is also the most accurate test for primary sclerosing cholangitis. (PSC)

in sclerosing cholangitis there are multiple intrahepatic and or extrahepatic structures of the billiary tract on cholangiography that give the billiary radicles a beaded appearance.
The strongest indication for ERCP is whe the pt has high fever, chills, right upper quadrant abdominal pain and jaundice. ERCP i both diagnostic and therapeutic because it can remove the stone. Fpr Primary sclerosing cholangitis look for a pt with history of ulcerative colitis, complaining of pruritus and jaundice with negative hepatitis profile and negative abdominal sonogram.
__________________________________________________ ____

esophageal Manometry
is used to establish the diagnosis of dysphagia or unexplained chest pain where there is no mechanical obstruction.
esophageal manometry consists of a transducer placed in the esophagus to record pressure.
Look for dysphaga with inconclusive barium radiograph or upper endoscopy, or barium prior to ding manometry; for example achalasia, diffuse esophageal spasm, or nutcracker esophagus.
_____________________________________________

Fecal occult blood testing

(FOBT) is the screening of normal brown stool for microscopic amounts of blood not grossly visible. Fross blood or dark stool such as melena doesn't occur until >100ml of blood has been lost in the bowel. FOBT will detect as little as 5-10ml of blood
FOBT is screening test for occult blood in the stool. FOBT is an option for screening for colon ca, in those above the age of 50. In colon ca screening the three methods of screening are
colonoscopy every 10 yrs
sigmoidscopy every 3-5 yrs
FOBT after age of 50
a positive FOBT must be investigated with a colonoscopy.
___________________________________

Fecal leukocytes

used to assess the etiology of acute dirrhea
invasive and inflammatory dirrhea can give blood as well as fecal leukocytes. Diseases such as salmonell, shiegella, campylobacter, etc can give both blood as well as white cells in the stool.
look for pt with dirrhea with no blood in whom you are tryin to see is there is an invasive form of dirrhea. Fecal leuckocytes are not found with dirrhea secondary to viruses, most protozoans or pre-formed toxins as Bacillus cereus or staph.
__________________________________________________ ____
Gastrin Level

serum gastrin should be measured wen you suspect Zollinger Ellison syndromes
look for ulcers tat are large >1 cm, recurrent, distal, and multiple. Gastrin levels greater than 1000pg/ml is diagnostic.
Remember that everyone on PPI has high gastrin level.
The secretin stimualtion test can confirm ZES when serum gastrin concentration testing is nondiagnostic. In ZES gastrin levels go down with secretin
ZES is associated with MEN I
Hyperparthyroidism
pituitary adenomas
pancreatic islet cell/Gastrointestinal adenomas.
________________________________________________
Reply

جوري
09-14-2008, 12:28 AM
Today's 5/5
still on oncology in Pharm.
_________________
Riuximab
is the best therapy to administer in addition to cyclophospamide, doxorubicin, vincristine, prednsione, CHOP as chemotherapy when the patient has non-hodgkin's lymphoma. Rituximab is also used to treat Rheumatoid arthritis when the disease is described as severe and not responsive to other disease modifying drugs. such as methotrexate or tumor necrosis factor inhibitors such as infliximab.

Rituximab is an antibody that depeltes T-cells. It bunds to the CD20 antigen and prevents cell activation and diferentiation.

adverse: asscoaited with the development of pneumonia and septic arthritis. IV infusion can cause surface urtricaria, angioedema and bronchospasm.
_____________________________________
All -trans Retinoic acid (ATRA)
is the best initial therapy for acute promyelocytic leukemia (M3) ATRA is added to the usual cytotoxic agents daunorubicin and cytarabine.

ATRA works by accelerating the terminal differentiation of malignant promyelocytes to mature neutrophils. This leads to apoptosis and complete remission without myelosuppression and bone marrow hypoplasia.
25% of pts develop retinoic acid syndromes' which consists of fever, edema, pulomnary infiltrates and hypoxia that may be from capillary leak. Hyperleukocytosis with extremely high white blood cell count may occur, it is treated with dexamethasone.
________________________________________

Trastuzumab

is an additional adjuvant therapy in case of metastatic Br. Ca.. it is used in addition to surgery and chemo.

it is a monoclonal antibody against the HER2/neu(ErbB-2) member of the epidermal growth factor famiy of cellular receptors. Those are exposed in increased amounts in some patients with breast cancer.

Trastuzumab is relatively benign when compared to cytotoxic chemotherapy. There is no bone marrow supression and no cardiac toxicity. its side effects are akin to those of other monoclonal antibodies and ma include fever, rash, nausea, dyspnea and rashes. allergic reaction may also be observed.
_______________________________________
Anti-thymocyte globulin (ATG)
is the treatment for aplastic anemia when the question describes panctopenia of unclear etiology and bone marrow transplant is not possible then the answer is ATG. ATG is used to treat aplastic anemia in combination with cyclosporine for those unable to undergo transplanation. ATG can also be used to prevent graft vs host disease and to prevent renal transplant rejection.
ATG works by removing cytotoxic T-lymphocytes from circulation this allows the bone marrow to regrow.
ATG can cause serum sickness, rash, anaphylaxix, thrombocytopenia and myalgias.
_______________________________________________
Azacytadine

this is the treatment for myelodsplastic syndrome
older pt with anemia elevated mean corpuscular volume (MCV) and a small percentage of blasts. The B12 and folate levels will be normal. the bone marrow hypercellular, spenomegaly and hepatomegaly are common.
azacytadine works by incorporating itself into DNA and RNA and inhibiting methylation of DNA. It is a pyrimidine nucleoside analog of cytadine. It is direcly cytotoxic t abnromal bone marrow hematopoietic cells.

major toxicities include myelosuppression and rater severe nausea and vomiting.
_____________________________________________

part two diagnostics after tarawee7 insha'Allah
__________________________________________

ok back with Gi
__ _________
Gastrograffin
water-soluable form of a radio-opaque contrast material is when where there is the possibility that there has been organ perforation and the material may leak through into the peritoneum. Gastrograffin is used as the diagnostic test of choice for Boerhaave's syndrome or esophageal perforation, because barium going through a perforation causes inflammation.
Look for a patient presenting with a recent history of severe retching followed by severe chest pain, dyspenea, shock, and possible crepitation. Initial chst x-ray would show free air in the mediastinum or pertoneum
most accurate test however i a ct scan
_____________________________________
HFE gene and C282Y mutation both lead to the over absorption of iron in the dueodenum. They are confirmatory tests for hemochromatosis. These genetic tests and an MRI can eliminate the need for liver biopsy to diagnose hemochromatosis.

HFE gene and C282Y mutation are found in pts wit characteristics of hemochromatosis and abnormal iron studies consistent with hemochromatosis. The patient will have liver disease, skin hyperpigmentation, joint pain, diabetes, infertility, the iron and ferritin levels will be elevated and the iron binding capacity will be markedly diminished.
______________________________________________

HIDA scan
is used when you suspect acute cholecystitis with a nondiagnostic ultrasonography.
Answer HIDA when you see either
Gallstones without gallbladder edema and ultrasound Murphy's sign or
Gallbladder edema and ultrasound Murphy's sign without gallstones.

HIDA is taken up by hepatocytes and excreted into bile concentrated in the gallbladder. If the Gallbladder does not opacify, it means that there is an obstruction of the cystic duct. leakage into the pericholecystic space suggests perforation. On a normal scan the gall bladder should opacify.
The HIDA is not recommended in critically ill pts who should be taken to the operating toom. If the diagnosis is obvious, the HIDA scan is not necessary, surgery is necessary.
__________________________________________________ ___________

PAS staining

PAS staining on a small bowel biopsy is the most accurate test for Whipple's disease/
Look for chronic multisystem illness, initially rheumatological (80%) then with fat malabsorption (75%), fever (50%) and ocular and neurological complaints.

initial course of ceftraiaxone followed by trimethorprim sulfamethozazole for one year.
__________________________________________________ ________________

24 PH monitoring, is indicated to document abnormal esophageal acid exposure.
it is appropriate for the following
Persistent sx of GERD after at least 4 weeks of RX with a PPI
to excluse GERD in pts with chest pain
GERD sx with negative upper endoscoy when you are considering antireflux surgical repair.
to assess the efficacy of antureflux surgery
to evaluate pts with laryngitis, pharyngitis, chronic cough, non allergic asthma (reflux induced asthma)
______________________________
Reply

جوري
09-15-2008, 12:39 AM
ok.. today 7:56 PM 9/14/2008-- 5/5 still on with Oncology in pharm -- pls again bear in mind this is only the latest and few excerpts of high yield review and by no means is it a substitute for any course you are taking

Cyclophosamide

this is an alkylating agent, that is widely used as an immunosuppressive and cancer chemotherapeutic agent.
it works by cross linking DNA this 'breaks' the DNA and broken DNA can't participate in cell division. Instead, it is metabolized to acrolein.

cyclophosphamide can cause hemorrhagic cystitis from the acrolein
it is used for the following conditions:

Acute leukemia and lymphoma, retinoblastima and a wide variety of cancers
Wegner's granulomatosis
polyarteritis nodosa
lupus nephritis
nephrotic syndrome (particularly when it does not respond to steroids)
rheumatoid arthritis that is very severe.
_________________________________________
cyclosporine
cyclosporine is a T-cell inhibiting immunosuppressive agent that is used to prevent organ transplant rejection and to control autoimmune diseases not reponsive to another initial therapy.
cyclosporine inhibits calcineurin. calcineurin produces interleukin 2. interleukin 2 ativates T cells. Tacrolimus is the other calcineurin inhibitor.
The most common side effect of cyclosporin is nephrotoxicity. there is increased susceptibility to fungal and viral infections. consumption of grapefruit juice increases its level with potentially adverse effects. Grapefruit juice inhibits the hepatic p450 system
cyclosporin is the best initial therapy for the following conditions
organ transplant rejection
aplastic anemia
to prevent graft vs host disease
it is occasionally used as a second line therapy to treat rheumatoidarthritis, ulcerative colitis, Behçet's disease, and psoriasis. It is used to treat myasthenia gravis to spare the pt from chronic steroid use.
_________________________________________________
Thalidomide
is an effective treatment for multiple myeloma (this was taken off the market for extreme tertogenicity way back when, it is back again)
Thalidominde in MM is equal in efficacy to combination chemotherapy
other uses are
Leprosy skin lesions
apthous ulcerations
Behçet's disease
Thalidomide has anti-inflammatory effects that are mediated through its inhibition of tumor necrosis factor, it also inhibits angiogensis.
Thalidomide is extremely teratogenic, periperal neuropathy and sedation also occur. Dry mouth and skin are common.
__________________________________________________

cladribine and pentostatin

cladribine and pentastatin are the treatment for hairy cell leukemia if the question describes a pt in his or her 50's with pancytopenia, splenomegaly and a 'dry' tap of the bone marrow, then the answer is cladribine or pentostatin. in addition cladribine may have some efficacy in treating lymphma, cheonic lymphocytic leukemia (CLL) and waldernstrom's macroglobulinemia
cladribine and pentostain are purine analogs tha are naturally toxic to lymphocytes in both the resting and dividing cells.
Myelosuppresions is the most common adverse effect.
_______________________________
67 year old gent evaluated for markedly elevated WBC count of 85,000 of which 90% are normal appearing lymphocytes 'smudge cells' are visible on smear. he has anemia, thrombocytopenia, and an enlared spleen.

fludarbine is the best therapy for advanced chronic lymphocytic leukemia (CLL) the presence of anemia. thrombocytopena and splenomegaly indicate tha this is advanced stage disease. fludarabine can be combined with cyclophosamide or rituximan. chlorambucil is used only as palliative to decrease the white cell burden of older pts.

fludarabine works by interfering with DNA synthesis and DNA repair through inhibition of DNA polymerase. The triphosphate form can also be directly incorporated into DNA, resulting in inhibition of DNA synthesis and function. Chlorambucil is a bifunctional alkalyating agent that is rapidly and comletely absorbed from the GI tract. its major metabolite, phenyl acetic acid mustard, is phamarcologically active against CLL
both fludarabine and chlorambucil cause myelosuppression.
__________________________________________________ ____

Now diagnostics
_______________________

secretin stimulation test

this test is rarely used, but it tests for both Zollinger Ellison Syndrom and chronic pancreatitis.
for ZES you inject secretin IV and measure the level of gastrin in the blood. secretin should inhibit gastrin and lower levels of gastrin in a normal person. in ZES there is a failure to suppress secretin levels

chronic pancreatitis: normally secretin should induce a large release of bicarbonate from the pancreas into the duodenum. During the test you place a nasogastric tube into the duodenum and measure the release of bicarbonate from the sphincter of oddi. with chronic pancreatitis the pancreas is too burnt out and tired to do anything.

this test is the answer when a pt has large recurrent and mulciple ulcers, but has a nondiagnostic fasting serum gastrin concentration. for chronic pancreatitis, look for a pt with fat malabsorption with no evidence of celiac disease and a normal D-Xylose test.
_____________________________________________

screening upper endoscopy
should be perfomed in pts with GERD in whom the symtoms have been present for more than 5 yrs. you are screening to see if there is Barrett's esophagus. Which is whn the lower esophagus is transforming from squamous to columnar epithelium. screening for the presence of esophageal varices should be done in those with cirrhosis

Barrett's esophagus is treated with PPI with a repeat upper endoscopy every 2-3 yrs. Low grade esophageal dysplasia is treated with PPIs with repeat upper endoscopy every 6 months. those with varices should receive propanolol to decrease the risk of bleeding.
_________________________________________

72 hr fecal fat
this is the oldest test for intestinal mal-absorption, as the name implies, you are to collect 3 days worth of stool in order to have a very sensitive test for fat malabsorption. a single test of a sudan black for the presence of fat is usually sufficnet

72 hour fecal fat colletion is the single most accurate test for malabosprtion; however it is virtually never done.
________________________________________

string test

this is an old test for giardiasis, it samples duodenal mucosa without the need for endoscopy, a string is swallowed and left in place with one end in the duodenum the other hanging outside the pt's mouth.
this test is hardly done, stool ova and parasite testing is sufficiently sensitive and less invasive. stool giardia antigen testing by ELISA is the single most accurate test for giardia and considerably less invasive.
_______________________________________________

sudan black stain

stool staining with sudan black is uded to detect steatorrhea as a sign of malabsorption.
72 hour quantative estimation of stool fat is the most accurate test because of increased sensitivity.
this is done on pts with chronic pale, greasy, malodorous diarrhea and no evidence of infection

statorrhea >>intraluminal>>>problem= impaired digestion of nutrients<<<pancreatic exocrine insufficiency>>>or bacterial overgrowth>>>orcholesostatic liver disease


statorrhea >>mucosal malabsorption>>>>problem=small intestine>>>celiac sprue>>>ortropical sprue>>>or whipple's disease>>>o rintestinal resection..


statorrhea >>postmucosal>>>problem=transport>>>>intestinal lymphagiectasia>>>or whipple's disease>>>or lymphoma

_____________________________
Reply

جوري
09-16-2008, 01:02 AM
today's 5/5 finishing off oncology in pharm-- and continue on with GI in diagnostics and move on to pulm
______________________

Flutamide
Bicalutamide
Nilutamide

These are nonsteroidal antiandrogens that are used for prostate cancer and hair loss, they compete with testosterone for binding androgen receptors.

They can cause gynecompastia, galactorhea, hot flashes, impotence and hepatic injury

They are the best initial therapy for metastatic prostate ca. as well as hair loss and hirsuitism in women. They should be started before gonadotropin-releasing hormone )HnRH) agonists in metastatic prostate cancer to prevent a flare-up of disease from the initial bump up in testosterone levels. GnRH agonsts initially cause a bump in gonadotropin levels.

_________________________________________

47 year old woman with breast ca. presents with back pain, she has spinal tenderness, hyperreflexia of the lower extremities and decreased sensation below her umbilicus extending through the lower half of her body

steroid adminstration is the most important action in a pt with acute cord compression. This pt has back pain, tenderness, and hyperreflexia. the most important thing is to relieve the pressure on her spinal cord with steroids. Steroids can be given faster and act faster than radiation. steroids decreases the edema surrounding the tumor. MRI is the most accurate test of the compression, but it isn't as important as steroids. Tamoxifen and the aromatase inhibitors, such as anastrazole and letrozle are useful but they don't work acutely.
___________________________________________

68 year old man presents with back pain from metastatic prostate ca. The MRI shows large spinal lesions that are near the cord but not compressing it .

prostate cancer that is close to impinging on the spinal cord should first be treated with androgen blockade with flutamide, bicalutamide, or nilutamide. Gonadotropin releasing hormone (GnRG) agonists should not be used as first in the case of impending cord compression. because they will cause a brief flare up in luteinizing hormone (LH) and follicale stimulating hormone (FSH) level and subsequently in testosterone levels, perhaps enough to cause cord compression. Block the androgen receptor first. Orchiectomy is the fastst way to lower androgen levels but isn't easy, or as acceptable as medications. Ketoconazole provides near orchiectomy levels of andogens decrease but has much more liver toxicity.
____________________________________________

75 year old man has recently undergone chemotherapy with cisplatin containing regimen, and has developed severe nausea and vomiting
the most effective therapy for severe nausea and vomiting induced chemotherapy is ondansetron, granisetron, dolasetron, palonosetron or aprepitant, corticosteroids, such as dexamethasone, also inhibit nausea

ondansetron and others of this group work by selectively inhibiting 5-hydrohytryptamine (5HT3) receptors. Sertonin is released by the small bowel in response to chemotherapy. 5 HT antagonists inhibit stimulation of the vagus nerve that leads to the vomiting reflex. Aprepitant is an inhibitor of the neurokinin-1 receptor.
_____________________________________________

37 year old woman comes to see you for evaluation for breast cancer. Her mother and sister both had breast cancer before the age of 45. Her mammograph is normal
Tamoxifen should be recommended for patients at high risk of breast cancer. Pts. with two or more first degree relatives (mother/sister) with breast cancer have a markedly increased risk of breast cancer--ssomewhere four to six times higher than the average. Tamoxifen decreases the risk of br. ca. in these pts

Tamoxifen and raloxifene act as estrogen receptor agonists or anatagonists. depending on the tissue. They are estrogen receptor antagonists on breast tissue. This is the basis of their efficacy in treating br cancer. they act as estrogen receptor agonists on the liver and bone. This is the basis of the beneficial effects of raloxifene on osteoporosis. Tamoxifen is an agonist o the uterus, raloxfene isn't.
Tamoxifen can lead to thromboembolic disease, endometrial cancer, menstural irregularities, sexual dysfunction and hot flashes.
________________________________________________

diagnostics finish Gi move on to Pulmonary.



Fasting Transferrin saturation

serum measurement of fasting transferrin saturation is the best initial test to diagnose hereditary hemochromatosis (HH) as well the screening test for asymptomatic family members . Fasting transferrin saturation = (serum iron/TIBC)
this is the umbrella to tie a case of liver disease with joint pain, diabetes, cardiomyopathy and skin hyperpigmentation and infertility.
Liver biopsy is the most accurate test
the pathogensis is genetically determined abnromal increase in intestinal absorption of iton
the optimal timing for screening family members is between the ages of 18-30 before organ damage (e.g cirrhosis) has occurred.

_____________________________________________

serum assay of trypsinogen
is used when you suspect chronic pancreatits in a patient with steatorrhea. This is the only test that is specific for steatorrhea pancreatic in origin. Trypsinogen evels are low if the fat mal-absorption is from burned out pancreas.

this is the answer only after Sudan black stain has confirmed fat malabsorption. Antigliadin and anti-endomysial antibodies are normal. Low trypsinogen levels are specifc but not sensitive for chronic pancreatitis.

__________________________________________________ _____

urea breath test
urea breath testing is the measurement of radiolabeled carbon dioxide after oral ingestin of labeled urea. H. Pylori splits urease. The labeled carbon is in the urea and is released as labeled carbon dioxide

urea breath testing is best used to confirm the eradication of H pylori immediately after treatment. serology does not rapidly improve after treatment. If an ulcer is found at endoscopy, the test of choice will be biopsy, not the urea breath test.

this is the answer when a pt with epigastric pain that doesn't improve or that recurs after proton pump inhibitor and antibiotics have been given. There is no routine test of cure of H pylori. you don't do the test if the patient feels better after treatment.
________________________________________
Pulmonary



this is a bronchoscopy, smal thin fiberoptic scope through the nose, through the vocal cords, through the carina and into mid sized branching bronchus.
the most accurate diagnostic test for infections such as Pneumocystis or TB. In addition, bronchoscopy is often the most accurate test for lung cancer or other mass lesion of the lung, such as abscess...this is if they are located centrally and are too deep to be reached by a transthoracic ling biosy.
the most accurate ofcourse if an open lung biopsy.
____________________________________



this abnormality is a right upper love cavitary lesion with some post-obstructive penumonia. the most likely cause is a bacterial lung abscess. although TB or a cavitating lung ca. can also give a lung cavity.

the most accurate test to do is a lung biopsy. must have tissue to obrain a specific microbioloic diagnosis.

for lung abscess the pt will be febrile with a cough for the last several weeks, weight loss, and possibly malodorous sputum. Lung abscess is most common in those with an increased risk of aspiration, such as alcoholics and pts after a seizure, stroke or intubation.

>>>>>>>>>>>>>>>>>>>>>>>>>>>
Reply

جوري
09-17-2008, 12:47 AM
6:58 PM 9/16/2008-- few mins before breakfast here, so let's see what I can get down.. still on pulmo with diagnostics and today we start opthalmology in pharm
___________



this film shows infiltrate, as well major structures of the ling like fissures,. the most likely etiology of this infection is streptococcus pneumonia, although H flu is also possible

the most accurate test of any bacterial infection is a gram stain of the sputum and a culture. Culture is always the right answer as most accurate diagnostic.
Xray is best initial test whenever there is cough and shortness of breath
__________________________



this xray shows bilateral interstitial infiltrates.
interstitial infiltrates can be caused by viral pneumonia, pneumocystis, mycoplasma, chlamydia, coxiella, and sometimes legionella
the most accurate test depends on the etiology:
Mycoplasma, chlamydia, coxiella-serology (antibody titers)
Legionella-urine antigen testing, culture on specialized media
PCP-bronchoalveolar lavage, increased LDH
__________________________________



chest xray with right pleural effusion
the most common etiology is congestive heart failure or pneumonia, Any infection can cause an effusion, although bacterial infesction is the most common. Less common causes are truama, cancer and connective tissue diseases such as lupus.

the most accurate test for pleural effusions is a thoracentesis for fluid analysis. The MRI or CT is never accurate for an infectious of neoplastic process. A high protein level indicated infection or cancer. A high cell count an/or low ph (<7.2) indicates an infection.
___________________________________________



the xray shows a pneumothorax
pt will present with acute shortness of breath and decreased breath sounds on one side, hyperresonance to percussion and tracheal deviation. It maybe spontaneous, from a central venous cacheter placed in the subclavian vein, or from ruptured bullae from pneumocystis or COPD.
the next thing is to re-inflate the lung, is there is a 'tension' pneumothorax, then a needle decompression should be performed. All large pneumothoraces require the placement of a chest tube.
_____________________________________________



flow volume loops are graphical representation of inhalation and exhalation. the inhalation starts at the far right and the tracing moves down. Exhalation begins at the far left and the tracing moves up as you exhale. the loop tells not only how much you inhale and exhale, but also how fast.

this test is to assess various forms of obstructive and restrictive lung disease; for example to tell the difference between COPD an interstitial lung disease. The flow-volume loop has no use in assessing infections, mass lesions, or reactive airways disease.
The flow volume loop is a graphical representation meant to uncover subtle defects in vntilation not easily represented on the numerical values of the other parts of the PFT's.

_________________________________________________
now ophthalmology/ psychiatry in pharm.
____________________________________-

70 year old woman comes to ER with sudden onset of painful, red eye. The cornea is hazy and edematous. The pupil is fixed at the midpoint and is nonreactive. visual acuity is decreased...
acute closed-angle glaucoma is treated with medications to increase drainage of aqueous humor and to decrease the production of aqueous humor. Timolol and pilocarpine drops are the best initial therapy. They constrict the pupil.
Mechanism:
beta blockers (timolol, betaxolol) constrict the pupils and increase drainage of fluid as well as inflow.
Cholinergic agonists (pilocarpine) increase drainage of the aqueous humor by opening the canal of schlemm
Carbonic anyhdrase inhibitors (acetazolamide, dorzolamide) decrease the production of aqueuous humor.
Alpha adrenerigc agonists (apraclonidine, brimonidine) increase drainage
prostaglandin analogs (latanoprost or trvoprost) increase drainage.
if medical therapy doesn't control ocular pressure, a laser trabeculoplasty or iridotomy is performed. this puts a physical hole in the iris allowing for drainage of fluid.
__________________________________________________ _______

two pts present with progressive loss of vision. Both are older and generally healthy, without diabetes of HTN. One has atrophic or 'dry' macular degeneration with multiple drusen visible. The other has 'wet' or proliferative disease.

dry or atrophic, macular degeneration has no proven effective therapy. Laser therapy doesn't help. and the disease will progress slowely over time. Antioxidants, such as vit A,C, and E as well as Zinc may offer some benefits and should be used

wet (proliferative) exudative macular degeneration has more therapeutic options. Treatment is with vascular endothelial growth factor (VEGF) inhibitors such as ranibizumab, vevacizumab or pegaptanib. These deugs are unique in that they can actually restore vision, not just delay worsening. Laser photocoagulation can be useful; verteporfin is a photosensitizing dye that is injected and actvated by subsequent laser irradiation to produce vascular damage.
_____________________________________________

psych.
Resperidone

resp. is the rx for chronic schizophrenia.
the precise mechanism is unknown. Resperidone has an affinity for antagonizing both sertonin and dopamine receptors.

resp. can cause weakness (asthenia), sedation, difficulty concentrating, and hypotension. It can cause elevated prolactin levels and weight gain, sexual dysfunction. The incidence of tardive dyskinesia is greater with risperidone than with the other second generation antipsychotic medications.
________________________________

olanzapine, ziprasidone, quetiapine, aripiprazole, clozapine, and Risperidone
are all second generation 'atypical' antipsychotic medications. Clozpine the most efficacious but has the highest incidence of adverse effects such as agranulocytosis.
the precise mechanism of action is unknown, they all bind to sertonin and dopamine receptors. most also bind to central alpha receptors as well as the histamine receptors.

they can all cause hypotension, and somnolence, most cause weight gain, Ziprasidone can prolong QT interval the most. Rispridone has the highest incident of tardive dyskinesia and prolactin elevation. Clozapine is the only one that suppresses the bone marrow.
______________________________________________

Selective sertonin reuptake inhibitors
Fluoxaine
Paroxtine
Sertraline
Fluvoxamine
Escitalopram

are the most commonly used anti-depressants
SSRI"s work by blocking the reuptake of 5-hydroxytryptamine [5-HT] presynaptically. this leaves more 5-HT to stimulate the receptor.
SSRIs are the best initial therapy for sevre depression. in addition SSRI's treat obsessive compulsive disorder, eating disorders, anxiety, attention deficit disorder and dysmenrrhea.
there is a possible increased risk of suicide in the first few months after starting medications. SSRI's can also cause sexual dysfunction such as impaired ejaculation.
>>>>>>>>>>>>>>>>>>>>
Reply

جوري
09-18-2008, 01:17 AM
today we continue on 5/5
still on pulmonary in diagnostics and psychiatry in pharm
____________________________________



these are pulmonary function tests. the patient gives a max exhalation after a max inhaltion. that is the FVC, or forced vital capacity. A normal person should be able to exhale at least 80% of the air in one second or FEV1

in the obstructive pattern as seen on the graph, you see a decrease in the FEV1 and FVC, however FEV1 decreases even more. This gives an abnromally low ration of FEV1 to FVC of <90%. The residual volume goes up and that is why the pt gets a barrel chest and a flat diphragm. This is caused by COPD. in lay man's terms as I struggled with this, ppl in COPD have alot of air trapped that they can't get rid of that is what is called residual volume
FEV1 - Forced Expiratory Volume in One Second - this is the volume of air which can be forcibly exhaled from the lungs in the first second of a forced expiratory manuever. It is expressed as liters. This PFT value is critically important in the diagnosis of obstructive and restrictive diseases.
FVC - Forced Vital Capacity - after the patient has taken in the deepest possible breath, this is the volume of air which can be forcibly and maximally exhaled out of the lungs until no more can be expired. FVC is usually expressed in units called liters. This PFT value is critically important in the diagnosis of obstructive and restrictive diseases.
FEV1/FVC - FEV1 Percent (FEV1%) - This number is the ratio of FEV1 to FVC - it indicates what percentage of the total FVC was expelled from the lungs during the first second of forced exhalation - this number is called FEV1%, %FEV1 or FEV1/FVC ratio. This PFT value is critically important in the diagnosis of obstructive and restrictive diseases.
..
the other graph with the restrictive pattern
. all the volumes are diminished, but they are diminished proportionately to all the ratios stay normal. restrictive lung disease is caused by interstitial fibrosis.

the PFT is the most accurate way to distinguish obstructive and restrictive lung dz.
the most accurate part of PFT is the FEV1/FVC ratio.
__________________________________

serum Ace levels
are associated with sarcoidosis in 75% of pts. Ace levels are contributory to establishing the diagnosis of sarcoidosis --
Ace is best to do when an african american female presents with cough, dyspnea , chest pain and bilateral hilar adenopathy on chest xray.

the most accurate test is the presence of noncaseating granulomas on biopsy.
____________________________________________

aerum lpha 1-Antitrypsin level
(AAT) is measured in a young nonsmoker with emphysema and liver disease, it may also be measured in a pt with a strong fam hx of emphysema, low levels indicate AAT deficiency, in which the body lacks elastase.
________________________________

Arterial blood gas

directly measures ph, partial pressure of carbon dioxide, and partial pressure of oxygen. The bicarbonate level is determined based on a calculation from the ph and pCO2. This allows you to calculate the alveolar-arterial A-a gradient.. for actualy formula please refer to earlier post.

the saturation determined from an oximeter is within 1-2% of that obtained on an ABG, so we do not need to do an ABG for O2 saturation. An ABG allows you to know how hard someone is working in order to become oxygenated and where there is life threatening acidosis.
ABG is the answer f there is a COPD or another possibility of respiratory acidosis. ABG is the answer is there is low serum bicarbonate and there maybe severe metabolic acidosis.
____________________________________________
purified protein derivative PPD

the PPD is to screen for exposure to TB and the possibility of latent infection. PPD testing is only to screen asymptomatic pts
the test is positive based on the following
>or = 5 mm: HIV positive, recent contact with person with active TB, patient with changes on CXR suggestive of prior TB, patients with organ transplants or other immunesuppressed pts,
>or= 10 mm recent immigrants from countries with high TB prevlance, injection drug users, residents, employees of hospitals, nursing homes, and homeless shelters, and pts with diabetes, renal failure, or hematological malignancies.
>or = 15mm person with no risk factors for TB
all pts with +ve PPD should get a CXR to exclude active disease. Positive tests are treated with isoniazid for 9 months.
Previous vaccinations with BCG has no impact on these recommendations
a test result >10mm is not to be considered a false positive because of previous BCG.
__________________________________________________ _

pharm psychiatry/pulmonary

32 year old woman comes to ER because of acute onset of facial grimacing, toticollis, oculgyric crisis, and abnormal contractions of the spinal muscles. She has been started on the antipsychotic medication fluphenazine as well metoclopromide for nausea

the patient has an acute dystonic reaction from both the antipsychotic meds as well as the metocloprimide. her sx consist of an acute oculogyric crisis as well torticollis
the precise mechanism of acute dystonic rxn is unclear. it occurs because of the antidopaminergic effect of antipsychotic meds as well as metochlopromide.
Acute dystonic reactions are treated very effectively and rapidly with diphenhydramine and benztropine.
__________________________________
lithium
is an alakali metal that has no effect in a normal person but can treat sx of certain mood disorders. the proposed mechanism may be through suppression of inositol triphosphate. although lithium's precise mechanism is unknown
adverse effects
tremor, ataxia, seizures
nephrogenic diabetes insipidus through inhibition of the ADH receptor.
hypothyroidism by inhibiting thyroglobulin iodination and coupling
teratogenesis, particularly ebstein's anamoly
lithium is indicated for bipolar disorder, and acute mania. it is prophylactic against migraine headache and cluster headaches.
__________________________

a pt has been started on anti psychotic meds ziprasidone after being on risperidone at high dose. He becomes febrile to 105 degrees F and is somewhat catatonic with muscle rigidity, confusion and a elevated CPK level.

neuroleptic malignant syndrome is managed with dopamine agonists, such as bromocriptine. Dantrolene is used as well and can help relieve muscle rigidity
________________________________________

pulmonary

anticholinergic respiratory therapy
ipratropium
tiotropium

inhaled anticholinergics with ipratropium or tiotropium are the best answer to the question 'what is the best initial drug therapy for a person with chronic obstructive pulmonary disease. They offer additonal bronchodilation for pts with asthma. while they don't work as rapidly as inhaled beta agonists, they can be added to inhaled beta agonists when the pts needs an acute rescue medication for asthma exacerbation. tiotropium has superior efficacy to ipratropium.
ipratropium/tiotropium antagonizes the effect of ach at muscarinic receptors in the lung. this results in bronchodilation and decrease in the amount of mucus production. they dilate and dry bronchi
.
because they are virtually nonabsorbed these drugs almost have no adverse side effects. at high dosages may cause pupil dilation, tachycardia, constipation, dry mouth and urinary retention due to their antichoinergic effects.
_____________________________________________
prostacyclin analogs
Epoprostenol
Treprostinil
Iloprost
Epoprostenol/treprostinol/Iloprost are the correct treatment for severe pulmonary htn. look for pt with slowely progressive SOB that is worst on exertion with a loud P2 heart sound, clear lungs, and pulmonary HTN on echocardiography.

prostacyclin analogs cause potent pulmonary artery vasodilation, inhibit platelet aggregation and inhibit the proliferation of blood vessles.
adverse effects are hypotention, flushing, bradycardia, and dizziness.

>>>>>>>>>>>>>>>>>>>>>>>>>.
Reply

جوري
09-19-2008, 12:29 AM
Today's five and five-- finish pulmonary in diagnostics move on to hematology, and pulmonary in pharm
___________________________

sweat chloride test

is the gold standard for the diagnosis of cystic fibrosis.
Pilocarpine is given and the amount of sodium and chloride is measured in sweat.

this is done for child with chronic respiratory problems such as cough, wheezing and SOB.. there will also be signs of malabsorption such as bulky stools and fat soluble vits deficiency ADEK, also inability to have children, look for an infant with failure to thrive. blood gas will reveal hypoxemia.
__________________________________

Heme



this is a hypersegmented neutrophil. Normal neutrophils contain an average of 3.5 lobes per cell. When the average is above 4 lobes, or there are 5% above 5 lobes, or a single 6-lobed cell, hyper segmentation is present. this is the defining feature of megaloblastic anemia.

hypersegmented neutrophils or megaloblastic anemia is present with vit B12 and folate deficiency.
the most accurate test is B12 and folate levels. B12 deficiency is also confirmed with an elevated methylmalonic acid level. The etiology of B12 deficiency is confirmed with anti-intrinsic factor and antiparietal cell antibodies, which are diagnostic for pernicious anemia.

__________________________________________________ _



both slides show a blood smear, cells with cytoplasmic projections consistent with hairy cell leukemia.
look for middle aged man presenting with gradual onset of fatigue, splenomegaly is present and maybe massive. The liver is enlarged in 50% of cases. Pancytopenia is the hallmark of hairy cell leukemia.
the most accurate test is tartarate-resistant acid phosphatase (TRAP) this is done on bone marrow biopsy.
_____________________________________________



this peripheral smear of RBCs shows Heinz bodies. Heinz bodies are precipitated, oxidized hemoglobin. They are found in glucose-6-phosphate dehydrogenase deficiency (G6PD). G6PD is an x-linked recessive disorder affecting 10-15% of African American males..
pt is usually healthy until exposed to oxidant stress and suddenly developes intravascular hemolysis resulting in weakness, tachycardia, jaundice and dark urine. The most common type of oxidative stress is from infection, not drugs. Drugs associated wit this stress include sulfa, primiquine, dapsone, quinidine, and nitrofurantoin. Fava beans also cause hemolysis.
Heinz bodies are removed resulting in cells called bite cells.
The definitive test for this disorder is a G6PD level. You must wait two months to do the level because the G6PD level will be artificially elevated immediately following a hemolytic event. All the most deficient cells are destroyed.

__________________________________________


This is an Auer rod.. which is an eosinophilic needle shaped inclusion in the cytoplasm.
Auer rods are pathognomonic for acute myelogenous leukemia (AML)
Auer rods will be found in a pt with pancytopenia, and blasts on peripheral smear of greater than 20% blasts on the bone marrow exam. Histochamical stains demonstrating myeloid enzymes such as peroxidase may further aid in choosing AML as the answer
_____________________________________-

Pharm
pulmo/renal
____

Bosentan

Bosentan is the RX for severe pulmonary HTN. Look for a pt with severe, progressive shortness of breath and a high pulmonary artery pressure on echocardiogram on right heart catheterization
Bosentan is a potent inhibitor of endothelin-1. Endothelin-1 causes vasoconstriction of the pulmonary artery as well as proliferation of the smooth muscle artery.
Bosentan is hepatotoxic and extremely teratogenic.
__________________________________________________ _

a young woman has persistent asthma not controlled with albuterol inhaler.

inhaled steroids have the best efficacy of all the long term asthma controlling medications. Inhaled steroids also have the best effect on mortality.
Adverse effects and mechanisms:
Inhaled steroids: oral thrush and dysphonia
Salmeterol: tremor, heart block, and bradycardia
Moteleukast: headache
Cromolyn : stablizes mast cells
Theophylline: tremors, seizures and arrhythmias. Theophylline has a narrow therapeutic index. it is very toxic with limited efficacy.
use meds in the following circumstance
-for Asthmatics not controlled with albuterol inhalers, inhaled steroids have the best efficacy if these do not work long acting beta agonists such as salmuterol should be used.
Moteleukast is best when the pt. has an atopic allergic disorder such as rhinitis.
Cromolyn is best when an environmental allergen is at work
Tehophylline is rarely used.
___________________________________________
Omalizumab
Omalizumab is the treatment for an asthmatic patient with an extrinsic allergic trigger that isn't controlled by inhaled beta agonosts combined with inhaled steroids or a leukotriene inhibitor, such as monteleukast. Look for a patient with an elevated IgE level or a positive skin test for a specific allergen.
Omalizumab blocks IgE antibody. This prevents IgE from binding to the mast cell or esoinophil. In this way, it prevents asthma exacernation. Look for a pt who is refractory to inhaled steroids, whom you are trying to keep off oral steroids.
___________________________________

Renal..
Conivaptan and Tolvaptan
Conivaptan and tolvaptan are used in the treatment of severe hyponatremia from syndrome of inappropriate antidiuretic hormone secretion (SIADH) a pt with neurological symptoms and a very low sodium level with a normal volume status call for this answer. Conivaptan is used in conjunction with hypertonic saline.
These medications are vsopressin (Antidiuretic hormone or ADH) receptor antagonists. conivptan and tolvaptan increase free water diuresis and raise the sodium level.
adverse effects are orthostatic hypotension, peripheral edema, headache, increased thirst, and hypokalemia. serious adverse is A fib.
_________________________________
Aliskerin
Alisekerin is used to rx HTN
is a direct renin inhibitor. Aliskerin blocks the conversion os angiotensinogen to angiotensin 1. This action leads to a marked decrease in the level of angiotensin II and alodesterone
Aliskerin can cause hyperkalemia, It doesn't cause cough or angioedema as do ACE inhibitors...

>>>>>>>>>>>>>>>>>>
Reply

جوري
09-20-2008, 04:31 AM
5/5

start with pharm still on renal
_________________
a man comes in for a preoperative evaluation of his episodic htn, he has palpitations, headaches, surgical removal of his adrenals is planned

phenoxybenzamine is the best preoperative medication prior to surgical removal of a pheochromcytoma. this is followed by propanolol.

phenoxybenzamine is a nonspecific alpha blocker. it is superior to the alpha blocking agents prazosin, terazosin, or doxazosin, propranolol is also a nonspecific beta blocker.

____________________________________

chlorthalidone
is a thiazide diuretic, inhibits sodium re absorption in the early distal tubule.

chlorthalidone and thiazide diuretics are the best initial therapy for hypertension, when there are no other compelling indications such as coronoary disease, CHF etc.. chlorthalidone also prevents recurrent kidney stones in calcium overexcreters.
chlorthalidone causes hypokalemia , hyperglycemia, hyperurucemia and hypercalcemia.
_____________________________________

42 year old man comes in for evaluation of hypertension. blood pressure is 146/94
HTN is defines as blood pressure >140/90. the most effective life-style modification is weight loss. Sodium restriction, dietary modification, and exercise are not as effective as weight loss.

for specific indication, specific indications, the best therapies are as follows:
Diabetes: Angiotensin-converting enzymes (Ace) inhibitors, angiotensin receptor blockers ARB's
coronary artery disease: beta blockers
CHF, ace inhibitors, beta blockers
osteoporosis: Thazides they decrease calcium excretion
prostate hypertrophy, alpha blockers, doxazosin
depression: don't prescribe beta blockers
Asthma: don't describe beta blockers.
___________________________________________

Demeclocycline

demeclocycline is a tetracycline antibiotic that is used in the treatment of chronic syndrome of inapproriate antidiuretic hormone secretion (SIADH)

demeclocycline inhibits the effect of antidiuretic hormones at the collecting tubule.

When the question describes a case of SIADH in which the underlying cause cannot be corrected, demecloycline is the answer. It is not used as an antibiotic.
like all tetracycline antibiotics, demeclocyline can cause photosensitivty.
_______________________________________________

Sirolimus, Tacrolimus, and pimerrolimus

sirolimus, tacrolimus, and pimercrolimus are T-cell inhibiting immuno-suppressive medications used for a number of autoimmune diseases and to prevent organ rejection.

sirolimus inhibits interleukins. Tacrolimus is a calcineurin inhibitor like cyclocsporine. Pimercellimus' mechanism is not known.
These medications are used to treat the following conditions:
sirolimus: prevents renal transplant rejection, prevents coronary stent restenosis. Treats graft versus host disease.
Tacrolimus: prevents liver transplant rejection and is used to atopic dermatitis, treats graft versus host disease
pimercolimus is only approved for atopic dermatitis.

Tarcolimus causes both renal and neurological toxicity. Just as cyclosporine. Bot sirolimus and tacrolimus cause lymphoma..

>>>>>>>>>>>>>>>>>>>>>>>>>>>

Diagnostics
____







these are bite cells. They are formed when Heinz bodies (see previous post) denatured hemoglobin are removed from the cells by the spleen. Macrophages in the spleen remove denatured hemoglobin.

bite cells are seen with GPD deficiency, which is an x-linked disorder. It is the most common enzymatic disorder of the red blood cells in humans.

Patients with G6PD deficiency are prone to devloping hemolytic anemia in response to sulfonamides such as dapsone and sulfasalazine. Other precipitating factors are infections, diabetic ketoacidosis, and favism.
________________________________________



Schistocytes
are fragmented red cells and are seen in a variety of shapes and sizes the individual pieces called helmet cells. collectively this is known as intravscular hemolysis or microangipathic hemolytic anemia.
Fragmented cells are seen in the thrombotic thrombocytopenic purpura (TTP), hemolytic uremic syndrome, major blood grou incompatibility, disseminated intravascular coagulation (DIC), paroxysmal nocturnal hemoglobinuuria, artificial heart valves, and snake bites.
All forms of hemolysis are associated with elevated LDH, indirect bilirubin, elevated reticulocyt count, and decreased levels of haptoglobin.
__________________________________________________ _____



this is ringed sideroblasts. Prussian blue stain is necessary in order to visualize it. The ringed sideroblasts is caused by iron accumulation within mitochondria in the red cell. This is the main findings in sideroblastic anemia...

look for case of microcytic anemia with an elavted serum iron level in an alcoholic. Acquired sideroblastic anemia can occur as a result of the ingestion of drugs such as alcohol, isoniaizid and chloramphenicol, or toxins such as lead or zinc.
ringed sideroblasts are also a feature of myelodysplastic syndrome..

__________________________________________________ __________


Rouleaux formation occurs when red blood cells form stacks or rolls..Rouleaux formation may form due to presence of abnromal globulins or fibrinogen. This formation of red blood cells is found in multiple myeloma and macroglobulinemia. other clues that suggest multiple meyloma might be anemia, hypercalcemia, renal failure and abnormal SPEP or UPEP.
__________________________________________________ _________

last and not least.. sob7an Allah.. it has been a long day..


Spherocytes are red blood cells that are almost spherical in shape. They have no area of central pallor like a normal red blood cell...
the most accurate test for heriditary spherocytosis is the osmotic fragility test. MCHS is not an accurate test..
the defect is in the ankyrin gene. which leads to spectrin deficiency, which results in membrane instability.
look for a pt. with recurrent hemolysis, a big spleen, and a family hx of anemia. The CBC reveals anemia and an elevated mean cell hemoglobin concentration (MCHC )

>>>>>>>>>>>>>>>>>>>>>>>>>
Reply

جوري
09-21-2008, 12:21 AM
today's 5/5 still in hematology for diagnostics
__________________________



these are sickled cells.

only sickle cell disease that is homozygous (SS) will produce sickled cells. Homozygous disease (AS) will be 'hematologically' normal

the most accurate test for sickle cell disease is a hemoglobin electrophoresis.

________________________________

this image is really cool, it shows target cells being ingested by macrophages.. anyhow, this is what a target cell looks like



these cells are associated with liver disease and certain hemoglobinopathies such as sickle cell disease, thalassemia and most notably hemoglobin C disease, iron deficiency can also have target cells.

the most accurate test if a hemoglobin electrophoresis.

despite their abrnomal appearance, they don't have a shorter life cycle compared to normal cells.

_________________________________________



these are tear drops cells.. and it because they hurt that they are crying :D.. just checking to see if you are reading lol

when tear dro cells are either shown or described, you should always think bone marrow disease such as myelofibrosis. look for a case of pancytopenia and massive splenomegaly in which hairy cell leukemia has been excluded. The bone marrow biopsy will show reticulin fibers.

__________________________________________________

anti-intrinsic factor antibodies, antiparietal cell antibodies

are highly confirmatory for pernicious anemia. They are nearly 100% specific for the disease. Pernicious anemia is an autoimmune disease in which you become allergic to you own IF, and gastric parietal cells.

Answer anti-IF and antiparietal cell antibodies when you see a case of B12 deficiency and you want to determine the etiology. The case will describe a pt in whom B12 is low, or te methylmalonic acid level is high. These antibodies are not to diagnose B12 deficiency, they are to determine the etiology.

______________________________________________




burr cells, also called echinocytes, are RBC's with many blunt, regular sicules. They are present in end stage renal disease and liver disease...



compared to spurr cells or acanthocytes, which have only a few irregular spicules, they are also seen in severe liver disease and in some forms of hereditary spherocytosis, nd myelodysplasia.

________________________________________________

ok Now Pharm.. still on Renal

_____

Clonidine

this is an old hypertensive in the archives lol.. no longer recommended for the rx of HTN because of the large number of adverse side effects and limited evidence of mortality benefit

clonidine is a centrally acting alpha 2 agonist. This decreases the release of vasconstrictive neurotransmitters, such as norepinephrine.

clonidine can cause sedation, and dry mouth and rapid withdrawl which can lead to hypertensive crisis.

clonidine has no room in rx of HTN however it is used as an adjunct to treat opaite withdrawl, diarrhea related to diabetes and occasionally tourette's syndrome.

________________________________________

a woman comes to the ER dept. after falling while running a marathon, she is also taking a statin medication. she has dark urine that is dipstick positive for large amounts of blood. There are no red blood cells seen on the microscopic examination. Her serum bicarbonate is low.

this pt most likely has rhabdomyolysis. The dark urine that is dipstick positive for blood with o red cells seen is myoglobin.

te best initial therapy for rhbdomyolysis is vigorous hydration, bicarbonate adminstration and possible mannitol.
the hydration and mannitol decrease the amount of time that the nephrotxic myoglobin is in contact with the kidney tubule. Bicarbnate prevents precipitation of myoglobin in the kidney tubule. In addition, it drives potassium intracellularly by causing an alkalosis.

The cause of death in rhbdomyolysis is hyperkalemia and acidosis. both of these can cause an arrhythmia.

__________________________________________________

a man has just undergone combination chemotherapy for widespread non-hodgkins lymphoma. The EKG shows peaked T waves

Hyperkalemia with EKG changes should be immediately treated with intravenous calcium chloride or calcium gluconate. This will protect the heart from an arrhythmia. Insulin and glucose should be used to drive potassium intracellularly, but they take 15-20 minuted to work. The calcium will protect the heart from an arrhythma while the Insulin and glucose take time to work.

Kayexcelate is a sodium/potassium cation-exchane resin and is adminstered orally to remove potassium from the body vua the gastrointenstinal tract. It takes several hours to work.

Tumor lysis syndrome can be prevented by adminstering intravenous hydration and allopurinol before chemotherapy is given.

__________________________________________

Azathioprine
is a purine inhibitor that is used to control autoimmune diseases.
azathioprine is converted to 6-mercaptapurine, whose metabolites inhibit purine metabolism. Purine synthesis is needed for the proliferation of B and T lymphocytes. This redices antibiody production.

Azathiprine can be bone marrow suppressive and can cause infection. Allopurinol interfers with azathiopurine, because the active metaboolite is metabolized by xanthine oxidase. Pancreatitis happens from direct toxicity.

Azathioprine is used for the following conditions

Inflammatory bowel disease
Vasculitis and mysthenia gravis to spare the pt from chronic steriod use
lupus nephritis that isn't responding to steroids, cyclphasapmide, or mycophenolate
renal translanation
rehumatoid arthritis

_________________________________________

mycophenolate mofetil
is an immunosuppressive agent with less toxicity than cyclophosphamide, azathioprine or cyclosporine.

mycophenolate inhibits purine synthesis, this prevents the proliferation of lymphocytes.
mycophenolate has a few adverse effects.
It can cause infection via leukopenia

mycophenolate is the drug of choice for lupus nephritis. it is superior to cyclophasmide, oter indications are the following
organ transplant rejection prevention.
Nephrotic syndrome, not responsive to sterioids and cyclophosamide
it can be used as a second line treatment for myasthenia gravis, atopic dermatitis, vasculitis and crohn's disease to prevent dteroid dependence

>>>>>>>>>>>>>>>>>>>>>>>>>>>
Reply

جوري
09-22-2008, 01:55 AM
today's 5/5 still on heme in diagnostics, renal in pharm and will move on to rheumatology insha'Allah
____________________________________

Bleeding Time

a test for the ability of the platelet to adhere to the endothelial lining of the capillary. Do not use it when the platelet count is abnormally low. Everyone with a low platelet count will have an abnormal bleeding time.

a blood pressure cuff is inflated to 40mm Hg to encourage the blood vessles, then small cuts are made, the test is the amount of time it takes for the bleeding to stop.

bleeding time is the answer, when the pt seems to have a bleeding disorder, examples are von willebran's disease or uremia induced platelet dysfunction.

the most accurate test for von willebrand factor level, ristocetin cofactor assy, or in vitro platelet aggregation studies..

__________________________________________________

decay-accelerating factor (DAF) or CD55/CD59 assay

Cd55/59 is a test for paroxysmal nocturnal hemoglobinuria (PNH), also knows as decay accelerating factor. DAF is a membrane component of the complement system.

Assay for DAF is the most accurate test for diagnosing PNH

DAF when you see a pt with intermittent dark urine, particularly in the morning, pancytopenia, and large vessel thrombosis..

_____________________________________________

Cold Agglutinins

are IgM autoantibodies directed against RBC antigens. Titers are measured when Cold Agglutinins disease, which is a type of autoimmune hemolytic anemia, is suspected.

high titres have been found to be associated with malignancies such as lymphoma, CLL, and Waldenstrom's macroglobulinemia, as well as mycoplasma infections and infectious mononucleosis.

Answer cold agglutinins when you see a patient with hemolytic anemia as well as skin discoloration when exposed to cold temperatures. Do not confuse this with cryoglobulins. which give purpuric skin lesions and glomerulonephritis.
________________________________________________

Coombs' Agglutinin test

coombs test measures the presence of antibodies against red blood cells.

Coombs tests is used for the diagnosis of autoimmune hemolysis. A positive coombs tests shows the agglutination of the patient's red blood cells.
a total of 20-50% of coombs positive hemolytic anemia is iodiopathic. the most common identifies etiologies are PCN, quinidine, alphamethyldopa, sulfa drugs and ASA.

_____________________________

D-Dimer/Fibrin split products

D -dimer is the by=product of the degradation of fibrin by plasmin. Fibrin split products (FSPs) are produced by the activation of fibrinogen by thrombin. They are abnromal only if they are present in increased amounts.
D-dimer can be measured by levels either latex agglutination (more rapid) or Elisa (more accurate). Elisa is far more sensitve.
elevated levels indicate pathology involving clot formation and lysis from activation of the coagulation cascade. They provide no information about platelet function.
Elevated D dimer/FSP levels are evidence of DIC, a negative result from the ELISA rules out DVT and PE but a positive results does not confirm the diagnosis.
order these tests to confirm suspected DIC and to rule out PE in pts with a low pre=test probability

____________________________

pharm/Renal

Man comes in with severe headache, confusion, blood pressure of 190/124, head CT is normal
Hypertensive emergencies are treated with IV agents that allow rapid lowering of BP with greater control han oral agents. Labetolol is the best initial therapy for hypertensive crisis. Labetolol is a mixed alph and nonspecific beta blocker that is very effective. Nitroprusside is used in the intensive care unit and usually requires monitoring with an arterial line. Other choices are enalaprilat, hydralazine or nicardipine.

in a hypertensive emergency, you should not lower the blood pressure more than 25% over the first several hours.
blood pressure lowered too rapidly can result in a stroke.

__________________________________________________ _

a patient with end stage renal disease is preparing to start dialysis. his phosphate level is markedly elevated ..

hyperphosphatemia from renal failure is treated with calcium carbonate, calcium acetate, sevelamer or lanthanum . Cinacalcet is a calimimetic agent that treats secondary hyperparathyroidism. it is used is there is hypercalemia from secondary hyperparathyroidism.

Calcium carbonate, calcium acetate, sevelamer and lanthanum all bind phosphate in the bowel. the calcium containing medications form complex with phosphate, which is then excreted in the sool. Sevelamer is a nonabsorbed polymer that also binds phosphate in the bowel. Cincalcet acts like calcium on the parathyroid gland and inhibits the release of parathyroid hormone.
All of these medications case some gastrointenstinal discomfort. calcium carbonate and calcium acetate can lead to hypercalcemia. if this happens, treatment should be switched to sevelamer or lanthanum.

__________________________________________________ ____

Pharm rheumatology

Tumor necrosis factos (TNF) inhibitors
infliximab, etanercept, adalimumab

tumor necrosis factor inhibitor are used to treat the following conditions:
inflammatory bowel disease --particularly crohn's with fistula formation

Rheumatoid arthritis, that is not responsive to methotrexate as a DMARD
psoriatic arthritis, when it is moderate to severe, as an alternative to systemic therapy such as methotrexate or UV light

ankylosing spondylitis
TNF inhibitor are immunosuppressive but less toxic than steroids.
TNF inhibitors can reactivate or worsen serious bacterial infections by inhibiting the immune system. TB reacivation in PPD positive pts is te most important adverse effect.
All pts who go on TNF inhibitor should have a PPD test done first. TNF inhibitors are also associated with the development of lymphoma.

___________________________________________

methotrexate..

this is the answer as to which slows down the progression of disease, in virtually all pts wit rhumatoid arthritis. It is the number one DMAR for rheumatoid arthritis.
it is also used for the following
severe psoriasis, particularly psoriatic arthritis
leukemia, lymphoma and certain solid tumors.

Methorexate causes
liver toxicity
pulmonary fibrosis
bone marrow suppression (myelosuppression)
kidney damage (precipitation of methotrexate crystals)

______________________________________
rheumatoid arthritis and alternate DMARDS

hydroxychloroquine
sulfasalazine
anakinra
abatacept
leflunomide

Alternative DMARDS to methotrexate are the correct answer to the following
pt is intolerant to methotrexate
methorexate fails to control disease
cases of mild disease where it is preferable to avoid the toxicity of methotrexate, sulfasalazine ad hydroxycloroquine can be used initially in this way..

adverse effects are as follows..
hydroxychloroquine-- renal damage, hemolysis
sulfasalazine--rash, heptaitis, agranulocytosis
anakinra--interluekin-1 antagonist-causes neutropenia
abatacept-- inhibits t cell activation-->infections
leflunomide-- inhibits pyramidine synthesis--causes rash, alopecia, mylosuppression and liver dysfunction...

>>>>>>>>>>>>>>..
Reply

جوري
09-23-2008, 01:41 AM
Rheumatology pharm and on to tox.

NSAIDS
Naproxen
Sulindac
ibuprofen
diclofenac
etodolac
indomethacin
ketrolac
piroxicam

Cox-2 inhibitors:

Rofecoxib
celecoxib-- removed from market
Valdecoxin (removed)

NSAIDS and COX-1 inhibitors are indicated as analgesics.. also useful for the folowing

Inflammatory syndromes , such gou, pseudogout, rheumatoid arthritis, and ankylosing spondylitis.
cystic fibrosis
fever
still's disease
Both NSAIDS and COX-2 inhibit prostaglandins

NSAIDS cause peptic ulcer disease, and renal insufficiency, such as interstitial nehritis and nephrotic syndrome . Although COX-2 inhibitors have less effect on the gastic mucosa, they hae very severe cardiac toxicity. The COX-2 inhibitors rofecoxib, and valdecoxib were removed from the market because of excess cardiac deaths, only celecoxib remains on the market.

___________________________________________

1 48 year old man comes to the ER with severe sudden pain in left knee after a beer binge
over the weekend. on examination, he has a fever. The knee and toe are red and swollen. Joint aspiration shows 25,000 white cells that are predominantly neutrophils, and crystals are present. The crystals are needle shaped and negatively birefringent . creatinine is 2.4

colchicine is the best initial therapy for acute attachs of gout, particularly when NSAIDS are contraindicated. In this case, a creatining elevation is contraindicated to NSAIDS. Colchicine is also used to treat familial mediterranean fever.

Colchicine inhibits leukocyte mobility, decreasing the white cells ability to phagoctytose within the joint space and decrease lactic acid within the joint. This action reduces the deposition of the urate crystals that perpetuate the inflammatory response.

the most common adverse effect of colchicine is diarrhea. in fact you should give colchicine to relieve pain until it produces diarrhea, rarely, colchicine may produce aplastic anemia.

If there is no response to colchicine then the next therapy is intracrticular steroids.

______________________________________

Allopurinol

Allopurinol is a drug that lowers urate synthesis and ecreases the serum uric acid level.

Allopurinol is a xanthine oxidase inhibitor. this reduces the uric acid level in both blood and urine.
Answer allopurinol when the question describes a patient with recurrent gouty attacks, tophi, and uric acid stones and who has failed pobencid or sulfinpyrazone. The patient should be between attacks. Allopurinol has no benefit during acute attack of gout, because it is not anti-inflammtory.

AAllopurinol is highly allergenic and can cause rash, eosiniphilia and interstitial nephritis.

__________________________________

Toxicology Pharm

two drunk men come to the ER. one has visual disturbances, the other hyperemia of the retina. The other has developed kidney stones, he has envelope shaped crystals in his urine, both have a metabolic acdosis with an increased antion gap

the first drunk with visual disturbance and an abnromal fundoscopic exam has methanol intoxication. The second patient who is drunk with oxalate crystals in the urine has ethylene glycol intoxication. oxalate crystals appear in the shape of an envelope. both cause metabolic acidosis.

Fomepizole is the best initial therapy for ethylene glycol or mathanol poisoning. Fomepizole is considered superior to ethanol infusion, because it isn't intoxicating. Definitive therapy is with dialysis to remove the substances.

Fomepizole inhibits alchol dehydrogenase. This prevents the production of the toxic metabolite and gives time for dialysis to be effective.

__________________________________________

32 year old woman comes to the ER dept. six hours after having ingested a bottle of 50 extra-strength (500 mg each)
acetamiophen tablets

best initial therapy for an overdose of acetominophen is acetyl-cysteine (NAC) and charcoal. Giving NAC is more important than getting a specific level of acetamiophen when the pt states she took a potentially harmful amount.

NAC works by replacing the glutathione reductase that is depleted from metabolites of acetominophen. When glutathione reductase has been depelted, the liver cells start to necrose.

charcoal can be used at the same time as the NAC. Charcoal doesn't bind enough of the NAC to result in a clinically significant impairement of its effect.

__________________________________________


Diagnostics still in heme

Factor V leiden mutation

factor V leiden predisposes to thrombosis by resistance to the antithrombotic effects of activated protein C. Protein C normally slows the clotting cascade by inhibiting factor V. the mutation allows factor V to ignore the natural anticogulant action of protein C. Factor V leiden i the most common cause of inherited thrombophilia.

Answer factor V leiden mutation as the most accurate test in a young person with an unprovoked DVT or PE. Thrombotic events after plane flights should evoke and investigation for thrombophilia.

the other tests of hypercogulable state are
protein S
Protein C when you see skin necrosis
lupus anticoagulant when you see an elevated PTT or spontanous abortions in the case
antithrombin III mutation, when you see resistance to heparin in the case.

________________________________

philadelphi chromosome

represents a genetic translocation between 9 and 22. it is associated with chronic mylogenous leukemia. It is also known as a BCR/Abl and can be detected by PCR.
answer Philadelphia chromosome when presented with a case pf probable CML. the white cell count will be very high, mostly neutrophils, and the LAP score will be low. The Philadelphia chromosome also has prognostic value. If you give imitanib (geelvec) the philadelphia chromosome goes away the prognosis is good.

__________________________________

Haptoglobin levels

used to determine hemolysis. It is a proten that binds to free hemoglobin. so when we ave hemolysis, RBCs will release free hemoglobin that will bind to haptoglobin. This will result in decreased haptoglobin levels. In hemolysis we also find elevated LDH, reticulocytes, and indirect bilirubin

typical scenario is sudden anemia without Gi blees. the presence of Jaundice is also highly suggestive. Acute enamia minus the Gi bleeding equals hemolysis.

____________________________________________

Hemoglobin electrophoresis,
is the most sensitive test to diagnose hemoglobinopathis such as SCD or thalasemia. The most accurate way to diagnose the presence of the hetrozygou forms of these disease or the trait
with respect to SCD, clinical presentations include ulcerations of the skin, of the legs, recurrent infections with pneumococcus or haemophilus, retionpathy, aseptic necrosis of the femoral head, osteomyelitis, growth retardation, and splenomegaly, Typically, the pt will be African American with a possible fam hx of the disease, Sickle cell trait will be in a pt who is asymptomatic with a family member with sickle cell disease or with unexplained hematuria.
with respect to thalassemia, clinical rpesentations range from normal to severely symptomatic with growth failure, hepatomegaly, jaundice and bone deformities..

____________________________________

Leukocyte alkaline phosphatase LAP score
this is an enzyme in WBC. If the cells are elevated in number and the function os normal, the LAP score will go up in proportion to the elevated count.
LAP is a test for CML, LAP white count is extremely hight and the differential shows mostly neutrophils. The case is likely to have a big spleen, giving upper quadrant pain and early satiety. LAP score should be LOW in CML, and are used to differentiate CML from leukemoid reaction..

>>>>>>>>>>>>>>>>>>>>>>>>>
Reply

جوري
09-24-2008, 12:58 AM
ok today's 5/5..
still on heme for diagnostics
lymph node biopsy
is used to detect infections such as TB, fungi and staphylococcus. Infections are suggested by nodes that are warm, tender and sometimes red.

excisional lymph node biopsy is a single lymph node that is most accurate to diagnose a lymphoma. A needle biopsy is a wrong answer. you need to see architecture. In addition, the individual lymphocytes will appear normal on a needle biopsy. The diagnosis of lymphoma requires the visualization of the architecture of the entire node. Noses with lymphoma are nontender, not red, not warm as they usually are in an infection.
_________________________________
methylmalonic acid (MMA) builds up when vit B12 is deficient. MMA has greater sensitivity than Vit B12 levels. Homocystine is elevated in both B12 and folic acid deficiency.

this is the test for a pt with macrocytic anemia and hypersegmented neutrophils but a normal B12 level.

______________________________________

Mixing studies

are done to distinguish between a clotting factor deficiency and an inhibitor of the clotting factor as the cause of an abnromal partial thromboplastin time (aPTT) you mix normal pooled plasma with the patient's plasma.
this is the best test wen you have a pt with an abnromal APTT.
if the test normalizes after mixingg, then the elevated aPTT is caused by a clotting factor deficiency.
The next best test if the mixing normalizes is individual clotting factors assyas of the patient's plasma to determine which factor is deficient.
if the test doesn't normalize it means that an inhibitor is present i.e factor VIII inhibitor of the lupus anticoagulant.

__________________________________
osmotic fragility test

is the test for heriditary spherocytosis.
it measures the amount of emoglobin released from red blood cells RBCs placed in hypotonic solution. RBC's with a reduced surface to volume ratio will lyse at concentrations that not affect normal cells. This is because they don't have enough cell membrane. They 'stretch' an then 'pop' when put into hypotonic solution.

this is the test for a case of hemolytic anemia and splenomegaly with spherocytes on the peripheral smear. The other features that push the diagnosis and this test are:
family history of recurrent episodes of anemia and bilirubin gallstones.

______________________________________
Ristocetin cofactor assay.
is a test for the function of von willebrand's VWF. Along with a VWF level. it is the most accurate test for Von willebrans's disease. Ristocetin acts and an artificial endothelial lining. if VWF is present and is functioning normally then platelets should adhere to Ristocetin.

this is the test for a pt with platelet type bleeding and a normal platelet count and normal VWF level. Platelet type of bleeding is superficial. Examples are epistaxis, petechiae, purpura, and gum or gingival bleeding.

_________________________________________

in pharm, I will finish the last of tox and start cardio, like infectious disease cardio will be a large review..

tox.

70 year old woman with hx of osteoarthritis comes to the ER because of SOB, tinnitus, and decreased hearing. Her blood gas is initially alkalotic but changes to a ph of 7.28, wit a PCO2 of 23. Her Serum bicarbonate is low nd the anion gap is increased

this is classic salicylate toxicity... pts presents with hyperventillation secondary to direct stimulation of the respiratory center in the brain. In addition, ASA tox can cause tinnitus, encephalopathy, and with severe toxicity, pulomary edema and yperthermia. In early tox there is respiratory alkalosis from direct stimulation of the brain, followed by metabolic acidosis, because ASA poisons the mitochondria and you lose te aerobic metabolism of oxidative phosphorylation. Lactic acidosis accumulates from anerobic glycolysis.

bicarbonate is the best initial therapy for ASA poisoning.
Bicarbonate will alkalinize the urine, resulting in increased excretion of ASAP at the kidney tubule
_________________________________

An alacoholic pt with seizure disorder is admitted with drowsiness, dysarthria, and difficulty walking. She does not know which meds she is on for her seizures. On. P.E there is ataxia ad vertical nystagmus. The head CT is normal

phenytoin toxicity is the agent most likely to cause drowsiness, ataxia, and nysstagmus. The tye os nystagmus correlates with the severity of the phenytoin tox. Mild tox leads to horizontal nystagmus. in fact, horizontal nystagmus maybe present even at the upper end of normal therapeutic leve of phnytoin .. severe tox leads to vertical nystagmus.

there is no specific antidote for severe tox.

________________________________

a man with a HX of depression and an empty bottle of amitriptylene comes to the ER dept with obtundation, dry mouth, dizziness, flushing and dilated pupils.

and EKG is the most urgent ste in the evaluation of tricyclic antidepressants overdorse. The EKG will show prolongation or widening of the QRA.
the most common case of death from TCA are arrhythmias and seizures.

TCAs inhibit the fast sodium channels in the His-Purkinje system as well the atrial and ventricular myocardium. this decreases conduction velocity, increases the duration of repolarization, and prolongs the absolute refractory periods. This effect is similar to that of quinidine.

Bicarbonate should be given if the QRS prolongation is above 100millisecons.
________________________________________

a guard is being evaluated after a nerve gas attack, he is found to be lacrimating, urinating and defecating, in addition he has respiratory distress and bradycardia


Atropine is the best initial therapy for organophosphate poisoning. Atropine reverses the effect of the anti-cholinsterase inhibitor and the massive amount of acetylcholine that accumualtes. Organophosphates are absorbes through the skin. Further absorption is prevented for removing the patient's clothes and washing the patient.

Pralidixime is the definitive treatment for reversing organphosphate toxicity. pralidoxime reactivates acetylcholinesterase.
______________________________________________

Angiotensin-Converting Enzyme (ACE) inhibitors:

Ramipril, Lisinopril, Fosinopril, Enalapril, Perindopril, Captopril, Trandolopril
ACE inhibitors are indicated for the following conditions:

Congestive heart failure with low ejection fraction (prevents left ventricular (LV) remodeling
hypertension (particularly in diabetics)
proteinurea, including microalbuminurea
Acute myocardial infarction prevents worsening LV function
hypertensive crisis

Ace inhibitors block the production of angiotensin II (AT II) in the lung. At II is a potent vasconstrictor. AT II also stimulates the release of aldosterone from the zona glomerulosa of the adrenal gland.

The most common adverse effect of ACE inhibitors is a dry cough. Angioedema and hyperkalemia may also occur. Ace inhibtors transiently decrease the gloerular filtration rate (GFR) but are renal protective in long term.

>>>>>>>>>>>>
Reply

جوري
09-25-2008, 02:44 AM
Still on heme in diagnostics

Russel's Viper Venom Clotting Time (RVVT)

RVVT is a phospholipid-dependent coagulation test.
it is used in detection of antiphospholipid antibodies or lupus anticoagulant. Think of this in a patient who has a prolonged PTT that doesn't correct with a mixing study. The dilute Russell viper Venom test maybe indicated to confirm that the inhibitor is a lupus anticoagulant.

Clinical scenario may be of a woman with or without features of SLE, with repeated abortions or giving birth to an infant with heart block or presenting with venous or arterial thrombosis.

_________________________________________

Shilling's test

is ded to find the etiology of a B12 deficiency. The most common cause of B12 deficiency is pernicious anemia, where there is decreased intrinsic factor due to antibodies against IF

_________________________________

in the first stage, radiolabeled cyanocobalamine is given orally, followed by an intramuscular injection of cyancobalamine 1 hour later, urine is collected for determination of the percent excretion of the oral dose. In pernicious anemia, or malabsorption, excretion in the urine is low. The test is repeate with added oral intrinsic factor, Adding IF should normalize cyanocobalamine absorption and urinary excretion in pts with pernicious anemia, but not in those with intenstinal malabsorption.
schillings's test is hardly ever used, except in cases with dx of B12 deficiency where the diagnosis isn't pernicious anemia..

_________________________________________________

Serum protein electrophoresis (SPEP)

the SPEP is the separation and fractionation of the pt serum proteins in order to assess the individual component, i.e how much os the protei is albumin how much is mmunoglobulin

SPEP is used to evaluate a high total serum protein. SPEP is the best initial test to diagnose myeloma, particularly if an x-ray of the bone looking for lytic lesions has already been done, or is x-ray is not one of the answer choices.
the most common reason to have an abnormality on the SPEP is a monocolonal gammopathy of unknown significance.
the most accurate test for an IgG abnromality detected as an SPEP spike is a bone marrow. A total o >10% plasma cells is indicative of myeloma. Waldenstrom's macroglobulinemia will also give an abnromal SPEP with an IgM spike.

__________________________________________________ __

Sucrose lysis test
is a screening test for paroxysmal nocturnal hemoglobinuria (PNH) a sample of the patient's blood is tested with sucrose and observed for evidence of hemolysis from complement activation. The hemolysis of PNH is caused by increased sensitivity of the patient's red blood cells to lysis by complement and evoked hemolysis.
sucrose lysis is key when the t presents with dark urine in the morning, with or without evidence of venous thrombosis and pancytopenia. The coomb's test will be negative,

the most accurate test for PNH is flow cytometry for the presence of CD55 and CD59, also known as decay accelerating factor. (DAF)

_________________________________________

last is on opthalmology



this is a photograph of proliferative diabetic retinopathy (PDR). There is neovascularization over the disc, covering much of the surface area of the retina. Neovascularization differentuates PDR from non PDR. the aberrant blood vessles are fragile and leaky causing recureent retinal and vitreous hemorrhages. Dot blot hemorrhages, micoaneurysms, hard exudates, cotton wool spots, and intraretinal microvascular abnromalities (IRMA) are present in both PDR and none PDR

PDR can lead to retinal detachment and neovascular glaucoma
the treatment is with pan retinal photocoagulation.
___________________________________________

Now Pharm cardiac
___________

64 year old woman with a HX of HTN comes to the ER dept with palpitations, she is found to have atrial fibrillation and pulse rate of 125/mins

atrial fibrillation and atrial flutter with a rapid ventricular rate are best treated with either a calcium channel blocker, such as verapmil or diltiazem, a beta blocker such as metoprolol or digoxin. The target rate is less than 100 BPM

calcium channel blockers cause hypotension, constipation, peripheral edema and heart block.
beta blockers, cause hypotension, bronchospasm, depression, erectile dysfunction and worsening dyslipidemia
dignoxin causes dysrhytmia, hyerkalemia, confusion, diarrhea and visual changes.
chronic atrial arrhythmias should be treated with coumadin with an international normalized ratio (INR) of 2-3 to avoid an embolic stroke.

Electrical cardioversion is indicated when the patient is hemodynamically unstable, exhibiting such symtoms as hypotension, hemodynamically related confusion, shortness of breath or chest pain.

_______________________________________

calcium channel Blockers
Diltiazem, Verapamil, Nifedipine, Felodipine, Nicardipine, Amlodipine, Nitrendiine, Nislodipine, Isradipine
Calcium channel blockers are correct for the following conditions
HTN (in pts with diabetes or high risk coronary disease)
Atrial arrhythmias
ulmonary hypertension
Hypertrophic cardiomyopathy
Raynaud's phenomena
Subarrachnoid hemorrhage

CCB's work by causing vasodilation by relaxing smooth muscle in the vascular lining. CCB's also inhibit conduction in the AV node of the heart
All CCB's cause postural hypotension, flushing, constipation, and edema, diltiazem and verapamil can cause AV block. The others particularly nifedipine can cause tachycardia.
___________________________________________

Spironolcatone/Eplerenone
Maybe appropriate treatment for the following conditions
congestive heart failure (CHF): as one of the medication that will lower mortality. Eplerenone is an alternative for CHF only
Ascites: as the best initial diuretic therapy
Acne, especially for women because of its anti-androgenic qualities
HTN-rarely
Amenorrhea
Adrenal hyperplasia or aldosterone producing adenomas
this is best for CHF however along with ACE inhibitors and beta blockers.

Spirinolactone is an aldosterone antagonist, it has antiandrogenic effects, which is why it helps with hirsutism, acne and amenorrhea. but it can also cause gynecomastia and hyperkalemia.
Eplerenone is used for CHF and HTN as well but does not inhibit testosterone receptors, so it can cause hyperkalemia but not gynecomastia

________________________________________

Propanolol

is indicated for the following conditions
Cluster migraine headache prophylaxis: Must be taken for several weeks to prevent headaches
Portal HTN, decreases the frequency of bleeding from esophageal varices.
Thyroid storm, propanolol decreases symptoms acutely
Essential tremor
pheochromocytoma

propanolol is a nonspecific blocker of beta -1 and beta -2 receptors
associated with worst side effect profile than beta one specific, such as atenelol, metoprolol. propanolol can cause bronchospasm and depression, bradycardia and hypotension and erectile dysfunction. it can also cause hyperkalemia by inhibiting sodium/potassium Atpase. Propanolol can have adverse effects on glucose and peripheral arterial disease which rarely occurs with selective blockers.

________________________________________

34 year old woman comes to the ER with palpitation. An EKG shows supraventricular tachycardia at a rate of 160 min, there is no response to vagal maneuvers, such as carotid sinus massage.

adenosine is the drug of choice for SVT that is not responsive to vagal tone carotid sinus massage.
Adensine reduced calcium currents and is antiarrhythmic by increasing AV nodal refractoriness. It transiently slows the sinus rate and the AV nodal conduction velocity. It is thought to open potassium channels, hyperpolarizing nodal tissue and making it less likely to fire.
Adenosine causes transient asystole, but this usually lasts less than 5 seconds, a bolus can precipitate a bronchospasm.

>>>>>>>>>>>>>>>>>>>>>>>>>>>>
Reply

جوري
09-26-2008, 02:05 AM
I am going through a rough time folks wal7mdllah, so If you are reading this, pls remember me in your du3a

ok.. on with opthalmology



this isn't a clip from the incredible hulk, it is fluorescein stain of cornea

after manul staining with fluorescein, the eye is exposed to blue light tht detects foreign bodies or damage to the cornea. fluorescein staining i used in the evaluation of the following conditions
Corneal Abrasion
infection
injury or trauma
foreign bodies
abnormal tear production
kertoconjunctivitis sicca
abnromalities in the surface of the cornea will be stained and appear green
normally the dye stays in the tear film (water, mucus, oil) and doesn't adhere to the cornea
the most likely question in which fluorescein stain is the answer, is whe you are shown a person who had an ocular truama and there is severe pain, and you need to exclude corneal abrasion .
_______________________________________________


pts with retinal detachment often describe an ascending/descending vein in front of one eye with flashes, floaters. The unilateral loss of vision is sudden and painless.
retinal detachment is usually caused by a retinal tear
retinal detachment is a medical emergency. Laser surgery or cryoplexy is always used to help reattach the retina. A scleral buckle may be placed like a belt around the outside of the eye to push the wall against the detached retina, or a vitrectomy may be done where the vitreous is removed and replaced with a gas that pushes the retina against the wall of the eye.

______________________________________



Roth spot is actually a cotton wool spot (infarct) with surrpunding hemorrhage.
Subacute bacterial endocarditis is commonly associated; however, roth sports are not specific for subacute bacterial endocarditis, and maybe present in other disorders, such as diabetes, Leukemia, anemia, hypertension and HIV

Roth spots in subacute bacterial endocarditis are an immunological phenomenon.

_______________________________________



slit lamp exam is a microscopic examination of the anterior half of the eye by projecting a thin beam of light through a slit. The anterior half includes lids, lashes, adnexa, conjunctiva, sclera, cornea, iris, pupil, and lens. When combines with special lenses, the slit lamp may also be used to examine the posterior eye (cup/disc, macula, vessles, periphery)

in this photo



you see kayser fleischer ring in Descemet's membrane on slit lamp, which are basically copper deposits. This finding is diagnostic of Wilson's disease, a disorder of copper excretion.

Slit lamp is the test for a pt. with red painful eye with pain worsened by shinning a light in the eye or photophobia. When severe photophobia isn't associated with meningitis, it can be from iritis or uveitis, which is an inflammation of the iris. Slit lamp is also a good way to detect cataracts or corneal injury.

___________________________________________



Gonioscopy is visualization of the angel between the cornea and the iris. This is performed to assess for narrow angle or angle closure in glaucoma

in Gonioscopy, light from the slit lamp is deflected obliquely through the cornea using a prism or mirror in order to get a view between the cornea and iris. The trabecular mshwork and schlemm's canal, the drainage system of the eye, lie in thi angle.

Answer Gonioscopy when you see a case of glucoma with a red, painful eye with fixed, mid dilated pupil and tonometry has already confirmed glaucoma. Gonioscopy is used to determine the types of glaucoma after the diagnosis has been made.

________________________________________

now cardio in pharm

________________

Aspirin
used for the following

Acute coronary syndromes (myocardial infarction \[MI} and unstable angina as well as post stent and post surgical bypass pts.
Stroke and transient ishchemic attack (TIA)
Peripheral arterial disease
fever reudction (antipyretic)
rheumatoid arthritis
essential thrombocytopenia
kawasaki's disease
Arthrtis, gout and in general as an analgesic

Asa irreversibly inhibits platelets b inhibiting cycloxygenase
the most common adverse effect is bleeding. Aspirin also causes peptic ulcers, asthma, renal insufficiency and rash. In toxic amouns, aspirin leads to metabolic acidosis, tinnitus, encephalopathy, renal insufficiency, and increased anion gap.

________________________________________________

Dipyridamole

is never used as first line therapy for anything.
but can be used for the following conditions
Stroke in combination with Aspirin
preventing heart embolic complications
peripheral arterial disease with asa
as diagnostic testing in myocardial perfusion stuies with thallium

dipyridamole inhibits adenosine deaminase and phosphodiesterase, which increase levels of cyclic adnosine monophosphate (cAMP) cyclic AMP inhibits platelets
adverse effects can include the following
dizziness and headache
gastrointenstinal bleed
____________________________________________

Clopidogrel and ticlopidine
are not the best initial therapy for anythng, clopidogrel is the right choice after ASA can't be tolerated or has faled. clopidogrel is used for acute coronary syndromes, stroke, peripheral vascular disease and post stent placement in combination with ASA

clopidogrel and ticlopidine inhibit platelet aggregation, they prevent adenosine diphosphate (ADP) from stimulating platelet and fibrinogen binding

the most common adverse effect for both is bleeding, Ticlopidine is always the wrong answer therapeutically. Ticlopidine is most often associated with neutropenia and thrombotic thrombocytopenic purpura.

++++++++++++++++++++++++++++++++++++++++++++++++++ +++

Cliostazol
is a phosphodiesterase inhibitor that is used or peripheral arterial disease
cilostazole increases levels of cyclic adenosine monophosphate (cAMP) it irreversibly inhibits platelet aggregation by inhibiting thrombin, adenosine diphosphate (ADP), collagen and epinephrine. It is also a vasodilator that is greatest in the femola bed and less in the vertebral carotid and mesenteric arteries.

a patient with intermittent claudicaton from vascular disease should be treated with cilostazole. It is the single best therapy for PAD. It is used in addition to ASA, dipyridamle and exercise.
adverse
it canses edema, dizziness, and vertigo. The most serious effect is Afib, ventricular tachycardia and CHF.

>>>>>>>>>>>>>>>>>>>>>>>>>>>>>>>>>>>>>>>>>

73 year old man with CHF recently started onramipril, metoprolol and spirinolactone, and furosamide. He has developed chronic dry cough that makes it difficult for him to sleep..

ACE are the most likely medication to be causing chronic cough, the sx is secondary to their effect on bradykinin levels

Angiotensin receptor blockers (ARBs) should be started in pts who are intolerant of ACE inhibitors. ARB's have the same indications as ASC, such as CHF, HTN, and acute mycordial infarctions. They seem to have an equivalent mortality benefit as well. ARBs include losartan, olmessartan, valsartan, irbesartan, candesartan, telmisartan and eprosartan.
The most common side effect o ARBs are hypotension and hyperkalemia.
_______________________________
Reply

جوري
09-27-2008, 03:01 AM
ok.. still on cardio today and almost done with opthalmology we'll be moving on to other endeavors

Pharm cardio
_____________
Glycoprotein IIb/IIIa
inhibitors (GPIIb/IIIa)

abciximab
tirofiban
eptifibitide
lamifiban
Orbofiban

Glycoprotein IIb/IIIa inhibitors (GPIIb/IIIa) are platelet aggregation inhibitors that are useful in keeping the coronary artery open in acute setting

Glycoprotein IIb/IIIa work by reversibly antagonizing the IIb/IIIa receptor on the platelet. This prevents fibringoen and Von Willebrand's factor from binding to the receptor on the platelet and thus, prevents the platelet from binding to the endothelial lining of each other.

Glycoprotein IIb/IIIa inhibitors are an adjunctive therapy for angioplasty and other percutaenous coronary interventions. They are also useful for non-ST segment elevation myocardial infarctions when thrombolytics are not indicated.

Glycoprotein IIb/IIIa inhibitors (GPIIb/IIIa) can cause bleeding, thrombocytopenia, and coronary artery dissection.

_____________________________________________

Thrombolytics
tPA
Anistreplase
streptokinase
Alteplase
Tenecteplase

Thrombolytics are recombinant versions of tissue plasminogen activator tPA that are used to reopen acutely thrombosed coronary arteries.
tPA cleaves plasminogen to plasmin. plasmin will dissolve fibrin that has been freshly deposites. After several hour, fibrin is cross linked by factory XIII or 'clot stablizing factor' this makes fibrin refractory to dissolution by plasmin.

Thrombolytics are contraindicated when there is a major bleed occuring. Such as melena or intracranial bleeding. It is also contraindicated with aortic dissection, head trauma or BP > 180/110

tPA is the answer when the question describes chest pain within 12 hrs and a 1mm St elevation of a new left bundle branch block. tPA has less efficacy than primary angioplast for an acute infarction. tPA is also indicated for ischemic heart disease within 3 hours as well as pulomnary emboli with hemodynamic instability, such as hypotension.
___________________________________________

Digoxin

dig has two main indications. rate control of Afib and symptomatic control of CHF
Dig DOESN'T LOWER MORTALITY, in CHF but it does decrease the severity of sx and frequency of hospitalization. dig will not convert afib to sinus rhythm, but it will slow the rate.

dig inhibits the Na/K Atpase. this increases cystolic calcium levels and increases the force of contraction of cardiac muscle..

dig at toxic levels can cause nausea, vomiting, arrhythmias, confuscion, hyperkalemia and visual disturbances.
________________________________________

a pt with CHF is maintained only on a diuretic and digoxin is admitted because of confusion, hypotension, nausea, bradycardia and visual disturbances. His dix levels is markedly elevated


Severe dg toxicity should be treated with dig binding antibodies (digibind) the indications for dig specific fab fragments (d-fab) are hyperkalemia, arrhythmias, encephalopathy, or hypotension.

The most common electroradiographic abnromality is ectopy, hwever the most common serious rhythm disturbance is atrial tachycardia with variable block. Sinus bradycardia may occur.

dig toxicity may lead to hyperhalemia because dig inhibits the Na+/K Atpase.

____________________________________

Amiodrone

is a potent antiarrhythmic medication with excellent efficacy but multiple side effects from long term use. It is the drug of choice for venticular fibrillation in an acute resuscitation.
Amiodarone is structurally similar to thyroid hormone. It blocks inactivated Na+ and ca 2+ channels and has a beta blocking effect. It potently inhibits abnromal automaticity.

Adeverse effecs are
Hyperthyroidism, hypothyroidism, pulmonary fibrosis and non sight threatening corneal deposits.

this is the DOC for ventricular fib and ventricular tachy
treating a fib with systolic dysfunction


____________________________________

Diagnostics

Tonometry

this is the test to measure intraocular pressure. it tests for glaucoma

there are several ways of performing tonometry: pneumotonometry is used as a screening tool. It measures IOP with a puff of air to flatten the cornea. Golmann tonometry is the more accurate way to measure IOP. It combines a slit lamp with a special probe that flattens the cornea and a weight that determines the amount of pressure

the most accurate test is electronic indentation tonometry where a small pinlike instrument is laces directly on the cornea and the pressure is read back on a monitor

it is the best initial and most accurate test for glaucoma .

______________________________

all right we are now up to infectious disease in diagnostics






Trichomonas vaginitis


this is treated with oral metronidazole with concurrent treatment of sexual partner. Single dose therapy is perferred because of increased pt adherence

The pelvic exam will show a strawberry cervix which is cause by multiple punctate hemorrhages visible on cervix

trichomonas is the only organism known to be mobile on wet mount, as the flagealla has it swim across the slide.

_________________________________________

alright the other three after I come back from a break
___________________
aight back from break



this is s india ink of the cereberal spinal fluid showing cryptococcus neoforman. The organism has a mucopolysaccharide capsule that provides a 'halo' around it that pushes away the india ink.
this is the best initial test for cryptococcus neoforman, when you see a pt with AIDS with <100 CD4 cells who presents with fever, headache, and possibly a stidd neck. CSF will show a mild elevation of lymphocyts. The presentation is a 'subacute' which means it is slower in onset and milder than bacterial meningitis. Caution: Normal CSF protein and white cell counts do not exclude cryptococcal meningitis.

The most accurate test for cryptococcal meningitis is either a cryptococcal antigen or fungal culture of the CSF.

__________________________________________________ _______________



the smear is a wet mount and shows clue cells. they are abnormal epithelial cells with ground glass appearance and a hazy border. The tiny dark dots are bacteria stick to the surface of the epithelial cells.

there will be a thin discharge and itching, also dyspareunua. fishy odor positive for the whiff test adding KOH to slide. The PH is usually > 4.5
pregnant women with BV are at an increased risk for miscarriage, early preterm delivery and infection after the pregnancy.

++++++++++++++++++++++++++++++++++++++++++++


ah the kit for a bone biopsy
look for a pt with peripheral vascular disease, with leg pain, warmth and an ulcer, wo has an abnromality on x-ray or MRI consistent with osteomyelitis.

needle aspirate is gold standard
for
osteomyelitis
bone tumors
metastatic bone disease

radiological evidence of osteomyelitis and positive blood cultures can replace the bone biopsy if osteomyelitis is suspected.

>>>>>>>>>>>>>>>>>>>>>>>>>>>>>>>
Reply

جوري
09-28-2008, 03:02 AM
on with pharm
_________________

68 year old woman is being evaluated in your office for worsening sob with minimal exertion. her echo shows ejection fraction of 32%

The medications that have a clear mortality benefit in CHF are as follows

beta blockers, metoprolol, and carvedilol
ACE
ARBs candasartan and valsartan
spironolactone

An implantable cardioverter defeibrillator has benefit for reducing mortality with ishcemic cardiomyopathy and an ejection fraction <35%

____________________________

27 year old woman from ecuador is being evaluated because of increasing shortness of breath. she shows rales on exam. she has an early diastolic decrescendo murmur. The symptoms have become worst because of pregnancy

the patient has mitral stenosis
mitral stenosis is most often from rheumatic fever in the past.

best initial therapy for mitral stenosis is pre-load reduction with sodium restriction and a diuretic. This is problematic however, because of her pregnancy. Diuresis can potentially cause IUGR.
The most effective therapy for mitral stenosis is baloon valvuloplasty. this procedure is ideal for a pregnant woman. pregnancy increases plasma volume by 50% and worsens sx. baloon valvuloplasty is safe in pregnancy and is by far preferable to open heart surgery.

____________________________________________

Your pt has recently been diagnosed with coronary artery disease by stress test. He is a nonsmoker, he doesnt have diabetes or htn, his LDL is 145 after three months of life style modifications. including diet, exercise and attempts at weight loss

the best initial therapy for high levels of LDL despite life style modification is a statn. LDL above 130 needs drug therapy.

adverse effects: hepatotxicity, LFT's should be checked for thabdomyolysis or myositis though neither is as common as liver tox.
The goal therapy for CAD is an LDL <100.

________________________________________________

a diabetic hypertensive obese smoker is found on angiography to have CAD, her LDL is 110

a diabetic is found to have an LDL of 122

a man with CAD has an LDL of 170 despite rx with a statin and life style modifications for the last 6 months. his triglcerides are elevated and his HDL is low

the best therapy initially is always a statin

the goal therapy for a person with CA and diabetes or CAD and multiple risk factors is an LDL <70. established risks are diabetes, smokin, htn, and an age >45 in men and >55 in women

case one statin
2 the goal in a diabetic is an LDL <100

when the goal can't be met with a statin, seconf medication should be added, cholestyramine bind cholesterol in the bowel but leads to bloating, abdominal pain and flatus. The best therapy specifically for triglycerides is a fibric acid derviative such as gemfibrozol.

Niacin is the best med to raise HDL. Niacin causes flushing and elevations of glucose and uric acid. Ezeetamibe lowers levels but has no proven mortality benefit.

the goal is an LDL < 100. the for for triglycerides is <150

_________________________________________

53 year old experiences syncope while at the opera. she loses her pulse, she is found to have ventricular tachycardia. Electrical cardioverson restores her to sinus rhythm

an implantable cardioverter/defib is the best initial therapy to prevent sudden death from either ventricular tachycardia or ventricular fibrillation. Electrophysiologic studies aren't necessary. she has already sustained v tach with loss of pulse. inducing an arrhythmia is not necessary. Beta blockers may be helpful to prevent the arrhythmia from developing but the life threatening nature of this rhythm disturbance makes having a defirillator essential.

_______________________________________________


infectious dz in diagnostics

Bone scan or technetium pyrophosphate nuclear bone scan

Bone scan is a method of detecting occult disease that has deposited in bone

technetium is picked up by osteoblasts and deposited in the bone as they lay down new matrix. You must have osteoblastic activity in order for it to light up

bone scans can be abnromal from both infections and malignancies and cancer are detected by bone scans. Bone scans lack specificity and often can't distinguish soft tissue infections from nearby involvement of the bone.

The rpecise etiology of the abnormality on a bone scan often requires a bone biopsy to confirm dianosis

____________________________________

CD4 (t-Cell) counts

CD4 count is used to monitor HIV positive pts. serum measurement of CD4 (t-Cell) markers is representative of the number of helper T lymphocytes in the blood count. CD4 is used to monitor the response to reatement. start antiretroviral therapy: CD4 <350 or viral load >55,000.
answer CD4 count as the best test to determine the severity of immunosuppression in HIV/AIDS

prophylaxis should be initiated in the following scenarios

CD4 count less than 200 cell/ul: TMP/SMX for pneumocystis jirovrci
CD4 count less than 100 cells/ul: TMP/SMX for toxoplasma
CD4 counts less than 100 cells/ul: itraconazole for histoplasmosis in endemic areas.

CD4 counts less than 50 cells/ul: azithromycin for mycobacterium avium complex (MAC)

___________________________

will finish the last three after a break insha'Allah
_________________________

Darkfield microscopy

the most accurate test for primary syphilis
a scraping of swab from the syphilitic cancre is rubbed on a slide

answer darkfield microscopy for primary syphilis when the questions asks for most accurate test. primary syphilis there is a false negative rate of about 25% for the RPR or VDRL. The darkfield is more sensitive than an RPR
___________________________________
Gallium scanning

is a nuclear scan that detects infection and some cancers. Gallium builds up in areas where white blood cells are present and there is increased iron metabolism. Gallium is transported on transferrin. Gallium is a nonspecific test that can help localize the site for subsequent CT or MRI scanning

Gallium is considered on pts with persistent fever without localizing symptoms and the initial blood cultures, chest x-ray and urinalysis are negative. Gallium detects the source of fever os unknown origin, lymphoma, and abscesses.

Ultimately, a biopsy is the most accurate way to determine the etiology of an abnromal gallium scan. Cancers require histology and infections require culture to be confirmed.
__________________________________________

herpes simplex PCR
is done on CSF and is the single most accurate test for herpes encepalitis.

look for a pt with fever, headache, and confusion of less than 2 weeks duration. The PCR of CSF is more accurate than an EEG, head CT, MRi or even brain biopsy.

>>>>>>>>>>>>>>>>>
Reply

جوري
09-29-2008, 06:03 AM
today's 5/5



indium labeled leukocyte scan
is a nuclear medicine test of occult infection or FUO. A sample of the pt's blood is drawn and incubated with indium. The indium tags the white blood cells then localize to the site of infection.

look for a case of occult infection not found on routine testing such as blood cultures, chest x-ray, or urinalysis. Indium is useful when gallium is inaccurate such as with intrabdominal infections.

a biopsy is more accurate than an indium scan.

_______________________________________



this is a KOH prep of candida
is is the best initial test to diagnose superficial fungus. KOH is applied to a wet mount samle of vaginal discharge/secretions or to a skin scraping to identify the fungus. This is seen under the microscope. Epithelial cells dissolve an the fungal structures remain behind and are visible

KOH is used for the following

Tinea (pedis, manus, corporis, cruris, capitis)
onchomycosis
tinea versicolor
candidiaisis

applying KOH to a wet mount and smelling/whiffing immediately after yeilds fishy odor (amine) is bacterial vaginosis.

_______________________________________________



Monospot test is the initial test used to diagnose infectious mono .. it detects hetrophile antibodies that are charcteristic of Epstein barr virus

monospot is the best initial test for a pt with a sore throat, fever, lymphadenopathy, and malaise, a total of 50% of pts will have splenomegaly, rash is present in 15% unless they have been given ampicilin. When it is present closer to 90%. Exudative pharyngitis may occur

the most accurate test acutel is the IgM to EBV viral capsid antigen (VCA). antibodies to EBV nuclear antigen (EBNA) rise in 3-4 weeks
_______________________________________



legionella urine antigen is found in legionella pneumonia. it is the best initial test
legionella urine antigen has a 100% specificity with L. pneumophila type i. Only 70-80% of disease is caused by L- pneumoophila type 1. sputum culture or a culture of the tracheal aspirate on specialized charcoal/ yeast extract remains definitive.

suspect legionella antigen when seeing a nursing home resident admitted for pneumonia that presents with fever, confusion, diarhhea and hyponatremia. CPK, liver function test and creatinine can also be elavated

_________________________________________

.

polymerase chain reaction HIV RNA viral load

the PCR HIV viral load is a quantitative measurement of the amount of virus circulating in the pt's blood

this test is the first thing to change in response to treatment. PCR HIV viral load is also the first thing to become abnromal is the patient stops taking medications or the treatment fails. The viral load test tells how fast the CD-4 T-elper cells will drop. The higher the viral load the faster the disease process.

measurements should be made at the time of the HIV diagnosis and every 3-4 months thereafter. The PCR viral load is also the basis of HIV sensitivity testing in order to determine the presence of resistance in failing regimen.

________________________________________

Now Pharm, finish cardio and move on to endocrine

_______________________

A man with metastatic cancer develops DVT/ woman with metallic heart valve presents for routine care.

those with deep venous thrombosis (DVT) can receive either low molecular weght heparin or IV-unfractionated heparin, their efficacy is identical. The measurement is followed by warfarin (coumadin) to international normalized ratio (INR) 2-3

heparin potentiates the effect of antithrombin on the clotting cascade

those with a first DVT should be maintained on warfarin for at least 6 months

most common adverse effect of both therapies is bleeding. Haparin can result in thrombocytopenia

metal heart valves maintain lifelong coagulation with warfarin. pts with metal heart valves are the only pts n which you routinely maintain target INR above 2-3. the target INR is 2.5-3.5
warfarin inhibits vitamin K dependent clotting factors (II,III,VII,IX and X)

__________________________________

a 32 year old woman in her third trimester with HTN, edema and proteinurea, se is being prepared for dleivery.

67 year old man develops torsades de pointes post infarction, he is hemodynamically stable.

Magnesium sulfate (Mg2+) is the best medical therapy for pre-eclampsia and eclampsia as well torsades de pointes. Magnesium prevents seizure in exlapmpsia.

Mg2+ works by decreasing ACH in motor nerve terminals and acting on the myocardium by slowing the rate of SA node impulse formation an prolonging conduction time. Mg2+ appears to inhibit calcium uptake into smooth muscle cells, reducing uterine contractilty . in general Mg2+ relaxes excitable neural and muscular tissue

Magnesium sulfate can lead to muscular weakness and loss of reflexes it causes diarrhea by promoting bowel evacuation through osmotic retention of fluid, which distends the colon. Severe magnesium toxicity can lead to respiratory paralysis
__________________________________

31 year old woman comes to the ER dept with palpitations, the EKG reveals a short PR interval, supraventricular teachycardia develops. When diltiazem is adminstered, the pt develops ventricular tachycardia. The pt remains hemodynamically stable .

Wolff=parkinson white syndrome


is best treated with procainamide or amiodarone. if an acute arrhythmia such as SVT or VT develops. these agents are effective against both atrial and ventricular arrhythmias, making them drugs of choice.

Cardioversion for a pt hemodynamically unstable. this is defined as chest pain, SOB from CHF, confusion, hypotension defined as systolic BP <90

radiofrequency catherter ablation after electrophysiologic studies is curative. The accessory conduction pathway is destroyed in an electrophysiology laboratory.

_________________________________________

Propylthiouracil (PTU) and Methimazole

PTU and methimazole are the best initial therapies for hyperthyroidism. They are used to lower the level of thyroid hormone.
they don't block the target organ effect. PTU is safe during pregnancy.

PTU and methimazole inhibit the peroxidase enzyme. This blocks the oxidation of iodine, inhibits the incorporation of iodine onto tyrosine, and stops coupling of these compounds into T3 and T4. hence these meds block three essential steps in the formation of thyroid hormone.

the most important adverse effect of these compounds is neutropenia. Both lower granulocyte levels by inhibiting bone marrow. Purpuric skin lesions may also occur.

______________________________

50 yea old man with psostate ca comes to ER after having experienced lethargy, constipation and generalized weakness for four days. His calcium is markedly elevated at 14.5 and the WKG shows a short QT, hydration with saline and loop diuretics have been given

Bisphosphonates are the best initial therapy to lower calcium levels after fluids and diuretics have been initiated. Bisphosphonates (pamidronate, zolendronic acid, alendronate, ibandronate risendeonate) are also used for osteoporosis when the T score is 2.5 standard deviation beloe normal (t-scre-2.5)

bisphosophantes work by inhibiting osteocalsts, prevnting bone resorption. they bind calcium hydroxyapatite in the bone and prevent its dissolution.

oral admin can cause esophagitis and for this reason they should be taken sitting up while drinking lots of water. rarer adverse is hypocalcemia and osteomalacia . osteonecrosis of the jaw has been associated with the use of bisphosphontes.

>>>>>>>>>>>>>>>>>>>>>>>>>>>>
Reply

جوري
09-30-2008, 04:52 AM
thought I'd forget about this thread today.. it has been a bad day sob7an Allah..
maybe tomorrow will be better insha'Allah

on with pharm /endocrine
___________
Calcitonin

is indicated for the following conditions
acute hypercalcemia
paget's disease
osteoporosis

calcitonin works predominately by inhibiting osteoclast activity.
calcitonin can cause flushing, rash, and constiption it rarely causes depression and bronchospasm. The most common adverse effects are rhinitis, and flulike sx.

________________________________

ok we now move on to derm

minocycline
is the best oral antibiotic for severe acne, that is not controlled by topical antibiotics or topical vitamin A derivatives such as terintoin. Other indicaations for minocycline include the following
nocardia
actinmycycosis
rosacea

minocycline is a tertracycline antibiotic that inhibits production in bacteria

minocycline can cause blusing discoloration of the skin but doesn't cause photosensitivity rash as doxycycline can. miocycline can bring on vertigo by causing vestibular dysfunction.

________________________________

a woman comes in with mild acne, she has a few comdones with an occasional inflammed papule or pustule

a man has failed initial theray for acne and has numerous papules and postules with mild scarring

your pt is very distressed and depressed because of numerous laege cysts on the face and trunk he is severely scarred

case one mild comodonal acne, is treated with topical benzoyl peroxide and a topical atibiotic such as erythromycin or clindamycin. Benzoyl peroxide is both antibacterial and comedolytic. topical antibiotics will eliminate the causative organism proionbacteium acnes from the comoedones.

case two, severe acne resulting in scarring in addition to benzoyl peroxide. the patient should be treated with a topical vitamin A derivative (tretionin) an oral antibiotic such as minocycline should be used.

case three is severe cystic acne. an oral antibiotic and oral vitamin A are needed, isotrentinoin will decrease sebum producton but is extremely teratogenic and can cause severe depression, dry skin and hyperlipidemia
_______________________________________

an adolescent boy comes to see you for rx for a long standing itchy eczematous rash on his face, hands, and feet in the flexural areas, the skin is lichenifies from scratching, he also has seasonal rhinitis and occasional utricaria

Atopic dermatitis is treated with antihistamines such as fexofendaine, cetrizine or loratadine. hydroxyazine and diphenhydramine are more potent but much more sedating. Doxepine is a tricyclic with extensive antihistamine effects.

Atopic dermatitis sx can be lessedned in the long terms by moisturizing skim, avoiding harsh sopas and treating skin infections. although topical steriods are effective to control symptoms acutely, in the long term they lead to skin atrophy. Topical calcineurin inhibitors such as tarcolimus and pimercrolimus can control atopic dermatitis in the long term without using steriods.

Antihistamines, such as fexfenadine are also used in the following condiions

Allergic/ seasonal rhinitis
angioedema
utricaria

_______________________________

a woman comes to the offic for pruritic, silvery, scaly, lesions of the knees, elbows, and hands, the lesions are on the extensor surfaces

localized psoriasis is treated with topical steriods to prevent skin atrophy. treatment with calcipotriene a vitamine D analog should be used. topical tazarotene, a vitamin A analog is also used. Tacrolimus and pimercrolimus are also used

widespread disease is treated with UV light. the most effective systemic therapy is methotrexate, but it also has the most adverse effects. Biological agents such as etanercerpt, alefacept and efalizumab are used as alternatives to methotrexate.

______________________________________

diagnostics infectious dz

VDRL/RPR

serume measurement for these nontreponemal examinations are used to diagnose or screen for syphilis

in the primary stage, you would expect the patient to present with a chancre and painless regional lymphadenopathy. in the secondary stage rash, lymphadenopathy, condylomata lata and alopecia can be expected.
teritiary syphilis is charcterized by cardiac or neurological dz.

false positives connective tossue disease, infectious mono, malaria, leprosy, IV drug use, hep C, infective endocarditis and pregnancy

false negatives prozone phenomenon- very high antibody titers are prestnt

when your clinical sspicion is high yet the nontreponemal tests are negative, you would want to do a darkfield examination in primary syphilis and FTA-ABS test for other stages..

The RPR and VDRL decrease or disappear in response to RX, the FTA can remain positive life long..

_____________________________________


Tzanck smear

are used to dx the skin manifestation of either herpes simplex of varicella zoster. You are looking for multinucleated giant cells. Tzanck smear is not necessary if the diagnosis is clinically clear from the presence of vesicles.

the most accurate test for a herpetic infection of the skin or genitals is a viral culture.

_____________________________





arthrocentesis is a needle aspirate of synovial fluid for microscopic analysis. The cell count is the most accurate ways of telling is there is an infection vs inflammation.

counts from 0-2000 are normal
from 2000-20,000 are from inflammatory disorders such as gout. counts above 50,000 are from infections, counts between 20,000, to 50,000 are intermediate

look for a pt with new onset of joint that is warm to the touch, painful, and swollen from an effusion, and decreased with mobility. serum uric acid maybe elevates

gout is negatively birefringent needle shaped crystals, these are monosodium urate crystals

pseudogout is positively birefringent rhomboid shapes crystals

___________________

aldolase

is released from damages skeletal muscle and is elevated in inflammatory myopathies such as polymyositis and dermatomyosistis. Creatine and CPK are also elevated.

look for a case of an older woman with difficult rising from a chair, there maybe a heliotrope rash and Gottron's papules. Aldolase or CPK is the answer to the question best initial diagnostic

the most accurate tests are the electromyogram and the muscle biopsy. these confirm the diagnosis of polymyositis and dermatomyosisits when the alodolase is elevated.
_______________________________________

Allergy testing

is a subdermal exposure to trace amounts of suspected allergens with subsequent visual inspection for signs of hypersensitivity.

this is contraindicated when there is a high risk of anaphylaxis.

beta blockers such as propanolol should be stopped prior to testing for allergens for which there might be severe reactions during desensitization. If anaphylaxix occurs, we want to use epinephrine to reverse it, and beta blockers will make the epi less effective..
>>>>>>>>>>>>>>>>>>>
Reply

جوري
10-01-2008, 04:25 AM



was out for Eid today.. insha'Allah will resume tomorrow, might add another sections of most likely diagnosis if my time permits insha'Allah...

:w:
Reply

جوري
10-02-2008, 01:42 AM
I am going to introduce a new most likely dx here, will do it at a rate of 15/5/5 to catch up with the other topics..

starting with cardiology

________________________________

1- Man comes into the Er with chest pain, that changes with respiration. the pain is sharp, and is worsened by inhalation. He is short of breath, as well but the symptoms are hard to assess because a deep breath causes pain, so he takes short fast shallow breaths...

most likely dx
a- te pain changes with bodily position. it is worse when lying flat, and better when sitting up.

b-cough, fever, hemoptosis
c- sudden onset of SOB with a normal lung exam
d-hx of astham or COPD with sudden onset of SOB, decreased breath sounds on one side

pericarditis is associated with pleuritic chest pain that also worsens with changed in bodily position. typically the pain of pericarditis is releverd when the person sits up, and stretch is relieved from the pericardium. only 30 % of pts have a pericardial friction rub. if it is present, it helps answer, the most likely diagnosis question. if the rubs are absent, this excludes nothing EKG shows PR depression and diffuse concave ST elevation.

Pneumonia is associated with cough, sputum, and hemoptysis. fever is nonspecific, every cause of pleuritic chest pain is asspciated with fever.

Pulomnary embolus presents with the sudden onset of shortness of breath and clear lungs on exam. There is no charcteristic physical findings of PE to allow you to resolve the dx in question.

sudden onset plus normal lings = PE. pt maybe on a long trip

pneumothorax, when large breath sounds decrease on one side. obstructive lung disease ca predispose to pneumothraces, particularly when there are blebs with COPD.

__________________________________

A man is brought to the ER dept after losing consciousness at home, he wakes after a few mins

a- sudden loss of consciousness and rapidly regaining consciousness, he is fully intact when regaining consciousness.
b- sudden loss but was disoriented for an hour or two on regaining consciousness
c-gradual loss of consciousness with shaking, sweating, palpitatons, and nausea

d- has a 10 point rise in pulse and a 20 point drop in systolic pressure when going from lying to upright posture.


crdiac syncope such as an arrhythmia or obstructive cardiac lesions results in the sudden loss and regaining of consciousness. Ventrcular rhythm disturbance such as a ventricular tachy or fib will result in syncope

seizures can result in sudden loss of consciousness but the regaining of alertness is slow because of being post-ictal results in a gradual regaining of consciousness described as post ictal state.

metabolic probs such as hypoglycemia, hypoxia or drug intoxication lead to a gradual loss of consciousness. this is often accompanies by signs of autonomic hyperexcitability such as tachycardia, palpitations and diophoresis. you may see metabolic respiratory acidosis or alkalosis.

orthostatic instability leads to syncope in association with a >30 pt drop in systolic blood pressure on changing position with a 10 point rise in pulse.

________________________________________


a patient comes to the office with palpitations for the last several weeks. she denies chest pain or SOB, the sensation is like her heart will flutter away at her chest..

1- the pulse is irregularly irregular

2- she drinks lots of coffee and the EKG is normal

3- she is losing weight and has diarrhea, her eyes are bulged forwars (exophthalmos)

4- there is episodes of flushing and low BP

A fib presents with palpitations and an irregularly irregular pulse. arial rhythm disturbances rarely result in syncope.

caffeine can easily lead to the feeling of palpitations even with normal EKG

hyperthyroidism results in weight loss, anxiety, tachycardia, diarrhea, and palpitations, about one third have ocular findings such as exopthalmus


carcinoid syndrome leads to papitations from the oversecretion of the neurotramsitter seratonin, episodic flushing, diarrhea, and episodes of HTN are common. palpitations with HTN should make you think of pehochromocytoma

________________________________________________


patient comes to ER with palpitations found to have an SVT after admin of filtiazem his rhythm deteriorates to Vtach

Wolff-Parkinson-white syndrome can present with an Atrial arrhythmia, alternating with ventricular arrhythmia. The key to answering the question is worsening of the rhythm after giving a calcium channel blocker CCB such as dilitizem or verapamil. the rhythm may also worsen with dig. CCB and DIg block conduction through the normal AV nodal pathway and force conduction through the abnromal aberrant tract. resulting in a deterioration of the rhythm.

EKG showing a short PR interval or delta wave. EKG is the best initial test.

elecreophysiological studies are the most accurate for pre-excitation syndrome.

___________________________________


a pt comes for toutine visit on PE he is found to have a pulse of 45

1- he is asymptomatic, he runs five miles each day
2-he has canon 'a' waves in the his neck. occasionally he is light headed.

sinus brady is common finding in well trained athelets. You can't be sure if bradycardia originates at the sinus node without an EKG

third degree complete heart block is associated with canon 'a' waves in the neck. it is often associated with symptomatic hypotension or syncope, and that is why a pacemaker is necessary. canon 'a' waves result from atrial systole against a closed tricuspid valve. the only condition to have bradycardia and conon 'a' waves is a complete heart block.

________________________________________


62 year old man is the ICU after an MI he is no suddenly lightheaded and hypotensive

1- there is a holosystolic murmur at the apex radiating to the axilla, the lungs are congested.

2-oxygen saturation increases from 40% in the right atrium to 82% in the right ventricle

3- he had an inferior wall infarction. he has tachycardia and clear lungs

4-bradycardia and canon 'a' waves are present

mitral valve rupture leads to acute pulmonary edema, the murmur of mitral regurgitation is holosystolic and radiates to the axilla

valve rupture leads to a step up in oxygen saturation as you go from right atrium to the right ventricle, this is from left to right cardiac shunting

right ventricular infarction accompanies 30-40% of inferior wall infarctions. This is because they are both supplied by the right coronary artery. The lungs are clear, patient may also have rupture of both left ventricular free wall, but outcome then is immeiate death.

complete heart block leads to bradycardia. hypotension and canon 'a' waves.

____________________________________________

a man is admitted for an MI of the anterior wall, he suddenly loses pulse

1- most likely DX

2- best initial dx test

3- initial therapy

sudden loss of pulse can be from asystole, Vfib, vtach or pulseless electrical activity

EKG is the best initial DX

in Asystole start with epinephrine and atropine

Vfib and Vtach-unsyncrhonized cardioversion
pulseless electrical activity, correct the underlying cause such as tension pneumothoraz, PE, hypovolemia and tamponade.

____________________________________________

28 year old woman seen for severe HTN, pressure is repeatedly elevated
1- abnromal sound auscultated in the flanks of the abdomen
2-hypokalemia
3-episodic with palpitation
4-upper extremity blood pressure is greater than lower extremity pressure
5-hirsuitism, clitromegaly


1- enal artery stenosis is most likely when bruits are heard on examination and most common cause of secondary hypotension
2- when you see hypotension combined with hypokalemia, think Conn's sndrome, or primary hyperaldosteronism.
3-pheochromocytoma is the only for of htn that is episodic
4-coarctation of the aorta results in higher blood pressure in the arms compared to the legs. the pressure can also differ between the arms, if coarctation occurs before 'offshoot' of left subclavian artery.
5-CAH with 11-hydorxylase deficiency leads to HTN when pt is under 30 and the HTN is very hard to control such as needing more than two antihypertensive meds.

________________________________________________

Man presents wih SOB and TIA, he has intermittent fever. The murmur changes markedly with bodily posotion. The SED rate is high

1-most likely DX
2-Most accurate diagnostic
3-RX


atrial myxoma is a benign cardiac tumor that is charcterized by a murmur that changes with bodily positions. This is also called a tumor plop. Myxoma presents with a murmu that is similar to mitral stenosis because it obstructs diastolic filling. There are also systemic sx such as fever, elevated SED rate, and anemia. TIA is likely due to embolization of myxoma, myxoma can be friable

Echo dignoses myxoma

treated with surgical removal only.

____________________________________


Man comes in with CP, Dyspnea and diaphoresis, occurs on exertion

1- pain happens every time he walks one or two flights of stairs. EKG normal
2-pain occured wih much less exertion. EKG shows ST depression
3-EKH shows ST elevation

stable angina is Cp occuring with the same level of exercise, stable angina is pain with exertion and relieved by rest, with a normal EKG

unstable angina is a type of acute coronary syndrome with a worse pattern of chest pain or pain occurin at rest. Acute coronary syndrome is the proper name, because you can't tell if the cardiac enzymes such as the troponins will be elevated until later. this may turnout to be a nonstemi infarction if cardiac enzymes are elevated
3- acute MI is assumes where there is chest pain with st elevation even before cardiac enzymes results are obtaines. The majority of pts with chest pain and stemi develop elevated troponins and CPK-MB levels. The pT would be a candidate for thrombolysis and or immediate cardiac cathe.

_______________________________________________


a man comes with pain his his leg, the last several weeks. the pain occurs while he is walking and is relieved when he sits down.

1- pain is unilateral and occurs with any forms of exertion of the leg, the skin is mooth with hair loss and skin appendages.
2- the pain is bilateral and made worse when walking downhill, he has no pain when bicycling.


PAD occurs as pain with any form of exertion of the lower extremity and is relieved by rest. as it wosens, loss of skin appendages such as hair follicles and sweat glands. sx may improve when dangling the head of the bed, gravity increases blood flow to legs.

spinal stenosis results in bilateral leg pain that is highly dependent on bodily postion. it is much worse with anything that has the pt leaning back. such as walking downhill. it is relieved by leaning forward such as sitting or bicyling. it isn't exertion that leads to pain, it is the pressure on the spinal cord on the ligamentum flavum in the spinal canal. MRI of the spine, likely lumbar will demonstrate stenosis.

___________________________

34 year old woman comes to the office with palpitations and atypical CP. The pain has no fixed patter. PE reveals a mid-systolic murmur. The murmur worsens with valsalva and improves by a leg raise

what is the dx and rx

MPV is the most likely DX when the question describes atypical CP in a young female. There is a midsystolic click followed by a murmur. Valsalva will worsen only the murmus of MVP and hypertrophic obstructive cardiomyopathy

MVP is confirmed with echo, and treated with beta blockers. Endocarditis prophylaxis prior to dental procedures is no longer recommended
________________________

a healthy young man experiences SOB with exertion. he has an episode of syncope while playing basketball. exam reveals
a systolic murmur worsens with valsalva and improves on squatting

hypertrophic cardiomyopathy most often presents with SOB, it can also cause syncope and may lead to sudden death. The murmur has the same crescendo/ decrescendo pattern of aortic stenosis but is heard best at te lower left sternal border. Aortic stenosis is heard best at the 2nd right intercoastal space and radiates to the carotid arteries.

HOCM shouls be treated with beta blockers. if Syncome occurs, an implantable cardioverter/defibrillator should be placed. Endocarditis prophylaxis prior to dental procedures is no longer recommended.

__________________________________________

woman comes with progressively worsened SOB on exertion and a murmur. there is edema

1- pregnant woman with a diastolic extra sound followed by a murmur. she has dysphagia and hoarseness

2-older man with angina and a systolic murmur radiating to the carotid arteries.

3- diastolic decrescendo murmur with a wide pulse pressure

mitral stenosis, often becomes symptomatic during pregnancy because of marked increase in plasma volume during pregnancy.

dysphagia hoarseness happen from enlargement of the left atrium pressing on the esophagus and recurrent laryngeal nerve. another clue is opening snap

aortic stenosis is a systolic murmur radiating to the carotid arteries and is the most common presentation of aortic stenosis

aortic regurgitation presents with SOB, but this is a nonsepecif finding, the key to the answer is the diastolic decrescendo murmur at the left lower left sternal border and th wide pulse pressure.

__________________________________________________

A man with a hx of HTN comes to the ER dept with a sudeen onset of sharp CP radiating to his back, there is a 15 point difference in BP between the left and right arms. a dastolic decrescndo murmur is present .

Aortic dissection presents with sudden onset of CP radiating to the bck, particularly the shoulder blades. HTN by far is the most common risk factor. the key to diagnostic is the pain radiating to the back, the wide pressure from aortic regurg and the difference in pressure between the arms.

the best initial test is a CXRAY, which may show a widened mediastinum

transesophageal echo, CT angiogram and MRI each have about a 90-95% sensitivity. aortic angiography is the single most accurate test. Transthoracic echo is not the test of choice due to its limited accuracy \; management is aggressive control of systolic BP (100-120mmhg)

__________________________________________________

ok.. now one with pharm endocrine
Sitagliptin, exenatide, and parmlintide

sitgaliptin, exenetide and pramlinitide are used as adjunctive therapies for diabetes. in addition to sulfonylurea, metformin or both. prmlinitide can also be combined with insulin.

mechanism

exenatide is an incretin mimetic. Incretin is released by the intestinal tract and increases after you eat. incretin increases insulin synthesis and release by pancreatic beta cells. Incretin decreases glucagon production

sitagliptin- blocks degradation of incretin
pramlintide- suppresses glucagon release via undetermined mechanism , delays gastric emptying, and has CNSS mediated anorectic effects

most common adverse effect is hypoglycemia.

________________________________________
Orlistat

used to RX obesity
is a reversible inhibitor of pancreatic lipases. This action blocks the absorption of fat in the intestine. fecal excretion is increases, this may cause mild decrease in fat soluble vits such as ADEK
advers, oily stools, diarrhea, flatus, abdominal pain and other sx, similar to celiac d and steatorrhea.

___________________________________________

fludrcortisone

is a steroid with very potent mineralocorticoid activity. being adminstered, is is the closest thing to eating aldosterone.

fludrcortisone causes sodium rention. it also causes hydrogen ion, acid excretion, leading to metabolic alkalosis

fludrocortisone is the rx for the following conditions

Addison's dz. (adrenal insufficiency)
orthostatic hypotension
septic shock rx (adjunct to hydrocortisone)
type IV renal tubular acidosis

fludrocortisone can lead to hypokalemia, edema, and HTN. like all steroids, fludcortisone can lead to osteoporosis, impaired wound healing and easy bruising
_______________________________________________

Alpha glucosidase inhibitors

acarbose
miglitol

these are oral hypoglycemic agents that are used in the RX of Dm
alpha glucosidase inhibitors are an adjunct for DM when sulfonylurea, metformin, or both are not able to control pts glucose level/ the pill must be taken with each meal to prevent glucose absorption

the adverse are bloating, diarrhea and flatulance.
________________________________________________

Metformin

is a biguanide used in rx of DM
it decreases hepatic gluconeogensis, and increases peripheral sensitivity to insulin. it doesnt require beta cells and will not therefore cause hypoglycemia.
metformin may be associated with lactic acidosis. Metformin is renally cleared and therefore a rise in creatining can cause the levels to rise, metformin is contraindicated in RF

metformin is the initial therapy for type II Dm, this is particularly true for obese pts who have peripheral insulin resistance.
______________________________________

now diagnostics still in rheumatology

ANA, double stranded DNA, and anti-SM antibodies

ANA is more than 98% sensitive for SLE but ANA isn't specific for SLE.
a negative ANA excludes SLE with 98% negative predictive value. Anti- DS-DNA antibodies are 99% specific for SLE. anti-smith antibodies are extremely specific for lupus.

Answer ANA, anti-SM, and ds-DNA when you see a young female with multiple spontaneous abortions (lupus anticoagulant) rash, joint pain
__________________________________________________ __

Anticentromere antibodies

Anticentromere are specific for CREST syndrome.
CREST is calcinosis, Rynaud's phenomenon, esophageal dysmotility, telangeictasia, Antitopoisomerase (SCL-70) is specific for systemic sclerosis)

anticentromere is the best initial test in a female 30-50 with positive rheumatoid factor positive DNA

__________________________________________________ __

Anti-histone antibodies
are often associated with drug induced lupus SLE. The most common drug that cause lupus are procainamide, hydralazine, sulfonamides, INH, and pencilamine.

stop meds
____________________________________


anti-Ro and anti-La antibodies are associated with Sjögren syndrome presents with a positive ANA, anti RO/Anti La and positive for rheumatoid factor.

best used when you see a triad of kertoconjunctivitis (sicca syndrome) xerostomia and arthritis, or dry eyes, dry mouth and arthritis
_________________________________________________

Anti-scl 70 antibodies

are directed against topoisomerase-1 and are ordered when systemic sclerosis, particularly the diffuse type is suspected.

look for polyarthralgia; Raynaud's phenomenon, thick, hidebound skin, dysphagia, and GI tract hypomotility as well as pulmonary and renal abnromalities.

Anti-SCL 70 has a low sensitivity 30%. hence negative test will not rule out dz.

it is however, strongly correlated with severe lung involvement, such as pulmonary fibrosis.

>>>>>>>>>>>>>>>>>>>>>>>>>>>>>>>>>>>>>>>>>>>>>>>> >> >>
Reply

جوري
10-02-2008, 08:22 PM
:sl:
1- oh I know.. but when you are doing medicine you get no time off whatsoever
2-You can't possibly remember everything, you need to always refresh. if you know the basic pathophysiology of things that you can apply it to any disease state, because the body only responds in a a certain number of predictable ways to any stressor.. but there is so much going on that you have to pay attention too, far to expansive for me to sum up in a couple of words here.
3-I get my info mostly from books, but I am subscribed to two journals at this time. It is important to keep on top of them, because you'll find yourself answering incorrectly. for instance as a quick example. prophylaxis for endocarditis was mandatory for many a conditions, one which was valvular heart disease, the type of prophylaxis itself also had conditions, now you'll find the guidelines have changed. Also many meds go obsolete, some new ones are in.. you have to do continuing eduction yearly, plus need to always renew your ACLS every couple of yrs
4- Do I enjoy it? well long gone the time when it was fun.. after a while it is just a very stressful life style and job, and alot of debt, loss of social skills, alienation from most family and freinds, lack of sleep, and inability to relate to others outside the confines of what you are taught, and how your own brain functions. basically life passes you by while you watch it.. but it has its rewards.. (I am yet to find them out) so don't let me discourage you if this is something you really want to do.. go in with a level head.

for me I had to fight with my family to let me do this, my dad always thought women should be pharmacists, and in fact that is what my sis is, she has a doctorate in pharmacology (but she is miserable) I couldn't forsee myself in pharmacy (though it is in fact an excellent career for women. the pay is good and you can always work on a part time basis), I also had to fight with my own fears. and I had to balance the two, because I knew they were all waiting for me to fail to come out with the I told you so. I'd advise you get married before hand with someone going in the same field, because I don't think people outside of medicine are compatible for many reasons.
1- if you are very stressed for any number of reasons, someone is suing you, the hours are bad, your liability insurance is up, your boss is an SOB and you come home to a nagging husband or a nagging wife, you are already in very bad shape.
2-most of the time you want someone who can listen to you who actually appreciates what you mean.. you can't discuss how you've erroneously misdiagnosed someone in aortic dissection with a myocardial infarction thereby leading to their immediate demise, with someone who hasn't the faintest clue what you are talking about.
They will judge you harshly, and unsympathetically on top of your own guilt and a heap of legal troubles you are already in.
3-sometimes you want to come home and say nothing at all, and would also appreciate someone who can appreciate how silence is heavenly, or someone willing to take you on a car ride at 3 am simply because you want to have a good cry away from people's prying eyes and not have it be thought of a completely outlandish request.
4- as for having kids.. I really can't comment on that any more than I can on a
'good' marriage. It is something you'll have to work out with your uber understanding husband if such a man exists, hence I stress finding someone in your exact field.
I'd advise in the end, that you go into a woman friendly field, and by that I promise you I don't mean Ob/gyne.. I mean something like dermatology, radiology, opthalmology/ pathology maybe peds.. all with the exception of peds are extremly competitive to get into.. you'll find 400 applicants competing for three spots (annually)

so my advise, is genuinely have an earnest session with yourself and see if this is right for you. And if it is, then don't let anyone or anything deter you..
in the end, I believe it to be the most noble profession and the highest level of education anyone can have, if those two are the only satsifactions one can get out of it..

Allah knows best

:w:
Reply

جوري
10-02-2008, 09:48 PM
I am doing this a bit early today.. 15/5/5

questions ending cardio and moving on to others

________________________________________________

a patient comes in with SOB on exertion, ortopena, bilateral lower extremity edema, and JVD. There is marked improvement with furosamide

what is the DX
1-multiple infarctions, alcoholism, and low ejection fraction on echo
2-long HX of HTN and ejection fraction of 70
3-hemochromatosis, sarcoid, amyloidosis hx.


1-dilated cardiomyopathy presents with a HX of multiple infarctions or alcholism. The low ejection fraction and systolic dysfunction are key to the diagnosis. All forms of cardiomyopathy lead to SOB, dyspnea on exertion, edema, and orthopnea.

2-Hypertrophic cardiomyopathy retains normal or hyperdynamic ejection fraction. Long standing HTN leads to impaired systolic dysfunction.

3-restrictive cardiomyopathy is by far the least common cause of CHF, sarcoidosis, amylodosis and hemochromatosis are the key diagnosis...

________________________________________________

A patient comes with a long HX of SOB, edema, Ascites, and hepatosplenomegaly. The pt. is an Immigrant. There is a rise in JVD distention with inhalation. There is a third heart sound

1- what is the most likely diagnosis
2-what is the most accurate diagnosis
3-Most effective RX

1- constrictive pericarditis presents with SOB and edema. This is nonspecific. The key to answering the diagnostic question is the presence of Kussmaul's sign, which is a rise in Jugulovenous pressure on inhalation. Constrictive pericarditis is most often a form of TB. which causes chronic inflammation of the pericardium. The third heart sound is the pericardial 'Knock'
2- Most accurate test is a CT or MRI scan of the chest.
3-the only effective therapy for constrictive pericarditis is surgical removal of the pericardium. The presenting sx of constrictive pericarditis can mimic those of restrictive cardiomyopathy. One of the distinguishing characteristics is the presence of equalization of the left and right sides heart pressure in constrictive pericarditis.

__________________________________________________ ______

moving on to ER medicine

pt brought in after having lost consciousness and is now awake

1- he has no focal neurological deficits and the CT scan is normal.
2- he has weakness of the arm and CT scan shows a collection of blood and one dilated pupil
3- his CT scan shows ecchymosis

1- concussion is caused by head trauma. There can be loss of consciousness or altered mental status. There is no anatomic damage to the brain.
The CT scan is normal.
2-Subdural and epidural hematoma both lead to a collection of blood around the brain, visible on head CT. There can be focal neurological defects and a dilated pupil on one side.

3-Ecchymosis of the brain results from head trauma. This is also called a 'contusion' Most of the time there are no focal neurological deficits. No surgery is required. Blood mixed in with brain. an ecchymosis is visible on brain CT and is, essentially, a bruise. No surgery is needed.

______________________________________________

A patient has a sudden onset of high fever and a change in mental status. All cultures are negative and there is no neck stiffness. His CPK is elevated.

1- he has recently been started on risperidone in addition to haloperidol
2-he has just undergone major surgery
3-He is outside playing volleyball in the summer

1-Neuroleptic malignant syndrome presents with a high em and altered mental status in relation to starting neuroleptic medications. This is probably related to the antidopaminergic effects of these meds. CPK and potassium elevation can occur. treatment is with dantrolene and dopamine agonist medications such as bromocriptine.

2-Neuroleptic melignant syndrome is caused by general anesthetics or succinylcholine. CPK elevations can also occur. Treatment is with dantrolene
3- Heat stroke occurs in relation to dehydration and increased ambiebt temp. There is high fever and confusion. RX is with hydration, and physical measures to cool the pt. Aersolizing water and evaporation is the most precise method of cooling the patient.

____________________________________________

Endocrinology

a female child is brought in because of abnormal hair growth, she has not developed menstruation and has acne, hirsutism of her face, and abnromal balding.

1-Hypotension, hyperkalemia, hyponatremia and elevated levels of 17-hyroxyprogesterone with diminished 11-deoxycortisone
2- hypertension, hypokalemia and metabolic alkalosis. levels of 11-deoxycortisone are elevated

1- 21-hydroxylase deficiency presents with hypotension, hyperkalemia and metabolic acidosis because of the loss of sufficient mineralocorticoid activity. Both aldosterone and 11-deoxycortisone levels decreased. Adrenal hormones are shunted into the excess production of DHEA, which accounts for ambiguous genitalia in females, such as clitromegaly. in addition, there is acne and hirsuitims. All forms of CAH have elevated ACTH and low levels of cortisol. 17- hydroxyprogesterone levels are increased because this is the precursor that should be converted by 21 hydroxylase.


11-hydroxylase deficiency presents with HTN and hypokalmia so exact opposite of 21-hydroxylase deficiency.. because of increased level of 11-deoxycortisone (11DOC). 11 DOC has mineralcorticoid activity, which accounts for the HTN and metabolic alkalosis . Adrenal hormones end up shunted into the production of adrenal andrgens such as DHEA, Conn's syndrome would present similar but with elevated levels of aldosterone and decreased Renin.

__________________________________________________ _______

A man comes to ER with weakness, and orthostatic hypotension, he has hyperpigmented skin lesions, hyponatremia, hyperkalemia and metabolic acidosis. Dark lines visible on gums and teeth.

1-Most likely DX
2-Next step in management
3-most accurate diagnostic test

Addison's disease. and addisonian crisis are the loss of alodosterone from the adrenal gland resulting in loss of sodium and water and the development of hypotension, in addition, hypoaldosteronism results in hyponatremia, hyperkalemia and metabolic acidosis.
Hyperpigmentation results from the high ACTH levels and high propiomelamocortin. Which also gives the dark lines in the gums.

2- the most urgent step is to draw a cortisol level and administer saline and hydrocortisone! A specific mineralocorticoid such as fludrocortisone is often necessary. Acute treatment is more important than waiting for the results of specific endocrine diagnostic tests because of the risk of death from hemodynamic compromise.

3- Cosyntropin stimulation testing is the most specific test. This is the measurement of cortisol levels before administering artificial ACTH. In a normal pt. the cortisol level witll rise with cosyntropin

____________________________________________

a young man being evaluated for HTN. he has episodes of headaches, palpitations, tachycardia, and sweating along with HTN

1-DX
2-Diagnostic testing

pheochromocytoma presents with episodes of HTN, palpitations, tachycardia and headache. The clue to the dx is the episodic nature of the HTN. the other sx are nonspecific.
The best initial test is to obtain blood levels of free metanephrines. This is the more sensitive than levels of epinephrine and norepinephrine because the catecholamines are secreted in an episodic fashion and have a short half life. a 24 hour urine for catecholamines and metanephrines is highly sensitive and specific. as well a CT scan or MRI scanning of the adrenal glands if the catecholamine levels are elevated in order to localize the tumor.

__________________________________________________ _____

a patient comes in with muscular weakness, polyuria, and polydipsia. There is a metabolic alkalosis and the potassium level is profoundly low at 2.5 mEq/L

1-HTN with low renin activity without edema.
2-High renin and high aldosterone activity with an elevated level of urinary sodium until the body is depleted of sodium. urinary ca2+ is high. Normal BP.
3- pt has a box of licorice in his hands. The renin level is low. BP is high.

primary hyperaldosteronism, or Conn's syndrome, presents with HTN, hypokalemia and metabolic alkalosis. The plasma renin activity is suppressed because of HTN. high aldosterone levels with low renin is a hallmark of primary hyperaldosteronism. The pts muscular weakness is from low potassium. The polyuria is nephrogenic diabetes insipidus from hypokalemia.

2- Bartter's syndrome


Both patients have the typical facial appearance with prominent forehead, triangular face, drooping mouth, and large eyes and pinnae.

this is a form of genetic defect in the loop of henle. pts lose sodium, chloride, and ca2+ resulting in volume depletion and secondary elevation of renin and aldosterone levels, with normal or low BP.

3- Licorice contains a substance that is similar in function to aldosterone. Locorice ingestion will present in an identical fashion to primary hyperaldosteronism. Anything that gives a low potassium leads to muscular weakness. BP is high.

________________________________________________

A man comes in with a long history of episodic flushing of his head and neck. the flushing is associated with strong emotions and the use of alcohol. He is hypotensive and tahycardic with the episodes. He has abdominal cramping and diarrhea, on PE, there are telangiectasia and the murmurs of tricuspid insufficiency and pulmonic stenosis.

1-DX
2-Best diagnostic

carcinoid syndrome most often presents with episodes of cuteanous flushing in association with diarrhea nd tachycardia and abdominal cramping. Hypotension and tachycardia occur with the episodes. The recurrent episodes of flushing lead to vascular telengiectasia, Long standing disease is associated with right sided cardiac lesions from chronic exposure to sertonin. some pts have wheezing.

2- best initial is urinary hydroxyindoleacetic acid (5-HIAA) levels. The tumors are localized in the intestinal tract with abdominal computed tomography (CT) and pentetreotide imaging (indium 111 octreotide imaging)



______________________________________

a woman comes in for an offensive body odor and excess sweating, but she is unable to explain or resolve it.
Her ring, hat, and shoe sizes have been increasing in the last few yrs. Her voice is thick and jaw protruding and enlarged. she also has joint pain

1-what is the most likely DX
2-Best initial test
3-Most common cause of death

Acromegaly is most often from a pituitary tumor secreting growth hormone. This leads to enlargement of hat, show, ring and glove sizes. The pt is in 30's and 40's. Arthropathy occurs from excessive articular cartilage proliferation. Entrapment neuropathies such as carpal tunnel syndrome can also occur. Diabetes occurs in 10-20% of pts. Amenorrhea can result from excess secretion of prolactin.
2- best initial test is a level of Insulin like growth factor-1 (IGF-1) this is confirmed by findings of of a failure or GH suppression by the infusion of glucose.
3-The most common cause of death is from the effects of GH on the heart, and HTN. There is also an increased risk of colonic polyps and cancer as well.

___________________________________________

a woman comes in because of infrequent periods, Her menstrual abnormalities have been going on for several months. on PE she has galactorrhea. Her urine HCG is normal

1-Dx
2-Diagnostics

Galactorrhea is an abnormally increased flow of milk from the breasts, It is caused by hyerprolactinemia. Medications such as alpha-methyladopa, tricyclic antidepressants or phnothiazines and beta blockers can cause it. It can occur normally from pregnancy and the first step is always a pregnancy test. head trauma can rupture he pituitary stalk and remove the normally inhibitory dopamine that comes down from the hypothalmus. if these levels have been excluded, MRI of the brain may show a pituitary tumor, prolactin inhibits the release of LH, and inhibits menstruation.
2- measure the prolactin level. If it is markedly elevated in the absence of pregnancy, then an MRI is the most accurate test to detect a pituitary lesion.

___________________________________________

a patient comes in with anxiety, unexplained weight loss, diarrhea, tachycardia, and palpitations. PE shows, tremor, thin hair, moist skin. The thyroxine level (T4) is elevated

1- exophthalmos, skin abnormalities above the knee, and proptosis.
2-An elevated thyroid stimulating hormone (TSH) level
3-A tender gland
4-Normal appearing gland, low TSH, low radioactive iodine uptake.

Graves disease is charcterized by ocular and skin findings. The radioactive iodine scan reveals hyperfunctioning gland. The TSH is low, treatment is with methimazole or propylthiouracil (PTU) followed by radioactive ablation and hormone replacement if hypothyroid.
2- TSH producing pituitary tumors are the only form of hypothyroidism associated with an elevated level of TSH. Perform and MRI of the brain to confirm dx.
3-subacute thyroiditis is associated with a tender gland. The TSH level will be suppressed and the radioactive iodine uptake (RAIU) will be diminished
4-silent thyroiditis is associated with normal appearing gland, low TSH, and low RAIU. The gland is nontender.
_____________________________________________

A young woman comes in because of failure to undergo menarche. She has normal breast development, but a paucity of pubic hair, vagina ends in a blind pouch and cervix is absent.

dx
rx

testicular feminization or complete androgen insensitivity often comes to light when there is failure to achieve menses. The pt appears female, normal breast development, but there is a marked diminishement in pubic and axillary hair. the vagina is short, and the cervix, uterus, and ovaries are absent. testicles can be found in the abdomen or labia
surgical removal of the gonad with estrogen replacement and dilation of the vagina in the management. these pts are emotionally and socially and functionally female (think jaime lee curtis)

__________________________________
on to GI

72 year old man comes to ER with a sudden onset of severe abdominal pain, he has a HX of severe mid abdominal pain. He has a HX of aortic stenosis, and a fib. he has been losing weight. his abdominal exam is relatively benign compared to his sx, his stool is heme positive,

dx, accurate diagnostic, effective therapy


mesenteric ischemia presents with severe abdominal pain that is far more intense than the relatively benign exam, It is mid-abdominal. it often occurs in association with valvular heart disease. CAD, and Afib. Mesenteric ischemia is often an acute embolic event to the mesenteric artery.

mesenteric arteriography is the most accurate diagnostic test.
treatment is by exploratory laprotomy for possible resection of the affected segment of bowel. pt with signs of peritonits should go directly for laparotomy. if infarction occurs death is highly probable.

______________________________________________

a man is brought to the ER with multiple episodes of vomiting blood, he also has diarrhea and black stool

1- bleeding preceded with severe violent retching
2- mid epigastric pain relieved by food
3- he is an alcoholic with low platelets ad spider angiomata. The volume of hematemesis is enormous

1- Mallory weiss tears are non-transmural tears in the esophageal mucosa. This is preceded by repeated episodes of retching or vomiting for any reason. Any form of upper GI bleeding can result in melena f more than 100-200 ml of blood is lost.
2-duodenal ulcer is the most common cause of upper GI bleeding
duodenal ulcers present with epigastric pain. the pain can be relieved by food. Endoscopy is necessary for specific diagnosis.
3-Esophageal varices leads to the highest mortality of any form of GI bleeding. The case will describe severe liver disease, severe cirrhosis is often associated by splenomegaly with splenic sequestration of platelets.

_____________________________________________

now pharm
______
still on endocrine
Sulfonylureas

Glyburide
Glipizide
Gliclazide
Glimpiride
Sulfonylureas are sulfa derived oral hypoglycemic medications used for DM
they lower blood glucose by simulating pancreatic beta cells to release insulin. They bind ATP-dependent potassium channels on the beta cell, which inhibits the outflux of potassium. This depolarization opens voltage gated ca2+ channels leading to increased fusion of insulin with cell membrane. unlike metformin sulfonylureas require functioning beta cells.

all sulfonylureas can cause hypoglycemia and weight gain. The increased insulin release prompts adipocytes to pick up more glucose. They are teratogenic and can cause SIADH . They are sulfa derivatives and are contraindicated in sulfa allergies.

Sulfonylureas are first line oral hypoglycemics. metformin us preferred in obese pts.

__________________________________________________ _
1 63 year old woman with DM is seen for routine eval. Her urine shows microalbuminuria. Her dilated nose exam shows background retinopathy as well as some neovascurization

what will protect her kidney, and eyes

microalbuminurea in a diabetic should be treated with an ACE inhibitor
Diabetic retinopathy can be controlled with tight control of her glucose level. Laser photocoagulation is indicated for proliferative retinopathy. defined as the presence of neovascularization of vitreal hemorrhages. Diabetic retinopathy is not controlled with or lipid control. Asa will not control diabetic retinopathy

________________________________

a man with diabetes has glucose levels > 200 md/dl despite the use of maximal doses of several oral hypoglycemics agents. His Hemoglobin A1c is >9 %. in addition, his glucose levels fluctuate during the day

Insulin glargine should be given as a once a day injection combined with one of the rapidly acting insulins (glulisine, aspart, or lispro) with meals

insulin glargine rapidly reaches a peak level and maintains a constant level for 24hrs. Glargine is similar in effect to an insulin infusion pump. the short acting insuling (glulisine, aspart and lispro reach a peak effect in 50-60 mins and last for four hours, they seem to be equal in efficacy.

______________________________________

ok now in GI pharm

Interferon
interferon alpha is used to rx the following
chronic hep C in combo with ribavirin
chronic hep c
melanoma
cryolgobilinemia in combo with ribavirin
multiple sclerosis
the mechanism of interferons are not clear, they are a cytokine that assists the immune response by inhibiting viral replication within the cells. they are immune modulatory and benefit multiple sclerosis
interferons cause flu like sx, such as myalgia, arthralgia, depression, thrombocytopenia and leukopenia.
_________________________________________________

Ribavirin
is a purine nucloside that is used as an antiviral
rivavirin inhibits viral mRNA synthesis. it competitively inhibits cellular inosine 5'phosphate dehydrogense and interferes with the synthesis of guanosine triphosphate (GTP) and thus nucleic acid synthesis in general. it inhibits both RNA and DNA viruses.

Ribavirin is the answer when the question describes a case of chronic hep c. the hepatitis C antibody should be positive with an elevated PCR RNA viral load and possible liver inflammation.. It is used in combo with interferon. Ribavirin is also the answer for respiratory syncytial virus.
ribavirin can cause anemia

_____________________________________

diagnostics

Rheumatology

C-Anca (antineutrophil cytoplasmic antibody)

is the best initial test to make a specific diagnosis of wegener's granulomatosis.
C-ANCA is a blood test. A pattern is seen on indirect immunofluorescence microscopy that indicated antibodies directed against proteinase 3 of neutrophils.
answer C-ANCA when you see upper and lower respiratory probs combined with renal dx, as hematuria and red cell casts in the urine. there will be hemoptysis, cough and sputum with abnormal chest xray not responsive to antibiotics. otitis and sinusitis will also be present.

the most accurate test for wegner's granulmatosis is a biopsy of the lung and kidney
_____________________________________________

DEXA scanning
is used to dx osteoporosis. it is used to detect bone density, when compared to general population
this is used on post meopausal women and pts with high cortisol like cushing's syndrome, exogenous steroids, hyperthyroidism or with hyperparathyroidism.
rx is with estrogens, bisphosphantes and SERM such as raloxifen and tamoxifen.

__________________________________________________ __

muscle biopsy
is used to dx suspected myopathy such as
glycogen storage disease





mitochondrial gene malformation


dystrophinopathies


dermatomyositis


Gottron's papule
and


heliotrope eyelids

and polymyositis


lymphocyte in myofibber
.. this is done when you see proximal muscle weakness, weak shoulder abduction, weakness rising from a chair, this is usually associated with myalgia and muscle soreness. other signs of myopathy is recurrent myolysis.

it isn't the answe for endocrine disoders like mysthenia.

_________________________________

RF factor
present in 70% of pts with RA. RF isn't specific for RA
.. do this test with a woman with symmetric inflammatory arthritis that s worse in the morning. affecting wrists MCP, PIP and look for elevated ESR or synovial fluid between 2000 and 20,000 cells.

_____________________________




this is an upright xray of the chest. pt is sitting up so it can show air under the diaphragm

perforated abdominal organ x ray, can be associated with penetrating trauma, rupture secondary to diverticulitis, appendicitis, other infection or iatrogenically from endoscopy
>>>>>>>>>>>>>>>>>>>>>>>>>>>>>>>>>>>>>>>>>>>>>>
Reply

جوري
10-03-2008, 11:48 PM
well, I had a bad day wal7mdlilah...
did you have a bad day?

will spare you nuances of mine if you spare me yours :)

questions on with GI
_____________
Man brought into ER with multiple episodes of red blood in his stool

Low GI bleeding is most commonly caused by diverticulosis and angiodysplasia. Other caused are polyps, colon ca. and ischemic colitis

colonoscopy is the most accurate diagnostic test of lower GI bleeding. There is no definitive way to determine the precise etiology of colonic bleeding without endoscopy, barium studies, angiography, and CT scanning cannot lead to specific diagnosis.

Hemorrhoids may also lead to red blood in stool. Often, the history will mention that the patient notes hematochezia and/or red blood on wiping.

___________________________________

32 year old with one day of diarrhea, no blood per still
1- there is vomiting, he recently ate chinese food
2-he has recently been camping. He has bloating and flatulnce
3-He is HIV positive with <CD4 cells.
4-There is flushing and wheezing. He ate fresh fish the same day.

1- Bacillus cereus, is associated with refried chinese rice. As with staphylococcus aureus there is no blood per stoll because it is a p preformed toxin. both organisms often present with vomiting

2-Giardiaisis is associated with unfiltered water, such as found on camping trips. Bloating and flatus are common, Giardiasis can mimic fat malabsorption.
3- Cryptosporidiosis is an organism that is common in those with AIDS and profound immunosuppression. The diarrhea is often chronic and responds to treatment of the underlying HIV disease.
4-Scombroid is histamine fish poisoning. Bacteria that produce histamine infect tuna, Mackerel or mahi-mahi. resulting in the rapid onset of diarrhea, vomiting, flushing and wheezing.

____________________________________________

35 year old woman comes to the office with several months of crampy lower abdominal pain. but there is never blood in the stool or weight loss.
1- the diarrhea alternates with constipation. The pain is relieved with a bowel movement. Symptoms are less at night.
2-she has episodes of flushing and hypotension
3-dietary change relieves all the symptoms within 24 hours.

1-Irritable bowel syndrome is a pain syndrome that is often associated with diarrhea alternating with constipation. sx are less at night. Key is abdominal pain with completely normal tests.
2-carcinoid syndrome presents with episodes of flushing and diarrhea, hypotension. Urinary 5-HIAA confirms the diagnosis.
3-Lactose intolerance presents with diarrhea in the absence of weight loss. Removal of milk products and cheese relieves the sx. Celiac disease would lead to weight loss that would need several weeks for sx to resolve. celiac disease would also be related to gluten containing products.
_____________________________________________

A generally healthy 40 year old teacher comes to the ER with several days of bloody diarrhea which occurred four times today. He has a temp of 102F pulse 105, and BP 112/78
1-has has been eating raw oysters and clams
2-he has mussels and has a hx of liver disease. Physical exam shows bullous skin lesions
3-anemia and thrombocytopenia, and an elevated creatinine are present. The retic count, bilirubin and LDH are elevated, haptoglobin is low

1- vibrio parahemolyticus is transmissited by shellfish. such as oysters, clams. Shellfish filter feeders that concentrate microorganisms as they feed themselves.
2-Vibrio vulnificus is associated with diarrhea in pts with liver disease who consume contaminated shell fish. there is also incidence of developing bullous skin lesions.
3-E.coli 0157:H7 is associated with the development of hemolytic uremic syndrome.

________________________________________________

a patient comes in for epigastric discomfort that radiates up into the chest. He has cough and hoarseness and bad taste in his mouth, like he is sucking on pennies.

Most likely DX?
Most accurate test
management?

GERD leads to epigastric pain that radiates under the sternum. In addition, the acid hits the back of the tongue, leading to bitter taste in the mouth, when acid hits the vocal cords there is hoarseness and sometimes coughing and wheezing.

the most accurate test is a 24hour PH monitor
The first therapy is a proton pump inhibitor (PPI) is both diagnostic and therapeutic.

______________________________________________

A man is evaluated for several weeks of epigastric discomfort and pain

1- He is an alcoholic and there is epigastric tenderness
2-he has no other sx. All lab tests are normal
3-He has two episodes of black stool, the pain is better with food

1-Pancreatitis is the only form of acute epigastric pains that is reliably associated with tenderness. Gastritis and ulcer disease are rarely associated with epigastric tenderness unless a perforation has occured
2-Non-ulcer dysphagia is the most common cause of epigastric discomfort. There is epigastric pain with an entirely normal examination including a normal endoscopy and the etiology is unknown.
3-Ulcer disease is the most common cause of upper GI bleding. Ulcers are not as common as non-ulcer dyspepsia as a cause of epigastric pain.
duodenal ulcers are more often improved with eating. Gastric ulcers are worsened with eating. An upper EGD can be diagnostic.

_____________________________________________

a 22 year old woman comes to the office with recurrent episodes of diarrhea, fatigue, and abdominal pain. There is occasional blood in addition she has joint pain, erythema nodosum and uveitis. The hematocrit is 32, MCV is 90 and ESR is elevated. Alkaline phosphatase is elevated but the bilirubin is normal. Stool culture and ova/parasite exam show nothing.
1-rectal bleeding is common. Antineutrophil cytoplasmic antibodies ANCA are present and antisaccharomyces cerevisiae (ASCA) are negative.
2-perianal and small bowel disease is present. A fistula was present in the past. Granulomas are present on biopsy. ANCA is negative ASCA is positive. A mass is palpable in the abdomen.

Ulcerative colitis with recurrent episodes of bloody diarrhea and pus from the rectum. the extra-intestinal manifestations of both forms of IBD are identical. Both give joint, skin, and ocular sx. Both give sclerosing cholangitis. UC gives positive ANCA and negative ASCA.

Crohn's disease gives small bowel disease, fistulae, and perianal disease, in addition to 'skip' lesions. Granulomas are characteristic of CD. CD gives a negative ANCA and positive ASCA. Anemia, low albumin levels and a high sedimentation rate can be found in both disease. Crohn's transmural inflammation whereas UC is limited to mucosa.
________________________________________

A woman is evaluated in the office for moderate hepatomegaly and elevation of the AST, ALT and bilirubin. A few spider nevi are present on skin.

1- The antinuclear antibody (ANA) and anti-smooth muscle antibody are positive. Gammaglobulins are also elevated and there is a brisk response to prednisone
2-Hepatomegaly is the main finding in a woman with diabetes, obesity and hypertriglyceridemia. the ALT is slightly higher than the AST. Fatty liver is seen on imaging. She doesn't drink alcohol.

1-autoimmune hepatitis presents with hepatomegaly and the stigmata of chronic liver disease. The ANA is often positive and the gammaglobulin levels are elevated. less reliable findings are the presence of antismooth muscle antibodies and the liver-kidney microsomal antibody. Autoimmune hepatitis responds briskly to prednisone use.

Non-alcoholic steatohepatitis (NASH) is associated with obesity, diabetes, hyperlipidemia. The liver biopsy shows fatty infiltration you would see in a pt with alcoholic use. NASH is associated with an ALT slightly greater than AST. this is opposite in a person with alcoholic liver disease. There is no definitive treatment for NASH besides losing weight and controlling the diabetes and hyerlipidemia .

_______________________________

a 38 year old man comes in with weight loss and flatulence, diarrhea, malodorous stool and weakness, he bruises easily, and his ca2+ is low. Sudan black stain is positive

1- iron deficiency is present. Folate level is low. He has a skin rash with vesicles. Anti-gliadin and tissue transglutaminase antibodies are positive.
2-Chronic alcoholic with epigastric pain and normal folate and iron levels.
calcification of the pancreas on CT scanning. Lipase and amylase level are normal

celiac disease and chronic pancreatitis both present with steatorrhea and weight loss. Both diseases lead to malabsorption of fat associated with the loss of ca2+ and vitamin K, easy bruising and malabsorption of vit B12. Only celiac disease leads to malabsorption of iron and folate. Iron and folate need an intact bowel to be absorbed. but don't need pancreatic enzymes to be absorbed. The most accurate test for celiac disease is small bowel biopsy ( :D do you see a theme here?.. I might be partial, but a pathologist is the arbiter of medicine) with them lie true diagnosis!

Chronic pancreatitis is most often from alcohol. The iron and folate levels are normal. Lipase and amylase levels are normal in most patients with far advanced chronic pancreatitis. Calcifications are present on CT of the pancreas in 70-80% of pts . The most accurate diagnostic test is a secretin stimulation test. Secretin should provoke the release of bicarbonate -rich pancreatic enzymes in a normal person.

_______________________________________________

Man presents with dysphagia and weight loss
1-pt is<50 with dyshagia for both solids and liquids at the same time
2-65 year old man with a clong hx of ETOH and tobacco use. Dysphagia begins with solid and progresses with difficulty for liqueds.
3-Foul breath and regurgitated food on the pillow the following morning
4-HX of scleroderma with reflux sx
5-CP that comes and goes. it is very severe but not associated with eating

1- Achalasia is associated with dysphagia for both solids and liquids at the same time. it is not progressive. There is no association with smoking or drinking.
2-Esophageal ca. is dysphgia first for solid food, then liquids. ca gets progressively worse, Achalasia is not.
3-Zencker's diverticulitum is associated with foul smelling breath. Do not use nasogastric tube or endoscope because of the risk of perforation.
4-Scleroderma esophgitis leads to reflux disease because the esophagus is ot capable of contracting. The answer is most likely DX question is easy scleroderma + esophagitis = scleroderma esophagitis. give proton pump inhibitors. look for CREST syndrome.
5-spastic disorders of the esophagus present with pain not related to eating or exertion. to answer the question, it must include a negative EKG, and stress test so as to not be classified as angina. Esophageal manometry can be diagnostic.
_______________________________

A man comes in or evaluation of weight loss, diarrhea and easy bruising, the ca2+ level is low and the sudan black stain is positive.

1- He has arthralgia, fever, and cognitive defects. There are ocular abnormalities such as nystagmus. Adenopathy is present. Biopsy of the duodenum shows PAS-positive organism
2- a patient from the Caribbean has severe folate deficiency and VIt B12 deficiency. Biopsy shows abnromal villi with lymphocytic infiltration. Antigliadin and anti-endomysial antibodies are negative.

Whipple's disease is a cause of malabsorption with arthraligias, fever and CNS abnormalities. Presence of PAS positive organisms, best initial therapy is a year of trimethoprim/sulfamethoxazole

2- tropical sprue is the answer when malabsorption in association with severe folate and B12 malabsorption. The Q must give a hx of someone from the Caribbean or India. On biopsy the villi are abnromal with inflammatory cells but they are not as flat as those seen in celiac disease. treatment is with tetracycline and folate.
__________________________________________________ _________

An alcoholic man is admitted with severe epigastric pain, abdominal pain, nausea, and vomiting, he is restless with mild fever.
1-Pt has an elevation of his amylase and lipase levels, as well as urinary trypsinogen activation peptide. CT scan shows inflammation.
2-CT scan shows necrosis of >30%
3-CT shows necrosis and biopsy grows gram negative organisms

1-Acute pancreatitis occurs in alcoholics, and those with gallstones obstructing the ducts. Epigastric pain, nausea, and vomiting are present. the key dx is epigastric tenderness in an alcoholic. Trypsinogen activating peptide is elevated. treatment is pain control, IV fluids, and NPO until pain is resolved.
2-Necrotizing pancreatitis on a CT scan of the abdomen is much more important as a prognostic factor than Ranson's criteria.. pts with severe necrosis should undergo biopsy to see is an infection is present. Necrotizing pancreatitis may benefit from antibiotics such as imipenem to prevent infection
3-onfected necrotizing pancreatitis can be diagnosed only by biopsy or surgery. these pts have nearly 10% mortality without surgical debridement.

__________________________________________________

a woman comes in with severe itching, hepatomegaly and elevation of alk phos and GGTP.

1-HX of IBD. overtime the bilirubin begins to elevate
2-middle aged woman with xanthomas, fat soluble vitamin malabsorption, hyperlipidemia and skin hyperpigmentation.

1-primary sclerosing cholangitis occurs in those with IBD. The alk phos. is elevated and the bilirubin only elevates much later in the disease. RX with ursodeoxycholic acid, but it is of limited effect
2- primary billiary cirrhosis occurs in middle aged women who present with itching elevated alk phos. The most most accurate test is the anti-mitochondrial anti-vody. RX is with ursodeoxycholic acid, but it is of limited benefit.
_____________________________
a young man is referred by psych for eval of tremor, and choreiform movement disorder. He was admitted for paranoia and psychosis, but was found to have an elevation of his transaminases and a coombs-negative hemolytic anemia.
dx, diagnostic test. therapy

Wilson's disease is a form of copper deposition in the bran, liver and kidneys. In addition, there is coombs negative hemolytic anemia. look for liver disease with a movement disorder and psychosis.
Wilson's disease is diagnosed by finding kayser fleisher rings on slit lamp exam. as well a low level of ceruloplasmin, which is the copper carrying protein in the body. There is increased urinary copper excretion, although the single most accurate test is an increased copper level on biopsy.
penciliallmine is the treatment that removes copper from the body.
__________________________________________________ ____

Middle aged man comes in for evaluation of joint pains and fatigue. He has hepatomegaly on examination and skin hyperpigmentation. diabetes has developed over the past few months. He has lost libido and has developed ED. LFT's are elevated. Echo shows restrictive cardiomyopathy.
dx.diagnostic. therapy

hemochromatosis is a form of iron deposition in multiple organs in the body, especially the liver. Cirrhosis will develop if untreated in 60% of pts with hepatocellular ca. in 15-30%. Another 15% will die of cardiac involvement. iron depostion also leads to diabetes, pseudogout, skin hyperpigmentation (bronze diabetes) and erectile dysfunction. the latter is a form of iron deposition in the pituitary and the gonadotropins.
the best initial tests is iron studies, with an elevated iron and ferritin level and low iron binding. This is a high iron saturation. This prompts the most accurate tests, which are the HFE gene mutation. liver biopsy with increased iron is the is single most accurate test.
Phlebotomy is the most accurate way to remove iron from the body.

_____________________ ok on with diagnostics

ER medicine



this is a skeletal survey
used in suspicion of child abuse, multiple myeloma and metastatic bone tumor.

parents might say he fell, but you see bruises.. also a clavicle is a hard bone to break unless by physical abuse..
_____________________________



voiding cystourethrography
is a radiological visualization of bladder filling and emptying through the urethra. not to be confused with cystometrics, which is a pressure measurement of the bladder, useful in evaluating incontinence. cystourethrography is an evaluation of the anatomy of the urogenital system

for trauma
neoplasia in and surrounding the bladder
UTI (in children)
akk children younger than 5 yrs old with UTI
children of any age with febrile UTI
boys of any age with UTI
__________________________________________________ ______

diagnostic peritoneal lavage

look for drunk or unconscious person who can't have his abdomen examined reliably with multiple trauma and unexplained shock. a catheter is inserted into the abdomen and saline infuses, aspirate is examined.
the following aren't appropriate for a DPL.
pts with signs of acute abdomen, an indication of a perforated viscus or intrabdominal bleed is an indication for immediate surgical exploration.
pts with gunshot wounds to the abdomen. is indication for immediate surgical exploration
hemodynamically stable pt with hx of abdominal trauma but no definite signs of viscus perforation on PE. abdominal CT is indicated here.

to be considered DPL positive DPL should show RBC count >100,000/mm3, WBC>500/mm3, bile or feces.
DPL is inadequare to dx retroperitoneal injuries .
______________________


intravenous pyelogram (IVP) is an old test of virtually no clinical utility. was used to dx pts with renal colic when no stones were found on radiograph.

IVP has been replaced with a US as the best initial, and CT as most accurate. IVP uses contrast which is a risk for both renal failure and allergic rxn.
________________________

pelvic laparoscopy
direct visualization of pelvic structures, most accurate for

ectopic pregnancy
PID
endometriosis
pelvic cyst
chronic pelvic pain.

for pt who presents with pevic pain vaginal bleeding, positive beta hCG, particularly if beta hCG is low for gestational age.
also in PID where pregnancy is excluded.
___________________________________
Pharm on GI
43 yo with chronic hep B in the clinic evaluated for rx. her surface antigen has remained (+) for more than 6m. her hep HBe antigen is (+) as well.

chronic hep B is defines as persistence of surface antigen for longer than 6 month. Entecavir, lamivudine, telbivudine, adefovir and interferon are all approved for use. therapy is most effective for those who are (+) for hep B e antigen (HbeAg) or hepatitis B DNA polymerase.

Enecavir, lamivudine, adefovir, and telbivudine all inhibit viral reverse transcriptase (DNA) polymerase. They cause DNA chain termination.

Entecavir, lamivudine, adefovir, and telbivudine all cause lactic aciddiosis, and can cause acute hep exacerbation. Adefovir is neprotoxic. interferon causes flu like sx, myalgias, arthralgia and headache.

___________________________________________

40 year old woman w persistent epigastric pain, has large gastric ulcer, there is no malignancy on biopsy. but (+) for H-pylori

initial therapy for H pylori is PPI, such as lansoprazole, omeprazole, pantoprazole, rabeprazole or esmeprazole combined with two antibiotics, the preferred are clarithromycin and amoxicillin.

if initial therapy fails, rx with PPI combined with bismuth subsalicylate and two different antibitics tetracycline and metronidazole.
___________________________________________

56 year old man with acute variceal hemorrhage from alcoholic cirrhosis comes to the er, he has been started on IV fluids, he received a blood transfusion and ffp

1-octreotide (somatostatin) is used for variceal bleeding to dec the severity of the hemorrhage
octreotide is a synthetic somatostatin, it decreases portal pressure and splanchnic blood flow, because there are numerous somatostains receptors throughout the portal circulation. it is superior to vasopressin or sclerotherapy. Propranolol would decreases frequency of recurrent bleeding but not has no effect on acute bleeding.
no sign adverse of octreotide
if octreotide doesn't work, endoscopic band ligation is the next best step. transfuse as needed, however massive transfusion can cause coagulopathy.

somatostatin is used to control growth hormone release in pts with acromegaly who are not candidates for surgery. it is also used to contrl dirrhea and carcinoid syndrome. it helps treat glucagonoma.

_______________________________
48 year old with altered MS secondary to hepatic insufficiency

Lactulose is the most effective theray for hepatic enecphalopathy. it can also be used to RX constipation. it is superior to neomycin as the best initial therapy for hepatic encephalopathy
lactulose is a nonabsorbed disaccharide. Bacteria is the colon consume the lactulose and lower the ph of the bowel. the acidification changes ammonia (NH3) to ammonium (NH4+) ammonium isn't easily absorbed and more readily excreted.
lactulose causes bloating, diarrhea and flatulance. it can result in hypernatremia and hypokalemia
__________________________________________________ ___

pencillamine is a PCN derived chelating agent used for a number of toxicities

pencillamine chelates, copper, mercury, zinc, and lead, it also decreases T cell activity and rheumatoid factor.

it can cause nephrotic syndrome and bone marrow suppression.
pencillamine is the drug of choice for wilson's disease, sometimes used for lead poisoning and severe RA. not responding to other therapies. It is used to rx cystunuria, can also be used to rx arsenic and mercury poisoning.
>>>>>>>>>>>>>>>>>>>>>>>>>>>>>>>>>>>>>>>>>>>>>>>> >> >>>>
Reply

جوري
10-05-2008, 01:58 AM
:sl:
today it will be 10/5/5 in lieu of 15/5/5.. I find this a bit time consuming, more so than I'd like it to be, and I still think we can be up to speed at some point

ok still on GI in most likely dx
________________________________________

A man comes to the ER with abdominal pain, tenderness and fever..

1- hx of alcoholic cirrhosis and ascites. BP and pulse are normal
2-HX of peptic ulcer dz. He has a BP of 86/60 and a pulse of 120, and there is rebound tenderness on PE.

1- spontaneous bacterial peritonitis (SBP) occurs with ascites. The Dx is based on ascitic fluid cell count of >250 neutrophils. Culture of the fluid should be injected into blood culture bottles. Most commonly SBP is from a single organism, such as Ecoli. The Ascitic fluid protein level is low. RX is with CEFOTAXIME. Ascitic fluid should be sent for gram stain, culture protein, albumin, CDH, amylase, and cell count
2- secondary peritonitis occurs from perforation of a viscus, is associated with signs of severe sepsis such as hypotension and tachycardia. peritoneal signs such as rebound or guarding are common. The ascitic fluid protein is elevated. This form of peritonitis must be treated surgically in addition to antibiotics. Look for air and the diphragm on an upright cxray.

__________________________________________________

A man comes for evaluation of recurrent peptic ulcers. The ulcers are multiple, >2CM in size and located in the distal portion of the duodenum. RX of H pylori has resulted in no benefit. he also has diarrhea..
DX, Diagnostic test, and best therapy


Zollinger Ellison Syndrome (ZES) is the most likely DX when the question describes a pt with ulcers that are large, distal, multiple and recurrent after RX for H pylori. Most ulcers are , 1cm in size. Diarrhea is from the inactivation of lipase from the high acid level.
Most accurate diagnostic test is an elevated gastrin level when off H2 blockers or PPI. Secretin should normally suppress gastrin. In ZES, secretin causes a rise in gastrin levels.
local dz should be resected. Metastatic dz should be treated with life long PPI.
__________________________________________________ _____

An elderly man is brought to the ER dept with SX of tachycardia, diaphoresis and lightheadedness that begin 15-30 mins after eating, he had surgeries in the past for nonresolving ulcers. Another hour or two after eating the sx recur.
Dx. therapy

this is dumping syndrome, occurs in those with vagotomy and gastrectomy as a part of surgery for ulcers. There are two phases with similar sx. A rapid release of gastric contents into the duodenum, resulting in an osmotic draw of fluids into the intestine, that results in hypotension, lightheadedness, tachycardia, palpitations and sweating. Later there is a rapid release of insulin resulting in hypoglycemia, which produces many of the same sx.

Dumping syndrome is managed with multiple small meals, devoid of carbs. Dumping syndrome is seen in those with morbid obesity that have undergone gastric bypass surgery.

_______________________________________

A pt with longstanding diabetes comes to the office for evaluation of nausea, vomiting, anorexia with a sense of early satiety, and abdominal bloating, sometimes there is diarrhea and sometimes constipation..
most likely DX, most accurate DX, what is the therapy?

diabetic gastroparesis is a form of autonomic neuropathy occurring in pts with longstanding diabetes and its effects on the nerves in the stomach. There is bloating with early satiety. The major stimulant to gastric motility is stretch. Longstanding diabetes results in a neuropathy that reduces he ability of the GI tract to perceive stretch. DX is made with nuclear gastric emptying study.
promotility agents such as metoclopramide and erythromycin will relieve sx.
___________________________________

an elderly woman comes in for her evaluation of urinary incontinence
1- there is an irrepressible need to void. it often happens at night, she leaks urine before she is able to go to the bathroom.
2-The pt is obese. Episodes of incontinence are brought on by laughing, sneezing, coughing or lifting heavy objects.

1-urge incontinence presents with sudden and irrepressible urge to urinate that results in the passing of urine before the patient is able to make it to the bathroom. There is often associated pain over the bladder. The Most accurate test is urodynamic studies in which a catheter with a pressure transducer is placed in the bladder with the bladder half full to measure pressure. treatment with urge incontinence is with anticholinergic agents that are more specific to the bladder like oxybutynin, tolterodine, darifenacin, solifenacin and occasionally tricylic antidepressants..

Stress incontinence presents with leakage of urine associated with coughing, laughing or sneezing, which increase intrabdominal pressure. RX is with kegel exercises or topical estrogens cream. Estrogen creams increases the growth of the distal third of the urethra.
_______________________________

most likely dx for
1-pt with an elevated alk phos but no itching and a normal anti-mitochondrial antibody test.
2-bowed legs worsened slowly over time with an abnromal gait. Back pain and an enlarged skull with headaches. The alk phos and urinary hydroxyproline levels are elevated.
3-75 y/o woman has a pruritic eczematous rash over her nipple. It is progessive and now has a custing and a discharge that is sometimes bloody.

1-Paget's disease of the bone is often asymptomatic with just a marked elevation of alk phos.. Abnromalities of the x-ray can be found when x-rays are done for other reasons. Asymptomatic paget's disease doesn't need therapy.
2-paget's disease is the answer when there is bone pain, headache and physical enlargement of the head, and bowing of the tibia secondary to softness. pain is the first sx. When very severe, there is warmth palpated over the bone. rarely extra bone growth is so severe that high output CHF develops. the alk phos is markedly elevated with normal ca2+ and phosphate levels. X-rays is the best initial test. The best initial therapy is bisphosphonates or calcitonin.
3- paget's disease of the breast is a form of breast cancer in older women presenting with a prurituc, eczematous rash that sometimes develops a discharge. biopsy is diagnostic and rx is surgical.
____________________________________________

A generally healthy 40 year old man comes in with severe pain in the bottom of his foot, the pain is extremely severe as he gets out of bed. in the morning and it improved with the first few steps. Stretching improves the pain. there is marked tenderness at the midpoint of the heel.

dx, rx?

plantar fasciitis is an idiopathic disorder of severe pain in the bottom of the foot. The pain is extremely severe in the morning, especially with the first few steps. There is severe point tenderness at the heel where the fascia inserts.

plantar fasciitis improves gradually over time. stretching the foot with a towel or with a small wall stretch will improve condition. occasionally steroid injections or surgical release are necessary.

_________________________________________________

A young mother comes to see you because of pain in her wrist. the pain on the thumb (radial) side of the wrist occurs when she is gripping and object. the finkelstein test is abnormal

dx, finkelstein? RX?

De Quervain's tenosynovitis is pain in the tendons of the wrist. The etiology is unknown. DeQuervain's is the answer when there is pain, swelling, and tenderness on the radial side of the wrist.
Finkelstein is pain in the wrist when the thumb is placed in the closed first and the hand is tilted toward the little finger (ulnar deviation)
there is no proven therapy. NSAIDS and splinting are the mainstay therapy.

______________________________________

a 54 year old woman comes to the clinic for a follow up because of fatigue, CBC reveals a decreased hct of 32% and an MCV is low as 68fl

1- an elevated red cell distribution width RDW and a high PLT count
2-a low serum iron, low TIBC, LOW retic, hx of RA
3-A profoundly low MCV with very few sx, and an elevated red cell count, iron studies are normal


The most common cause of microcytic anemia is iron deficiency. iron deficiency is associated with elevated RDW because the cells become progressively smaller as the iron deficiency worsens over time. Iron deficiency is associated with thrombocytosis. this is benign and requires no additional rx. the best initial test is a low iron, low ferritin and increase TIBC. hx of blood loss.
2- anemia of chronic disease is characterized by a low serum iron, low TIBC, normal ferritin levels. Any chronic or inflammatory condition can lead to anemia of chronic dz. and is extremely common in RA
3-Thalassemia is associated with very few sx because the red cell count is elevated. This can maintain the total HCT close to normal. Thal gives normal iron studies. the most accurate test is hemoglobin electrophoresis. Don't treat with iron.
__________________________

an alcoholic comes to the ER because of fatigue, his only med is isoniazid. stool negative for occult blood. Hematorcit is 32% and the iron is elevated.

DX, diagnostic, therapy

sideroblastic anemia is most commonly associated with alcoholism. and is the only anemia associated with a high circulating iron level.
The MCV is most often increases, but it can be elevated or normal. Although leads poisoning is commonly associated with sideroblastic anemia, there are more people who drink alcohol than exposed to lead.
Sideroblastic anemia is also associated with isonizid use and myelodysplasia.
the most accurate test is a prussian blue stain. iron built up in ringed sideroblasts (see previous pages for smear). iron is built up in mitochondria.
there is no specific therapy, either remove the toxic exposure or treat the myelodysplasia.

________________________________________

ok pharm GI

an alcoholic is admitted for an episode of vomiting blood from esophageal varices. he has has several episodes before and has been twice treated with esophageal band ligation. he is already being transfused and hydrates and has a normal PT time.

acute esophageal bleeding can be treated with endoscopic band ligation. if this has been done, then transvenous( or transjugular) intrahepatic portosystemic shunts (TIPS) is useful. A catheter is placed into the hepatic vein, and a shunt is created through the liver to the portal veing. It can result in immediate decrease in portal HTN. the most common wrong answer is sclerotherapy, since it is inferior to band ligation.

most common adverse effect of TIPS is hepatic enecphalopathy.
propanolol is used to prevent recurrent variceal bleeding.
___________________________________________

pt is very distressed by persistent sx of epigastric pain from GERD. still has SX in spite of PPis

Nissen fundoplication is the treatment most likely to benefit a pt with persistent reflux GERD sx despite PPIs. this is surgically places narrowing of the distal esophagus in which the stomach is sewn around the esophagus.

H2 blockers are never as effective as PPis.
Cisapride is a promotility agent that has been removed from the market for causing arrhythmias. (caused torsades de pointes.
pylor doesn't cause GERD, treating it won't help.
distal esophagectomy is the rx for high grade dysplasia from Barrett's esophagus.
________________________________________________
38 year old woman comes to the office with dysphagia with ingestion of both solis and liquids. she waked up with regurgitated food particles on her pillow. Barium studies show narrowing of the esophageal sphincter to relax

Achalasia is treated with esophageal dilation. Pneumatic dilation is performed. In those refusing Pneumatic dilation. Botulinum toxin injection can be performed. the problem with botulinum toxin injection can be performed. The problem with Botulinum toxin injection is that it is not a permanent procedure and the symptoms can recur after 6-12 months, requiring additional injections or therapy.
if pneumatic dilation or botulinum toxin injection repeatedly fail, surgical intervention is performed. The Heller myotomy is the surgical cutting of the lower esophgeal sphincter.
_______________________________________________
27 yr old healthy GI fellow has developed severe, bloody diarrhea and abdominal pain over the last day. He is having 10 bowel movements a day, his pulse is 125, temp 103F. He is orthostatic and has abdominal tenderness. he is vigorously hydrating himself.

severe bloody diarrhea from food poisoning is treated with antibiotics, if the pt shows signs of sepsis. the question will describe hypotension, bloody diarrhea and abdominal pain.
the best empiric therapy for severe infectious diarrhea is fluroquinolone such as ciprofloxacin. Cipro has the greatest range of coverage of invasive pathogens such as campylobacter and salmonella.
__________________________________________________

53 year old diabetic for chronic abdominal pain and bloating, and a sense of fullness, she also has nausea with occasional vomiting. she sometimes has constipation and sometimes diarrhea. Upper endoscopy is normal.
diabetic gastroporesis is characterized by abdominal bloating, fullness, and early satiety and nausea, the best therapy for diabetic gastroparesis is with erythromycin or metoclopromide.

Erythromycin increases the release of motilin in the gastrointestinal tract. This increases the force and frequency of contractions in the stomach and intestines. In a normal pt with diabetic gastroparesis, this increases the forward flow of gastric contents. Metoclopromode increases of gastric contractions as well.

__________________________________________________ _____


diagnostic abdominal laparoscopy is used for direct visualization of abdominal structures used for
Ascites of unclear etiology
liver dz with inconclusive biopsy
staging of abdominal ca .

in trauma pt.
maybe used in stable pts as a less invasive aalternative to a laparotomy. DAL is especially useful in investigating injuries of the diphragm.

look for a victim of MVA in whom the CT is nondiagnostic and organ damage is suspected
________________________________________________
prental screening

tests include
cervical cytology
CBC, urinalsis, blood group Rh, serology for syphilis, rubella antibody, glucose screening, fetal ultrasouns, cervical cultures for Nisseria, group B strep, chlamydia trachomatis.
in a woman over 35 chromosomal screening.
_______________________________________________


renal ultrasound
best for
kidney stones, Congenital renal malformations, pediatric tumors, recurrent UTI, chronic renal failure, renal artery stenosis, polycystic kidney disease

in a pt with unexplained hemauria and flank pain and tenderness
renal US in a pt with dysuric sx such as frequency and burning, the urinalysis has WBC and flank pain.
_______________________________________



right upper quadrant ultrasound
used in pts with right upper quadrant pain with or without fever, the ultrasound will evaluate hepatobilliary structures for the folowing
liver pathologies
Masses, tumors, abscess, cysts
portal HTN
Gallbladder and billiary tree
cholelithiasis
cholecystitis
choledocholithiasis
cholangitis
the most accurate test for billiary pathology is ERCP or MRCP. most accurate for liver pathology is liver biopsy.
a (+) murphy's sign is suggestive of cholecystitis.
_________________________________________________


skull x ray
can detect fractires, ltic and blastic lesions frm bony mets or paget dz such as in above.
they are incapable however of detecting a bleed.
look for pt who has experienced head trauma. but it is almost always the wrong answer..
>>>>>>>>>>>>>>>>>>>>>>>>>>>>>>>>>
Reply

جوري
10-06-2008, 03:53 AM
10-5-5

heme

pt comes to the office because of fatigue and slowly progressive dyspnea on exertion. the HCT is low at 25% and the MCV is markedly elevated at 130fl. the peripheral smears shows hypersegmented neutrophils with an average of 5 lobes. The LDH and indirect bilirubin are also elevated. the retic count is low.
DX,
1- elderly pt with glossitis and peripheral neuropathy. both the methylmalonic acid and homocysteine levels are elevated.
2-malnourished alcoholic with an elevated homocysteine level

Vitamin B12 deficiency and folic acid deficiency are identical in their hematologic abnormalities. The most common abnormality of B12 deficiency is peripheral neuropathy.
2- Folic acid deficiency does not give neurological abnormalities. in addition folic acid deficiency elevates only the level of homocystine, whereas B12 elevates both homocysteine and the level of methylmalonic acid.

_______________________________________________

pt with sickle cell dz is admitted because of fatigue developing over several days. The HCT has dropped precipitously. the MCV is normal and the retic count is low. WBC count and plt are normal
dx, accurate, initial therapy

Parvovirus B19 is the most likely cause of pure Red-Cell aplasia in a person with a hemoglobinopathy. The retic count should be elevated in a person with anemia. This is particularly true in the case of sickle cell dz in which the retic count is usually 10-20%.
Parvo-virus invades the bone marrow and freezes the growth of precursor cells in the marrow. the retic count is abnormally low. Because a pt with SCD has such a high percentage of reticulocytes. the HCT can drop very precipitously when infection with parvovirus occurs.

the most accurate test for B19 is PCR for DNA. If this is not one of the choices, then test is IgM against the virus. Although bone marrow biopsy with an increased number of giant pronormoblasts does develop. this is obviously more invasive and not as specific as the PCR for parvovirus DNA.

RX is with IV immunoglobulins.

_______________________________________________
a pt comes to the er with sudden onset fatigue, SOB and HCT 20%. MCV is slightly elevated, the retic count, LDH and indirect bili are all elevated and the haptoglobin is low

1-HX of SLE. CLL, lymphoma or med such as PCN
2-Recurrent episodes with a large spleen. Often with fam hx. An elevated MCHC
3-Sudden onset of hemolysis in a male pt with an acute infection. occasionally happens after sulfa drug use.

1- All forms of hemolysis, lead to elevated LDH, indirect bili and retic. Autoimmune warm antibodies are found in association with SLE, lymphoma, and CLL. In addition, medications such as PCN, sulfa, and quinidine can provoke autoimmune hemolysis lead to elevated levels of LDH, indirect bili, and retic. Autoimmune warm antibodies are found in association with SLE, lymphoma, and CLL. In addition, medications such as PCN, sulfa medications and can provoke autoimmune hemolysis. The Most accurate test i a coombs test. the smear will be normal because the hemolysis is occurring in the spleen.
2-Hereditary spherocytosis presents as recurrent episodes of hemolysis. The MCHC is elevated because red cell membrane is to tight to contain the amount of hemoglobin present. The most accurate test is osmotic fragility.
3-Glucose-6-phosphate dehydrogrenase (G6PD) deficiency presents most often in males because it is X linked. Although primaquine, dapsone and fava beans have been classically associated with this disorder, the most common cause of acute hemolysis is an infection. the best initial tests are for Heinz bodies and bite cells.
__________________
A man comes to the office wit dark urine in the morning, his urinalysis shows hemoglobin but no red cells are visible, there are no white cells or protein. the CBC shows anemia and mild thrombocytopenia, the LDH, indirect bilirubin and retic count are elvated. He has a HX of a large vessle thrombosis. The Leukocyte alakaline phosphatase level is low.

DX, dignostic and common cause of death

PBH presents with episodes of dark urine in the morning. The hemolysis occurs over night with hemoglobin visible with the first morning urine. Pancytopenia often presents as a sign of hemolysis such as elevated bili, and retic count are present. the retic count maybe low.. A low Leukocyte alkaline phosphatase maybe present.
the most accurate test is a CD55/59 antigen test that is low. CD55/59 is the marker for decay accelerating factor. DAF removes complement from the cells before they are destroyed. older less accurate tests are sugar/water and Ham's test which look for activation of complement.

The most common cause of death is large vessel thrombosis. less common complications are acute leukemia, aplastic anemia, and myslodysplasia. This is because PNH is a stem cell disorder.

__________________________________________________ _____

An african american man comes to the ER wth pain in his back, chest and thighs, he has a hx of sickle cell disease. He is febrile 102f. cxray and urinalysis normal. o2, fluids, and analgesics are started his HCT is 28%

most urgent ste
best initial test to confirm parvo B19
most accurate diagnostic
initial to confirm SCD

the most urgent step in SCD when a fever is present is to start antibiotics such as ceftriaxone, levofloxacin or gatifloxacin. Don't wait for results of cultures. pts with SCD can die rapidly of overwhelming sepsis because they are functionally asplenic.
pavovirus B19 results in aplastic crisis particularly in those with an HX of hemoglobinopathies. The best initial test is a reticulocyte count. pts with SCD usually have a high retic count. PARVOVIRUS gives a low retic count.
the most accurate test for parvo is a PCR for the DNA virus.
the best initial test for sickle cell dz is a peripheral smar. the most accurate test is a hemoglobin elecrophoreis. pts with SCD often present with acute chest syndrome...

>>>>>>>>>>>>>>>>>>>>>
Reply

جوري
10-07-2008, 02:49 AM
yesterday was a disaster I set out to do 10/5/5 did only five.. the ac adapter of my new computer is dead.. finito so here we are on desktop.. let's make up what was lost..


Heme in most likely DX

1-An AA is taking a course in skydiving, he is on his first time up at high altitude, about to jump when he develops severe chest, back and thigh pain. When the plane returns to the ground, he feels well, his CBC including, peripheral smear, isnot normal. His only medical hx is of dark urine
dx, and diagnostic?

Sickle cell trait or hetrozygous (AS) sickle cell disease is present in 8% of African Americans. Acute painful crisis in SCT is extremely rare and occurs and occurs only under conditions of the most severe hypoxia or high altitude, such as would occur during parachute jump. The only significant manifestations of AS disease are renal concentrating defect (isosthenuria) and occasional episodes of gross hematuria. There is no specific therapy.
Most accurate test for ickle cell trait is a hemoglobin electrophoresis. there is no specific therapy!

__________________________________________________

2-A man from Miami recently moved to Chicago for his residency. He has an episode of pneumonia that is mild with a dry cough and bilateral interstitial infiltrates that resolve with azithromycin, while shoveling snow he suddenly develops pain and discoloration of his fingers, nose an ears. His hct is 28%, and the bilirubin, LDH and retic count are elevated.
dx, accurate test, therapy

Cold agglutinin disease or igM-induced antibodies is the most likely diagnosis when there is hemolysis in association with pain and discoloration of acral portions of the body such as the fingers, nose and ears on exposure to cold. in addition, although most cases of cold agglutinin disease are asymptomatic, look for a recent history of mycoplasma pneumonia such as suggested in this case. Epstein barr virus is also another clue.
the most accurate test is a direct coombs that is positive for complement only. all the usual findings of hemolysis are present, such as an elevated LDH , indirect bili, and retic count but they are nonspecific for cold agglutinin disease

no specific therapy is usually necessary, steroids aren't helpful. in severe cases, alkylating agents such as cyclophosphamide can be used. cyclosporine are also helpful.

________________________________________________

3-A man with diarrhea comes because of weakness and anemia, in addition, he has an elevated retic count, LDH, and indirect bili level. The haptoglobin is absent. the plt count is 38,000 but he isn't bleeding. the creatinine is 2.4

dx,cause of diarrhea, best test, most accurate test

Hemolytic uremic syndrome (HUS) is the triad of hemolytic anemia, renal insufficiency and thrombocytopenia. If neurologic abnormalities and fever are also present, this is thrombotic thrombocytopenic purpura TTP

HUS is often associated with E.coli 0157:H7.
the best initial test is a peripheral smear showing fragmented red cells such as schistocytes or helmet cells pls see previous page for slide. This is also referred to as microangipathic hemolytic anemia.

there is no specific test for either HUS or TTP. they are diagnosed bases on either the triad or pentad of laboratory abnromalities. Most of the time they will resolve spontaneously. Do not give plts or antibiotics.

______________________________________________

4- Man comes to the office with sx of dizziness, headache, fatigue, and blurred vision, he is very itchy after a warm shower, he gets nosebleeds, he has splenomegaly. His hematocrit is 58%. The MCV is low at 68fl the white cell count and plt are normal.
dx,best initialt diagnostic, most accurate, and common cause of death..

polycythemia vera is a neolasm of the bone marrow with markedly elevated hct. in the absence of hypoxia or an elevated level of erythropoietin. Polycythemia vera presents with signs of hyperviscosity such as headache, blurry vision, fatigue and epistaxis. pruritis after a warm show is common because of histamine release from basophils. the cells in polycythemia vera are small.

the best initial test is an arterial blood gas to exclude hypoxia as a cause of secondary polycythemia. if the hematocrit is markedly elevated above 60% and the white count and plt count are elevated. no additional tests besides a bone marrow biopsy are necessary because nothing else besides polycythemia vera will give an elevation of the three cell lines.

the most accurate is a nuclear red-cell mass test
the most common cause of death is large vessel thrombosis from hyperviscosity of elevated red cell mass.

____________________________
5- 52 year old man comes to the office with painful burning in his hands. the only lab abnromality is plt count of 1,500,000
dx, accurate diagnostics, common cause of death, best therapy



is a platelet cancer. this is a myeloproliferative disorder of the bone marrow in which the platelets are elevated to levels above 1 million. The white cell count can also be up.
ET ca present with erythomelalgia, which is painful, red burning of the hands; however, it may present with a high platelet count only.
there is no specific test, the bone marrow shows, nothing except increased number of megakaryocytes. Red cells are normal, there is a high frequency of mutation to JAK2.
essential thrombocytosis can result in death from either bleeding or thrombosis. thrombosis is more common.
the best initial therapy is hydroxyurea.

__________________________________________________

6-an elderly man is being evaluated for progressive fatigue. CBC shows pancytopenia. MCV is normal

dx
1-splenomegaly, nucleated red cells, rear drop cells, and leukoerythoblastosis.
2-splenomegaly, a noaspirable dry tap, and a positive tartarate resistant acid phosphatase
3-pancytopenia alone with a vacant bone marrow

1-myelofiboris is diagnosed by finding the combination of nucleated red cells, teardtop-shaped cells, and an immature white cel that forms on smear such as promyelocytes or myeloblasts. All together, this is caled 'leukoerythoblastic' presentation. the liver and spleen are big because progressive marrow fibrosis leads to extramedullary erythropoiesis.

2-hairy cell leukemia



presents in middle aged ts with pancytopenia, massive splenomegaly, and a dry tap. the most accurate test is the TRAP or tartarate-resistant acid phosphatase.

3-Aplastic anemia is simply pancytopena of unclear etiology. the marrow is empty and can be replaced by fat, there is no fibrosis in the marrw. splenomegaly is not present because extramedullary hematopoiesis is not occurring. the marrow has just simply died. Remember parvovirus B19 in pts with previous baseline anemia such as sickle cell, or thalassemia can cause transient aplastic anemia.

__________________________________________________ _
7- a 60 year old man comes in with fatigue, low grade fever and abdominal fullness, massive splenomegaly is found. His white cell count is markedly elevated at 175,000, they are normal and mature appearing on smear. the leukocyte alkaline phosphatase (LAP) score is low
dx, accurate diagnostic and common cause of death

Chronic myelogenous leukemia (CML) presents with fatigue, upper abdominal pain





presents with fatigue and left upper quadrant pain from really big spleen. the white cell count is markedly elevated but they look normal and the LAP score is low. A low LAP scores means that the cells maybe up in number but they are low in function.

the most accurate test for CML is the philadelphia chromosome. this can be called bcr/abl mutation.

\

without rx with imatinib, 20% of CML pts will transofmr into acute myelogenous leukemia each year.

__________________________________________

8- a 72 year old man comes in for progressive fatigue, he has splenomegaly on examination, his HCT is 30% with an MCV of 107fl. There are oval shaped cells. the retic count is reduced. the white cells show bilobed nuclei. There is a mild reduction in plt count. B12 and folate levels are normal.

dx, accurate diagnostic

myelodysplastic syndromes are a collection of pre-leukemic syndromes with macrocytic anemia, they are seen exclusively in elderly pts. they often have a bilobed neutrophils known as Pelger-Huet cells


the platelet countand retic count are often reduced. although a small number of pts progress to acute myelogenous leukemia most pts will die of bleeding or infection before that occurs.

the most accurate test is a bone marrow biopsy. the marrow is hypercellular despite the peripheral low counts. the prussian blue shows ringed sideroblasts .



___________________________________________
9- 34 year old man comes in with severe bleeding from his skin, nose and rectum, he has a fever, CBC shows pancytopena, there are blasts visible on the peripheral smear. The PT and PTT are elevated. Some of the neutrophils have an eosinophilic inclusion body visible.

dx, diagnostic

acute promyelocytic leukemia, or M3 leukemia, presents with the same pancytopenia as any other acute leukemia with blasts present. in addition, promyelocytic is always the most commonly asked question, because it is acute and has a very distinct presentation. The association with disseminated intravascular coagulation (DIC) is characteristic or promyelocytic leukemia. , this is because promyelocytic granules activate the coagulation pathway


the esopsinophilic body is an Auer Rod, which is characteristic of promyelocytic leukemia ( also AML)

the most accurate test for acute leukemia is a bone marrow biopsy. this is the most accurte way to assess the number of blasts, in addition, the most important test to determine prognosis are cytogenetic studies. these are best obtaines on actively replicating cells found in the marrow.

________________________________________________
10- a 60 year old man is found to have an elevated total protein on routine blood testing in the office. Electrophoresis reveals a monoclonal IgG spike. Calcium, CBC, urinalysis, and skeletal bone survey are all normal
dxm accurate diagnostic, initial therapy

this is monoclonal gammopathy of unknown significance (MGUS) is most found on routine testing of blood protein in an elderly pt. The pt is asymptomatic. all other tests are normal. There will be no Bence jones protein, no bone lesions and a normal uric acid level.
the most accurate test is a bone marrow biopsy, MGUS has <10% plasma cells on bone marrow biopsy.
there is no therapy for MGUS. only 1% of pts per year will progress to myeloma, and no therapy is known to prevent this.

__________________________________________

pharm.
a young woman with multiple bowel movements per day and abdominal pain, some BMs have blood, she was dxed with crohn's disease
Mesalamine derivatives are the best initial therapy for both crohn's and UC. Mesalamine is the most effective way of delivering 5-aminosalicylic acid (5-ASA) several formulations exist.
Asacol delivers the 5-ASA to the distal bowel
Pentasa delivers therapy to the entire bowel.
sulfasalazine gives 5-ASA but has many more adverse effects such as rash, hemolysis and renal tox, because of the sulfa component.
acue episodes of worsening inflammatory BD that are not controlled with 5-ASA derivatives are treated with oral steroid budesonide. Budesonide has enormous first pss effect on the liver, so it has limited toxicity. Azathioprine can be used to keep the disease nder control without the use of steroids.

_____________________________________________

44 year old woman comes for a follow up, she has has acid reflux for 5 yrs. Endoscopy reveals a 3 cm of columnar epithelium of the distal esophagus.

barret's esophagus is columnar epithelium extending up out of the stomach into the distal esophagus. Barrett's esophagus should be treated with a PPI. such as omeprazole, pataoprazole, lansoprazole, esmeprazole, or rabeprazole.

PPIs work by inhibiting the release of acid from the parietal cells of the stomach. they inhibit the K+/H+ ATpase. PPIs have no major adverse effects.

surveillance endoscopy for Barretts esophagus should be performed every 2-3 yrs. if low grade dysplasia is present, repeat endoscopy every 6 months.

____________________________________________

a man develops multiple ulcers in his duodenum. they recur after rx for H pylori. his gastrin level is elevated. he undergoes endoscopic US and an octreotide scan to determine therapy

an endoscopic US and octreotide scans are performed to determine the presence of mets. localized ZES or gastrinoma is treated by surgical resection
Metastatic ZES is treated with lifelong PPI.

_____________________________________________
a 55 yo woman comes in with pruritis, her alk phosp is elevated, her AST and ALT are minimally elevated. anti-mitochondrial antibodies are present in increased amount.

this is primary billiary cirrhosis, treated with ursodeoxycholic acid. Cholestyramine maydecrease itching but should not be given at the same time as ursdeoxycholic acid, because it may decrease absorption.

the mechamism or ursdeoxycholicacid is poorly understood, it decreases plasma and endogenous bile acid conc. this reduces hepatotoxicity, because endogenous bile acids are more toxic to the liver. ursodeoxycholic acid decreases esiosinophil activation and may decrease the immune destruction of hepatocytes.
urodeoxycholic acid generally devoid of adverse effects.

__________________________________________________ ____

43 year old man comes with patiguem joint pain, darkened skin, ED, serum iron and ferritin levels are markedly elevated. the MRI shows abnromal liver.

2- pt with SCD comes in for rx, she has been transfused for severe sickle cell crisis 5 tomes a year for 10 yrs

case one is heriditary hemochromatosis from overabsorption of iron in the duodenum. the pt should undergo periodic phlebotomy to dec iron overload.

case two has developed hemochromatosis from repeated transfusion. can't use phlebatomy on someone who is anemic, thus chelation therapy is used. Deferasirox is an oral iron chelator, Deferoxamine has to be given by injection and is harder to use
chelating agents bind to iron and allow it to be excreted..

________________
diagnostics

pelvic us, vaginal US

Pelvic US used for
pelvic or lower abdominal pain with amenorrhea or vaginal bleeding
suspected pregnancy
suspected ectopic pregnancy

transvaginal US
superior to pelvic used in
placenta previa (painless vaginal bleeding in the third trimester)
fetal monitoring in the first trimester
obese women in examining pelvic anatomy.

____________________________



the crystals are enveloped shaped oxalate crystals
they precipitate in renal tubules when toxic dose of ethylene glycol, a component of antifreeze is ingested.
most specif test is to determine blood level. urine can be examined under fluorescent wood's light, however, urinary fluroescene lasts only a few hours after ingestion. bcause the oxalic acid precipitates with calcium the blood level of calcium is always low.
answer etheylene glycol overdose and oxalate crystals when you see an elevated anion gap and metabolic acidosis . the question most often describes a depressed pt with possible attempted suicide, Hypocalcemia can lead to QT prolongation and cardiac arrhythmias. Oxalate crystal deposit in the renal tubules can cause acute tubular necrosis.
_____________________________

Anti-basement membrane antibodies (Anti-BM)
are indicative of goodpasture syndrome



pt with hematuria, and HTN, accompanied by a cough and SOB, hemoptysis. Renal insufficiency and red cell casts in the urine are characteristic..
the most accurate test for goodpasture is a lung or kidney biopsy as in above.
___________________________

total complement level (ch50)
complement deficiency leads to recurrent bacterial infections. CH 50 or total complement assay, is performed on young pts with recurrent unexplained infections such as pneumonia, sinusitis, meningitis with encapsulated organisms such as Neisseriae, s. pneumonia, and H flu. the pt will have normal lymphocyte and immunoglobulin levels. an abrnomal CH 50 assay indicated inherited or acquired complement deficiency.

______________________________________
urine microalbumin

this is a sensitive screening test for diabetic nephropathy

look for a case with diabetes for >5 yrs who has normal renal function and a negative urinalysis for protein. if the dipstick is already positive for albumin, there is no reason to look gor microalbuminuria. screening is done on yearly basis .

if the test is positive use ace inhibitors, or ARBs to prevent rogression of disease.
a negative test for microalbumin is <30mg/day, levels between 30-300md/day are called microalbuminuria. levels above 300 would register positive on the normal urinalysis.
renal biopsy is the only test of the effect of diabetes on the kidney that is more sensitive than microalbuminuria.

___________________________________
Reply

جوري
10-07-2008, 11:08 PM
a 70 year old man comes to the hospital with blurry vision, SOB, confusion, vertigo and nausea, He is anemic and the white cell count is normal. The serum viscosity level is increased to 1.5 times that of water. He has engorged sausage shaped blood vessels in his eyes
dx, diagnostic

Waldenström's macroglobulinemia ( lymphoplasmacytic lymphoma)


is caused by hyperviscosity from the overproduction of IgM from lymphocutes and plasma cells. IgM is larger than IgG and therefore presents with a hyperviscosity syndrome that obstructs blood vessels in the brain, lungs, and eyes and results in SOB, vertigo also occurs. GI bleeding may occur from engorged vessels. This is the same short of presentation as leukostasis reaction in acute leukemia, however, the white-cell count in Waldenström's macroglobulinemia is generally normal.

the most accurate test is a serum protein electeophoresis with an elevated IgM spoke and a bone marrow biopsy showing increased plasma cells. Bone x-rays are normal.

_____________________________________________

a pt. is admitted for a PE. two days after starting IV heparin the plt count starts to dec.
dx, diagnostic

heparin indusced thrombocytopenia (HIT) occurs several days after the start of heparin. The most common presentation is an asymptomatic decrease in the plt count. Occasional episodes of thrombosis may occur. Venous thromboses are generally three times more common than arterial thrombosis. In general, a 50% decrease in the number of platelets after starting heparin is considered a threshold criteria.
The most accurate test for HIT is for antibodies to platelet factor 4. these are heparin induced anti platelet antibodies. Serotonin release is very sensitive. RX is to sop all heparin products immediately.

_______________________________________________

Pt is admitted to psych for an acute episode of hallucination, psychosis and hysteria, she also has abdominal pain and dark urine. She recently started phenobarbital for seizures. The attack accompanied the onset of menses. Despite the severity of her abdominal paon, the exam is benign.

dx, initial diagnostic, initial therapy

acute intermittent porphyria (AIP) presents with severe abdominal pain, neuropsychiatric disturbance, and dark urine. Episodes often happen around menses and or after the start of meds such as phenobarbital.
AIP is confirmed with urinary levels of aminolevulinic acid and porphobilinogen.

AIP is treated acutely with dextrose and IV heme infusion.

____________________________________

a woman comes in with increased bleeding after a dental extraction. she has noticed increased bleeding such as epistaxis and petechiae for many yrs. The plt count is normal. the aPTT is modestly elevated.
dx, diagnostic, most accurate

Von Willebrand's disease presents with an increased mucosal type of bleeding, particularly after minor trauma, surgery, or ASA use.
the APTT can be elevated because factor VIII antigen (von willebrand's factor, VWF) and factor VIII coagulant ( hemophilia A factor) travel bound to each other. This can't be hemophilia because the type of bleeding is hemophilia would be deep bleeding into a joint or into muscle, such as a hematoma. in addition hemophilia does not express itself in women.
the best initial test for plt function is bleeding time. Do not do a bleeding time is the plt count is low. if the plt count is low, the bleeding time will be abnormal.

the most accurate test for von willebrand's dis is a combination of VWF and ritocetin testing. Ristocetin testing determines the function of the VWF if the level is normal.

________________________________

a pt comes in with bleeding into his joints after minor muscle trauma. the plt count is normal.

1-male child with an elevated aPTT and a normal PT
2-Pt who has recently has an IV antibiotics. There is elevation of both PT and aPTT.
3-an alcoholic pt with low albumin, who also has varices. both PT and aPTT are elevated.

1- Hemophilia is the most likely dx with hemarthrosis n a male child after a minor trauma. only the aPTT will be elevated. The best initial test is a mixing study and the most accurate is a specific level of factor VIII or IX. The mixing study s the first test to perform to determine the presence of a clotting factor deficiency. if the aPTT is elevated from a clotting factor deficiency, the lab value will return normal when mixed 50:50 with normal plasma. if there is a clotting factor inhibitor, it will not correct.
2-Vit K deficiency is suggested by the recent antibiotics which deplete the levels of vit K in the body. Both PT and aPTT will be elevated. The diagnosis confirmed by looking for an improvement after administering supplementary vit K.
3-Liver disease presents in the same manner clinically as vit K deficiency, but there will be no improvement after adding supplementary vit K.
__________________________________

a pt comes in with a elevated aPTT found on routine screening prior to a minor procedure. The PT is normal
1- There is no bleeding at any time, the pt completely asymptomatic.
2- the pt has minor bleeding occasionally in the past, but only with trauma or surgical procedures.
3-there has been clotting such as a DVT, in the past. The VDRL is positive.

1- Factor XII deficiency produces an elevation in the aPTT with no evidence of bleeding even under conditions of additional trauma.
2-Factor XI deficiency results in a prolonged aPTT and gives abnromal bleeding under conditions of trauma or surgery. such as a dental extraction. Factor 11 deficiency is common in Ashkenazi Jews.

3-Lupus anticoagulant is a type of antiphospholipid antibody that results in increased clotting but gives a prolonged aPTT as a lab artifact . It is associated with a false positive VDRL. on mixing studies, the aPTT will not correct on a 50:50 mix normal plasma because it is a circulating antibody. the antibod will be presents in the mix. Deficiencies correct to normal when mixed, antibodies will not.
________________________________________

a pt is admitted with a pulmonary embolus, he is not obese or elderly, there is no malignancy or increased risk of clotting that can be identified.
1-Most common cause of thrombophilia
2-there is skin necrosis with the use of warfarin
3-the aPTT does not rise after the use of heparin

1- Factor V leiden mutation is the most common cause of thrombophilia. This is a genetic defect that results in resistance of factor V to inactivation by protein C.

2- Protein C is associated with skin necrosis with the use of warfarin. protein C is a natural vitamin K-dependent anticoagulant with a very short half life. When starting warfarin, there is a transient hypercoagulable state that is produced for a short time before the other clotting factors are inhibited.

3-antithrombin III deficiency is a cause of thrombophilia that results in resistance to heparin. Heparin works by potentiation of the effect of antithrombin. If there is a low level of antithrombin then heparin will no work. There will be no rise on aPTT afte a bolus of heparin.

_____________________________________________
pls diagnose each of the following transfusion reactions

1-Mild febrile reaction with the first unit of blood. with the second unit there is SOB and Pulmonary infiltrates that resolve in 24hrs
2-immediate anaphylaxis after transfusion
3-mild urtricarial reaction after transfusion, no evidence of hemolysis
4- A single degree centigrade with no evidence of hemolysis.

1-Leukoagglutnination reaction occur from donor antibodies attacking and agglutinating recipient WBC, resulting in SOB, this is also known as transfusion associated ling injury (TRALI) no rx necessary.

2-IgA deficiency leads to anaphylaxis and occurs from IgA in the donor blood. This occurs in IgA deficient recipient. Use blood from IgA deficient donors in the future.
3-Urticarial rxn occur as an allergic rxn to donor plasma proteins. urticarial reactions can be prevented by transfusing washed red cells.
4-febrile non-hemolytic reactions occur from a reaction against donor white cells. prevent this by filtering blood.

________________________________________________

a young pt comes in with multiple sinopulmonary infections. He has had sinutitis, bronchitis, pneumonia and otitis media

1- the pt is an adult with normal lymph nodes. B-Cell numbers are normal. Immunoglobulin produced is markedly low.
2- A male child has infections in the first year of life, lymph nodes and tonsils are hypoplastic, Immunoglobulin levels and B cells are absent.
3-T cells are absent. there are cardiac defects, facial abnormalities, and hypocalacemia with low PTH level IgG levels are normal

1-Common variable immunodeficiency (CVID) presents in adults with normal numbers of B cells but markedly low immunoglobulin levels.
RX is with replacement of immunoglobulins.
2-X-linked agammaglobulinemia (Bruton's) presents in make children at an early age. Not only is no immunoglobulin produces, but the B cells and normal lymphoid structures are missing. RX is with immunoglobulin replacement.
3-Di George's syndrome is an isolated T-Cell deficiency from thymic hypoplasia. DiGeorge's syndrome is associated with cardiac and facial defects. hypocalcemia results in the inability to develop parathyroid glands. Bone marrow transplant is used in severe cases.
_____________________________________________
a young pt comes in with repeated episodes of otits mesia and pneumonia. in addition there is eczema and dermatitis.

1- Allergic disorders, asthma and urticaria also occur. There was a severe persistent diarrheal illness from Giardia lamblia. A blood transfusion resulted in anaphylaxis.
2-a male child presenting at a very early age with infections also has a bleeding disorder. the platelet count is low and platelets are small in size.

IgA deficiency most commonly comes to attention because of frequent sinopulmonary infections. There are multiple allergic disorders with IgA deficiency such as asthma, urticaria, rhinitis and atopic eczema. Chronic infections can result in anaphylaxis. i the cells are not washed because of an allergic reaction to IgA in the donor blood. There is no specific therapy. Transfusion should only be from IgA deficient donors or with washed red cells.

2-Wiskott-Aldrich syndrome is the combination of increases susceptibility to infection combined with eczema and thrombocytopenia. Atopic dermatitis occurs with increased frequency of otitis media, pneumonia, and thrombocytopenia. bleeding is common.

_______________________________________
\pharm

25 year old woman comes to see you for the fifth time in two months because of abdominal pain, she has periods od diarrhea alternating with constipation, the sx are less severe at night. Her abdominal CT scan and colonoscopy are normal.

IBS is initially treated with dietary modifications such as increasing fiber content. the best sources of fiber are bram, psyllium, husks, and methcellulose.
Antispasmodic medications such as dicyclomine, or hyoscamine may decrease the pain, they have modest anticholinergic effect. diarrhea can be treated with dihenoxylate or loperamide which inhibit gi motility.
if fiber and antispasmodic agents aren't effective, tricyclic antidepressants may work, tricyclics are anticholinergic and have a beneficial effect of neuropathic pain.

______________________________________________

Sildenafil, tadalafil and vardenafil
these are the best initial therapies for erectile dysfunction. sildenafil has also been approved for rx of pulmonary htn.
these meds are phosphodiesterase inhibitors. they decrease vasculat tone. and increase flow into the penis by increasing local concentrations of NO
phosphodiesterase inhibitors can cause hypotension, headache and facial flushing. they should not be used in combo with nitrates for pts who have CAD. grapefruit juice can increase the levels of these meds because of its effect on inhibiting cytochrome p450.

__________________________________________________ _

a 72 year old man comes to the office because of a delay in his ability to urinate. He has a decreased urinary stream, increased urinary fequency, there is no burning pain when he urinates. urinalysis is normal.

the best therapy for benign prostatic hypertrophy is a combination of a 5 alpha reductase inhibitors, such as finersteride, and peripheral alpha blockers, such as doxazosin, prazosin or tamsulosin.
finersteride blocked production of dihydrotestersterone, which stimulates growth of the prostate, opening the bladder neck, prostate capsule, and prostatic urethra, prazosin, terazosin, doxazosin and tamsulosin increase the volume of urinary stream.
peripheral alpha blockers can cause dizziness and orthostatic hypotension, Tamsulosin has the same therapeutic benefit with less incidence of hypotension.
if medical therapy fails, the treatment of BPH is a transurethral resection of the prostate TURP.

_________________________________________

Finasteride and dutasteride
are te best therapies for benign prostatic hypertrophy. they decrease the volume of the prostate and increase the volume of urinary flow. Finasteride is also approved to treat male pattern hair loss.
hese meds inhibit 5 alpha reductase, which converts testosterone to dihydro-testosterone. dihydro-testosterone is responsible forthe growth of the prostate as well male pattern baldness.
Finasteride and dutasteride have a very small effect of decreasing libido, decreasing the volume of ejaculate, and causing a mild increase in the incidence of erectile dysfunction.
________________________________
ezetimibe

is used to rx hyperlipidemia. it is added to HMG-CoA reductase inhibitor (a statin) if the low density lipoprotein (LDL) is not sufficiently controlled. The major advantage of ezetimibe it is limited side effects.
Ezetimibe inhibits absorption of cholesterol at the brush border it blocks the sterol transporter.
Ezetimibe causes diarrhea and abdominal pain because of its ability to block fat absorption.

____________________________

diagnostic

water deprivation test

is used to distinguish between the causes of diabetes inspidius (DI) to objectively assess pts with hypernatremia and high urine output.
you restrict water intake for 12-24hrs with observation for the volume of urine, urine osmolality, and change in body weight to evaluate whether the kidneys are conserving or excreting water properly, if the urine volume stays elevated after the serum osmolality goes up, then it is DI. If there is a decrease in response in response to giving vasopressin or ADH, then it is central DI, if no response then it is nephrogenic

if this is a bit confusing to you, and you like anecdotal learning, then I have covered the tread entirely earlier here
http://www.islamicboard.com/health-s...dus-siadh.html

________________________________________



diascopy is the best initial test for erythematous lesions. It is a fancy way to see of the lesion blanches (turns white), a lear plastic slide is pressed over the skin, used to determine if an erythmatous skin lesion is due to vasodilation or extravasation of RBCs via damaged blood vessels.

if the skin blanches with diascopy, the erythema is causes by vasodilation, simple vasodilation is caused by viral diseases, drug reactions and insect bites. if the lesion persists under diascopy, the redness is caused by extravsation of erythrocyte via damaged blood vessles, as seen with vascuilitic and purpuric lesions.

__________________________________________________ ____


excisional biopsy is a skin technique where the full thickness of the lesion is removed preferably intact.
this is best for such and malignant melnoma see above.
can't be staged properly otherwise is not fully removed.

_____________________________________________

direct immunofluorescence (DIF) of skin biopsy samples is used to diagnose skin disorders of immunologic origin. The DIF test looks for the presence and staining pattern of immunoglobulins 9IgG, IgM, IgA), C3 and fibrinogen in the skin. for examle it is useful for differentiating between pemphigus vulgaris and bullous pemphigoig


pemphigus vulgaris immunofluorescence ( IgG and C3 between epidermal cells)



bullous pemphigoid immunofluorescence (IgG and C3 at dermodermoepidermal junction)

________________________________

fungal culture
is the most accurate diagnostic test superficial fungal infections of the body. examples, tinea, onychomycosis and fungal vaginits.
not the best initial test though, fungal infections are primarily diagnoses by visual appearance and confirmed by KOH prep.
onychomycosis (nail bes) and tinea capitus (scalp) need to be treated with oral antifungals. (terbinafine or itracaonazole). all of the other fungal infections of the skin may be treated wit topical meds such as ketoconazole, clotrimazole, nystatin or ciclopirox.

>>>>>>>>>>>>>>>>>>>>>>>>>>>>>>>>
Reply

جوري
10-09-2008, 02:01 AM
we start with infectious disease in most likely dx

1- pt comes in with fever, headache, and vomiting, he experiences a seizure..

1- confusion is the main compliant
2-stiff neck (nuchal rigidity) and photophobia are present.
3-He has focal neurological deficits and projectile vomiting.

1- encephalitis is characterized predominately by confusion and fever for a few days. Although there is a headache, nausea, vomiting and seizures. these findings aren't specific to encephalitis. Encephalitis is best diagnosed with a head CT followed by a lumbar puncture. The most accurate diagnostic test for herpes encephalitis is a PCR of the CSF, not a brain biopsy.

2-meningitis presents with stiff neck and photophobia

3- brain abscess presents with focal neurological findings in addition to confusion, and focal findings--

______________________________

pt comes to the ER dept with dever, headache, neck stiffness and photophobia
1- six hours of sx with 3,500 white cells that are predominantly neutrophils.
2-neutrophilic predominance and recent neurosurgery.
3-the CSF protein is markedly elevated, there are 175 lymphocytes and the adenosine deaminase level is elevated.
4-Petechiae and rash are present on the wrists and ankles that move toward the body. CSF lymphocyte count is mildly elevated.

pneumococcus is the most common cause of bacterial meningitis. you can't tell the difference between pneumococcus, hemophilus, gram negative meningits, and staphylococcus for sure without culture of the cerebrospinal fluid.

2-staphylococcus is the most common organism after recent neurosurgery.
3-TB is suggested by a very high protein, high adnosine deaminase level, and lung lesions.
4- Rocky mountain spotted fever presents with a salmon colored rash that is vascuilitic in nature and moved toward the body. A tick bite is recalled in 60% of cases. CSF shows a modest elevation in lymphocyte count.

__________________________________________________ ________

A patient comes in with fever, headache, photophbia and neck stiffness. The cerebrospinal fluid (CSF) protein level and white cell count are elevated.

1- an alcoholic, elderly patient who is HIV positive and is on steroids for lymphoma with 2,300 neutrophils in the CSF
2-Generally a healthy patient with mild lymphocyte elevation.
3-Adolescent with a petchial rash and terminal complement deficiency. Neureophil count is elevated.
4-HIV positive pt with mild sx over several weeks. CD4 count of 20. Mild CSF lymphocyte elevation...

1- Listeria monocytogenes presents with increased neutrohils in the CSF in pts who are immunocompromised, elderly, neonates. steroids, alcoholism, chemotherapy, and leukemia all predispose.
2-Viral meningitis occurs in healthy pts and is self-limited. there is no past medical hx and the lymphocyte count in the csf is mildly elevated.
3-Neisseria meningitis occurs more often in adolescents in conditions of crowding such as dormitories or in military recruits. a petechial rash is characteristic. Splenoectomy and terminal complement (C5-C9) deficiency are very strong risk factors. A vaccine against N.meningitis exists.
4-Cryptococcus in the case of HIV/AIDS with low CD4 counts (<50). cryptococcus gives a modest elevation of lymphocytes in the CSF.

__________________________________

A man comes in for evaluation of fever, cough and sputum production.
1- fever is minimal. the chest x ray is normal.
2-there is discolored sputum with hemptysis. CXRAY shows an infiltrate in one lobe.
3-an alcoholic with poor dentition. the sputum is foul smelling.. there is weight loss, with persistent sx over several weeks..
4-immigrant with weigt loss and a cavitary lesion on cxray

1-Bronchitis presents with mild fever, cough, sputum, and a normal chest xray.
2-Pneumococcal pneumonia is the most common cause of community acquired pneumonia. There is discrete infiltrate seen in individual lobes of the lung. Hempptysis is a nonspecific finding. Hemoptysis will not help you answer the Q. anything that makes you cough will give hemoptysis.
3-Lung abscess is an answer when the sx are chronic over several weeks. the sputum smells bad and there is an increased risk of aspiration such as intoxication, seizures, stroke, or intubation. poor teeth predisposes to higher volume of infected material to aspirate.
4-TB is most often in immigrants. there is chronic cough, fever, weight loss and night sweats with cavitary lesion.

_______________________

a pt comes to the ER with fever and a cough for the last several days. the cxray is abnormal with bilateral interstitial
1- HIV positive pt with 110 CD4 cells on no medications. The cough is dry and the LDH is elevated.
2-an 82 year old man with COPD with diarrhea and altered mental status. Sodium is low.
3-pt is a sheep farmer. Bilateral interstitial infiltrates are present.
4-Generally a healthy young person, hemolysis is present.

pneumocystis pneumonia (PCP0 is the answer when the pt is HIV positive with <300 CD4 cells on no prophylactic medications. The LDH is elevated.

2-Legionella pneumonia is associated with GI and CNS abnormalities in elderly pts with a hx of lung disease. The sodium level is often especially low in pts with legionella pneumonia

3-Coxiella brunetii causes Q fever. Coxiella is transmitted from animal exposure.
40 Mycoplasma pneumonia is the answer when the question describes a generally healthy pt with mild sx and interstitial infiltrates. Occasionally there is autoimmune hemolysis from IgM cold agglutinins.
_____________________________
32 year old woman comes to the ER with lower abdominal pain and lower abdominal tenderness. cervical motion tenderness
1-temp is 101F and the white cell count is 16,000, NL 4,500-10,500
2-pregnancy is positive

1-pelvic inflammatory disease is the dx when there is lower abdominal pain and tenderness with cervical motion as well as fever and leukocytosis. A pregnancy test should be done to exclude ectopic pregnancy. cervical cultures and DNA probe for gonorrhea and chlamydia should be performed. the most accurate test is a laproscopy. in most cases think about admission for parenteral antibiotics when febrile.
2-ectopic pregnancy presents with cervical motion tenderness and a positive pregnancy test. A pelvic ultrasound should be performed. if negative than a transvaginal.

__________________________

A man comes to the clinic with a genital ulcer and enlarged inguinal adenopathy
1-ulcer is firm and painless with heaped up and indurated borders.
2-the ulcer is soft and painful.
3-large nodes that are tender is the main finding
4-the ulcer started as vesicles that lost their roof

1-primary syphilis presents as a genital ulcer with adenopathy. the ulcer is firm but painless. the most accurate test is darkfield exam of a scraping. RPR and VDRL are only 75% sensitive in primary syphilis. rx single IM PCN
2-chancroid is soft and painful, specialized culture media are necessary to diagnose Hemophilus ducreyi. rx is a single dose azithromycin.
3- lymphogranuloma venereum presents with matted enlarges lymph nodes. the nodes may develop a draining sinus tract and often tender. dx is with complement fixation testing of a smaple of blood or with aspration of the node.
rx is with doxycyline for three weeks.
4-Herpes simplex begins as a vesicular lesions that may ulcerate. if the dx isn't clear, viral culture confirms the dx.
___________________________________________
a pt comes in with dysuria such as urinary frequency, urgency and burning

1- urethral discharge is present.
2- there are 50 white cells in the urine, suprapubic tenderness is present.
3-fifty white cells are present in the urine. temp is 102F an there is flank tenderness.
4- after rx for pyelonephritis for 7 days, fever, flank tenderness and pyuria persist.


1-urethritis presents with urethral discharge, although a discharge by itself is sufficient to suggest the diagnosis. A urethral swab for gram stain shows gonorrhea. urine for nucleic acid amplification testing is the standard of care. RX is with a single dose of azithromycin and ceftriaxone. Always treat for chlamydia as well, since rate of coinfection is very high.

2-cystitis is suggested by dysuria >5 white cells in a urinalysis, and suprapubic pain. Three days of trimerthoprim/sulfamethoxazole or quinolone is the rx
3-pyelonephritis is diagnoses with dysuria, fever, flank pain and tenderness and white cells in the urine. sonogram or ct of the kidney will sow, possibly hydronephrosis or abcess.
4-perinephric abscess is diagnosed with persistent sx of pyelonephritis despite rx. imaging of the kidney will show a collection of infected materla. Biopsy is the most accurate diagnostic test.
______________________________________
pt comes wth pruritis of his genital area.
1-there is also an itchy rash of the web spaces of his fingers, elbows, and axilla. narrow burrows are visible in web space
2-the itching is limited to hair containing areas of the pubic and axilla. Live organisms are visible near the hair.

Scabies presents with pruritic lesions of the genitals. There are itchy area in the hands, elbows, wrists in the web spaces. Narrow burrows may be visible where the sarcptes scabiei has dug underneath the skin.
2-pediculosis or crabs are much larger than scabies and are visible on the skin surface in hair, an pubic region and the axiilla. scabies and pediculosis are best treated with prematherin.

______________________________________

pt comes in with swollen, red, immobile joints

1- single joint involves in an elderly pts with a hx of arthritis, and there is effusion present.
2-young pt has multiple joints invlved. there are petechiae, rash, and tenosynovitis present. there is pain on moving the fingers and toes.

1-Septic athritis from staph or strp presents with involvement of a single joint. Most often the pt has a hx of underlying joint abnormality such as arthritis. the more deformed the joint is the more likely the pt to have septic arthritis. most accurate test is aspiration of the joint for cell count and culture.
2-disseminated gonorrhea presents with polyarthritis, tenosynovitis and petechia, the most accurate method of establishing a dx is to culture the joint as well as the urethra, cervix pharynx, rectum and blood.

____________________________________________


__________________________________________________ _

diagnostics



skin scraping mineral oil mount is used to aid in the diagnosis of scabies. mineral oil is applied over suspected lesion to enhance the viewing of a burrow. The skin is scraped and viewed under a microscope. The skin cells remain intact, mite feces are preserved, and the mites remain motile and alive in mineral oil.

look for pt with itchy pruritic rash


visible particularly in the web spaces of hands and feet. there may also be a hx of sexual contact and genital or breast involvement as well.

A KOH wet mount is prepared from the skin scrapings and the slide is heated, breaking up the tissue to reveal any mites hiding under a lump of squamous cells.

__________________________________



patch testing is used to determine the allergen responsible for allergic dermatitis after conservative measures like removing the supposed offending agent and a trial of topical therapy fail.

patch testing isn't useful for irritant contact dermatitis because irritant dermatitis is not an immunologically mediated response.

_______________________________________



wood's light examination used a YV light to diagnose certain skin infections caused by yeast, bacteria, or dermatophytes. it is also used to detect pigment changes in the skin. for instance it detects loss of pigment in fair skin individuals affected with a depigmentation disorder such as vitiligo
the following organisms fluoresce

pityrosporum ovale

will fluoresce yellow
corynebacterium minutissimum

will fluoresce bright red.

microsporum canis will fluoresce blue/green like this you get the pic


in the scalp but not on glabrous (no hair) skin
in the U.S tinea capitis is most commonly caused by Trichophyton tonsurans

which doesn't fluoresce under wood's light..

_____________________________________




these are CT scans of the head without contrast for the latter two.
they show intracranial hemorrhage., one with epidural hematoma, the last with a subdural hematoma. the epidural is from an arterial bleed and has higher pressure and more readily pushed in the brain, the blood forms in the shape of a lens. the subdural if a form of venous bleed that forms the shape of a crescent .
CT is always the best initial test for the brain, especially in trauma when looking for blood. the cr is always best for a stroke in which we are looking for blood.
in head trauma and intracranial bleeding such as subdural and epidural hematoma, there is no test more accurate than head ct scan.
_____________________________________



this is an MRI. it is the most accurate test for MS, strokes, lesions in the brain stem and cerebellum.
the more accurate test than an MRI for a mass lesion is a brain biopsy.

___________________________

pharm

54 year old woman comes to the office for eval of sudden overwhelming urge to urinate and pain in her bladder. this happens frequently and results in incontinence. she failed behavior modification for this problem

the best therapies for overactive bladder or 'urge' incontinence is tolterodine, oxybutynin, trospium, darifenacin or solifenacin.
These meds are muscarinic receptor antagonists and have significant antichoolinergic effect.
they are associated with dry mouth, dry eyes and constipation, secondary to their anticholinergic effects.

___________________________________

Varenicline ** ask your doc if this is right for you, if you are a smoker
is a therapy for smoking cessation. it is an adjunct therapy to behavioral modifications, nicotine replacement, and bupropion. It is most useful in people who have signs of nicotine withdrawal. it has greater efficacy than nicotine replacement therapies, such as patches, gum and bupropion.
Varenicline is a nicotinic ACH receptor partial agonist. It blocked nicotine from binding..

______________________________________

bupropion
is an antidepressant that can also be used to help with smoking cessation.
bupropion inhibits the reuptake of norepinephrine and dopamine. it is an aromatic aminoketone that has amphetamine like qualities.
bupropion can cause HTN, insomnia and seizures but does not have adverse side effects like SSRIs and is contraindicated for ppl with eating disorders because it can suppress appetite. it is also contraindicated for those with a seizure disorder and those using MAO inhibitors.

___________________________________

Thrombin inhibitors

Argatroban
Lepirudin
Hirudin
Bivaliridun

direct thrombin inhibitors are used as anticoagulants for pts with clots who have developed heparin induced thrombocytopena and can no longer use haprin. in a pt with heparin drip whose platelet count dropped by half. may also describe a clot or a positive test for plt factor four therefore Argatroban and Lepirudin are the best initial therapy. Bivalirudin and Hirudin can also be used for acute coronary syndromes.
these meds directly inhibit thrombin
common adverse effect is bleeding
they are monitored by checking the activated aPTT.

__________________________________________

32 year old man is admitted with his third painful crisis from Homozygous SCD. his crisis has resolved on discharge
Hydroxyrea is used to decrease the frequency of painful sickle cell crisis. However, hydroxurea does not terminate in an acute crisis.

hydroxurea increases the level of fetal hemoglobin, preventing subsequent crisis. it also increases the amount of water in the red cells preventing sickling.
Hydroxurea causes myelosuppression and leukemia with long term use.
Hydroxyurea is also used to lower cell counts in essential thrombocytopenia and polycythemia vera.
__________________________________________________ __
Reply

جوري
10-10-2008, 03:56 AM
pharm heme
_____________

Angrelide

is a platelet reducing agent.
angarelide is a phospohdiesterase inhibitor

Anagrelide has a single indication as the rx of thrombocytosis. secondary to myeloproliferative disorders. while anagrelide is used to lower the pt count in essential thromocyteopenia (ET) phone home ;D, chronic myelogenous leukemia (SML) and polycythemia vera. it isn't as effective as hydroxyurea for ET and not as effective as imatinib for CML.

_____________________________________

1- a 24 year old woman comes in with epistaxis and petechiae. the plt count is 27,000. she is generally healthy, and her spleen is normal size.
2-a 45 year old man comes i with melena and intracranial bleeding . his plt count is 8000. his bone marrow shows increased megakaryocytes. He has antibodies present against glycoprotein IIb/IIIa receptor.

the pt has idiopathic thrombocytopenic purpura (ITP) she is generally a healthy person with only low plt. there is no splenomegaly. treat with steroids. If the thrombocytopenia is repeatedly recurrent aftet stopping steroids, a splenectomy should be performed.
2-life threatening bleeding with ITP is treated with IVIG or rhogam in combination with steroids. The fastest way to bring up the plt count is with IVIG. this rx is faster than plt transfusion in which the plts will be consumes.. Plasmapheresis isn't helpful.

__________________________________________

plasmapheresis
is the best initial therapy for the following

Guillain Barre syndrome. equal in efficacy as IVIG
Goodpasture's syndrome, in combo with steroids
Myasthenia gravis crisis in which there is an overwhelming svere muscle weakness. IVIG can be used in this case as well
Thrombotic thrombocytopenic purpura TTP
waldenstrom's macroglobulinemia. used to dec hyper viscosity from the high levels of IgM

_______________________
22 year old man comes to the office for increased bleeding after using asa. he has some petechiae on his legs, bleeding time is prolonged. the ristocetin cofactor assay test is abnormal. He is about to undergo a tooth extraction

Desmopressin DDAVP is the best initial therapy for von willebrand's dz. desmopressin is an artificial version of ADH or vasopressin.
Desmopressin works by causing the release of subendothelial stores of von willebrand's factor VIII

Desmopressin can lead to HTN, facial flushing, and headache. Desmopressin can also be used for mild hemophilia and with central diabetes insipidus.

_____________________________________

a woman with factor V leiden mutation develops a PE. she is hemodynamically stable

2- man with rate conteolled Afib is stable at home, he is 68 with dilated left atrium on echo.

1- PE are best treated with heparin; IV-unfractionated heparin and low molecular weight heparin have equal efficacy. warfarin should also be started and heparin continued until warfarin is in therapeutic range.
2- warfarin is continued for 6 months. the duration of therapy doesn't change because of the presence of factor V mutation with the first episode of clot.
IV heparin is monitored with the activated aPTT. LMW heparin is occasionally monitored with factor xa levels

case II

chronic afib is anticoagulated with warfarin to an internatioalized normalized ratio INR of 2-3, there is no need to start heparin before starting warfarin.
warfarin is continued permanently.
warfarin is monitored with INR.

______________________________________

45 year old male GI has afib. he has a heart rate of 60-80 bpm, no htn, dm, previous strokes.. the echo is normal.

afib can be rx with asa alone. this is a long afib asa alone for alone afib. the pt is without accompanying risks, such as dm, htn, previous stokes. and has a normal echo. low risk pts would have al the bleeding risks of the drug without the therapeutic benefit.

____________________________

A pregnant woman develops PE in her 2nd trimester

thromboembolic dz during pregnancy is treated with heparin throughout the pregnancy. because of the possibility of teratogenicity with warfarin. LMW heparin can be administered safely by subcutaneous injections.
heparin works by potentiating the effect of antithrombin on the clotting cascade.

Besides bleeding, heparin is associated with thrombocytopena, long term use can be associated with osteoporosis

_________________

43 year old man comes to the hosp. w back, thigh, and chest pain and weakness, he has a temp of 102F. he has enamia and an elevated bilirubin, LDH, and retic count. fluids, pain meds and o2 have been started,

this pt has sickle cell dz with an acute painful crisis. He is febrile. the most important med for is at this time is antibiotics. Ceftriaxone, levofloxacin, gatifloxacin or moxifloxacin are all appropriate to cover encapsulated organisms such as strep pneumo, klebsiella, salmonella, or hemophilus.

fever in sickle cell pt is life threatening, because the pt is functionally asplenic. do not wait for results. pt will die of sepsis before the results are known.
admin of antibiotics for fever lowers mortality as well hydroxyurea to prevent crisis.

___________________________

Anti-Ach-R antibodies
Myasthenia gravis is characterized by anti ACH receptor antibodies. that are made when autoimmune dz causes antibodies that bind to postsynaptic ach receptors: there is no neuromuscular transmission if there is no receptor on the muscle because it has been eaten away by an antibody.

Anti-Ach-R antibodies are the best test to dx Myasthenia gravis. they are more accurate than tensilon or edrophonium testing.
single fiber electromyography EMG is the most sensitive diagnostic test for MG.

____________________________



carotid doppler is a sonographic eval of the carotid arteries that allows a precise quantification of the degree of stenosis of these vessels. it is used to see whether a pt needs a carotid endarterectomy or angioplasty.
carotid doppler is used on pts who present with signs of ischemia in the anterior circulation of the brain, such as amaurosis fugax of focal extremity weakness. it is the initial test to determine the etiology of embolic strokes or TIAs

the most accurate test for carotid arteries is the angiogram.

_____________________________________
cervical spine x ray


for someone diving and landing on his head or having somethin heavy land on his head. gives increased axial loading.

_________________
ct myelogram


is to detect compression of the spinal cord from mass lesions, such as cancer and occasionally from hematoma or an infection such as epidural abscess.
a spinal needle is introduced into the subarachnoid space. the pt is hung nearly upside down in order to allow contrast material into the subarachnoid space to move and detect the lesion at the place it stops flowing. The MRI has essentially replaced the CT myelogram. though it may has an advantage in detecting leptomeningeal cancer.

MRI will always be the better answer, if you suspect cord compression.
if MRI isn't possible b/c pt can't lie still or has a metal pacemaker and the magnet of the MRI would be dangerous to such a patient.

____________________________

Edrophonium tensilon test

is a very sensitive test for MG. Edrophonium chloride is an ACH esterase inhibitor with rapid onset and short duration of axn. it prolongs the presence of ach in the neuromuscular junction and results in an immediate increae in muscle strength
elecromypgraphy is the single most accurate test to confirm the dx of MG
thymomas are associated with the dx of MG 10% of the time.

____________________

EEG


is the measurement of brain wave patterns by putting electrodes on the scalp.
done in pt with new onset of seizure, also elpful in pts with sudden onset syncope if there is no cardiac etiology identified.

EEG maybe contributory in the dx of Creutzfeldt-Jakob dz. but is isn't as good as a brain biopsy. EEG doesn't necessary determine brain death if the brains tem reflexes are clearly absent.

Sleep deprivation EEG has the greatest sensitivity.. this is the answer is the pt has been seizure free for 2-3 yrs and you want to stop anti epileptic meds.

____________________

EMG


is a test of muscular strength and electrical activity. EMG's are used to dx neuromuscular dz. to distinguish between neural and muscular disorders.
Needle is places in the muscle. The electrical activity during rest and activity is compared. An abnormal pattern or amplitude tells there is dz.
an EMG in the case should make you think of MG or Guillain Barre syndrome and muscular dustrophy.
single fiber EMG is the single most accurate test for MG. combined EMG and nerve conduction velocity testing is the single most accurate test for GBS. EMG combined with muscle biopsy are the most accurate tests for polymyositis.
____________________________

Lumbar puncture


is insertion of a spinal needle into the subarachnoid space to aspirate CSF fluid.
for the following
suspected meningeal infxn
pt with worst headache of her life, whom CT is negative to exclude subarachnoid hemorrhage.
new onset seizures with fever
any infant less than 6 weeks old with fever if other studies are negative,
LP's are contraindicated for mass lesion. do use a head CT before LP when there are focal findings, papilledema, or a new seizure, or when pt is confused as to make the neurological exam inaccurate.

____________________________________

oligoclonal bands



detection of elevated bands of igG (oligoclonal bands) in the csf is possible in a pt presenting with signs and sx of MS. the presence of these bands isn't specific.

look for a young pt with weakness, numbness, unsteady gait, spastic paraparesis, diplopia, or sphincter disturbances (urinary urgency_ exxamining the CSF for oligoclonal bands is recommended only when MRI is nonconfirmatory but your clinical suspicion for MS remains high.

MRI of the brain is the most accurate test to dx MS wit a sensitivity of 90-95%

>>>>>>>>>>>>>>>>>>>>>.
Reply

جوري
10-11-2008, 02:28 AM
Infectious disease
___________________________

a pt comes to the hospital with fever and a murmur

1- 67 year old woman with four months of fever and fatigue. she has an HX of MR
2-a 27 year old injection drug user, the murmur is best at the lower left sternal border.
3- A man whose aortic valve was replaced three weeks ago
4-a pt who was recently diagnosed with diverticulitis and colon cancer.

Virdians group streptococci is the most important organism in subacute bacterial endocarditis. They occur most often in those with underlying valvular disease.
2-staphylococcus aureus is the most common cause of endocarditis in the injection drug user. this is often MRSA as well Oxacillin resistant. injection drug users often have involvement of the right side of the heart such as the tricuspid.
3-Staphylococcus epidermidis and other coagulase negative staphylococci are the most common cause of endocarditis when a heart valve has been replaced. This is presumably from seeding of the valve during surgery.

4-Staphylococcus bovis is most often associated with endocarditis in those with evidence of colonic pathology.

_____________________________________________

an HIV positive man with 25 CD4 cells comes in for blurry vision x a few days. He is not on any HIV meds

CMV retinitis occurs exclusively in pts with CD4 counts under 100. CMV presents with blurry vision.
2-Dilated ophthalmologic examination is the best initial method of diagnosing CMV retinitis. It is basically diagnosed on how it looks. CMV antibody testing in the blood has no value. It is a clinical dx based on direct retinal visualization.

Ganciclovir, foscarnet or valganciclovir is the standard of care in rx.
__________________________________________________ __

pt is admitted to the ICU because of severe metabolic acidosis. The serum bicarbonate is low at 14. the pt is disoriented and can't offer an adequate hx. no records are available.

1- fever, hypotension, tachycardia, and an elevated white count
2- hyperglycemia and hyperkalemia
3-oxalate crystals in the urine with a low serum calcium
4-elevated CR.
5-Normal anion gap

fever, hypotension, leukocytosis, and tachycardia imply the presence of sepsis as a cause of metabolic acidosis. The first step in evaluation of an metabolic acidosis is the eval of the anion gap. An anion gap (na+ minus Cl- and Hco3-) that is >12 is consistent with lactic acidosis, salicylate OD, Methanol, uremia, diabetic ketoacidosis, and ethylene glycol OD.
2-Diabetic ketoacidosis (DKA) gives hyperglycemia and hyperkalemia, although the total body levels of potassium are depleted.
3-Ethylene Glycol OD results in oxalate crystals in the urine. The formation of calcium oxalate crystals lowers the calcium level. look for the term envelop shapes crystals
4-Renal failure causes metabolic acidosis because of the kidneys inability to excrete acid.
5-normal anion gap implies either RTA or diarrhea, the urine anion gap is positive. With diarrhea, the urine anion gap is strongly negative. The lower the urone anion gap number, the greater the kidney's ability to excrete acid.

___________________________________

A man is admitted to the hospital with renal failure developing over a few days
his cr has risen from 0.8mg/dl to 2.5 mg/dl. His bun has risen even more, going from 15 to 54. his serum bicarbonate is slightly low. the urine sodium is low and the urine osmolality is high..

Blood pressure 92/56 and pulse is 124.
2-serum albumin is 2.2 and the PT is elevated. there is splenomegaly
3-Ha has an ejection fraction of 24% with edema. a diuretic was recently started.
4-a bruit is present at the flanks and he has just started an ACE inhibitor.


will take care of the rest of this tomorrow insha'Allah.. am feeling a bit tired today..

:w:
Reply

جوري
10-13-2008, 04:42 AM
picking up where we left off

A man is admitted to the hospital with renal failure developing over a few days
his cr has risen from 0.8mg/dl to 2.5 mg/dl. His bun has risen even more, going from 15 to 54. his serum bicarbonate is slightly low. the urine sodium is low and the urine osmolality is high..

1-Blood pressure 92/56 and pulse is 124.
2-serum albumin is 2.2 and the PT is elevated. there is splenomegaly
3-Ha has an ejection fraction of 24% with edema. a diuretic was recently started.
4-a bruit is present at the flanks and he has just started an ACE inhibitor.



1- pre-renal azoremia from any cause leads to an elevation of the BUN and CR, wit the BUN rising more than the CR in a ratio greater than 15:1. The tachycardia and hypotension in the first case suggest hypovolemia or any other form of chock. FeNa <1% also indicated a pre-renal etiology.

2- low oncotic pressure for any reason results in pre-renal azotemia because of decreased renal perfusion. In addition, liver dz such as cirrhosis can lead to hepatorenal syndrome, which is renal failure entirely based on liver failure.
3-CHF from any cause leads to rpe-renal azotemia. It can become suddenly worse with the volume depletion from a diuretic. pre-renal azotemia leads to low urine sodium and high urine osmolality
4- Renal artery stenosis is associated with decreased renal perfusion. ACE inhibitors can precipitate acute renal failure. Think about fibromuscular dysplasia in a young woman.

_____________________________________________

pt with worsening renal function over the past few days. His cr is 3.5md/dL and the BUN is 28 units. The urine Sodium is 45meg/L and the urine osmolality is 290 mosm/L. His serum bicarbonate is low.

1- pt has been on gentamicin for last 8 days
2-he was on pipercillin for a few days. but stopped yesterday. He has fever and rash and there are eosinophils in his urine.
3-Chemotherapy for lymphoma was started 2 days ago
4- there is an empty bottle of anti-freeze by his bedside..

1-aminoglycoside induced RF generally occurs after 5-10 days of exposure to the medication. As with all forms of acute tubular necrosis, the BUN and CR will rise in about 10:1 ratio.
the urine sodium will be high (>40) and the urine osmolality will be low (<350) because of the inability of the damaged kidney tubules to concentrate urine . Amphotercin and any other renal toxic meds will result in the same numbers.
2- Allergic interstitial nephritis presents with fever, rash and eosinophils in the urine. The presence of esosinophils in the urine is more frequently than in the blood.
3- Hyperuricemia from tumor lysis syndrome will lead to acute RF
4-Antifreeze contains ethylene glycol, which leads to acute RF from oxalic acid accumulation in the kidney tubule. Look for 'envelope shaped oxalate crystals' in the urine. Formic acid accumulates with methanol ingestion and causes blindness.
____________________________________________

a man comes to the ER dept after sustaining a prolonged seizure. He has dark urine which is strongly positive on the dipstick for blood but in which no red cells are seen on microscopic exam. His serum bicarb level is low.

Rhabomyolysis presents after crush injury or severe exertion of any kind with dark urine in the absence of visible red cells. This is indicative of urine myoglobin. Rhbdomyolysis leads to metabolic acidosis, hyperkalemia and eventually renal failure.
2- Urine myoglobin is the most specific diagnostic test for rhbadomyolysis. The potassium level and EKG are probably the most urgent diagnostic steps because they determine who is most likely to die. the CPK level will be significantly elevated. admin of IV fluids and alkalinazation of the urine are important. an elevated CPK is not specific ofor indicating the cause of renal failure.
__________________________________________________

pt with confusion. His sodium is at a low 122 mEq/L he has no edema, clear lungs, and no JVD. There is no orthostasis.

1- pt has lung ca with mets to brain. Urine Sodium is 70 (high) and urine osmolarity is 450 high.
2-the pt is bipolar, with frequent urination all day and less at night. urine sodium is 10 low and urine osmolarity is 75 low.
3- the pt has diabetes with a glucose level of 850 (NL 80-110)
here are some lab values for reference

Albumin 3.2 - 5 g/dl
Alkaline phosphatase (Adults: 25-60) 33 - 131 IU/L
Adults > 61 yo: 51 - 153 IU/L
Ammonia 20 - 70 mcg/dl
Bilirubin, direct 0 - 0.3 mg/dl
Bilirubin, total 0.1 - 1.2 mg/dl
Blood Gases

Arterial Venous
pH 7.35 - 7.45 7.32 - 7.42
pCO2 35 - 45 38 - 52
pO2 70 - 100 28 - 48
HCO3 19 - 25 19 - 25
O2 Sat % 90 - 95 40 - 70
BUN 7 - 20 mg/dl
Complete blood count (CBC) Adults

Male
Female

Hemoglobin (g/dl) 13.5 - 16.5 12.0 - 15.0
Hematocrit (%) 41 - 50 36 - 44
RBC's ( x 106 /ml) 4.5 - 5.5
4.0 - 4.9

RDW (RBC distribution width) < 14.5
MCV 80 - 100
MCH 26 - 34
MCHC % 31 - 37
Platelet count 100,000 to 450,000
Creatinine kinase (CK) isoenzymes
CK-BB 0%
CK-MB (cardiac) 0 - 3.9%
CK-MM 96 - 100%
Creatine phosphokinase (CPK) 8 - 150 IU/L
Creatinine (mg/dl) 0.5 - 1.4
Electrolytes

Calcium 8.8 - 10.3 mg/dL
Calcium, ionized 2.24 - 2.46 meq/L
Chloride 95 - 107 mEq/L
Magnesium 1.6 - 2.4 mEq/L
Phosphate 2.5 - 4.5 mg/dL
Potassium 3.5 - 5.2 mEq/L
Sodium 135 - 147 mEq/L
Ferritin (ng/ml) 13 - 300
Folate (ng/dl) 3.6 - 20
Glucose, fasting (mg/dl) 60 - 110
Glucose (2 hours postprandial) (mg/dl) Up to 140
Hemoglobin A1c 6-8
Iron (mcg/dl) 65 - 150
Lactic acid (meq/L) 0.7 - 2.1
LDH (lactic dehydrogenase) 56 - 194 IU/L
Lipoproteins and triglycerides

Cholesterol, total < 200 mg/dl
HDL cholesterol 30 - 70 mg/dl
LDL cholesterol 65 - 180 mg/dl
Triglycerides 45 - 155 mg/dl (< 160)
Osmolality 289 - 308 mOsm/kg
SGOT (AST) < 35 IU/L (20-48)
SGPT (ALT) <35 IU/L
Thyroid Function tests

Free T3 2.3-4.2 pg/ml
Serum T3 70-200 ng/dl
Free T4 0.5-2.1 ng/dl
Serum T4 4.0-12.0 mcg/dl
TSH 0.25-4.30 microunits/ml
Total iron binding capacity (TIBC) 250 - 420 mcg/dl
Transferrin > 200 mg/dl
Uric acid (male) 2.0 - 8.0 mg/dl
(female) 2.0 - 7.5 mg/dl
WBC + differential

WBC (cells/ml) 4,500 - 10,000
Segmented neutrophils 54 - 62%
Band forms 3 - 5% (above 8% indicates left shift)
Basophils 0 - 1 (0 - 0.75%)
Eosinophils 0 - 3 (1 - 3%)
lymphocytes 24 - 44 (25 - 33%)
Monocytes 3 - 6 (3 - 7%)
Laboratory test
Values if prerenal cause of acute renal failure
Values if intrarenal cause of acute renal failure

FENa, percent*
<1
>1

BUN to creatinine ratio
>20:1
10 to 20:1

Urine specific gravity
>1.020
1.010 to 1.020

Urine osmolality, mOsm per kg
>500
300 to 500

Urine sodium concentration, mEq per L (mmol per L)
<10 (10)
>20 (20)

Urine sediment
Hyaline casts
Granular casts

FENa = fractional excretion of sodium; BUN = blood urea nitrogen.

*-FENa is calculated as follows:

FENa =
Urine sodium ÷ plasma sodium X 100

http://www.globalrph.com/labs.htm

SIADH is caused by an abnormality of the brain or the lungs. This can be a cancer, infarction or infection. SIADH is associated with an inappropriately high urine sodium and osmolarity. The normal response to a low serum sodium should be a low urine sodium and low urine osmolarity. SIADH is a case of euvolemic hyponatremia. Free water restriction is the treatment .

2-psycogenic polydipsia is associated with bipolar disorder. there is a normal urinary response to hyponatremia. The normal response is low urine sodium and osmolality. decrease in sx at night is the key to the dx. when he goes to sleep he stops drinking, so he stops urinating.
3- pseudohyponatremia is from an elavted glucose for any reason. For every increase in glucose of 100 above normal there is a 1.6pt decrease in sodium.

___________________________________
Reply

جوري
10-15-2008, 01:39 AM
on with nephrology
______________________

on routine screening, a pt is found to have low sodium of 127. He has no SX of the hyponatremia, and the neurologic exam is normal

1- pt has CHF with peripheral edema
2-he has 7g of protein every 24hrs and the serum albumin is 2.4 (normal is 3.5-5.5)
3- the potassium level is elevated at 6.2mEq/L (normal (3.5-5.2) and there is mild metabolic acidosis

1- CHF results in hyponatremia because of a decreased intravscular volume. The same effect occurs in cirrhotic pts. This is an appropriate increase in ADH because of decreased intravascular volume.

2-Nephrotic Syndrome results in hyponatremia because of a decrease in intravascular volume from low oncotic pressure. Nephrotic syndrome here is the most likely DX because of the low serum albumin levels as well as the marked increase in protein in the urine.

3-Addison's disease or hypoadrenalism of any cause results in hyponatremia. the loss of aldosterone results in th urinary loss of sodium and the retention of both potassium and hydrogen ions.

__________________________________________________ _______


a pt with severe hypernatremia is admitted to the ICU for confusion, there is polyuria despite increase in serum sodium. The pt is dehydrated.

1- the urine volume markedly decreases in response to the admin of vasopressin
2- there is no response to the adminstration of vasopressin. The urine volume remains high and the urine osmolality remains low.
3- the pt has diabetes and the glucose level is markedly elevated, but the serum bicarbonate is normal


1- Central diabetes inspidus is an insufficiency of antiduiuretic hormone ADH due to damage to either hypthalamus or posterior pituitary. There is a marked response in urine volume to the administration of vasopressin

2- with nehrogenic diabetes inspidus, there is no response to the admin of ADH. NDI is often from hypokalemia or hypercalcemia. There may a HX of lithium admin

3-Nephrotic hyperosmolar coma results in severe hypernatremia when there is a marked osmotic diuresis from hyperglycemia.

_____________________________________________

A patient is seen because of Muscular weakness. There is also an elevated serum bicarbonate of 30. The potassium level is decreased at 2.9

1-vomiting is severe
2-the pt is on a loop diuretic because of CHF


vomiting from any cause can cause hypokalemia. This is because the metabolic alkalosis from vomiting causes a transcellular shift of potassium intracellularly. This is also because the loss of chloride from the stomach leads to an increase in bicarbonate resorption from the kidney. This state is hypochloremic hypokalemic metabolic alkalosis.

2- Diuretics cause hypokalemia because the volume depeltion leads to increased aldosterone secretion. All volume contractions lead to metabolic alkalosis by this mechanism. All cases of hypokalemia result in Muscular weakness.

___________________________________________

A man has mild proteinurea found on routine urinalysis

1- he is a healthy athlete undergoing intensive physical training.
2-he is a waiter. when split 24 hour urine is measure, the morning urine has no protein but the afternoon urine shows trace proteinuria
3-He is generally healthy and the repeat urinalysis shows no protein

Mild proteinuria can be found in healthy young athletes undergoing physical training. this is a benign finding and needs no further testing.

2-orthostatic proteinuria can occur in those who stand up all day long. when urine is split into morning and evening protein measurement. there is more in the first 8 hrs of the day. this is benign

3-between 1 and 10% of the population can have transient mild proteinuria. if protein is not found on repeat testing it needs no further follow up. if it persists, a 24 hour urine measurement of protein/ creatinine ratio is performed. Only if the proteinuria is persistent or the ratio is elevated should a renal biopsy be performed.

___________________________________________

a woman is in your clinic because of edema developing over months. she has a normal echo. her urinalysis shows 4+ protein and the spot protein/creatinine ratio is 7:1. triglycerides are elevated.

1- there is a hx of diabetes and htn. the eyes show background retinopathy.
2-she has been an injection drug user of heroin in the past.
3-she was recently diagnoses with lymphoma

Nephrogenic syndrome is a combination of edema, a 24 hr urine protein greater than 3.5g and hyperlipidemia. A spot protein/creatinine ratio greater than 3.5 is the same as a 24 hour urine protein. Diabetes and HTN are the most common causes of nephrotic syndrome. A ratio of protein to creatinine is equal of the amount found on a 24 hr urine.

2- injection drug use and heroin both cause focal segmental glomrulonephritis. HIV is also associated with focal/segmental disease.

3-the most common cause of nephrotic syndrome is a primary disease limited to the kidney is membranous glomerulonephritis. Membranous glomerulonephritis is also associated with cancer such as lymphoma.

______________________________________________

A man comes to see you because of persistent hematuria. The urinalysis shows red-cell casts and a 1+ proteinuria. The urine sodium is low.

1- the pt is asian with recent viral infxn. there are no systemic manifestations.

2- he has had lifelong eye probs and ear probs with deafness.
3- he had pharyngitis a week ago and has periorbital edema
4-he has multiple systemic probs such as petechiae, joint pain, abdominal pain, and GI bleeding. There is neuropathy and lung involvement.

1- IgA nephropathy, or Bergers disease presents as isolated hematuria at the same time as a viral illness. It is more common in Asians. and is the most common cause of acute glomerulonephritis.

2- Alport's syndrome presents with glomerulonephritis in association with eye and ear probs. Such as deafness. All forms of glomerulonephritis give red cell casts and mild proteinuria.

3- poststreptococcal glomerulonephritis leads to tea or cola colored urine which is proteinuria and hematuria. perorbital edema is characteristic. The blood will show anti-streptolysin O antibodies as a sign of strep infection,

4-Polyarteritis nodosa (PAN) presents as a systemic vascuilitis with skin, joint, GI, CNS and neurologic probs. PAN spares the lung.

_____________________________________________

a pt is in your office for evaluation of blood in his urine x a few days

1- he has burning on urination and must urinate frequently
2- he also has pain going from his sides to his groin. the pain is extremely severe.
3- Red cell casts and protein are found in the urine as well. urine sodium is low.
4- has recently undergone chemo

1- Urinary tract infxn of any kind such as cystitis or pyelonephritis, can lead to hematuria. Definitive diagnosis on the location of the pain described in the question. urinalysis and urine culture should still be obtained.
2-Nephrolithiasis, or kidney stones, present with severe flank pain radiating to the groin, also known as renal colic.
3-Glomerulonephritis of any kind can present with hematuria. when red cell casts. red cells and mild proteinuria are present the most likel diagnosis is glomerulonephritis. the urine sodium is low because of vasoconstriction of te afferent arterial, which is present in all forms of glomerulonephritis.
4- cyclophosphamide leads to hemorrhagic cystitis.

____________________________________

a pt comes with hematuria, joint pains and purpuric skin lesions. urinalysis reveals red cells, red cell casts, and mild proteinuria. the spot protein ratio is 1.2

1- hx of hep c and an IgM present in the blood
2- a child with abdominal pain

Cryoglobulinemia is most often associated with chronic hep c. Cryoglobulinemia leads to renal dysfunction. skin lesions and joint pains. Neuropathy is common, both Cryoglobulinemia and cold agglutin disease are from IgM antibodies in blood. Cold agglutinin disease however, leads to hemolysis, not renal dysfunction, an is associated with mycoplasma

3-Henoch-Schönlein purpura, is the most likely dx when the pt is an adolescent or a child. presenting with GI sx in a combination with renal, joint and skin findings. Palpable purpura of the lower extremities is key. The most accurate test is a skin biopsy with IgA deposited in the skin, but routine biopsy is not necessary.

_____________________________________________

a 27 year old woman comes because of hematuria and flank pain as well left lower quadrant abdominal pain with diverticuli found on colonoscopy. Auscultation shows a mid systolic click. There are cysts found on the ovary as well as the liver

polycystic kidney disease presents with hematuria and can present with kidney stones that occur with increased frequency. In addition to kidney disease, there are also cysts of the liver and ovary with diverticulosis, mitral valve prolpase, and aneurysmal disease in the circle of willis.

The most common cause of death from kidney disease is renal failure. renal failure occurs from chronic repeated infections such as pyelonephritis. in addition there are recurrent kidney stones secondary to significant anatomic abnromalities. Aneurysms rupture is not the most common cause of death in polycystic kidney disease.

_______________________________________________

a patient comes in with sudden onset of flank pain and hematuria
1- there is a hx of sickle cell disease. the patient has taken extra doses of multiple pain meds, including NSAIDS. there is some necrotic material in the urine.
2-the pain radiates to the groin in an otherwise healthy person.

1-papillary necrosis occurs in pts who have underlying kidney disease such as sickle cell disease or chronic pyelonephritis. The presentation is similar to nephrolithiasis in that there is sudden flank pain and hematurua. However, it often occurs from the use of extra NSAID medications and is associated with necrotic material in the urine. The most accurate test is a CT scan. There is no specific therapy.

2-Nephrolithiasis presents with sudden flank pain radiating to the groin. the most accurate test is a spital CT scan. kidney stones do not need contrast material to be visible on x rays of the abdomen have poor sensitivity. the IVP is always wrong, it is slow and associated with potential for renal insufficiency and allergy from contrast.

_________________________________________

Man comes to the office, and is found to have casts in his urinalysis.
1- white cell casts
2-red cell casts
3-eosinophil casts
4-hyaline casts
5-muddy brown or granular casts

pyelonephritis is associated with white cell casts, if they are there , they are specific for the disease, generally, casts ass little to help the diagnosis, which is usually obvious in the presence of fever and dysuria, and flanl pain and tenderness.

2- red cell casts are specific to glomerulonephritis.

3- Eosinophil casts are specific to allergic interstitial nephritis. they are not present as often as individual eosinophils.
4-Hyaline casts are found on dehydration or any other form of pre-renal azotemia. They are the accumulation of normal protein which sludges because of decreased renal flow.
5-granular or muddy brown cast are found in acute tubular necrosis, the 'granules' are sloughed off, necrotic epithelial cells from the kidney tubules..
_________________________________________________

Neurology

a pt comes to the office for evaluation of headache
1- woman with unilateral headache that is throbbing at the time of menses. she is nauseated and sees bright flashes of light. Light hurts her eyes and sounds are painful.
2- bilateral squeezing pain like a belt tied around her head
3- a man with unilateral tearing and redness and nasal stuffiness, there are several short headaches..

1- migraine headaches are common, but not always unilateral with autonomic probs such as nausea and vomiting. Visual problems such as bright flashing lights, zigzags of lights or visual field defects occur. there may be photophobia and phonophobia, migraines can be precipitated by mensuration, physical or emotional stress or lack of sleep.

2- tension headaches are bilateral and bandlike, there are no associated neurologic probs.

3- cluster headache are ten times more common in men, there are multiple short hedaches in a limited period of time, they are very severe with redness of the eye, lacrimation, rhinorrhea and nasal stuffiness. Horner syndrome sometimes occurs...

______________________________________

Man comes with severe facial pain that occured while his wife was gently stroking his face. the pain is extremely severe, started at one side of his face, and is like a nail being driven in his cheek..

trigeminal neuralgia or 'tic douloureux' is an idiopathic disorder of the fifth cranial nerve. there is sudden severe pain of the face brough on by touch, chewing or movement. the pain is lancinating and unilateral.

2- trigeminal neuralgia is treated with carbamezepine. if medical therapy is not effective, surgical resection of the nerve may be necessary.

__________________________________________

an elderly man is brought to the er with a sudden onset of weakness over the right side of his body, dysarthria, and loss of his right visual field. his head CT scan is normal

1- the sx began with unilateral loss of vision on the left side. All sx resolve within 6 hrs. MRI os the head is normal

2- sx persist. MRI of the head is normal in 24 hrs

TIA begin with loss of sensory and motor function that resolves in <24 hrs. All imaging studies are normal, TIAs often begin with 'Amurosis fugax" which is a transient loss of vision. The visual loss is on the contralteral side from other sensory and motor loss. this is from a carotid embolus on the same side as the visual loss.

2-stroke is a permanent neurologic loss, often from a non hemorrhagic embolic or thrombotic episode of the middle cereberal artery. There is loss of motor ad sensory function on the opposite side from the lesion. This is frequently accompanied by a 'homonymous hemianopsia' which is the loss of the optic radiation of fibers through the parietal lobe. A stroke on the left eliminates the visual field on the right. pts look toward the side of the lesion.

_______________________________

a man presents to the ER with severe vertigo, he is found to have hemifacial anesthesia, dysarthria, dysphagia, and sensory loss of his body on the opposite side from the hemifacial anesthesia. He is ataxic and there is Horner's syndrome present.

this is Wallenberg or lateral medullary syndrome is a stoke of the posterior inferior cerbellar artery (PICA). this results in ipsilateral facial sensory loss, contralateral body sensory loss , vertigo, atazia, dysarthria, dysphagia, and horner's syndrome.

2-MRI of the brain is the most accurate way to assess the cerebellum and brain stem. CT scanning does not effectively look at the posterior fossa or the brain stem.

__________________________________________________ ___

a pt comes in with sudden onset of weakness. the weakness is unilateral and is worse in the lower extremity compared to the arm. Sensory loss is also present that is worse in the leg, he is confused and there is urinary incontinence.

Anterior cerebral artery stroke presents with unilateral loss of motor and sensory function. These sx are worse in the lower extremity compared to the upper extremity. There is also confusion and urinary incontinence.

MRI of the brain is the msot accurate method of determining the presense of a stroke. Echo and carotis doppler studies are used to determine the etiology of the origin of the stroke, specifically looking for evidence of vegetation or intra cardiac thrombus.

_______________________>>>>>>>>>>>>>>>>___________ ___
Reply

جوري
10-16-2008, 06:18 AM
Neurology
____

A man comes to the ER with sudden, extremely severe headache, the first such episode he has ever had

1- there is photphobia and neck stiffness, fever, and a loss of consciousness from which he recovers.

2- he has unilateral loss of vision which persists.

1- subarachnoid hemorrhage (SAH) results in a sudden severe headache with meningeal signs such as nuchal rigidity, fever and photphobia. The two key features is the most likely dx would be the loss of consciousness in 50% of ts. CT scan without contrast is 95% sensitive in detection of SAH. Lumbar puncture will detect the rest, showing blood and/or xanthochromia the yellow appearance of cerebrospinal fluid (which envelops the brain) (this headache is described as a thunderclap headache)

2- temporal arteritis leads to severe unilateral headache associated with loss of vision as well as tenderness of the scalp and the artery. The answer is always to give steroids rather than wait for a temporal artery biopsy, there maybe Jaw claudication and onset is in the elderly.
_____________________________________________

a woman comes in with severe back pain
1- hx of cancer, spine tenderness, hyperreflexia, urinary incontinence, los of sensation in the lower extremities.
2-no tenderness and no focal neurologic defecits.
3- spinal tenderness, leukocytosis, and fever.

1- Spinal cord compression from metastatic disease is thought to be present when back pain is accompanied by tenderness, hyperreflexia, sensory loss below the level of the comrpession, and sometimes urinary or fecal incontinence, steroids are critical to prevent worsening sx.
2- low back pain or lumbosacral strain has no accompanying focal neurologic probs. the straight leg raise may elicit pain suggesting disc herniation. This doesn't chanfe the answer for initial management, which is to give analgesics and not perform routine imagine testing. Do not advise of bed rest.
3-Spinal epidural abscess is the answer when there is fever, leukocytosis, and spinal tenderness. Imaging such as an MRI should be performed if there is spine tenderness. which suggests compressive mass.
_____________________________________

a child is brought for eval of mental subnormality and seizures

1- there is a port wine stain on the face and leptomeningeal angiomas.
2-facial adenoma sebaceum, renal lesions, and 'Shagreen patches' are present, with leathery plaques of subepidermal fibrosis, usually situated on the trunk. retinal hamratomas are present, pale hypopigmented 'ash-leaf' patches are present.

1- Sturge-weber syndrome presents with seizures and mental subnormality in association with port wine stain and leptomeningeal angiomas.



2-Tuberous sclerosis gives hamartomas of the retina in association with ash leaf hypopigmented areas. there are also lesions of the heart and kidneys. Adenoma sebaceum is reddened nodules on the face.



_________________________

a pt comes in with loss of pain and temp sensation of the lower extremities
1- the loss of pain and temp is bilateral, there is also loss of bilateral motor function. There is striking sparing of position and vibratory sensation bilaterally.
2-a knife wound is sustained to the back. the loss of pain and temp is on the opposite side from the injury. there is loss of position and vibratory sensation of the same side as the injury

1- Anterior spinal artery infarction results in the bilateral loss of all pain, temp and motor function below the level of the infarction. There is sticking preservation of position and vibratory sensation which has another vascular supply on the posterior portion of the spinal cord..


2-Brown Sequard syndrome is a hemisection of the spinal cord. Pain and temp are not lost on the opposite side from the lesion. Position and sense are lost on the same side as the injury.


___________________________________

a pt comes in some time after being involved in a motor vehicle accident, there was spine trauma the patient has lost pain and temp sensation in a capelike distribution across the neck and down both arms. Touch, position and vibratory sensation are intact. Over time, there is motor loss below the level of the injury

1- syringomyelia presents with the loss of pain and temp in a capelike distribution across the neck and arms. There is sparing of tactile sensation, position and vibratory sense. Reflexes are lost. There maybe lower motor neuron manifestation at the lesion with upper motor neuron signs below the lesion enlarges. Syringomyelia is caused by tumor and trauma.
MRI is the most accurate test.
Surgery is the rx.
_________________________________________

an obese young woman comes in for an eval of severe headache and double vision. she has recently started OCP. on PE she has six cranial nerve palsy and papilledema. Head CT is normal

Pseudotumor cerebri is an idiopathic increase in intracrnial pressure that occurs more often in obese women who are using OCP or teracycline antibiotics. the key to the answer is the presence of a headache in association with diplopia, papilledema, sixth cranial nerve palsy and a normal CT.
Lumbar puncture is the most accurate diagnostic test.
treatment is with the loss of weight combined with actazolamide and diuretics. Steroids and surgical shunting are sometimes necessary.
____________________________________________

pt with multiple bruises on her legs. she is accompanied by her husband. whom she insists is kicking her every night. he denies this, he does say his legs are uncomfortable at night and that this discomfort is relieved by moving his legs. His legs feel 'creepy crawly'. he tries to avoid the prob by drinking cofee.

restless leg syndrome is an idiopathic disorder of discomfort in the legs at night relieved only by movement. it is worsened by restlessness and caffeine use.
Dopamine agonists such as ropinirole or pramipexole are the rx of choice.

_______________________________

A man comes to office for progressive muscular weakness. the weakness is diffuse and accompanied by dysarthria and difficulty chewing and handling saliva, with a decreased gag reflex. there is spasticity, hyperreflexia, muscle wasting and fasciculations

Amytrophic lateral sclerosis (ALS) is the only disease to combine uppr and low motor neuron dysfunction. Pts come with progressive motor weakness, dysarthria, dysphagia, loss of gag reflex and difficulty handling oral secretions. Only yhe motor system is affected. Upper motor findings are sasticity and hyperreflexia. Lower motor neuron findings are wasting and fasciulations. Mental function remains intact.
Death results in inability to handle oral secretions and recurrent pneumonia progressive respiratory insufficiency etc etc..
>>>>>>>>>>>>>>>>>>>>>>>>>>
Reply

جوري
10-17-2008, 06:01 AM
A man comes to the ER with a seizure Head CT scan shows a 'ring' or contrast enhancing lesion. there is surrounding edea and modest mass effect..

1- HIV negative Pt.
2-HIV positive pt. CD 4 count is <100. the repeat CT shows a smaller lesion after two weeks of pyrumethamine and sulfadiazine
3-HIV positive pt. CD4 count is <100. the repeat CT shows the lesion is unchanged after two weeks of Pyrimethamine and sulfadiazine..

1- Ring or contrast enhancing lesions can be either neoplastic or infections. in HIV negative a brain biopsy must be preformed to confirm the dx.. there is no clear way to determine a precise histological type without a biopsy.

2- toxoplasmosis occurs in HIV positive pts with <100 CD4 cells. the response to RX wih pyrimethamine and sulfadiazine for two weeks is sufficiently specific to confirm the dx.

3- lymphoma presents as a constant contrast enhancing lesion in HIV positive pts with <100 CD4 cells. There will be no response to therapy for toxoplasmosis. A brain biopsy must be performed to confirm.

_______________________________________________

A man comes to the office for eval of a tremor

1- the tremor is in the hands and occurs both at rest and when he is moving them. it is worse with caffeine. An alcoholic drink improves it.

2- he is an older pt. the tremor is only at rest and does not occur when he is reaching for an object.

3- there is no tremor at rest. When he reaches for something. his hands wobble considerably


1- Benign essential tremor occurs both at rest and when reaching for objects. caffeine and beta agonists make it worse. Alcohol improves the treatment. treatment with propranolol.

2- parkinsonian tremor occurs at rest and isn't present on intention such as when reaching for objects.

3- cerebellar disorders such as a stroke result in a tremor only when reaching for things. This is similar to an abnromal finger to nose test. There is no tremor at rest.

_______________________________

A man is being evaluated for dementia. He has poor short term memory

1- He has parkinsonian features such as tremor, rigidity, and gait abnormalities in addition to the dementia.
2-The dementia has been rapidly progressive over several months. He has myoclonus.
3- there is gait ataxia and urinary incontinence.
4-Social inappropriateness and emotional lability preceded the loss of memory.

1-Lewy body dementia is accompanied by features of the movement disorder of parkinson's disease.
2-Creutzfeldt-Jakob disease is characterized by rapidly progressive dementia and myoclonus jerks.
3-Normal pressure hydrocephalus is the triad of dementia, gair ataxia and urinary incontinence.
4-Frontotemporal dementia or pick's disease, starts with abnormalities of social appropriateness and emotional lability prior to the loss of memory. There is inappropriate anger, laughing or crying. MRI of the brain shows focal 'lobar' atrophy of the brain. Alzheimer's disease is slowely progressive loss of memory with no focal neurologic abnormalities.

__________________________________

A man in his 30's is brought by his fam for cognitive abnormalities. He has developed progressively worsening emotional outbursts such as anger, depression, and paranoia. There is a profound movement disorder similar to chorea. He is now showing memory loss

Huntington's disease consists of personality changes such as emotional instability, paranoia and depression combined with a movement disorder and dementia. the disease is AD
the diagnostic test is for a specific DNA sequence abnormality consisting of CAG trinucleotide repeat sequences.
there is no therapy.
________________________________

A man is brought to the ER for weakness. the weakness begins in his feet and has progressed to bilateral severe weakness of both legs. Knee jerk and ankle reflexes are absent. He recently has an episode of gastroenteritis

Guillain Barre syndrome consists of ascending weakness that progresses from the feet upward. The weakness may involve the diaphragm, at which time the respiratory failure develops. Deep tendon reflexes are lost in an ascending fashion. There is an association of Guillain Barre after an episode of campylobacter gastroenteritis.
the diagnosis is most accurately confirmed with nerve conduction studies which sow a decrease in conduction velocity. CSF shows an elevated protein with no cells. PFT test are crucial to determine who is most likely to develop respiratory paralysis.
treatment is with IV immunoglobulins or plasmophoresis.
________________________________

an alcoholic man is brought to the er dept with confusion, agitation. on exam there is paralysis of the extraocular muscles and gait ataxia

Wernicke's encephalopathy is characterized by the development of confusion, gaze plasies, and nystagmus as well as ataxia of the gait. It is most commonly found in Alcoholic pts. It is caused by a thiamine deficiency.
rx is admin of thaimine
______________________________

72 year old man is evaluated in the office for rigidity, tremor, micrographia and hypomimia (reduced degree of facial expression)
1- orthostatic hypotension is the most significant abnormality
2- vertical gaze plasy is striking
3-Ataxia such as an abnromal heel to shin and finger to nose tests is the chief complaint.

1- Shy-drager syndrome: parkinson's disease with orthostatic hyptonsion as the main finding.
2- supranuclear palsy: vertical gaze plasy is the most important feature
3-olivopontocerebellar atrophy: Ataxia is the main feature
_____________________

A man with metastatic prostate ca. comes in for eval of pain and motor weakness of the lower extremities. There is bilateral leg weakness and sensory neuropathy. "saddle" anesthesia or loss of sensation in the perineal area is striking. Bowel and bladder abnormalities are present

cauda equina compression is a peripheral nerve injury that presents with urinary retention, saddle anesthesia and progressive leg weakness. Saddle anesthesia is numbness in the perineum, genitals, buttocks, and upper inner thigs. Urinary retention with over flow incontinence may occur. Anal sphincter tone is decreased in 60-80% of pts.

MRI is the most accurate diagnostic test. Surgical resection of the compressive lesion should be ASAP ..
____________________________

A chronic smoker comes in with an abnromal X-ray with a lesion in the superior sulcus. On PE, he has dropping of his eyelife on one side. the pupils are unequal in size (anisocoria) the pupil remains constricted in dark light. He doesn't sweat on one side of the face.

Horner's syndrome is unilateral presence of ptosis, with diminished elevation combined with anisocoria from the inability to dilate the pupil; as well as anyhydoris, which is the loss of the ability to sweat on one side. Horner's is the combination of miosis, ptosis and anyhrodsis
Horner's syndrome is from loss of sympathetic stimulation. This can be congenital or from cervical adenopathy, from carotid dissection of from a pancoast tumor or superior sulcus tumor.

________________________________________

A pt comes in with weakness of the legs progressive over several months, there is loss of bladder control and abnormalities of the deep tendon reflexes. Hyperintense lesions of the white matter of the spine seen on MRI

1- the pt from the caribbean motor defects are limited to the legs. There are antibodies to HTLV-1 in the serum
2- Motor defects are present in the arms as well. She has an episode of optic neuritislast year. MRI of the brain shows lesions as well

1- Tropical spastic paraparesis (TSP) is from an unclear effect of HTLV-1 on the white matter of the thoracic spine. Defects of the motor and sensory system are limited to the legs. urinary abnormalities are present as well. There is no proven treatment. Resolution does not occur, and the condition is chronic and progressive. There are no ocular abnormalities in TSP.

2- Multiple sclerosis presents with multiple motor, sensory, urinart and autonomic abnormalities of the entire nervous system. Defects tend to relapse and recur. They occur in different areas over time. the most common abnormality is optic neuritis ...

>>>>>>>>>>>>>>>>>>>>>>>>.
Reply

جوري
10-18-2008, 05:13 AM
still on neurology

A man comes in with muscular weakness. the weakness makes it difficult for him to chew his food, and he has difficulty swallowing.
1- the weakness is worse at the end of the day. repetitive exercise makes it worse.

2- He has lung cancer and repetitive exercise makes it better .

3- the weakness occurred only after an infusion of gentamicin.

Myasthenia gravis with worsening weakness with repetitive exercise. There is ptosis and difficulty swallowing. The best initial test is antibodies to ACH receptors. The most accurate test is an electromyogram. Tensilon (edophonium) test confirms the DX

2- Eaton Lambert syndrome is a mysthenia like syndrome to in association with small cell lung ca. Repetitive exercise makes it better..

3- Aminoglycoside use can provoke muscle weakness by inhibiting the neuromuscular junction.

________________________________

An HIV positive man with 25 CD4 cells comes in for eval of Multiple motor, sensory and cognitive defects. MRI reveals white matter lesions in multiple places. the lesions don't enhance with contrast. There is no mass effect and no surface edema

1- progressive multifocal leukoencephalopathy (PML) results in multiple white matter lesions wth no ring enhancement and no mass effect. PML is a viral infection that causes disease only for those with the most profound immunosuppression. such as AIDS with CD4 cells <50. toxoplasmosis and lymphoma both give mass effect and contrast enhancement. PML is from the polyoma virus knows as the JC virus.

2- there is no specific antiviral therapy known to be effective for the JC virus causing PML. the lesions will resolve if antiretroviral terapy is used that raises the CD4 cell count.

______________________________

A man comes to the ER because of a sensation of the room spinning around him as well as nausea. Nystagmus is present on examination.

1- changes in position of his head precipitates the vertigo. Hearing is normal and there is no tinnitus or ataxia.
2- hearing loss, tinnitus and ataxia are present.
3-Hearing loss and tinnitus are present this is the first episode.
4-There are multiple episodes of hearing loss and tinnitus in his ears.
5- there has been recent head trauma

1- benign positional vertigo is isolated vertigo brought on by changes in the position of the head, there are no other findings

2-acoustic neuroma or eight nerve tumor can have prominent ataxia in addition to hearing loss and tinnitus.
3- labrynthitis is a viral infection of the inner ear that leads to a single episode of hearing loss, tinnitus and vertigo
4-Ménière's Disease presents with recurrent and persistent episodes of hearing loss, tinnitus, and vertigo, this is like persistent or recurrent labyrinthitis.
5- perilymph fistual occurs from head trauma resulting in anatomic damage to the inner ear. all forms of vertigo are associated with nystagmus.
_________________________________________

ob?gyn

a woman comes in for eval of pelvi pain that began several days before her period and continues until the menstrual flow slackens. she also has dyspareunia. Pelvic exam shows some tender nodules in the cul de sac.

endometriosis presents with pelvic pain related to the timing of mensuration. the pain begins a few days before the period and continues until the flow stops. Dyspareunia and infertilit are often associated with it.
Ultrasound or MRI can be done as the best initial test. the most accurate test is a laproscopy.
treatment for endometriosis is with combination estrogen/progesterone contraceptives. Androgens such as danazol can also be effective. GnRH agonsts such as leuprolide or nafarelin can be used to inhibit ovulation. Mild discomfort is best treated with NSAIDS.
___________________________________________

a primigravid woman comes to L&D because of abdominal pain she is found to have a blood pressure of 150/92 proteinuia and a headache peripheral smear shows schistocytes and fragmented cells. The AST, ALT and bilirubin are 3 times the upperr limit of normal. plt count is 87,000

HELLP syndrome.. is comprised of hemolysis, elevated liver function tests and low plts. in a woman in her second or third trimester o pregnancy. Eighty five percent of pts have HTN and proteinuria, hence it is a variant of preeclampsia. Many pts will complain of abdominal pain. Look for abnormal blood smear, elevated transminases, and thrombocytopenia.
delivering the baby is the mainstay therapy. there is no doubt that rapid delivery is the best management if the pt is >34 weeks of pregnancy. for those at earlier stages of pregnancy, steroids should be given.

________________________________

A woman comes in because of abnormal vaginal bleeding, she has not had a period for 16 weeks. there is no fetal movement or heart sounds. the uterus is considerably larger than it would be at 16 weeks of gestation. She has passed grapelike clusters of material from her vagina. Her HCG level is >40,000

Gestational trophoblastic dz or hydatidiform mole presents with abnormal uterine bleeding. Vomiting and a uterus enlarged to a size greater tha would be expected 10-16 weeks of gestation.. the HCG level is markedly elevated and there can be passage of grapelike clusters.
sonography is the most accurate test.
surgical evacuation is the best initial therapy.
___________________________

37 year old woman in her 7th month of pregnancy comes in because of vagina bleeding. she is a smoler with a HX of HTN
1- the uterus is painful and tender, there is no fetal distress noted on the monitor. abnormally increased numbers of uterine contractions are present, sonography is inconclusive.
2- the uterus is not tender or painful, there is no abnormal uterine contractions. abdominal sonography confirms the diagnosis

1- Abruptio placenta is painful, late trimester bleeding. the uterus is ender. abruption is the separation of the placenta from the uterine wall. mild abruption will have no fetal abnormalities. More severe cases will present with fetal distress. If there is no fetal distress, the patient can be managed with conservative in hospital observation. More severe cases with fetal distress requires delivery of the child. Sonography will ot confirm the dx of abruption. HTN and smoking are risk factors for abruption,

2- Placenta previa is painless later trimester bleeding. sonogram confirms the diagnosis. Do not do a digital pelvic or speculum exam for abnromal bleeding without a sonogram to exclude placenta previa. the uterus is nontender.
___________________________

a pregnant woman in her 27th week of pregnancy comes t the hospital for edema and HTN protein is present in her urine

1- blood pressure is 148/96 there is 2 g of protein in a 24 hr urine.

2- blood pressure is 162/112 and 7 g of protein are present in 24 hrs. PLT count is 85,000. she developed a seizure

preeclampsia presents with mild HTN, edema, and moderate proteinurea. plt count is normal. the management is conservative. rapid delivery is not necessary. Magnesium sulfate doesn't need to be given immediately.

2- Eclampsia is characterized by seizures, severe HTN, marked proteinuria and possibly thrombocytopenia. Rx is with Magnesium sulfate, labetolol, or hydralazine to control BP and urgent delivery of the baby. Diazepam can be used to manage seizures.

_________________________________

Opthalmology

a pt comes to the ER with the sudden loss of vision in one eye. the eye isn't red, painful or tender

1- visual loss occurs 'like a curtain coming down' in from of his eyes, there are bright flashes of light.
2-pale milky retina wit a 'cherry red macula' the arteries have areas of pallor interspersed with blood in a 'boxcar' pattern
3-collections of blood are visible in the retina
4-Headache and jaw claudication on chewing
5-examination is normal. Visual loss resolves in a few hours

1- retinal detachment presents 'like a curtain coming down' with flashes and floaters. The Question may describe a hx of head trauma.
2- retinal artery occlusion gives pale retina with a cherry red macula the eye is not red or painful
3-retinal vein occlusion presents the same way as retinal artery occlusion, but acute hemorrhages are present on retinal examination .
4-Giant cell or temporal arteritis gives headache, jaw claudication and tenderness of the temporal area. occurs in pts older than 50.
5-Amurosis fugax from an embolus gives a normal physical examination. the eye is not red, painful or tender.
___________________________

A man comes to the ER dept with sudden onset of a painful red eye.
1-a pupil is nonreactive and is fixed at mid dilation. Visual acuity is decreased.
2- a discharge is present. Lymph nodes are enlarged.
3-Photophobia is present there maybe hx of sarcoidosis or Reiter's syndrome.
4-He sustained ocular trauma earlier today. he feels like 'Sand caught in his eyes'

1- Acute angle closure glaucoma gives a painful tender, hard eye with a noreactive pupil fixed at the midpoint, Tonometry shows increased pressure and the cup-to-disc ratio is >0.3
2- conjunctivitis is the only form of 'red eye' that gives an ocular discharge. viruses cause bilateral disease. enlarge the pre-auricular nodes, and cause itching of the eyes. Bacteria cause unilateral disease
3-Uveitis is associated with photophobia, the diagnosis is confirmed with slit lamp exam, and the best initial therapy is with topical steroids. Inflammatory bowel dz may be described in the case.
4-Corneal abrasion gives a feeling of foreign body sensation from trauma. the most accurate test is a fluorescin staining. contact lenses are likely described in this case..

>>>>>>>>>>>>>>>>>>>>>>>>>>>
Reply

جوري
10-18-2008, 06:57 PM
before I go on with tonight's stuff let me get out of the way a brief caveat on how to handle hyponatremia! electrolyte/acid/base imbalances in the human body is a biggie and you need to know proper algorithm on how to handle a pt in such states.

someone comes in with hyponatremia measure their serum osmolality see if it high, low or normal

if it is high, the problem is glucose remember for every hundred over glucose is you lose 1.6 sodium
if it is normal, the problem is lipids or proteins
if it is low you must figure out their volume status
how do you check out for volume status? you'll check out their state, if they have edema it is high, check for orthostatic hypotension tachycardia to see if high it is an edematous condition like heart failure, liver failure or nephrotic syndrome ..
if the extracellular fluid volume is low, they are fluid depleted then you must measure urine sodium
if the urine sodium is low i.e <10 then they are probably losing fluid from the skin or the GI tract..
if the urine sodium is high >20 they are probably losing fluid from their kidneys
if the extracellular fluid volume is normal, it is almost always SIADH or one of the drugs causing SIADH, could be water intoxication, could be a thyroid problem
Reply

جوري
10-19-2008, 12:34 AM
a bit about acid base disorders


Let's talk about acid base disorders
specifically
Alkalosis = High Ph
Acidosis = Low PH
then move on to renal tubular disorders
definitions for starters, Alkalosis is a situation when the Ph is high
first step is to determine whether this is alkalosis or acidosis
first start by looking at PH
so if PH = 7.4 +/- 0.3 that is normal
PH < 7.37 = acidosis
PH > 7.43 = alkalosis

then once you know whether acidosis or alkalosis
you need to determine whether it is respiratory acidosis or alkalosis or whether it is metabolic acidosis or alkalosis
arterial blood Gas will give PH
PCO2 normal value is between 35 and 45 very dependent on your breathing rate, the faster you breathe the lower your PCO2 will be, the slower you breathe or if you hold your breath the higher your PCO2 will be these change very rapidly..
then there is the bicarbonate level NL (20-28) it is a metabolic buffer, so changes slowly... thus if you get acidotic and your bicarb is set to change that, it will happen slowly compared to PCO2 in mins
the next thing you get is Oxygen level which can vary depending on which altitude you live at and O2 sat level typically in the mid nineties
we mostly however focus on PH, HCO3- and CO2
respiratory alkalosis for starters
is caused by multiple reasons
hyperventilation, rapid, acute fast breathing
also anxiety, from acute pain, panic disorders
the PCO2 will go down but O2 will be normal
can happen in acute respiratory disorders, acute pneumonia or PE, also early ASA toxicity --
in an acute there is no time for metabolic compensation
PH High
PCO2 Low
and HCO3 will be unchanged (typical of respiratory)
24 year old female feeling suffocating, tightness of throat and chest
RR 36
ABG =
PH =7.52
O2=99% (no lung probs)
PCO2 low
HCO3= unchanged
alkalosis because high PH PCO2 low, breathing fast alkalosis respiratory alkalosis
Another case:
24 year old female comes to the ER feels like she is suffocating, RR 36 tightness in throat and chest
ABG
PH= 7.52
o2 sat = 72%
PCO2= 25 (very low) normal is (35-45)
Hco3- 27 (normal)~
not anxiety or panic, case of something wrong with lung, maybe has BCP acute respiratory alkalosis lung disease
________________
now metabolic alkalosis

the problem is that you are either gaining HCO3- or losing H+
how does this happen?
lose acid, with prolonged vomiting, or other NG tube for instance sucking fluid along with stomach content
or diuretics and make them alkalotic,
or rather than losing acid, you gain alkali
like with antacids, milk alkali syndrome build up of bicarb.
metabolic alkalosis is a slow chronic process, losing acid, or eating bicarb as slow gradual, as ou start to become alkalotic the body will have you correct by having you breathe slower, so PH falls from alkali closer to normal
PH increases
HCO3- increases
and respiratory rate slows

typical case
67 year old with HTN on HCTZ complains for fatigue, with orthostatic hyptenion
she is hypokalemic K down, Hyponatremic Na+ down and hypochloremic Cl is down
Ph 7.42 (a little alkali)
PCo2 is high
HCO3- is high
alkaline, highbase in her blood
metabolic alkalosis, body is trying to compensate, by raising PCO2 higher and higher
high PH, High bicarb, high PCO2 for respiratory comp.



____________________

Now Acidosis


Hypovnetillation could be acute could be chronic
unable to blow off PCO2.. hold your breath
or COPD, can;t exhale all the air all the time
do it long enough breathe poorly because of chronic obstructive lung disease
kidneys will retain bicarb to lessen the acidosis
Acute respiratory acidosis, taking narcotics, lowering respiratory rate, or acute asthma
on chronic basis, with chronically elevated
pickwikian syndrome, PCo2 rise get startles awaken
manifestations of high PCO2
blood gases are
decreased PH
High PCO2 breathing slow or poorly not ventilating well
if acute bicarb won't have time to rise, in time it will rise but will never be completely normal
typical
27 year old RR 7
needle tracks pupils are small narcotic overdose
Ph =7.28
O2 sat low
PCo2 =High not breathing
bicarbonate hasn't compensated it is normal

72 year old woman with COPD, comes in with gradually SOB
PH 7.37
O2 90% borderline of normal
PCo2 high 56
HCO3 at 32 also high
she is acidotic because breathing ineffectively
in attempt to correct bicarb rises to bring Ph toward normal

Ph 3.37
O2 Sat = 90%
PCO2 high at 56
HCO3- 32 high
not primary?
___________
metabolic acidosis most complicated
a whole bunch of causes but in general
relative increase to the quantity of acid due to the following
1- addition of acid
2-inability to excrete acid
3-loss of base Hco3-
mechanism low PH
HCO3- low (decreased)
RR increased for compensation in effort to blow off PCO2, thereby increasing PH back to normal, occurs rapidly.
sudden acidosis it will kick in right away..

typical case 17 year old with diabetes hasn't been taking his insulin -- presents with altered mental status
ABG
PH =7.29
O2 sat = 97%
PCO2 = 29
HCO3= 17
metabolic acidosis both PH and bicarbonate are low, as compensation body has been breathing fast to bring down PCO2 to bring that number down
metabolic acidosis
first step look at
metabolic acidosis with high anion gap or with normal anion gap
anion gap adding sodium and potassium subtracting from them the two engatives in blood
so
[NA+ + K] - [cl + HCO3-]
normal anion gap is 11+/-3 that is between (8-14)
high anion gap higher than 11 +/-3
some other acid has been added to the body
if normal anion gap excess HCL or loss HCO3 base
high anion gap acidosis
a few causes
anion gap is >14
acid other than HCL
chloride is unaffected
1- ketoacidosis (due to alcoholic or diabetic, starvation)
lactic acidosis, renal failure, intoxication
ethyelene glygol, salicylates
lactic acidosis
aspirin
methanol
uremia
DKA
Proylene glycol
Isopropyl
ethlene glycol
(La Mud pie )

high anion gap
ketoacidosis over production of ketone bodies
in RF unable to excrete organic acids
normal anion gap
all have in common, pt is losing bicarbonate from GI or urine
or can't excrete acid in urine
common causes are diarrhea from loss of HCO3-
or RTA inability to excrete acid in urine

normal anion gap acidosis
RTA several types
all of these types have one thing in common the inability to effectively acidify the urine.
the kidney ought to be rid of acid -- urine could even be alkalotic overtly wrong

type I- occurs in distal tubules of kidney, the primary defect, Urine PH high, seen in bunch of conditions
kidney stones, Amphotercin B toxicity
lithium use, Sickle cell disease
diagnose with acid challenge,
expecting to spill over in urine
still can't acidify then in urine so give bicarbonate
RTA II in proximal renal tubules
problem to absorb bicarbonate until the levels are very low and eventually when urine goes to distal tubules some will be resorbed, lots of bicarb in urine even when having acidic blood
this occurs in a bunch of conditions
like meyloma, fanconi syndrome, wilson's disease
so basic defect inability to absorb bicarb, this is diagnosed bicarbonate is lost in the urine even in the face of acidic blood, and the way you treat is, is by volume restriction-- volume restrict, enhances ability to absorb bicarbonate
type IV due to hypoaldosteronism due to adrenal insufficiency or diabetes, presents with hyperkalemia, restrict salt, urine sodium is still high must treat with fludacortisone, mineralcorticoid agent
essential basis about RTA
RTA- I
Defect: Distal tubule can't excrete acid
result urine is alkaline even when the blood is acidic
the potassium will be low
the test to do is acid load test-- the urine remains alkaline even when given an acid load
treat it: by giving oral bicarbonate

RTA II
defect: proximal tubule can't absorb bicarbonate
result: urine remains alkaline except in very low flow state
potassium here is :low
test for this: bicrabonate load test, renal tubules don't absorb the bicarbonate and urine remains alkaline
treat: volume restriction

RTA IV

Defect is adrenal or aldosterone deficiency, results in high urine sodium, aldsterone causes you to retain sodium and lose potassium, so if you have an aldosterone deficiency you'll lose sodium and retain potassium
when you lose sodium, sodium exchanges with hydrogen and potassium, you build hydrogen concentration in blood -- hydrogen is an acid will also retain potassium which makes this different from the other two states
test: salt restriction test urine sodium remains high because you don't have mineralocorticoid activity, the treatment is to replace mieralcorticoid with fludrocortisone
Reply

جوري
10-21-2008, 07:18 AM
opthalmology

an elderly man comes for eval of visual loss over last several months, peripheral vision is in tact. central vision is lost. Doesn't have DM or HTN. lesions are visible on the retina.

1- Multiple Drusen ( an eye disease resulting from small accumulations of hyaline bodies underneath the retina) are visible. they are dry without new vessels

2- straight line appear wavy or curves. there is sudden deterioration in vision over weeks.

1- Age related macular degeneration can be either dry or wet . Dry ARMD presents with loss of central vision and multiple yellow drusen on eye exam. This is very slowely progressive. no clear therapy

2- wet ARMD can have sdden, rapid prgression over several weeks to months. subretinal fluid hemorrhge, and lipid exudate are visible. neovascularization appears as grayish discoloration in the macular area. Flurescin angiography reveals the choroidal new blood vessles being formed. treatment is with vascular endothelila growth factor inhibitors such as ranibizumab or pegaptanib. they are given by intravitreal injection.

_________________________________

a 34 year old man comes in for evak of visual loss and eye pain developing over one to two weeks. he has diminished perception of red colors. The optic disc is swollen on examination. eye pain is worsened by movement of the eyes. The pupil constricts only when light is show in the unaffected eye

Optic neuritis presents with unilateral loss of vision peaking in one to two weeks. there is swelling of the optic disc and pain on movement of the eyes. color desaturation is a partial form of color blindness. the normal eye will see an object as dark red, the affected eye wil see it paler, such as pink. an afferent pupillary defect is present. An afferent pupillary defect is present. the affected eye will not constrict when light is shown directly into it, but it will constrict when a light is show directly into it. but it will constrict when light is shown in the normal eye. this is known as marcus gunn pupil

multiple sclerosis is the most common cause of optic neuritis, encephalitis and lupus also cause optic neuritis

most cases will respond to steroids.

___________________________________________

a pt comes in for a sudden onset of double vision and a headache. he has severe sinusitis that didn't respond to antibiotics. extrocular movements are markedly impaired. cranial nerves III, IV, and VI are paralyzed. There is ptosis, chemosis ( edema of the mucous membrane of the eyeball and eyelid lining) and proptosis

cavernous sinus thrombosis is an acute thrombosis of the venous sinus surrounding the sphenoid sinus. usually from a sinus infection. the key to the dx is the presence of opthalmoplegia from plasy of the third, fourth and sixth cranial nerves. there is marked redness and swelling of the eye, and bulging forward of the eye, proptosis. ptosis occurs from impairment of the third cranal nerve, which normally lifts the eyelid. when black materka is present on the palate in a diabetic the diagnosis is mucormycosis.
MRI is the most accurate test.
surgical debridement is critical in the management, in addition to atibiotics, without surgery mucormycosis is rapidly fatal.
________________________________

pediatrics

a child comes in with several days of cough, coryza and a low grade fever
1- barking, spasmodic cough and stridor is present. the voice is hoarse

2- after 7-10 days of upper respiratory tract infxn, paroxysms of coughing occur. there is striking gasp or whoop after the paroxysm of cough. there are typically five or mroe cough per episode.

1- croup is a viral infection of the upper respiratory tract that results in a barking cough and inspiratory stridor. the white cell count and temperature maybe mildly elevated. An anterior posterior neck xray will show sublottic stenosis. RX is with inhaled epinephrine and dexamethasone.

2- Pertussis or whooping cough presents with paroxysms of coughfollowed by high pitched inspiration, or whoop. the diagnostic test is a culture or PCR secretions for botdetella pertussis. Erythromycin and azithromycin are the antibiotics of choice. A hx of lack of vaccination maybe given..

_______________________

a chlld comes in with sudden onset of high fever, sore throat, drooling, dysphagia, and inspiratory stridor. swallowing is painful. the sx cause the child to sit up and lean fwd and hyperextend the neck. voice is muffled, cough is absent

epiglottitis is a respiratory emergency with a very ill, irritable appearing child with high fever, drooling and both pain and difficulty swallowing. the child leans fwd with a muffled voice to aid in handling oral secretion. difficulty breathing is common and may suddenly worsen. Cough is absent.

sudden aiway obstruction may occur with oral examination, venipunctur, or any cause of anxiety. for the msot critical intial step is to transfer the pt to an operating room or an area where emergency trachostomy can be perfomed.
Lateral neck x-ray may show swollen epiglottis as a thumbprint as the first test. direct visualization of the epiglottis should occur only after an airway is secure.
Intubation and ceftriaxone are the initial therapy after guaranteeingairway will not close off. Dexamethasone is useful.

__________________________________

a two year old child is brought in fo eval of shortness of breath resulting in irritability. the child squats to releive the SOB. A systolic ejection murmur is heard at the upper left sternal border. the S2 is single. A right ventricular heaven is present

Tetralogy of fallot is a common congeital heart defect. the pt may present at birth of later in life if the degree of pulmonary outflow tract stenosis is milder. toddlers will squat in order to increase venous return to the heart and improve sx of SOB. The S2 is single becase the P2 is not heard. right ventricular enlargement occurs because of pulmonic stenosis. the PS drives unoxygenated blood through a ventricular septal defect. transposition of the great vessles becomes symptomatic immediately after birth and soon as ductus arteriosus closes. The murmur of PS is heard at the upper left sternal border.

2- Echocardiography and cardiac cathe are the most accurate diagnostic tests.
Surgical closure is the rx
tetraology of fallot has a ventricular septal defect, pulmonary valve stenosis, over-riding aorta and a right ventricular hypertrophy.
______________________________

one Y/O child is brough with abdominal pain, blood is passed per rectum. there is nausea and vomiting.

1- the pain originally occured 15-20 mins apart, but has now become constant, lethargy has eveloped. A sausage shaped mass is palpable in the abdomen. the blood is mixed with mucus so it looks like currant jelly

2- there is repeated episodes of bleeding. tenderness is present to the left of the umblicus. upper and lower endoscopy are normal


1- intussusception is an idiopathic obstruction that occurs in the firsr year of life . the key to dx is abdominal pain progressing from episodic to constant combined with bloody stool and palpable abdominal mass.
Ultrasound or contrast enema are most accurate tests. Barium on air enema will successfully reduce 90% of pts intussusception. surgery is seldom needed.

2- Meckle's diverticulum presents with repeated episodes of lowe GI bleeding. it can mimic appendicitis. the dx is based on technetium bleeding scan/. surgical resection is necessary.

__________________

A male child comes in with hypogonadism. testosterone levels are low

1- at puberty extra long bones develop with gynecomastia and diminished sperm count. FSH and LH levels are abnromally high. the testes are atrophic.

2- A male infant has no testes palpable in the scrotum

3- Anosmia is present. there is renal agensis. The LH and FSH are markedly diminished
1- klienfelter's syndrome is hypogonadism associated with an abnromal karyotype. the pt is XXY. LH and FSH levels are elevated but the testes are nonfunctional with markedly low testosterone. these pts are normal until puberty. they develop extra long bones and gynecompastia. RX is with testosterone replacement.

2- cryptorchidism is usually apparent much younger when ne of both testes are missing from the scrotum. sperm and testosterone will be normal RX is surgically pull the testes down from the abdomen and attach them to the scrotum. this is important to do as early as possible because of increased risk of testicular cancer.

3- Kallamann's syndrome is a genetic defect resulting in low gonadotopin levels from hypothalmic deficiency og GnRH. Kallamann's syndrome is associated with anosmia and renal agensis.

_________________________________

a girl is brought in because of failure to achieve menarche. the pt is short in stature compared to her sis with a webeed neck and wide spaced nippled, she is hypertensive and has a murmur of a bicuspid aortic valve.

Turner's syndrome is a karyotypic abnromality with absence of a second second X chromosome in a phenotypic female. there is webbed neck, short stature, cardiac abnromalities.
Karyotype is X,O
treatment is growth hormone and estrogen replacement.

________________________________

a pt comes in with pain in his testicles

1- one testicle is higher than the other and lies in an abnormal horizontal axi. the entire testicle is tender and edematous and there is nausea and vomiting. the cremasteric reflex is absent

2- there is relief of pain with elevation of testis. fever and sx of dysuria are present. there is point tenderness on part of the testis.

testicular torsion presents as a surgical ER with sudden severe pain and swelling of the entire tests. the cremaster reflex is a absent. sonogram may help confirm the dx. surgical reduction of the testis is the rx.

2- epididymitis presents with painful testis that may show relief with elevation of the testis. both testis are at the same heigh and there maybe be fever and irritating sx on urination. there maybe redness of the testis. dx is with gram stain of urethral content. the most accurate test is a DNA probe or culture.
Ofloxacin or levofloxacin are useful.

__________________________

an infant is noted to have copious secretions shortly after birth. there is drooling, choking and respiratory distress, and inability to feed. Air is present n the Gi tract

traceoesophageal fistula and esophageal atresia presents with drooling and respiratory distress along with choking and cyanosis
DX is initially determined by inability to pass an orogastric tube. Contrast studies will confirm the dx.

rx is with surgical ligation of the fistula. if atresia is present, the ends of the esophagus maybe surgically re-anastomosed.

___________________
Reply

جوري
10-22-2008, 07:19 AM
1- a child comes with pain in his leg and a limp unrelated to trauma

1- a five year old child with progressive limp pain that is relieved by rest. X-ray of the hip shows widening by rest. X-ray of the hip shows widening of the articular space.

2- an adolescent pt who is obese, has groin pain radiating to the knee and thigh. X ry shows medial displacement of the epiiphysis and wide growth plates

!- Legg-Calvé-Perthes disease presents with a pain in the anterior thigh that is relieved by rest. the child is typically 5 and walks with a limp. pain is relieved by rest. This is probably from avascular necrosis of the femoral head. X-ray is the best initial test. the disorder is self limitin. range of motion exercises are appropriate

2- Slipped capital femoral epiphysis presents in older children who are obese. radiographic images show medial displacement of the epiphysis. surgical pinnin or external fixation are often necessary.

________________________________

2-A two year old Asian child presents with fever that isn't responsive to antibiotics. bilateral conjunctivitis is present with a rash, strawberry tongue, lips that are dry and cracked and cervical adenopathy. there is edema of the dorsum of the hands and feet. the superficial layer of the skin comes off in large sheets

Kawasaki disease, or mucocutaneous lymph node syndrom, begins with fever and progresses to bilateral conjunctivitis, rash, edema o he dorsum of hand and foot. Mucous membrane involvement is common. although sedimentation rate, C-reactive protein and plt count are all elevated, there is no specific test for Kawasaki's dz.
Coronary artery aneurysm and myocarditis with decreased myocrdial contractility are the most dangerous complications of therapy.
rx. IV immunoglobulin and ASA in order to prevent cardiac involvement.

__________________________________

3-A child comes in with markedly enlarged lymph nodes in his elbow, axillary and cervical areas. the nodes are tender, there is fever, the child has a kitten and a turtle, and fishes as pets

cat-scratch fever is the development of painful, tender nodes a few days or weeks after a scratch or bite from a cat. fever isn't always resent. a small number of pts develop ocular involvement, encephalitis, or seizures.
serologic testing is often helpful. the most accurate test is aspiration f a lymph node with PCR or warthin starry staining of the materla.
nor rx necessary if limited to lymph nodes.

_______________________________

4-a six year old child comes in with joint pain, there is also fever, multiple joints are red, swollen and painful. in addition there is a new heart murmur. the anti-strptolysin O titer is elevated. EKG shows a prolonged PR interval. throat culture is negative

Acute rheumatic fever is diagnoses with the presence of two of the major criteria (carditis, arthritis, subcutaenous nodules, chorea, and eryhema marginatum) the dx is also determines with the presence of one major criteria ( fever, arthralgias, high ESR, and prolonged PR interval on EKG) the diagnosis requires confirmation of the presence of recent streptococcal infection, such as a throat culture or anti-streptolysin I titer.

treatment is with antibiotics (PCN, erythromycin) for streptococcus and ASA. prophylactic pcn is used until the age of 21
__________________________________

5- a 12 year old child comes to the office for evaluation of progressive leg weakness. he is unable to keep up in running with his peers and has frequent tripping. PE shows a high arched foot (pes cavus) and hammer toes. he has relatives with the same foot shape abnormality

Charcot Marie-tooth syndrome is progressive peroneal muscle atrophy. It presents with progressive weakess of the muscles of the legs with a high arches foot pes cavus and hammer toes. vibratory sense and general sensation are also lost in a glove and stocking pattern. reflexes are lost, gait abnormality such as steppage gait develops
nerve conduction studies show marked slowing of conduction. biopsy of a peripheral nerve, such as the sural nerve, shows marked axonal degeneration.

________________________
6- a newborn with a fam hx of CF comes in with billious vomiting, abdominal distension, and failure to pass meconium. the pain is worst in the right lower quadrant. there is weight loss and poor appetite

meconium ileus occurs mostly in those with a HX of CF with tenacious meconium obstructs the terminal ileum.
pts present shortly after delivery with vomiting and right lower quadrant pain and distension.
Xray of the abdomen shows a soap bubble appearance f air bubbles mixed with meconium.
treatment is with enemas. Acetylcystine can be combined with the enema. if this isn't effective, surgery (such as a laparotomy) is performed.

_____________
7- a child is brought in with confusion, lethargy and intermittent episodes of disorientation. the pt had a recent viral synrome, and was given ASA. five days later, severe vomiting developed, followed by altered mental status. AST and ALT are 2-3 times normal. Bilirubin and CSF are normal

Reyes syndrome is enecphalopathy combined with fatty infiltration of the liver. a child who has had recent ASA for a viral episode, specifically chicken pox. clinical manifestations can be extremely severe, with deepening stage of coma manifested by progressive unresponsiveness, seizures, pupils that are fixed and dilated and respiratory arrest. SIADH and diabetes melitus can occur.
the dx is made based on enecephlopathy cmbined with fever, hepatic steatosis and recent ASA use. livr biopsy is the most useful test. blood CSF and glucose levels are frequently low. the bilirubin is normal but the PT time is often elevated.
there is no specific therapy for reye syndrome.

__________________________________________________ ____
8- A 12 year old boy comes in because of pain just below the knee. he is very active in sports and is generally healthy. there is tenderness and swelling of the tibial tuberosity a few inshces below the kneee at the patellar tendon insertion site

Osgoog-Schlatter dz is a chronic traction injury at the insertion pt of the paterllar tendon on the tibial tuberosity. the key is pain and swelling and tednerness on the exam below the knee. Osgood Schlatter is probably the most frequent cause of knee pain in children.. characterized by activity related pain. child rubbing his shinbones.
rx of osgood Schlatter dz is rarely necessary beside some analgesics, it resolves spontaneously over several weeks or months.

___________________________
9- a 12 year old child is brough in for eval of abnromal movements of hs face, and shoulders. there is facial grimacing and head jerking, and blinkin. he also produces cocal sounds that are barkin or grunting in quality, and seem involuntary, occasionally yells out obscene words

Tourette's syndrome is combination of motor tics which can be accompanied by involuntary use of foul language and barking and grunting sounds.
RX for tourette is with clinidine, or anti-psychotic meds such as haloperidol or risperidone.
____________________
10- a man with HX of depression is brought to the ER with muscular rigidity, myoclnus, fever, ataxia, confusion, tremor and sweating, he was recently started on paraoxetine . meperidine was used yesterday for pain. dextromethorphan was used yesterday for a cough

sertonin syndrome is a colelction of sx and PE findings such as muscular rigidity, mycolonus, fever, ataxia, confusion, tremor, sweating. tere is no specific test to confirm the dx. the clue is recent initiation of SSRU, dextromepthrophan and mepridine increase the level of sertonin. can preciptate the syndrome.

There is no specific therapy.. stop all med...
__________________
Reply

جوري
10-23-2008, 06:21 AM
pulmonary..
1- pt comes in for evaluation for SOB over the last several months. His physical exam and chest xray can't determine a clear diagnosis. Pulmonary function testing is performed

1- FEV1 52%, FVC 54%, DLCO 40%, TLC 58%
2-FEV 1 44%, FVC 70%, DLCO 45%, TLC 128%
3- FEV 1 94%, FVC 92%, DLCO 110% FEV 1 decreases by 25% with methcoline

FEV1= forced expiratory volume in one second
FVC, forced vital capacity
DLCO, diffusion capacity of the lung for carbon monoxide
TLC =total lung capacity

1- restrictive lung disease to interstitial disease gives decrease in both the FEV1 and the FVC but the proportion between them is normal. Everything is decreased, but it is decreased equally. Because of interstitial fibrosis, the DLCO is decreased. Carbon monoxide can't adequately diffuse across the membrane.

2- Obstructive lung dsease (COPD) decreased both the FEV1 an the FVC, but the FEV1 decreases far more. The TLC is increases bu the volume is not usable because it is residual volume that participates in gas exchange. Increasing the RV is what leads to flattened diahragms and a barrel chest. The DLCO is decreased in COPD, the parenchyma is destroyed in the lung and you can't exchange gas if the lung is destroyed.

3- Asthma appears similar to COPD except that there is reversibility with bronchodilators. if the pt is normal at rest, methacholine is used to provoke a >12% decrease in FEV1 to confirm the diagnosis. Because te lung parenchyma has not been destroyed. The DLCO is normal, it can also be increased due to hyperventillation.

___________________________

2- A man comes to the office for evaluation of persistent asthma despite the use of inhaled bronchodilators. in addition to eisodes of SOB of breath he has sputum with brown plugs, transient infiltrates on chest x-ray and eosinophilia on CBC. there are 'tram-track' markings on chest x ray

Allergic bronchopuloonary asergillosis (ABA) presents with many qualities similar to asthma. thee is SOB, hemoptysis and wheezing. In addition, the chest xray shows recurrent transient infiltrates and 'tram-tack' lines in the bronchi. these track tram lines are indicative of edema of the bronchial wall and bronchiectasis. An elevated esosinophil count is the main clue for dx.

the most accurate diagnostic are elevated level of IgE, Aspergillus precipitans in the serum, Aspergillus-specific IgE and IgG. And sometimes increases skin test reactivity to Aspergillus. Aspergillus can sometimes be grown from sutum.

treatment is with prednisone and itraconazole.
________________________________________

an obese, middle aged man comes to the office for excessive daytime sleepiness. He also notes impaired concentration. His wife says he snores alot. on PE exam he is obese and hypertensive

obstructive slee apnea is defined as the presence of excessive daytime somnolence combined with several additional findings, such as snoring, frequent nocturnal awakening, unrefreshing sleep and impaired consciousness. HTN is found is 50% of pts but is not part of the diagnostic criteria of the dz.

the most accurate test is polysomnography. this is the most accurate way to document periods of desaturation as well as periods of apnea and hypoapnea. By defintion, obstructive sleep apnea is a combination of the sx just described combined with >5 eisodes per hr of apnea and hypopnea.
treatment with continuous CPAP

______________________________________________

A man comes to the office with several month of cough productive of large volumes of sputum. he has fever and hemoptysis

1- fat malabsorption, intestinal obstruction and azoospermia. There are episodes of SOB as well

2- poor dentition, intoxication, seizures, stroke, or intubation. the sputum is foul smelling.

3- episodes of coughing and sputum production come and go it is chronic long term disease


1- Cystic fibrosis is charcterized by pancreatic insufficiency, leading to fat malabsorption, infertility from azoospermia, and intestinal obstruction. the azoospermia happens from imperfectly formed ducts in the male as well as blockage of sperm transport from insipisated secretions. the sputum ic colonized by multiple organisms.

Lung absecess occurs in tose with poor dentition and a reason for increased aspiration, such as seizures, stroke, intoxication or emergenc intubation. All of these impair the gag reflex.
Bronchiectasis is characterized by long term recurrent episodes of cough sputum production, infection. the dx is confirmed by high resolution CT scan.
____________________
a pt is on his third postoperative day when he is found t be suddenly short of breath, his lungs are clear to ausculaation. His pulse is 115 and his blood gas shows hypoxia

pulmonary emboli PE presents with sudden SOB with normal lung exam. there are no definite phusical findings conclusive of the dx of PE. pneumothoraz does not always give abnormalities on chest exam if it is small. that is why the best initial test is a chest xray.. chest xray is often normal on PE.

te most accurate test for a P is an angiogram. although this rarely is done. Spiral CT scan has become the standard of care in terms of testing. however the sensitivity of the est isn't ideal. low probability pts are often best tested with a D-Dimer tes by EliSA. the negative predictive valie of this test is greater than a negative spiral CT scan.

Starting with treatment with heparin is more important than waiting for a definitve diagnostic test such as the CT V/Q sca or angiogram. the presentation of the sudden onset of SOB with clear lungs is critical to the dx. CHF and asthma give clear abnormalities on exam.. pneumonia isn't sudden .
______________________

38 year old AA woman comes for sevral months of SOB, dry cough and tender patches on her skin, just below the knees. thre are crackles on lung exam. the x ray shows hilar adenopathy

sarcoidosis is characterized by SOB with dry nonproductive cough. Sarcoidosis is far more common in AA wmen. Sarcoid is characterized in almost all cases by some form of abnromal lung finding on chest xray. this can be hilar, paratracheal, or mediastinal adenopathy alone or in combination with parenchyma involvement. Although systemic sx such as fatigue, fever, weight loss can occur, the diagnostic question is strongly base on seeing a woman with chronic dry cough. enrlarged lymph nodes on xray is the most characteristic finding on sarcoidosis.
the most accurate diagnostic test i a lung hilar node biopsy looking for noncaseating granulomas.
predinsone is the rx of choice.

__________________________________

60 year old man with alcoholic cirrhosis has been admitted to the hospital with SOB. there is an increased alevolar arterial gradient. his SOB and hypoxia bexome worse when he sits upright. the chest xray and D dimer are normal

hepatopulmonary syndrome is a triad of cirrhotic liver dsease, hypoxia and worsening SOB upon sitting upright. worse dyspnea of being upright is referred to as orthdeoxia. It is presumed to be from vasodilatory substances that are not cleared by the diseases liver. This leads to abnromal pulmonary vascular dilation and right to left shunting.

hepatopulmonary syndrome can be confirmed by contrast echo. Technetium laveled albumin as a perfusion study can be diagnostic...

>>>>>>>>>>>>>>>>>>>>>>>>>>>>>>>>>
Reply

جوري
10-24-2008, 06:19 AM
A man comes to the office for progressive worsening SOB and cough, he has a barrel shapes chest. on CXRAY there are flattened diaphragms and bullae. Blood gas shows retention of carbon dioaxide and an elevated bicarbonate level

1- He is an elderly, long term smoker
2- he is youner than 40 and has never smoked. He also has unexplained liver disease

1- COPD generally occurs in long term smokers, particularly those over 60. all forms of COPD are associated with progressively worsening SOB. Cigarettes markedly accelerate the usual loss of lung function as the pt ages.

2- Alpha-1-antitrypsin deficiency results in premature emphysema in a nonsmoker under the age of 45. the findings can be identical to COPD that occurs in older smokers. There can also be evidence of unexplained cirrhosis as well.

_______________________________

A pt comes with slowly progressive SOB over the last year or two. He has a dry cough, dry crackles on exam and a loud P2 heart sound. His CXRAY shows bilateral interstitial disease

1-He worked as a rock blaster being a glass manufacturer
2-He makes underwear. His SOB is worse on monday and improves by the end of the week
3-He was a shipbuilder an he has pleural plaques on XRAY
4-He manufactured electronic equipment and he has granulomas that respond to steroids.

1-Silicosis is a form of interstitial lung disease that occurs in those exposed to sand. glassmaking, rock blasting or raw quartz. It presents with a ry cough, SOB, and interstitial infiltrates on CXRAY and chest CT scan. there is no therapy.
2-Byssinosis occurs in those exposed to raw cotton, such as in the manufacture of fabrics. It is a type of fabrics. It is a type of reactive airways disease that is worse on the first day of the work week.
3-Asbestosis is classically found in shipbuilders. it is associated with pleural plaques. The most common cancer in asbestosis is lung cancer, not mesothelioma.
4-Berylliosis is a rare cause of granulomatous lung disease in association with the manufacture of recycling of electronic equipment and fluorescent light bulbs in the past. Steroids have been effective ..

_____________________________

A man is brought to the ICU with acute SOB and hypoxia. he has been placed on mechanical ventilation. He has a stroke and you presume that he aspirated gastric content, resulting in pneumonia. he has bilateral infiltrates and his wedge pressure is normal. The ratio of PaO2 to inspired oxygen is <200

Acute ARDS is a disease of diffuse lung injury that results from sepsis, aspiration, trauma, pancreatitis. There is diffuse capillary leak. the CXRAY shows bilateral infiltrates, and there is marked hypoxia. The chest x-ray looks like CHF but the pressures are normal.

there is no specific diagnostic test for ARDS. it is presumptive diagnosis. the ratio of the arterial O2 to the fraction of inspired oxygen (paO2/FiO2) is <200

________________________

A usually sedentary 50 year old man comes to the office for evaluation of painful arms and legs with thickened, erythmatous, edematous skin. Limb movement is restricted by pain and skin thickening. There is an orange tinge to the skin that resembles an orange peel 'peau d'orange' this began just after he started a vigorous exercise program. the white cell count is elevated with 45% eosinophils

this patient has esosinophilic fascitis. which is characterized by thickened, edematous skin that can restrict movement and looks like scleroderma. the skin color is orange tinged. the key to the dx is skin changes combined with blood eosinophilia. It often begins with a sedentary person who being a new vigorous exercise program. The skin is thick to the point of resembling an animal's hide. less common features are joint pain and carpal tunnel syndrome.
most accurate test is a skin biopsy
scleroderma has shiny skin, usually has raynaud's phenomenon and esophageal involevement. Scleroderma doesn't give eosinophilia.
______________________

a 40 year old woman comes in with progressive musclar weakness occurring over months. She can't rise from seated position without using her hands. the muscles are tender There is a purplish perioribital rash and scaly lesions over the extensor surfaces of her knuckles. The CPK level and aldolase are relevated. the ANA is positive

polymyositis and dermatomyosistis present with proximal muscle weakness that makes it difficult to rise from a seated position or to walk on stairs. Half of the pts also have muscle pain. Dermatomyositis is associated with a helitoe rash. a purplish periorbital rash. Gottron papules are scaly lesions over the metacarpophalangeal joints. Elevation of the levels of CPK and aldolase are expected. A positive ANA is present in 80% of pts.

the most accurate test is a muscle biopsy. Antibodies to Jo-1 are resent in 30% of pts and are extremely specific for dermatomyositis. abnormalities in the EMG are expected.
Polymyositis gives an increased risk of malignancy and cardiac involvement.
____________________

a middle ages woman comes in with several months of dry eyes and difficulty chewing and swalloing, particularly with dry ood. She feels constantly thirsty and also has dysareunia. she feels like there is a 'sand under her eye lids' physical exam revealed markedly enlarges parotid glands and multiple dental caries
Sjögren's Syndrome is an autoimmune disorder caused by lymphocytes attacking the lacrimal and salivary glands. Pts complain of dry eyes and dry mouth. it is also known as Sicca syndrome. Vaginal dryness leads to dyspaeunia. Saliva is necessary to physically leads to severe dental caries as well as loss of taste and smell.
the diagnosis is confirmed with a schrimer test, in which filter paper is placed in the eye. A normal person can moisten 15mm of the paper. Those with Sjögren's Syndrome moisten <5mm. Anti SS-A and anti SS-B antibodies are present in 65-70%. The ANA is present in 90-95% and the rheumatoid facor is 80%. The ANA and rehumatoid facto are non-secific. Biopsy of the gland is rarely necessary.
pt with Sjögren's Syndrome are at risk for lymphoma..
_______________________________

A 24 year old woman comes in foe eval for recurring joint pain and a rash on her face, she has had episodes of fever that have never been diagnoses in addition to fatigue and some weight loss. she also reports intermittent chest pain that changes with respiration. the urinalysis shows red cells and proten

Systemic Lupus erythematosus is a multiorgan disease that is most commonly presents with joint pain and skin lesions. Nonspecific sx such as weight loss, fever, fatigue are common but are not part of specific diagnostic criteria. Even without lab testing. this pt has four manifestations of SLE, arthraligia, hematuria/proteinuria, serositis such as CP and rash. four of eleven criteria are the standard for dx of SLE

the full criteria is
Mala rash
discoid rash
photo sensitivity
oral ulcers
arthritis
serositis
renal disorder
Leukopenia (<4000/uL), lymphopenia (<1,500/uL), hemolytic anemia or thrmbocytopnia (<100,000/uL)
Neurologic disorder
Positive anti-DNA or anti smith antibodies or positive for antiphospholipid antibodies
antinuclear antibodies in raised titer..
____________________________

A woman comes to your office for substernal CP suggestive of reflux dz and dysphagia. she complains of episodes of severe pain in her fingers, associated with changes. On PE you note thickening of the skin and immobility of joints

1-Systemic sclerosis or scleroderma is characterized by thickening of one the skin that leads to immobility and pain in the joints. Raynaud's phenomenona is pain in the fingers with color changes from white to red to blue. Raynaud's phenomena is present in virtually all cases of scleroderma. Esophageal disorders are common. this can either be esophageal disorders are common. this can be either reflux, dysphagia or both.
The CREST syndrome is calcinosis, Raynaud's, esophageal dysmotility and telengiectasia.
there is no diagnostic test for scleroderma. the ANA is positive in 90% f ases of more. The anti-Scl-70 antibody is present in one third of cases and is directed against topoisomerase.
scleroderma results in death from involvement of the heart, lung, and kidney. pulmonary fibrosis and pulmonary HTN develop over time slowly

>>>>>>>>>>>>>>>>>>>>>>>>>>>>>>>>>>>>.
Reply

جوري
10-25-2008, 02:02 AM
Salaamz I needed to add this bit about FiO2 to accompany the last post

FiO2
From Wikipedia --yes


The FiO2 is expressed as a number from 0 (0%) to 1 (100%). The FiO2 of normal room air is 0.21 (21%).

A patient's FiO2 may be varied through the use of different Venturi masks, in combination with varying oxygen flow rates. In addition, most mechanical ventilators have controls for adjusting FiO2. An increased FiO2 is necessary in managing adequate oxygenation in patients who are critically ill due to causes such as major surgery, acute lung injury, sepsis, pneumonia, congestive heart failure, or other cardiopulmonary disease. The oxygenation to a patient on a ventilator can be manipulated by changing not only Fio2, but also the tidal volume,the respiratory rate and having a Positive end-expiratory pressure (PEEP). Generally the FiO2 is maintained at less than 40%. Higher settings can lead to oxygen toxicity.

Another common misconception is that the FiO2 changes with elevation. It remains at 0.21 at all altitudes within the atmosphere. What changes is the barometric pressure of air. At altitude, therefore, the partial pressure of oxygen delivered by that 21% of oxygen is lower. The partial pressure is the driving force to oxygenate the blood and therefore a lower partial pressure makes it that much harder to get O2 delivered to the tissues that require it, resulting in hypoxia.
It is important to note that the PaO2/FiO2 ratio:
The ratio should be greater than 286.

and anything below <200 could be considered ARDS
if you wish to know how to calculate oxygen needs etc in a patient then this is a good website

http://classes.kumc.edu/son/nurs420/...ygen_needs.htm

will follow up with high yeild notes later this evening insha'Allah

:w:
Reply

جوري
10-25-2008, 07:13 AM
a woman comes in with months of fatigue and tiredness, and sleep disturbances. She also complains of headache.

1- young woman with muscle tenderness in the neck and shoulders. Alla tests are normal. Eleven trigger pts in the neck, shoulders and hips are tender.
2-older woman with an elevated sed rate and normal CPK, she has temporal arteritis. All sx respond to steroids
3-fatigue for longer than 6 months with normal tests. there are no physical findings

1- fibromyalgia gives multiple trigger pts of excessive tenderness in characteristic areas around the neck, trapezius, hips and kneees. Alla tests in fibromylagia are normal. the pt is under 50 pain is more prominent than lassitude. tricyclic antidepressants and exercise help.

2- polymyalgia ehumatica (MR) gives pain withiut trigger point tenderness. PMR is in older women and is associated with giant cell temporal arteritis. the CPK is normal but the ESR is markedly elevated. Normocytic anemia is often present. there is an excellent response to steroids.
3- CFS is defined as more than 6 months of tiredness. it is often associated with headache, sleep disturbance, muscle and joint pain and tender lymph nodes. there are no physical exam or lab abnromalities. there is no proven therapy.

__________________________

A man comes to the ER dept with the sudden onset of pain, redness and swelling of a joint

1- the pain occurs after an alcoholic binge. the metatarsal phlangeal joint is involved.
2-the pt has a hx of hemochromatosis or hyperparathyroidism. the knee is affected.

1- gout occurs most often in the first metatasophalangeal (Podagra). it can occur after binge drinking. aspiration of the joint is the most accurate diagnostic test. Gout is from uric cid crystals, which are needlelike with strongly birefringence under polarized light..
2-psudogout of calcium pyrophosphte dihydrate (CPPD) deposition disease is more common with hemochromatosis, hyperparathyroidism and acromegaly. the knee is most commonly affected joint CPPD crystals are rhomboid in shape, are blunted and have weakly positively birefringence.

____________________________

A man of middle eastern origin comes in with shere recurrent oral lesions of unclear etiology, he also has geintal lesions and erythema nodosum like lesions. he develops a sterile skin abscesses whenever he has a needle stick

Behçet syndrome is an idopathic disorder that occurs in pts of middle eastern or eastern origin. consisting or recurrent oral and genital lesions. ocular lesions such as uveitis, optic neuritis, or retinal vasculitis. skin and joint lesions are frequent.
there are no specific tests for this syndrome
the worst complication of Behçet syndrome is blindness. Neurologic involvement occurs in 20% of pts. this consists of chronic meningioencephalitis and may also lead to brain stem lesions and psychiatric disturbances. peripheral neuropathy isn't a feature.
mild Behçet dz responds to colchicine. severe dz is treated with steroids.
pathergy phenomenon (PP) is ofen used as a criteria for Behçet syndrome. it refers to the hypersensitivity to needle sticks..

______________________

a 12 year old comes in with pain in multiple joins, fever and a salmon colored rash

1- there is also splenomegaly, tender lymphadenopathy, and pericarditis. the ESR is elevated. Iridocyclitis develops later. Joint fluid yields 8,000 white cells

2- the pt has a normocytic anemia, a profoundly low retic count, and giant pronormoblasts on the bone marrow. Generalized flulike sx are also present.

Jevenile RA presents with high fever, multiple large joint involvement, tender lymph nodes, splenomegaly, and complications of JRA such as pleuritis or pericarditis. iridocyclitis is a complication if JRA that may lead to blindness. A mildly elevated synovial white cell count is common.

2- parvovirus B19 presents wit a diffuse rash, flulike sx, and aplastic crisis. the alastic crisis is more common in those with a hemoglobinopathy such as sickle cell dz. parvovirus can also present as an isolated rash known as erythema infectiosum, or fifth dz. it looks like 'slapped cheeks'

___________________________________

a pt with a hx of osteoarthritis comes in with pain in the back of the kneww on exam there is palpable fluids filled mass, that is felt when the leg is in full extension

A baker's cyst is an outpocketing of the synovium of the kneww that causes pain in the back of the kneww. these cysts are often easy to dx by palpation. when they rupture, pain extends into the calf. and may mimic a deep venous thrombosis. they occur in pts with hx of arthritis.

on PE, including transillumination is not diagnostic. Baker's cyst is detected by sono or MRI
Most baker's cysts don't need specific therapy. severely symptomatic cysts can be treated with aspiration or steroid injection surgery is often not necessary.

___________________________

a 53 year old man has had several month of cough, SOB, fever, and weight loss. there is hemoptysis, joint pain and sinusitis. last month he has his first episode of otitis media. the chest xray shows a cavitation. there urinalysis shows red cells, red cell cass, and potein. there is no response to antibiotics, and all sputum testing including TB is repeatedly negative

the most likely dx is Wegner's granulomatosis.
wegner's is characterized by upper and lower respiratory tract infxns, as well renal abnormalities, such as hematuria an proteinuria. the upper respiratory probs are sinusitis and otitis. in addition wegner's is a systemic vascuilitis with involvement of the brain (stroke) skin (purura/petichiae) eye (uveitis/iritis) GI tract bleeding and joint pain and neural tissues.

the best initial test is c-ANCA. the most accurate test is a biopsy of the involved organ. CHurg-Strauss syndrome would present only with lung and renal sx and would not affect the body diffusely, as does wegner's...

_____________________

a woman comes in with several month of progressive worsening joint pain and swelling in more than three of the joints in her wrist and metaxarpophalangeal joints.

the pain improves over several hours as the day progresses. the xray of the hand is normal

RA is characterized by a least four of the following

1- morning stiffness for at least one hr and present for at least six weeks of three or more joints.
swelling of wrist, metacarpophalangeal, or proxima intephalangeal symmetric joint swelling.
hand xray changes typical of RA include erosions or unequivocal bony decalcification
ra subcutaenous nodules
positive RA factors

RA is also associated with pericarditis, lung nodules and effusion, anemia, vascuilitis, and peripheral neuropathy.
the RA factor is nonspecific. the most specific blood test for RA is antibody to cyclic citrullinated peptide (specificity 95%)
___________________________

a man comes in with pain in several joints in an asymmetric distribution. he has back pain and sacroiliac SI joint is involved, he has pain in the knee and ankle as well. his RA factor is negative but he is ositive for HLA-B27 antibodies
1- an adolescent with decreased lumbar spinal mobility and back pain. He also has uveitis. X ray shows that he has fusion of the SI joint.
2there is nonspecific urethritis, circinate balanitis, and conjunctivitis. A skin lesion knows as keratoderma blennorrhagicum is present
3- psoriasis with nail pitting is present. the distal interphalangeal joints are especially affected.

all three of these cases are seronegative sondyloarthropathies. the RA factor is negative and antibodies to HLA-B27 are frequently present. the first case is ankylosing spondylitis. which is present in a young man with back pain and decreasing spinal flexibility. exercise improves sx. 30% have uveitis and 3% aortisis. A fused SI join on X ray or MRI is required for the DX

2-reactive arthritis orr Reiter's syndrome is a triad of nonspecific urethritis, conjunctivitis and asymmetric arthritis. Skin lesions are common.

3-Psoriatic arthritis occurs in 10% of thosw with psoriasis, Nail pitting is characteristic. as is involvement of the distal interphalangeal joints (DIPs) RA does not give sacroilitis or DIP involvement.

___________________________

a young asian woman comes in with fever, fatigue, weight loss, arthralgia and night sweats. these sx resolve, she later has an episode of syncope and amaurosis fugax. Arm pain is present on exertion. pulses are diminished in upper extremities

Takayasu's arteritis is an inflammatory polyarteritis of unclear etiology that occurs most often in Asian women before the age of 50. after an initial period of nonspecific inflammatory sx such as fever, fatigue, and weight loss. the pt develops an occlusive vascuilitis of the aorta and subclavian artery. Lesions proxima to the branching off of the verterbral artery. Lesions proximal to the branching off of the verterbral artery can result in retrograde flow from the brain. these subclavian steal sx may result in syncope and transient ischemic attacks. HTN is present in 25%. Arm pain develops from vascular insufficiency, as does diminished and eventually absent pulses.
the most accurate test to examine the vasculature by CT, MRI or angiogram.
corticosteroids improve sx.
_________________________

a pt comes in with a hx of connective tissue dz , he has recurrent episodes of pain in his external ears. Hearing is normal, but the cartillage of the ear is red, inflmmed and deformed. He has recently developed the same problem in his nose. Joint pain, hoarseness, and ocular sx are present as well.

relapsing polychondritis is an idiopathic inflammation of the cartilage of the ears, nose, larynx and trachea . the episodes are recurrent and ofen occur in those with other connective tossue disorders. Such as SLE or RA. the nose can ebcome severely deformed. Laryngeal involvement presents as hoarseness. iritis is part of the syndrome. after the acute attacks subside, the cartillage becomes deformed.
Some pts develop aortic root dilation and aortic regurgitation
relapsing polychondritis is treated with corticosteroids.
________________________

>>>>>>>>>>>>>>>>>>>>>>>.
Reply

جوري
10-26-2008, 07:08 AM
a 53 year old man has had several months of cough SOB, fever, weight loss. here is hemoptysis. the CXRAY shows cavitation . urinalysis shows red cell casts and protein. there is no response to antibiotics, and all the sputum testing is negative repeatedly, including TB.

1- upper and lower respiratory tract involvement. there is also multi-organ involvement such as joint, skin, eye, CNS and GI

2-Eosinophilia and asthma with wheezing

3- Only lung and renal involvement with no additional organs

Wegner's granulomatosis is a systemic vasculitis with upper and lower respiratory tract involvement. there is renal involvement as well, but this isn't unique. Wegner's also has joint pain, purpuric skin lesions, iritis and uveitis, GI lesions, sroke, and neurologic involvement.


2-Churg strauss syndrome also a systemic vascuilitis but it is unique in that it is characterized by eosinophilia and asthma .

3- Goodpasture syndrome is not a vascuilitis, although there is ling and renal involvement, the dz is limited to these two organs.

_________________________

A woman comes to the ER with right lower quadrant abdominal pain. she has a temp of 101 F and an elevated white count of 14,000

1- the pain started around the umblicus and is worse on passive extension of the right leg

2- there is cervical motion tenderness on pelvic exam

appendicitis presents with periumblical pain that progresses to pain at the right lower quadrant midway between the umblicus and the anterior superior illiac spine of the hop (McBurney's point) there may also be additional pain with passive extension of the right leg. CT scanning can help confirm dx.

Pelvic inflammatory dz and salpingitis presents with lower abdominal pain in women. both of these are associated with cervical motion tenderness (CMT), ectopic pregnancy can also lead to these findings.

_______________________________

a man is in the hospital several days after abdominal surgery. he is nauseated with abdominal pain and is unable to tolerate feeding. He has not passed stool or gas. He his bloated and there are no bowel sounds on auscultation

A dynamic ileus can occur with any form of abdominal surgery that penetrates the peritoneum. Normally, peristalsis should return within 24 hrs. prolonged ileus produces abdominal pain, bloating, absent bowel sounds and the inability to pass gas or stool.

an abdominal xray will show multiple air/fluid levels
there is no specific therapy to restore bowel motility. Decompression of the stomach should be performed with nasogastric suction.

___________________________________________

an elderly man comes to the ER dept with left lower quadrant abdominal pain

1- he has a fever elevated white count. there is tenderness in the left lower quadrant. the dx is best made with CT scan

2- he has rectal bleeding and the diagnosis is best made with a colonoscopy


1- diverticulitis presents with left lower quadrant abdominal pain and tenderness in an older person, because it is an infection, there is fever, leukocytosis. because of an increased risk of perforation with colonoscopy, the dx is best made with CT scan. Antibiotics such as ciprofloxacin and metronidazole are standard care.

Ischemic colitis is a type of chronic intestinal ischemia. it presents with pain and rectal bleeding. colonoscopy best confirms the dx. there is no specific therapy.

____________________________________

an infant suffers nonbilious projectile vomiting after almost every feeding. He is dehydrated. A firm, nonmobile, olive-sized lesion is palpated in the abdomen. There are fewer stools that are smaller. a wavelike motion is visible on the abdomen after eating. Metabolic alkalosis is present

pyloric stenosis is an idiopathic narrowing of the pyloric sphincter of the stomach. an infant between two and eight weeks of age develops progressively worsening projectile vomiting. the stenotic sphincter may be palpable in the abdomen, about the same size as an olive. After eating, peristaltic waves maybe visible on the abdomen. Dehydration and metabolic alkalosis may occur from vomiting
the most accurate diagnostic test is first an ultrasound and best with barium studies of the abdomen
surgical correction is best therapy with a myotomy.

___________________

a 36 year old man presents to the Er dept with severe pain in the back of his lower lef from the heel to the back of the calf. This hapened with a 'popping sound' as he started a game of basektball, which he plays every few weeks. He has been on cipro for the last 6 weeks for prostatitis

Achilles tendon rupture presents as a sudden 'pop' or 'snap' but apparently no crackle



:haha:.. just checking to see you are awake.. anyhow, this happens upon exercise or when vigorously dorsiflexing the foot. this is seen more often in those who engage in viperous activity after prolonged periods of inactivity without adequate stretching or preparation. there is severe pain n the back of the foor and up into the calf
quinolones predispose to achilles tendon rupture becaise of their ability to inhibit chondroblasts and osteoblasts
surgical reattachment is necessary.

___________________

a man comes to the Er because of intoxication. he is disoriented with an unsteady gait and an alcohol odor on his breath. he has metabolic acidosis with respiratory alkalosis as a compensation.
1- He has visual disturbance. his retina is hyperemic on exam and the anion gap is elevated.
2-Envelope shaped crystals are found on urinalysis. his serum calcium is low. the aion gap is elevated
3- his anion gap is normal

1-methanol intoxication is associated with an elevated anion gap metabolic acidosis and intoxication. methanol is metabolized to formic acid which leads to optic nerve toxicity and visual disturbance.

2-Ethylene glycol intoxication results most often from ingestion of antifreeze. it forms calcium oxalate crystals in the kidney and they appear as 'envelopes' in the urine


the formation of calcium oxalate crystals results in a low serum calcium level. both methanol and ethylene glycol lead to elevated anion gap.

3- Isopropyl alcohol is rubbing alcohol and ingestion leads to metabolic acidosis with a normal anion ap. the osmolar gap is elevated in all three scenarios indicating ingestion of an abnormal substance.
______________________________

A man is snowed in during a storm. He has a wood burning stove. the patient and his family have been having lightheadedness, fatigue, SOB and headaches, he feels better when he is shoveling snow.

carbon monoxide poisoning presents with lightheadedness, shortness of breath, headache, faitgue, when it is more severe there will be confusion and CP. the two most important clues to answering the dx question is either a woodburning stove in a contained area, or pt rescued from burning building. 60% of deaths from fires on the first day are from smoke inhalation and CO poisoning.

carboxyhemoglobin levels are the most accurate diagnostic test. the blood gas will show metabolic acidosis with respiratory alkalosis as compensation.
the best therapy is 100% Oxygen. Hyperbaric Oxygen is an answer if there is cardiac of CNS abnormalities.

______________________________

a man with severe CHF is brought in because of confusion, blurry vision, vomiting, diarrhea and color vision abnormalities. His potassium level is elevated. EKG shows ventricular ectopy and paroxysmal atrial tachycardia

digoxin toxicity most commonly presents with GI disturbances, such as nausea, vomiting, and diarrhea. Neurologic toxicity includes confusion, blurry vision, yellow halos around objects, and color vision mispreception. Hyperkalemia occurs from inhibition of the sodium/potassium ATPase. the earliest EKG abnormalities include atrial or ventricular tachycardia. and or atrial trachycardia with variable block.

treatment for digoxin toxicity is with digoxin binding antibodies. the strongest indication for digoxin binding antibodies is cardiac or CNS toxicity.

_______________________________________

a man who works in the demolition/construction business comes in with abdominal pain. He has HTN, anemia, renal insufficiency. PE exam shows foot drop. Peripheral smear shows basophilic stipling

lead poisoning in adults presents with abdominal pain or 'lead colic' there is a direct renal toxicity against the renal tubules. lead blocks the production of heme, resulting in sideroblastic anemia and basophilic stippling on blood smear HTN develops for unclear reasons. Neurotoxicity takes the form of wrist or foot drop.

the free erythrocyte protoporphyrin levels is elevated. lead level is the most accurate diagnostic test.
rx is with chelating agents such as succimer, EDTA, or dimercaprol. Succimer is an oral agent.

___________________________

an elderly woman with osteoarthritis comes in with hyperventillation, tachycardia and nausea. she also complains of tinnitus. Blood gas shows a PH of 7.45, pCO2 of 22, and serum bicarb of 14. Cxray shows pulmonary edema

Salicylate tox presents with nausea, hyperventillation, tinnitus and metabolic acidosis as well as primary rspiratory alkalosis.
ASA is also renal toxic both from direct toxicity to the renal tubules and from inhibition of prostaglandins that dilate the afferent arteriole. Non-cardiogenic pulmonary edema may be visible on chest xray.

Alkalinzation of the urine is performed in order to increase urinary excretion.
___________________________

a depressed pt comes in aftr a suicide attempt confused and disoriented. he is unable to offer a coherent hx. His mouth is dry, and there is urinary retention, dilated pupils and decreased peristalsis.

tricyclic antidepressant OD presents with signs of anticholinergic side effects of the meds. which include dry mouth, flushed skin, twitching muscles, dilated pupils, tachycardia, and diminished bowel sounds, seizures can occur.

the most urgent step is to do an EKG to check the presence of widening QRS. seizures and arrhythmias are most common cause of death.
RX of cardiac tox is with bicarbonate.

______________________________

a police officer has just been exposed to a nerve gas. he comes in with excessive salivation, lacrimation, urination and diarrhea. in addition there is wheezing and bradycardia

organophosphate toxicity most commonly results from insecticide exposure, in addition, it is the basis of nerve gas. pts presents with signs of ACH toxicity, such as salivation, lacrimation, urination, defectation, bronchospasm and bradycardia.

Atropine is the most important initial step in addition, to remove the pts clothing and wash him to decontiminate his skin.
Pralidoxime is the specific antidote for organophosphates .

___________________________________



thank you for reading..


:w:
Reply

جوري
06-19-2009, 12:38 AM
approach to the patient with incontinence: most people believe they have this down (myself included) but have no clue how to approach the proble.. thus

the best thing to do is work systematically and develop a differential

1- irritatative incontinence
2- Genuine stress incontinence
3-hypertonic (urge incontinence)
4- Hypotonic (overflow incontinence)
5- Fistula or bypass incontinence


Now for your diagnostic plan:

cheapest first -- urinalysis looking for irritative incontinence -- what causes irritation of the bladder cause it to contact?
1- infection
2- tumor
3- foreign body
4- stones

your findings on urinalysis with an infection, white cells, and offensive agent.. with a tumor you'd find red cells (microscopic hematuria) as well with a foreign body-- if you have a urinalysis that doesn't show these three, then you've ruled out irritative incontinence .

Next cheap test is the Q tip test before cystometric studies..
Q tip test. pt lays supine put a lubricated cotton tip applicator in the urethra and have pt. increase intrabdominal pressure, if there is loss of support for the bladder, as the bladder goes down, the Q tip goes up, an increase in angel more than 40-45 degrees you have a positive Q tip suggesting increased mobility of the proximal urethra.

Now cystometric studies : ask the patient to empty as much urine as s/he can, then do a residual volume to see how much is left, leave the catheter in and infuse saline into the bladder, as the saline goes in, it empties from the cath into the tube and you can see the level of the fluid, see how high it goes and look for involuntary contractions by level of fluid going up and down and you'll instruct the patient on when the sensation of bladder fullness occurs, called the sensation of fullness volume, as well the sensation of wanting to void

the normal values for those:
a normal residual volume is <50cc
a sensation of fullness >250cc
sensation of wanting to void >400cc

dx genuine stress urinary incontinence:
loss of support of the urethral vesicle junction, so the proximal urethra is no longer an intrabdomionl structure

looking at plevic floor you can see that the pelvic urethra is an abdominal structure -- during coughing the intravesicle pressure goes up and the urethral pressure goes up as well = no loss of urine

however, with loss of support of the proximal urethra of the bladder neck, during coughing, the bladder pressure goes up but can't be transmitted to the urethra because it is no longer an abdominal organ.. the bladder doesn't contract, only the pressure.. so now coughing, running, sneezing jumping incontinence..
RX medical therapy, kiegel excer. strengthen the pelvic diaphragm is post menopausal maybe estrogen or elevate with urethropexy to become again an intraabdominal organ..


if there is a prpblem in bladder do cystoscopy..
if no urine lost at night then for sure stress incontinence..

involuntary bladder contraction.. is urge hypertonic bladder (irritable bladder syndrome) only 150cc and need to go.. you suppress contraction with anticholinergics...

if overflow hypotonic from multiple sclerosis or meds.. not a surgical rx, self cath. if non fixable neurological problem..

if continuious leakage because of a fistula because of pelvic radition or radical surgery because of a tumor, you put a tampon in and inject indigo carmine dye, the tampon turns blue.. fix that with surgery......
Reply

Musaafirah
06-28-2009, 04:09 PM
You know one thing that annoyed me so much.
I revised so much for acid base disorders, memorising the equations where necessary, with the possible chance of the topic coming up in this years clinical chemistry exam, as it wasn't in last years paper. I didn't however give too much thought to the renal functions, nor liver as they were in last years paper.
However, I went in to the exam to find that the paper was very similar to last years.
How awful that lecturers have no pattern to what they do!
Alas.
Do you have to continually take exams, even though you're qualified?
Reply

جوري
06-28-2009, 04:57 PM
If you want, I'll cover renal acid/base disorders here in in the upcoming couple of weeks in a really simple fashion.. one super wonderful book that was of much use to me, though it might go into things that aren't necessary for your purposes right now, is this one














anyhow, the answer to your question, is yes, you could be on the verge of retiring and still have to take exams to maintain your license.. but it is for the patient's own good and yours....imagine you are a pathologist or a radiologist or even a GP who has early dementia and continue to diagnose, manage and pass out surgical cases... yearly exams are made so you'd maintain your license and just about anyone in health care has to take them.. pharmacists have to do it to keep up with latest drugs, it is called CE credits.. you need a certain number per year.. once you graduate studying isn't over .. and I'd be worried about inertia in health care, I wouldn't want folks with dated info working on me... that as an aside, the exams aren't as tenuous as the licensure exams which are at least 9 hours long, and there is 4 of them..
step I, step II, CSA (clinical skills assessment with simulated patients) and step III which has a CCS component.. you take yearly exams during your residency to get to the next year, and once you specialize you have to take the board exam in your specialty and then yearly to maintain your CE (continuing education) and licensure ....

:w:
Reply

جوري
07-01-2009, 01:49 AM
childhood immunizations are another chapter most like to skip because they seem tenuous but we'll take the generalist's approach and make them simple:

1- Hepatitis B vaccine, initially given at birth, 2nd dose one month later, 3rd dose at 6 months of age-- if the mother is hepatitis B surface antigen positive, then that is a red-flag, the mother most likely has a chronic hepatitis, the baby has been exposed to that in utero, the child is vertically exposed, you must therefore not only give a hepatitis B vaccine because with active immunization it takes a while to make antibodies and mount and immune response you must also give the hepatitis B immunoglobulin. which is called HBIG -- this must be administered within the first 12 hrs of life -- if don't know the mother's status, you must get a surface antigen and if positive give HBIG along with the immunization in the first week of life.
2- DtaP (the acellular component invented by the japanese) :D
3- HIB.. three types of vaccine, different people making it, like toothpaste consider crest, Colgate and pepsodent for instance all different names to toothpaste.. how do I administer certain types from different company like PEDvax, or comvax, can be used in infancy 2m, 4m, but a dose also required at 6 months... however if you used Dtap + hib combo you can never use the combo products for primary immunization. manufature put different components into one syringe, you can never use combo in the first month of life.
4- inactivated polio, all children need four doses of IPV at 2m, 4 m and then one between 6 and 18 months, and then another booster before starting school from at 4~6 years.
5- MMR... important thing to remember is that you need to give a booster dose between 4~6 yrs of life, however if you miss it you can give it aferr that, you need a 2nd MMR.. if you have missed it between four and six yrs complete it by 11~12 yrs old
6- varicella at any visit after 12 months of age.. make sure the pt doesn't have a hx of varicella don't give it if they have had it.
7-if negligent parents, try to find out if pt had a varicella vaccine, or if 13 and never had a hx of varicella they'll need two doses four weeks apart.
8-pneumococcal vaccine we've two types heptaovalent PCV recommended to children between 2-23 month of age.. high risk pts like with sickle cell or nephrotic, they should also receive the peunmoccocal polysaccharide they need two types, especially sickle cell, because they are susciptible to various strains. by using the two you have a chance of broader protection.
as far as heptatis A only in areas that are endemic.

lastly, flu A vaccine, often confused with HIB, influenza vaccine isn't the same as hemophilus... who would get flu A, children greater than 6m with certain risk factors, if they had asthma, diabetes, lung dz like cystic fibrosis, sickle cell, etc
other things, vaccine is age appropriate. need two doses separated by four weeks.

_____________________________________
Reply

جوري
07-01-2009, 03:25 AM
On Bartter's syndrome:

patient with Bartter' syndrome defect in the ion transport protein in the thick ascending limb of the loop of Henle, such as the apical loop diuretic sensitive sodium potassium chloride cotransporter (NKCC2, Na-K-2CL) the failure to reabsorb NACL in the loop of Henle causes defects in the urinary concentrating abilities, NACL wasting, Hypercalciurea, hypomagnesemia, and decreased intravscular volume .. similar to the effects seen with loop diuretics such as Ethacrynic acid, furosamide and bumetanide. The renal response to decreased intravascular volume, sensed by decreased stretch in the afferent arteriole, is the increased production and release of renin by the juxtaglomerular cells -- increased renin increases the activity of angitensin II and aldosterone and downstream affects. increased aldosterone increases potassium and hydrogen ion wasting in the distal tubule and the collecting duct, causing hypokalemia and metabolic alkalosis, respectively. In the light of concomitant chloride loss in the urine , the resulting metabolic alkalosis is of the hypochloremic variety .. might get a metabolic panel like so
NA+ 130
K 3.2
ca2+ 7.8
HCO3- 30



Reply

Nσσя'υℓ Jαииαн
07-01-2009, 03:00 PM
:sl:

Sis I'm taking Organic Chemistry and Genetics, would that be a smart move to do at the same time? And will these notes help me in them?
Reply

جوري
07-01-2009, 05:19 PM
format_quote Originally Posted by Light of Heaven
:sl:

Sis I'm taking Organic Chemistry and Genetics, would that be a smart move to do at the same time? And will these notes help me in them?

:sl: sister

with all honesty organic chemistry was my downfall in undergrad, not even calculus I, and II were as bad as ochem -- sob7an Allah those who got it got it, but I wasn't one of them.. I had to drop it first time I took it and when I finally passed it, it was barely by the skin of my teeth, I was just so glad to have it behind me considering there were two courses of O.chem, and my happiness was short lived with one, on how I was to get by in the other..

my advise is to get a tutor early on, the first couple of chapters are deceptively simple but you can fall behind fast.. nothing on this thread pertains at all to O.chem.. maybe a few things on genetics, but undergrad is about understanding how things work together, graduate school is about their relevance to the real world, you can't get to one level without having completed the other as your knowledge base to build upon..

Now your experience might be completely different than mine -- I think if anything you should get out of this thread is to do at least just a little bit every day and not let it build up...genetics was fairly easy, it takes dedication so it is up to you.. the problem isn't with genetics it is with O.chem.. can you manage your time wisely or should you dedicate yourself to the difficult course get it behind you and take genetics later? only you are the best judge of your time and your ability..

and Allah swt knows best

:w:
Reply

Nσσя'υℓ Jαииαн
07-01-2009, 06:08 PM
:sl:

Wow that sounds tough. JazakAllah Khair sis. InshaAllah I'll try my best.
Reply

جوري
07-03-2009, 03:43 AM
a few skin conditions and later will go over dermatological pathology in common diseases of infants...




Erythrasma is a long-term bacterial infection that usually appears in the area between overlapping skin (skin folds).
Caused by a bacteria Corynebacterium minutissimum.
Erythrasma is more common in warm climates. in diabetics
appears link a ring worm infection but it isn't examine under woodslamp.. coral red because of coroporyphyrin III in the stratum corneum





inverse Psoriasis--Inverse psoriasis is an unusual type of psoriasis that occurs in skin folds. These patches look different than other types of psoriasis. They are usually smooth, deep red, and glistening without any scale



intertrigo refers to an inflammation of the body folds. This is usually located in the inner thighs, armpits, and underside of the breasts or belly. It is a form of contact dermatitis.




Seborrheic dermatitis is a papulosquamous disorder patterned on the sebum-rich areas of the scalp, face, and trunk. In addition to sebum, this dermatitis is linked to Malassezia, immunologic abnormalities.. when resistant to treatment think pt with undiagnosed HIV.
Reply

جوري
07-03-2009, 07:17 AM
corresponding coronary arteries that supply regions of the heart
1- inferior wall st segment elevation in leads II, III, and AVF= right coronary artery
2- posterior wall (tall broad R waves in V1, V2, ST depression; and tall, upright T waves in V1, V2, V3) =posterior descending artery
3-Anteroseptal wall ( ST elevation from V1-V3) left anterior descending artery
4- Anterior wall (ST elevation V2-V4)= left anterior descending artery
5-Lateral wall ST elevation in I, AVL, V4, V6) circumflex artery


Reply

جوري
07-05-2009, 02:15 AM
a Hodge bodge of review notes I know, but many are either high yields or areas that have caused me personal difficulty so for the sake of the greater good ...

a young woman presenting with nausea, diarrhea, abdominal cramps, vomiting for at least two weeks, her other hx is noncontributory, she does drink 'socially' has mild lower abdominal tenderness and these are her lab results

sodium 136/meq
potassium 3.0/meq
chloride 116/meq
bicarbonate 10/meq
BUN 16
cr 0.9m/dl
glucose 90
urinary sodium 12
urinary potassium 10
urinary chloride 110

the cause of her metabolic acidosis is?
diarrhea

because:

the patient has a non anionic gap metabolic acidosis
you calculate the anion gap as such :

(Na+ + K+) - (cl- +HCO3-) in this case gives us = 13 the normal value is (10-12)
anyhow,
the two most important factors of non-anionic gap metabolic acidosis are diarrhea where there is loss of bicarbonate in the stool and renal tubular acidosis (there are four) in which there is an inability to excrete sufficient amounts of hydrogen in the urine . the best way to differentiate between the two is to calculate the hydrogen ion excretion in the urine .. one expects hydrogen ion excretion in the urine to be high in diarrhea because the kidney attempts to get rid of excessive acid load -- and it will be low in RTA.
urinary hydrogen concentration is calculated as such: H+ = Cl- (K+ + Na+)
Reply

جوري
07-06-2009, 11:56 PM
covering the topic of nephrology today:



figure out what kidney is failing?
______________

your approach:

renal failure has three main causes
pre-renal azotemia .. kidney not to function not because the problem is in the kidney rather in the circulation.

the second is intra-renal azotemia, or intrinsic renal disease--
and the third is post renal azotemia

problem with inflow to the kidney, or the outflow being blocked or can have disease within the kidney.

Now we start with pre-renal azotemia:
pre-renal many think of as dehydration, it is true in part, but it is a problem that is caused by poor perfusion, cause occur from dehydration and volume depeltion, or lost fluid from burn, third spacing during pancreatitis, or bad burn, kidneys aren't getting enough blood flow in, can have also cardiovascular disease, hypotensive because of poor cardiac output, not enough circulation to the kidney will be read as poor blood volume. inadequate blood to the kidney.

the same thing can happen if you have a situation of low albumin level leading to decreased oncotic pressure, like in nephrotic syndrome or cirrhosis, dec. intravscular volume because they run a low vascular volume. Kidney thinks it isn't getting enough blood.

the final main cause, is overt circulatory problem, like narrowing of renal artery, stenosis. Or medication induced vasconstriction, NSAIDS for instance, some of the blood flow is prostogandin dependent. cause the kidneys not to function and cr. goes up.

these are the causes of pre-renal. Not enough blood flow and not just because of dehydration.

why is pre-renal azotemia important. Important, conceived of as ischemic situation, if stays untreated, it will progress to where the kidneys necrose.

what you'll need to do is tell pre-renal from intra-renal because treatments are completely different.

________________

Next post renal, outflow problem

1- outflow of bladder can be obstructed, tumor in pelvis, scarring, strictures, BPH, back pressure goes back to kidney, causing obstruction of outflow..

or at level of ureters, stones, tumors, papillary necrosis, also putting back pressure unto the kidneys.
significance is back pressure on kidneys and damage

_______________

intra renal failure.. problem with the integrity of kidney, the most complicated of the three kinds..

intrinsicrenal, often acute tubular necrosis are many names due to kidneys not working.
how do you distinguish:
specify tests or lab data to interpret:

tests for post renal azotemia
first prostate exam, obstructed urethral outflow.
the best test however, check for residual urine in the bladder, have them urinate put catheter or scan with bladder scan to see how much urine is left after they urinate, typically they should only have 50cc or less

the best of all is an ultrasound, that looks at ureters, bladder.. which will detect hydronephrosis

now pre-renal azotemia

bunch of tests, the most widely used, is the ratio of BUN/CR
Cr level of 1
BUN 10
BUN more than 10 more than 30 it is a good indication of pre-renal
so that is a key labtest

the other more accurate
is measurement of urine sodium or fractional excretion of sodium

if kidneys are in a pre-renal situation , the kidneys say I am not getting enough blood, they think you're bleeding to death, so when they think that will, start retaining sodium to keep vascular volume, so urine sodium will be very low. short term treatment is fluids

urine sodium less than 10 is low

other things, same principal.. kidneys will also concentrate urine, high specific gravity and high urine osmolalaity, they are corroboratory findings.

typical case.
62 yr old for nausea and vomiting
his crt is 3.6 was normal 3 months ago, he also has orthostatic hypotension
BUN 82
urine sodium 6
specific gravity 1.028
BUN is more than 20:1
kidneys trying to replete vascular volume.. faced with this, this is pre-renal azotemia.
rx: fluids

_______________

intra-renal will see the opposite
BUN/CR is normal not elevated 10:1 or even less
will see instead of low urine sodium it will be high, because the kidneys are failing, they start leaking, so high levels of urine sodium
only place that falls down is pt who is on diuretic, so the test then loses value.
look for low BUN/CR
look for sodium spillage in urine
urine specific gravity and osmolality is low

you might also see red blood cell casts or granular casts, the kidney is falling apart and pieces of it coming apart, released into urine.

algorithm: approach to Azotemia

1- faced with high creatine level first thing you want to do is to exclude pre-renal azotemia :
a- what is the BUN/CR if >20:1
b-Urine Sodium <10
c-FeNA <1
d-OSM >300
f-specifc gravity > specific gravity rising by.001 for every 35 to 40 mosmol/kg increase in osmolality. Thus, a urine osmolality of 280 mosmol/kg (which is isosmotic to plasma) is usually associated with a specific gravity of 1.008 or 1.009. so higher than those values and now you know how to calculate 'normal'

sounds like pre-renal

if pre-renal is the case, then treat the cause, whether volume depletion, renal artery stenosis or heart failure..

also need to exclude post renal, using sonogram and post void residual urine, if see hydronephrosis then treat obstruction with stent etc
if no evidence or pre or post renal, then the problem is intra renal

BUN/CR not high
urine sodium that is high more than 20 or more than 30, FeNA that is greater than 1 and osm less than 300 and urine specific gravity is low since urine can't concentrate since kidneys aren't working properly.. that will tell you intrarenal ..

intrarenal failure there are many causes:
but they will either occur in the tubules and interstitum and the other occur in glomeruli which is glomerulonephritis ...

main categories:

first category and most important is ATN
acute tubular necrosis ..
occurs in different settings.. the tubules start falling apart, for instance in prolonged ischemia (shock kidney) hypotensive, labs look pre-renal but then kidney falls apart.. prolonged above conditions, clots of A fib, occluding renal artery...
2nd which are also common are toxins like
radiological contrast, drugs like amonoglycosides and amphotercin B, pigments like myoglobin from muscle breakdown

ATN occurs in phases, prodromal period of time
from beginning of insult to effect on kidney..
then when you have outright ATN, stop making urine, drop to nothing or 2cc an hr (oliguric) then there will be a diuretic phase of ATN
metformin with contrast are at a great risk..

another category is acute allergic interstitial nephritis, an allergy to a drug.. common drugs cephalosporins, beta lactams, methcillin hence we don't use it, sulfas, allopurinol
acute allergic nephritis, takes offending drugs, they'll get fever, rash, high eosinophil count, especially in the urine.
rx steroids short term
other tubulointersistial disorders
deposition disease, any number of chemicals can get into kidney, like hemoglobin is hemolyzed and out in circulation can deposit, myoglobin from rhabdomylisis, rx with high flow fluid to keep from crystalizing.
pts with Multiple myeloma crystalize Bence John's proteins
or crystalization of crystals, like oxalate, ethylene glycol antifreeze, part of mechanism of toxicity, is oxalate crystals depositing in kidney, also vitamin C in very big doses, can cause stones and crystalization.
crystalization of uric acid, for any disorder, like someone with lymphoma and it breaks off giving out uric acid, also in gout as well.
hypercalcemia, hyperparathyroid and crystallize calcium in the interstitum etc etc.
other causes of tubular interstitial disorders, infections like pyelonephritis, develop intrarenal, with white cell casts in urinary sediment.
others drugs and toxins, various analgesics NSAIDs, antibiotics.. outdated tetracycline.
variety of cancer deugs, cisplatin, mitomycin, methotrexate etc.
radiation nephritis, heavy metal poisoning...

glomeruldisorders

vasculitis type disorders (not very common) like wegner's granulamtosis
disorder with granulomatous dz in lung, paransal sinuses who also get lesions in the kidneys, intrarenal failure, high sodium in urine, cr high, problems with chronic sinusists and nasal congestion, maybe lesion in the lung as well.. test here is ANCA

Henoch shcnolen purpura, thrombotic thrombocytopenic purpura, htn etc.

other glomerular disorders, glomuerlonephritis, post streptococal glomerulonephritis.. get hypertensive and edema
do ASO titer to prove that the glomerulonephritis in someone who had a sore throat== post streptococoal glomerluonphritis

IGA nephrpathy, deposition of IgA following a viral illness...

chronic, like with collagen vascular dz. like SLE, also goodpasture syndrome
antibodies against basement membrane and lungs, will show up with intrarenal failure, and they will have lesions in lung and cough blood. order antibasement membrane antibodies

another Alport renal failure with hearing failure.
__________________________________________________ _


Now, how will we decide who has what...

glomerulonephritis

high urine sodium >20
High fractional excretion of sodium >1
BUN/CR suggesting not pre-renal
urinalysis will be key
hematurea and RBC casts = glomerulonephritis
most definitive tests to prove Glomerulonephritis is a kidney biopsy

things that go along with glomerulonephritis is nephrotic syndrome, also occurs in vascuilitis..
1- massive proteinurea massive 3.5 g 4+ dipstick positive and more
they lose intervascular oncotic pressure and develope edema, will run albumin down and hyperlipdemia

best test for nephrotic syndrome is a 24hr urine protein
best test to determine the type of glomerulonphritis causing this nephrotic syndrome is a biopsy.

some nephrotic syndrome are treated with prednisone

best first test (initial test) in a pt. with high urine sodium and indicators of intra renal failure.. is urinalysis!!!


infection, drug hypersensitivity, glomerulopnephritis, nephrotic syndrom, meyloma, uric acid (gout) and myoglobinurea.

what do we see in each of these?

infection= WBC and bacteria
pyelonphritis WBC casts
Drugs/ eosinophillia
glomuerlonephritis = RBC casts
Nephrotic syndrome = Hyperlipedmia, heavy protenurea
Multiple myeloma _ proteinurea
uric acid = uric acid crystals
Myoglobinurea = High CK levels and dipstick positive test for blood but no blood cells.


a different way
pyuria = infection
RBC casts = glomerulonephritis test with kidney biopsy
blood urine dipstick but not micro = myoglobinurea
crystals in uine = gout
eosinophils = allergic drug reactions
massive proteinurea = nephrotic syndrome
__________________________________________

cases:

62 yr old nausea vomiting, creatinine is 3.6 was normal 2 months ago, takes no meds
has large prostate, pulse is high, BUN is 72, urine specific gravity is 1.023
urine sodium = 9
all else is WNL

best initial rx..
likely cause.. he has a large prostate gland, potential outflow obstruction
tempted to get sono to measure post void.
his other things.. BUN 72 and cr 3.2 ratio of 20
concentrated urine
urine sodium of 9
all point to pre-renal azotemia
best initial rx is fluids
_____________________

case II
62 yr old nausea vomiting. CR 3.6, taking NSAIDS
labs
urine specific gravity 1.004 dilute
BUN 41/ 3.6 almost 12:1 ratio
urine sodium = 42

renal problem spilling sodium, can't dilute urine even though dehydrated, he has renal disease, with many dx to consider..

what is next? do sonogram to exclude post renal azotemia, then go back and say renal dz and work it up.

________________________

boy with high BP, edema, urinalysis, proteinurea and RBC casts..
young boy with glomerulonephritis.. blood test to do is ASO titer.

__________________________________

ESRD

end stage renal disease
renal failure that is incompatible with survival..
need dialysis or kidney transplant

how does it show up?

chronic findings and acute findings
important points, need for dialysis isn't determined by creatinine

hypocalcemia
kidneys are supposed to make 1.25 hydroxy vitmin D, because they are not working, you don't have this chemical so don't absorb calcium in GI tract, thus your body makes more PTH to absorb calcium from bone, causing bones to become osteopenic. renal failure need their calcium and Vit D supplementation

people get hypermagnesemia because they can't secrete it.

also have accelerated HTN through renin angiotensin system, very difficult to control, combo of accelerated HTN and high lipids, causes death.

always anemia, since kidneys make erythropoeitin, pts on dialysis, will usually receive erythropoietin but anemia is concomitant to ESRD

impaired immune functions, as well presumably because of toxins not excreted get pruritic.

all of these things are common, become osteopenic and break hip, acceprated lipids and HTN and get MI.

Hyperkalemia, potassium high enough you'll die
potassium is too high from changes it makes on EKG, focus on T waves..
T waves are so sharp and pointed, you don't want to sit on them:






hyperkalemia, that is an acute manifestation that potentially needs dialysis..

metabolic acidosis, in non working kidneys, is severe enough fatal needs dialysis..

fluid overload to the point of pulmonary edema, if kidney can't urinate then dialysis

pericarditis, build up in pericardial sac needs dialysis

pts with high BUN, obtunded, encephalopathic, comtaosed needs dialysis.

renal dialysis, in blood, or direct connection in the blood, venous shunt, or pertoneal, fluid swish around and come back through cath.

________________________

62 yr old pt on dialysis, returns from 4 days trip, feels very poorly (pts need dialysis every two days) he has nausea, vomiting and SOB, he has tall peaked T waves. Cr. 5.2
Hco3- is 16
metabolic acidosis
best rx is dialysis immediately..

__________________

quick recap with key words:
Eosinophils in urine = drug reaction
RBC casts= glomerulonephritis
Heavy proteinurea= nephrotic syndrome
high triglycerides= nephrotic syndrome
Radiological contrast = ATN
Urine sodium < 10 pre-renal (kidney trying to retin its plasma volume)
BUN/CR > 20 = pre-renal
Reply

جوري
07-07-2009, 03:15 AM
Part II nephro.

Now with electrolyte disorders.. very important...
follow this recipe and you can't lose :X



let's start with hyponatremia which is the most common:

SX that folks have, when it is mild and gradual, pts are asymptomatic, sodium falls slowly, with severe, they gets headaches and confusion, and abnormal mental status changes, and at an extreme that is rapid, they can develop CNS coma and seizures ..

causes of hyponatremia, or pseudo hyponatremia
in couple of situations, sodium is low when the blood is hyper-osmolar, what is ahppening the patient is hyperglycemic, sugar is high and as an artifacts of balancing fluids, the sodium falls... the relationship for every 100 mg glucose goes up the sodium falls 1.6mg

pseudo hyponatremia with normal osmolality, artifcat with high triglycerides or hyper-proteinemia like with multiple myeloma

and there is real hyponatremia

so another way to look at it, is you can have hypeonatremia with high osmolality which is seen in hyperglycemia
you can have hyponatremia with normal osmolality which can be seen with lipids or proteins .
or you can have hyponatremia with low osmolality and that can be seen with multiple causes..

____________________________________

high and normal is a pseudo..

so first step to determine hyponatremia is to look at the osmolality

going to continue this review tomorrow insha'Allah since I am falling asleep...:-[

__________________________________________________ __________

picking up from where we left off:

low osmolality multiple causes real hyponatremia..
first step is to determine the osmolality to see if psuedo or real..
send blood plasma but sometimes you have to figure out by calculating
formula: 2xserum sodium + glucose/18 + bun/2.8 = osmolality
or better easier, 2 x sodium + 10 is glucose is normal
normal osmolality is between 280~300

_____________________

hyponatremia with high osmolality.. there is a new solute in high concentration usually glucose, causes fluid shift.
1.6mg% for every 100% rise in glucose
ex. if glucose is 342 and sodium is 126, glucose account for 3.2 sodium drop..
how much higher than normal is glucose of 342? that is 200 units higher than it should be, so for every 100 unit, the sodium drops 1.6
200 points higher, 200x1.6 =
adjusted sodium is 129 so doesn't account for all the drop in sodium thought..

another example
glucose 342
sodium 131

does the glucose explain hypontremia?
400-100 = 300 units higher
300x1.6= 4.8
4.8 + 131 = normal sodium

____________________

now hyponatremia with normal osmolality
coming from proteins and lipids..

certain amount of blood is made of protein and lipid..
conc. of sodium
in abnormal situation, the protein and lipid is higher leaving less space for water and sodium
the amount of sodium is normal but less of it per liter of blood.

patient presents with weakness.
sodium 128
glucose 142
serum osmolality 282
total protein 7.2

what test will show cause of hyponatremia? lipids
normal osmolality hyponatremia..

____________________________

real hyponatremia, measure extra-cellular fluid volume
divide into multiple causes of low osmolality

determine volume status
if have high volume status, then edematous condition

ecf is low
fluid loss, from diuretics, kidney, sweat, burns, etc.

if ecf is normal euvolemic..
multiple causes like
SIADH (ecf normal category)
if high hypervolemic
if low hypovolemic
if euvolemic there aremultiple causes.

low osmolality with hypovolemia, volume depeltion either renal or extra-renal, going to tell by urine sodium.
burn patients or marathoon runners.
or get volue depletion from losing fluid from kidneys

hypoaldosteronism, glucocorticoids deficiency

the most important of the three conditions, the one that does it with a normal volume..

common cause is SIADH
other causes are pathological water intoxication, psychogenic polydipsia, low osmolality hyponatremia but will retain normal volume
hypothyroidism

the most important thought is SIADH
the urine osmolality is abnromally high in face of hyponatremia and low serum osmolality-- if blood is hyponatremic diluted, appear to have extra water in the blood, the kidneys ought to protect me, pee out water so that the sodium level can come back up? expect to have dilute urine to get rid of dilute water in blood.. but here the antidiuretic hormone is abnrmal, actually excretes very concentrated urine high urine osmolality in urine with low osmolality in blood, which should be fixed.. but it is because I have an inappropriate ADH causing me to retain fluid and get rid of sodium ..
high urine osmolaity when serum osmolality is low.. need to also look for renal or thyroid function or drug problem too.

SIADH causes are lung disease, can cause brain to secrete inappoporiate amounts of ADH, concentrate urine and dilute the blood. any brain disease, tumor, subarachnoid hemorrhage, small cell lung cancer, or drugs, like TCA, MAOi, fluxoteine, carbamezapine, narcotics, phenothiazines, clofibrate, vincrsine, cyclophosaphamide, vinblastine.

drugs that increase ADH from brain or on kidney, oxytocin in labor acts like ADH.

SIADH dx. serum hyponatremic, low osmolality, and urine ought to have low osmolality but instead inappropriately high.

rx: either reverse the cause, but you can't always do that,if you can't reverse the cause, restrict fluid, or give furosamide with normal saline..

or two drugs that decrease the action of SIADH on the kidney
demclocycline and lithium
emerency rx for serious hyponatremia as convulsion and seziures is normal saline.

someone comes in with hyponatremia
measure serum osmolality and see if high low or normal.

figure out volume status.
if high, edematous conditions, heart, liver failure
if ecf is low, they are fluid depelted, measure urine sodium, if low they are losing from skin or GI
if high then probably losing it from the kidneys

if fluid volume is normal,
then SIADH or previousely discusses..

cases:

82 yer old vomitting and confusions, takes thyroxine, fluoxetine, glucose 342, has pneumonia

he is vomitting can cause dehydration
low ecf volume hyponatremia
taking thyroid med can cause SIADH like syndrome
as well fluoxetine and chlorpropmide
glucose is high running glucose level down
how to determine?

need osmolality and volume status as well urine sodium to figure out
if efc is normal then check urine sodium
_________________________________________

again

approach to hyponatremia



key points..
Determine serum osmolality high and normal are artifacts of glucose lipids and proteins
with low osmolality determine the ECF volume
High ECF edematous state
low, renal or extra renal depending on whether the sodium is greater than 10 or less than 10
normal ECF then SIADH, drugs, thyroid, cancer etc

_______________________________________________

Next topic is hypernatremia which is the least common of these metabolic disorders:

cause of hypernatremia
people typically dehydtrated, losing fluid more than sodium, thus sodium level rises, occurs through insensible water loss, people with bad burns, also common from GI tract in diarrhea, and become hypernatremic that drive to drink more when you go alot..

another cause is diabetes insipidus, no ADH action, ADH either not made or can't work on kidney, so either a tumor in pituitary or such or nephrogenic, ADH is made but doesn't work on kidney, due to lithium or kidney failure, spill water in urine like crazy..
lethargy, sezures and coma like hyponatremia

rx is fluids, treat the dehydration, think of diabetes inspidus like you'd think of diarrhea..
correct the problem slowly, as to not cause serious central myelnosis

if can't correct this problem right away think diabetes inspidus..
do water restriction test

with passage of time measure urine volume and urine concetration as we restrict their fluid intake..
in a normal person. the urine volume will be less with restriction and the urine comes out should be yellower and more concetrated.
if you have diabetes insipdus, you don't have any ADH, you'll mak urine at same volume, if give antidiuretic at that stage and are unable to respond because of kidneys it will keep right on then you can make dx of nephrogenic diabetes inspidus.. on the other hand if given antiiduretic hormone and it become normal then you've responded to it, you had a CNS lesion that prevented you from making ADH.

rx correct the underlying problem, if tumor or lithium, or replace antidiuretic by given internasal DDAVP
if nephrogenic because of kidney defect, you can give thiazide diuretic causing sodium depeltion helping keep water or NSAIDS impair ability of concentrating ability, neither of those methods are really all that great.

__________________________

now low potassium problm hypokalemia

common causes, GI loss, diarrhea.. various tube drainages in the bowel, medications, diuretics, people on lasix and furosamide, beta agonists like albuterol cause calcium to enter into cells, decreasing k+ in circulation.
Insulin, and drive sugar from circulation into cells potassium goes with it.
then there are rare causes

hypoaldosteronism (conn's syndrome) primary hypoaldosertonism, cushing's syndrome, bartter's syndrome, can't absorb sodium chloride, lose potassium along the way.

liquiorice, not common but has mieralcorticoid effect. Glyceric acid.. chemical in liquiorice with mineralcorticoid effect.


Hypokalemia

73 with COPD and CHF, has diarrhea for three days, he is on lasix, furosamide, and multiple albuterol rx in the ER

multiple causes of hypokelamia

diarrhea in stool
lasix in urine
multiple albuterol (potassium to go into cells

mild is specific t wave changes, severe can cause cardiac toxicity as well potentiate digoxin effects, because of hypokalemia

rx replace the potassium, orally, best if possible..
but in hospital pts at times given IV 10meq/hr

too much potassium can cause cardiac conduction defects, this is how you execute people in prison, you cause heart block and asystole
____________________________________

hyperkalemia
increased intake of potassium, error in hospital, or meds that causes hyperkelamia, like potassium sparing diuretic, ace inhibitor, potassium artificat from venipuncture...
the tourniquet and blood sample hemolyzed and caused potassium to go extracellularly..

also in situations where potassium moves from cells into blood, like familial periodic parlaysis, myoglobuniurea, renal failure, drugs. hypoaldosteronemia

adrenal inssufiency ..

tall pointed peaked t waves..
pt on ace inhibitor or on NSAIDS or spirnolactone or others and potassium is high, specify treatment

67 y/s with DM on glyburide, and ace inhibitr
cr 4
and potassium 6.6
number of reasons
ace inhibitor
because cr is rising renal failure can't excrete potassium
how will you fix it?

in stages:

first stage if you get an EKG changes protect the heart stabilize cardiac membrance
so give calcium chloride or gluconate infusion.

next lower the potassium level by driving potassium out of circulation by giving glucose and insulin 50% dextrose and insulin or give doses sodium bicarbonate create a situation of alkalosis, potassium will go into cells
or get rid of potassium from body keyexylate
or if in renal failure then dialyze them

number one with EKG changes protect the heart give calcium
number two lower serum potassium
if no EKG changes then lower potassium and get it out of the body

__________________________________________________ _______

Now for acid base disorders (RTA's) acidosis and alkalosis.
have a systematic approach to it all...interpret blood gases...

alkalosis= PH is high
acidosis = PH is low

first step is to determine acidosis or alkalosis..
7.4 is normal
less than 7.37 acidosis if more than 7.4 alkalosis
first level decision

now decide whether respiratory or metabolic acidosis or alkalosis..

arterial blood gases..
get PH, will get PCO2 normal is 35-45 and dependent on breathing rate, the faster the lower the PCO2, the slower the Higher

bicarbonate normal is (20-28)

normal mid twenties.. bicarbonate is a metabolic buffer
and changes develop slowely..
if bicarb is trying to fix acidosis, it will happen over days as opposed to PCo2 in a matter of minutes

O2 and PO2 level which varies by altitutde, and saturation which should be over 90

let's start with respiratory alkalosis

caused by a bunch of things
hyperventillation, rapid acute fast breathing..
respiratory alkalosis will drive PCO2 now, various causes, can be caused by anxiety, acute pain, panic attack, and if this happens, PCO2 will go down but there is no other problem

other acute prbs, like Pulmonary embolism and pneumonia, also early asa toxicity, breathe fast, breathe down their PCO2..

no time for metabolic compensation... will see high PH, and PCO2 that is low because of fast breathing, and HCO3 didn't have time to change so it will be normal..

24 year old female, feeling suffocating and tighteness in chest, RR 36
get blood gases, find out PH 7.52 alkalotic
O2 is 99%
PCO2 is low and HCO3-

thus respiratory lkalosis

__________________

another 24 year old everything the same
PH 7.52
however her PCO2 25 and her O2 sat is 76% which is low
HCO3- 27

there is a respiratory problem

ON BCP and had a PE
she is hypoxemic

___________________

metabolic alkalosis
the problem is you are gaining an alkali bicrbonate or you are losing an acid
how does it happen?
lose acid with prolonged vomiting or other GI loss, like NG tube, or diuretics or rather than losing acid, gain HCO3- people who use sodium bicarbonate pills (antacids) calcium bicarbonate for antiacids build up of bicarb, or H2 blockers or proton pump inhibitors..
it is a slow chronic process, losing acid, or gaining bicarb gradually.. as you start to become alakaline, the body will breathe slower so that PH falls from alkaline level to closer to normal..

increasing PH almost back to normal..

typical case..
67 yr with HTN on HCTZ complains of fatigue, her exam shows dehydration with orthostatic, hypokalmic, hypnatremic, hypchloremic... PH 7.42 (a little alkalotic)
PCO2 is high and bicarbonate is high

alkaline, because she has high base in her blood, metabolic alkalosis. body compensating by breathing slow.

________________________

respiratory acidosis, hypoventillation.. could be acute or chronic.. an inability to blow of PCO2
COPD, can't exhale all the air, will build up PCO2 in blood..
breathing poorly become acidotic.. kidneys retain alakli (hco3)
acute respiratry acidosis .. inability to breathe at normal level, taking narcotics that lower respiratory rate, or ashtma..
on chronic basis.. chronically elevated PCo2 from Obesity, or COPD

ABG
dec PH
high PCO2
if acute the bicarbonate won't have time to rise..

typical case
27 yr unconscious
rr is 7
pupils is pinpoint
ph 7.28
O2 is low
PCO2 is high (not breathing)
no time for bicarb compensation thus it is normal

72 yr with COPD getting worst, getting more SOB
ABG
PH 7.37
O2 sat 90%
PCO2 is very high
HCO3- is also high

acidotic because she is breathing ineffectively, in attempt to crrect HCO3 tries to correct it

________________________________

question on the same ABG
Ph. 7.37
O2 sat 90%
pCO2 56 (high)
HCO3- (34) high
metabolic alkalosis (HCO3-) high with high PCO2 as compensation?

how do you know that the alkali isn't the primary thing?
because PH is not alkalotic it is acidotic.. without over comepnsating

______________________________

metabolic acidosis the most complicated...

causes a relative increase in the quantity of acid, due to any of the following, addition of acid, inability to excrete acid, loss of base HCO3-
mechanism in blood gases
low PH decreases b/c acidosis
bicarbonate dec because less base
respiratory rate will increase to blow of PCO2 to increase PH back to normal..
sudden acidosis, the respiratory rate will kick in right away

17 yr old with DM and not taking his insulin presents with altered mental status.. PCO2 is low and Bicarb is low, metabolic acidosis because the bicarb is low, and as comp. body has been breathing fast to blow off PCO2

first step in sprting metabolic acidosis is it?
metabolic with high anion gap, or normal anion gap, called non anion gap..


what is an anion gap?
it involves
adding the postive cations and subtracting from them the two negative anions as such

[NA+ +K+] - [cl- + HCO3-]

normal anion gap is 11+/- 3 i.e between 8-14

high anion gap acidosis higher than above number means another organic acid has been added to the body.. normal anion gap acidosis, either excess HCL- or loss of bicarbonate...

high anion gap acidosis a few causes
over production or under secretion of acid, chloride levels unaffected

four causes
1- ketoacidosis ( alcoholic ketoacidosis, diabetic ketoacidosis, starvation
lactic acidosis
renal failure
or intoxication with any number of different intoxicants
ethylene glycol, methanol, salicilates

la mud pie
l lactic acidosis
a aspirin
m methanol
u uremia
D DKA
P propylene glycol
I isopropyl alchol
e ethlyene glycol

normal anion gap acidosis

they are either losing HCO3- or can't excrete acid..
common causes of these are diarrhea, loss of bicarbonate, or RTA inability to excrete acid in the urine___________________

normal anion gap (RTA)

they all have one thing in common, inability to acidify the urine..
renal tubular acidosis kidneys can't do that.. less acidic urine or overtly alkaline which is wrong...

type I RTA occurs in distal tubule of the kidney, you can't acidify urine, your urine PH will be high 5.6 or higher
seen in kidney stones, amphotercin B toxicity, lithium tox, sickle cell disease. give acid challenge test.. expect that acid spills in kidney and they should acidify urine, but they can't.. the rx is to neutralize the urine by giving bicarbonate.

_____________

RTA type II which occurs in proximal renal tubules and the problem here is inability to absorob HCO3- until the levels becomes very low, eventually in the distal tubule some bicarbonate will be salavged..so again inability to acidify the urine
occurs, in myeloma, fanconi syndrome, wilson's disease
RTA II inability to absorb bicarbonate..
bicarbonate is lost in urine even in face acidic blood, and the treatment here is volume restriction , the kidneys will absorb a little more

Type IV hypoaldosteronism, adrenal defienciy, some diabetes..
the presentation is key, presents with hyperkalemia..
restict salt, and even in restricing salt, the sodium is lost, it is treated with fludrocortisone.

_______________________

so let's do chart

RTA type I

defect: Distal tubule cannot excrete acid.
Results: urine is alkaline even when blood is acidic
K+: is low
test is acid load test -- urine remains alkaline
RX: oral bicarbonate
________________________
RTA type II
defect: proximal tubule can't absorb bicarbonate at normal urine volumes
Results: urine remains alkaline except at very low urine flow volume
K+= low
Test: do a bicarbonate load test renal tubules don't absorb alkali and the urine remians alkaline
RX: volume restriction
_________________________
RTA IV
defect: renal or aldosterone deficiency
Results: in high urine sodium -- aldosterone usually causes you to retain sodium and lose potassium, and when you have an aldosterone defcicney you'll lose sodium and retain potassium. increase in sodium excretion in the kidney exchanges with potassium and hydrogen -- thus you build up high hydrogen ion -- that is how you get acid in RTA type IV
K+: high
Test: salt restriction test.. the urine sodium remains high, spilling sodium in urine
RX: replace mineralcorticoid with fludrocortisone..

___________________________________________















Reply

جوري
07-26-2009, 09:43 PM
An interesting case of a 28 yr old woman who presents for eval of oligmenorrhea and infertility. P.E reveals acne, hirsuitim, and mild clitromegaly. Lab tests reveals mild hyponatremia, and normal levels of 17-ketosteroids and plasma dehydropiandrosterone levels and no change in aldosterone levels. The Pt is diagnosed with a defect in CYP21A2.. What best explains the late onset of her disease?

and the answer is an incomplete 21 hydroxylase defect.
The Patient described has disease resulting from partial dysfunction of CYP21A2 (21-hyroxylase P450C21). defects in CYP21A2 are the most common cause of CAH. There are two copies of each gene in the human genome, one derived from the mother and the other from the father. A complete deficiency of a gene product therefore, requires inactivating mutations in both copies of the gene in question. CAH evident in infancy thus requires that both genes for an enzyme required for adrenal steroid hormone synthesis be inactivated- Adult-onset CAH, however is most commonly the result of an inactivating mutation in one copy of a steroid synthesis gene. The remaining copy is able to code for sufficient enzyme to prevent catastrophic infantile presentation of CAH. In the case described, the patient's single remaining copy of CYP21A2 was able to synthesize sufficient aldosterone precursors to prevent extreme salt wasting and was able to prevent and extreme accumulation of steroid precursors, thereby preventing virilization in infancy. She does however, overproduce androgenic adrenal hormones on ACTH stimulation, and this is likely the cause of her oligmenorrhea, infertility, hisruitism and acne.


Now to look at other similar defects in the same pathway. A complete 3-hydroxysteroid dehydrogenase defect, would cause defects in aldosterone and cortisol and overproduction of adrenal androgens. Clinically male pts. would present with ambiguous genitalia and female pts with virilized genitalia. Hypertension isn't a feature of this disease..

11-hydroxylase (CYP11B1) deficneyc causes hypertension through accumulation of mineralcorticoid 11-deoxycorticosterone.
Reply

جوري
07-26-2009, 10:24 PM
70 year old woman with a long history of DM brought to the ER with a sudden onset of weakness on the right side of the body. She has a 20 pack year hx of smoking and is currently taking metformin and hormone replacement therapy. She is oriented to time and place and person, and seems to answer all questions appropriately. She was able to dress herself before showing up to the ER. Her vitals are BP of 140/90 mmHg and PR of 85/min. P.E confirms loss of strength in the right upper body and lower extremeties. There is a right sided facial weakness as well. The visual field is intact. A CT scan of the head is ordered. Occlusion of which vessel is the most likely cause of her SX?

and the answer is penetrating arterioles of the posterior limb of the internal capsule. Lacunar infarcts are small (<0.5cm) focal areas of parenchymal loss found in the basal ganglia, anterior and posterior limb of the internal capsule, pone and occasionally cerberal white matter. The five classic syndromes include pure motor hemiparesis, pure sensory stroke, ataxic hemiparesis, and clumsy hand dysarthria. They are presumed to be ischemic in origin and due to occlusion of the pentrating arterioles aforementioned structures. Lacunar infarcs are particularly frequent in persons suffering from hypertension or DM. The most common type is a pure motor lacunar infarct most often occur in the genu or posterior limb of the internal capsule (Aphasia, Agnosia, Hemianopsia, neglect etc) pure motor infarcts most often occur in the genu of posterior limb of the internal capsule where the descending corticospial and corticobulbar tracts are located. Infarction of the anterior limb of the internal capsule, most often results in ataxic hemiparesis where both the pyramidal (weakness) and frontocebellar (ataxia) are located.

Now the other major arteries like Anterior cerberal artery occlusion presents with contralteral weakness and cortical sensoty loss in the contralteral leg without weakness in the arm. Urinary incontenance, confusion and behavioral distrubances are likely.

The Middle cereberal artery occlusion presents with contralteral hemiplegia, hemisensory loss, aphasias and homonymous hemianopia. our pt here has intact visual fields and not suffering aphasia.
Posterior cerberal artery occlusion presents with contralteral homonymous hemianopia visual hallucination and agnosias.

vertebral artery occlusion may not cause clinically noticeable sx because of good collateral circulation when sx occur they are similar to basialar artery occlusion.






Lacunar infarcts



self explanatory...
Reply

جوري
08-04-2009, 07:20 PM
I'll be writing the step up to USMLE step 3 on here.. will save most of you a good $40 insha'Allah..

we'll start with cardio ..

little factoids first..

the left anterior descending is the most common site of coronary artery occlusion



coronary arteries fill during diastole. while systemic arteries fill during systole. conditions or drugs that reduce diastolic filling decrease coronary perfusion during a given period of time.

As heart rate increases, the time available for diastolic filling decreases.
During exercise, cardiac output initially increases due to increased stroke volume and then due to increased heart rate.


  • cardiac output is dependent on the rate of contraction. I.E heart rate, and the volume of blood forced out of the left ventricle per contraction. i.e stroke volume.
  • heart rate is the number of contraction per unit time and is expressed as beats per minute.
  • stroke volume is the change in volume from immediately before a contraction to the completion of a contraction. SV = (end diastolic volume) - (end systolic volume)
  • dependent on contractility i.e force of heart's contraction, preload i.e the amount of stretching force on cardiac muscle fibers at the end of diastole, and after load, i.e the vascular resistance which ventricles must overcome to produce outflow.
  • increases with catecholamine release e.g epinephrine, increased intracellular calcium, decreased intravscular sodium, digoxin use, and stressful events. e.g anxiety, exercise

decreases with beta blockers, heart failure, hypoxia with acidosis.
___________________________________

Vascular pressures
_______________________________

  • systolic blood pressure is the maximum vascular pressure expereinced during heart contraction, diastolic blood pressure. DBP is the baseline vascular pressure between contractions.
  • pulse pressure is the increase in blood pressure attributed to cardiac outflow during contraction
  • pulse pressure = (SBP)-(DBP)
  • mean arterial pressure (MAP) is the average BP considering the unequal amounts of time spent in systole and siastole
  • MAP = (DBP) + (1/3 X SBP)

_________________________________________



Aorta gives rise to the left and right coronary artery.

Aorta >>>>>>left coronary artery>>>>>>>left anterior descending branch >>>>>septal branch supplies anterior wall of left ventricle, the septal branch supplies anterior 2/3 of intraventricular septum.
Also from the left coronary we get the circumflex branch which supplies the left atrium lateral wall of left ventricle, posterior wall of left ventricle.

now from the AORTA TO THE right coronary we get three branches off the right coronary artery (the posterior descending branch which supplies the inferior wall of left ventricle, and posterior 1/3 of intraventrciular septum, the marginal branch supplies the right atrium, right ventricle, and the SA, AV node branches.. very important to pay attention to that infarction to the right coronary will spell disaster to the SA and AV nodes.

now the physiology of heart contraction.
Key principles


  • increasing the end diastolic ventricular volume causes an increaased stretch on cardiac muscle fibers, this leads to an increase in the force of contraction i.e the frank starling relationships
  • the end systolic volume and pressure generated by the ventricles are dependent on afterload

increased contractility i.e force of contraction independent of preload and afterload leads to increased muscle fiber tension during isometric contraction
_____________________________
Reply

جوري
08-06-2009, 10:20 PM



EKG
P wave, normal atrial depolarization
PR interval: conduction through the arterioventricular node (<0.2 sec)
QRS interval: ventricular depolarization (<).12 sec)
ST interval: isoelectric ventricular contraction
T wave: ventricular repolarization
U wave: relative hypokalemia


___________________

Normal cholesterol function
Cholesterols and triglycerides are carried by lipoproteins
increased low density lipoprotein (LDL) levels lead to an increased coronary artery disease (CAD) risk
increased HDL is protective
Increased LDL and decreased HDL result from a diet high in fatty foods, tobacco and obesity, alcohol use, DM, and certain meds like OCP and diuretics.

Pre-operative cardiac risk assessment:
for surgical pt estimating the liklihood of an undesired cardiac event occuring as a result of surgery and anesthesia

  • considers measurable cardiac function, pre-existing cardiac dz, age and important co-morbidities
  • young healthy pts. maybe cleared for surgery with a normal EKG
  • older pts with existing co-morbidities require and extensive workup by a cardiologis
  • factors suggesting and increased risk of an adverse cardiac event
  • age >70
  • pulmonary function, forced expiratory volume in 1 sec/functional vital capacity (FEV1/FVC) <70% of expected. Partial pressure of carbon dioxide (pCO2) >45 mmHG, pulmonary edema.
  • cardiac dz: MI within the last 30 days, poorly controlled nonsinus arrhythmia, pathologic Q waves on the preoperative ECG, severe valvular dz., decompensated heart failure with poor ejection fraction.
  • Renal insufficiency: creatinine (CR) > 2.0 or a 50% increase from baseline
  • surgery type: cardiac/vascular surgery or anticipated high blood loss
  • higher risk pts should have their cardiac function optimized prior to elective surgery and should be made aware of the risks is the surgery is emergent
  • perioperative B blockers and postoperative noninvasive cardiac monitoring are recommended for pts determined to be at an increased cardiac risk.
  • ______________________

Invasive cardiac monitoring

  • Arterial line: constant access to artery, e.g radial, femoral, axillary, brachial, dorsalis pedis) that allows accurate measurement of arterial BP and allows easy access to arterial blood for blood gas measurments
  • pulmonary artery cath I.E swan-Ganz Cath: inserted through the left subclavian or right internal jugula veins that run through the heart to the pulmonary artery; a transducer in the cath. allows the measurement of cardiac output, mixed oxygen (O2) saturation, systemic vascular resistance, and pressures in the right atrium and pulmonary artery; a baloon maybe inflated at the cath tip to fill the pulmonary artery lumen and to measure the wedge pressure equivalent to the left atrium pressure.
  • ______________________

fetal circulation.




  • Gas exchange occurs in the uteroplacental circulation
  • fetal HG has a greater )2 affinity than adult HGb and oulls O2 from maternal blood
  • umblical arteries carry deoxygenated blood to placenta, umblical veing carry oxygenated from placenta to the portal system
  • changes following birth
  • lung expansion causes an increased pulmonary blood flow leading to an increase in relative blood oxygenation
  • a decreasing serum level of prostoglandin E results in ductus arteriosus closure; umblical cord clamping results in the end of placental circulation and increase in systemic vascular resistance
  • this increased vascular resistance, in trn induces the ductus venosus closure and umblical artery and vein constriction
  • left atrial pressure increases due to increased pulmonary blood flow, and umbilical circulation decreases, causing a decrease in inferior vena cava pressure.
  • decrease in inferior vena cava pressures leads to foramen ovale closure.
Reply

جوري
08-07-2009, 08:12 PM
cardiac transplantation

  1. indicated for end stage cardiac disease (e.g. CAD, congenital DZ. cardiomyopathy with an estimated survival of <2 yrs
  2. contraindicated in pts. with pulmonary HTN. and Renal insufficiency. COPD, or other terminal diseases, smokers and pts > 70 yrs old are also excluded
  3. acute rejection is common
  4. most death occur in the initial 6 m after transplant, 5-yr survival is 70%

__________

case 1-
25 yr old man presents to PCP for a wellness checkup, he has been in good health. His only complain is recurrent Achilles tendon and hamstring pain that occurs following significant exertion. He has noticed some small bumps in the back of his heels and knees in the past yr in the regions of the pain. He denies medical conditions but states that his father died last year at a young age from a heart attack. He denies substance abuse, review of system and the remainder of his family is unremarkable. P.E is normal, but he has multiple small hard nodules behind his knees and heels.

Differential:
Chronic tendonitis, crystal arthropathy, hypercholesterolemia

labs unremarkable
ECG normal sinus
biopsy of the lesion shows a large collection of cholesterol laden material

DX. Familial hypercholesterolemia (hetrozygote)
RX

  1. the pt was placed on a regular exercise regimen and a low fat low cholesterol diet
  2. the pt was prescribed a regimen of simvastatin and ezetimibe

a repeat lipid analysis in 1 month found the LDL to be 155mg/dl
the tendon xanthomas gradually regressed
the pt was followed regularly to confirm an adequate reduction of lipid levels and cardiac screening.

The majority of cases of hypercholesterolemia are acquired

  1. steps to DX
  2. congenital hypercholesterolemia
  3. inherited form of hypercholesterolemia in which a genetic defect causes abnormally high levels of total cholesterol, LDL and/or triglycerides

common types:

  1. familial hypercholesterolemia autosomal dominant defect in LDL receptors with an associated increased total cholesterol and LDL : the dz is much more severe in homozygotes than hetrozygotes
  2. hypercholesterolemia and hypertriglycerdemia associated with an increased hepatic production of apolipoprotein B-100 protein
  3. Familial defective apolipoprotein B-100 similar to FH except that the defect is in the LDL particle and not LDL receptor
  4. history: tendonitis around xanthomas, xanthlesmas, cholesterol emboli are seen in the retina on fundoscopic exam
  5. total cholesterol and LDL >250 mg/dl in hetrozygotes and >600mg/dl in homozygotes
  6. triglycerides are elevate >200 in familial combined hyperlipidemia and a biopsy of xathomas will detect collections of cholesterol; genetic testing is available but typically uneeded for dx
  7. RX:
  8. healthy diet, exercise
  9. pt education
  10. lipid lowering agents , consisting of one statin and at least one other cholesterol lowering drug to achieve LDL levels <160 mg/dl or lower depending on cardiac risk factors
  11. trig drugs maybe added if required

Blood serum cholesterol levels should be collected from a fasting pt (12-14 hrs) to minimize post prandial influence.
Reply

جوري
09-09-2009, 08:21 PM
Media Tags are no longer supported
Reply

جوري
09-09-2009, 08:23 PM
can't go wrong with a couple of aunty ji's teaching you about the EKG? :lol:..
Reply

جوري
09-09-2009, 08:41 PM
Media Tags are no longer supported

Media Tags are no longer supported

Media Tags are no longer supported

Media Tags are no longer supported

Media Tags are no longer supported

Media Tags are no longer supported



Reply

Iris
10-31-2009, 08:27 PM
Wow G.S. you seem to have put a lot of effort into these notes mashaAllah. I will inshaAllah take my time to go through these...it'll be good revision for me. (I'm a fresh graduate from med school)
JazakAllahu khairan!
Reply

Raphael
11-07-2009, 12:47 AM
I'm impressed you took the time to actually do this.
Reply

OurIslamic
11-07-2009, 12:58 AM
Interesting...I hope to become a med student one day :)
Reply

جوري
11-21-2009, 07:03 PM
Media Tags are no longer supported


JV catheter

indications:
Major surgery
drug admin- vasoactive drip meds.
hyperalimentation ( TPN)
chemo
prolonged abx therapy
CVP pressure monitoring
pulmonary artery cath.
transverse cardiac pacing
temporary hemodylaysis
Aspiration of air emboli

relative contraindications
previous carotid surgery
documented carotid disease
superior vena cava obstruction
severe trauma of the neck
coagulopathies




Reply

Raudha
11-21-2009, 07:40 PM
Jazakillah Khair sis Skye. Somehow I never noticed this thread until today. I will defo make use of it in the future. It looks really good MashaAllah.

May Allah reward you abundantly :wub:
Reply

جوري
11-21-2009, 07:49 PM
Jazakoum Allah khyran

anyone with medical knowledge can contribute to this thread.. it isn't just mine..

:wa:
Reply

Silver
12-03-2009, 05:50 PM
I haven't been on this forum for quite a while. I'm a 3rd year med student now & we're starting to focus on pathologies in our courses. So this thread will be really useful for me. Thanks!! :)
Reply

Silver Pearl
12-17-2009, 07:59 PM
Mashaa'Allaah, excellent thread! Very comprehensive!

I'll try and add some CT, Ultrasound and X-Ray notes Inshaa'Allaah.
Reply

جوري
12-17-2009, 08:17 PM
^^ that would be amazing, I'd be very grateful for that..

Jazaki Allah khyran

:wa:
Reply

جوري
03-17-2010, 02:11 AM
Great lecture on Neuroendocrine tumors,


Media Tags are no longer supported

Reply

جوري
04-07-2010, 06:25 PM
Dr Najeeb is brilliant not to mention hilarious in his description of the renin-angiotensin-aldoesterone system



I am sticking a link to all 79 videos.. most folks have problems with renal physiology the most but it would pay to listen to all his lectures actually ..

http://www.youtube.com/profile?user=DoctorNajeeb#g/u

:w:
Reply

Banu_Hashim
04-09-2010, 11:08 AM
BarakAllahu Feek. Very useful.
Reply

جوري
04-09-2010, 03:50 PM
Jazaka Allah khyran, if you find something useful share it.. I am surprised at how many useful lectures I found on youtube..

:w:
Reply

BintAbee
04-11-2010, 05:07 PM
format_quote Originally Posted by Gossamer skye
Dr Najeeb is brilliant not to mention hilarious in his description of the renin-angiotensin-aldoesterone system



I am sticking a link to all 79 videos.. most folks have problems with renal physiology the most but it would pay to listen to all his lectures actually ..

http://www.youtube.com/profile?user=DoctorNajeeb#g/u

:w:
Jazakillah Khair. Very benefcial indeed. Dad was excited about these lectures too :)
Reply

جوري
04-11-2010, 05:14 PM
^^ this guy is remarkable masha'Allah he did for me in one day what an entire semester of physiology didn't..
al7mdlillah.. I like the way he incorporates 'mango' in the topic of filtration, I must admit I have never encountered an Asian professor who didn't incorporate that fruit into any topic somehow..;D

:w:
Reply

Raudha
04-12-2010, 04:46 PM
:sl:

My friend sent me this link to a movie of heart embryology which I found VERY useful:

http://video.google.com/videoplay?do...1776699704562#

I hope others find it beneficial as well.

:w:
Reply

جوري
04-12-2010, 05:07 PM
^^ jazaki Allah khyran that is indeed very useful..especially in understanding how some congenital heart diseases develop..

:w:
Reply

Raudha
04-12-2010, 08:33 PM
^Ameen. Wa Iyyaki dear sis.

I also have this link that my Dad sent to me. I haven't been through it in detail though. Its called the Auscultation assistant:

http://www.med.ucla.edu/wilkes/inex.htm

I hope its beneficial.

:w:
Reply

جوري
04-12-2010, 09:09 PM
Oh God yes.. I have been thinking of investing in one of those electrical stethoscope.. you know the art of the physical exam is really a dying one with all the technology we have .. although I wonder what that spells out to with the new healthcare reform

:w:
Reply

جوري
04-14-2010, 07:20 PM
iridectomy for narrow angle

Reply

Banu_Hashim
04-14-2010, 07:24 PM
YouTube is possibly one of the best inventions ever. I've found so many amazing organic chemistry lectures :D. Al7amdulillah.
Reply

جوري
04-14-2010, 07:30 PM
^^ organic chemistry was the death of me.. I didn't even have internet when I was in under-grad.. I really think the number of hours I have vested studying would have been so much better utilized and elucidated if I had all that technology at my finger tips.. but al7mdlillah 3la kol 7al.. still there are plenty of folks with the latest technology who manage to do nothing at all with their lives as such I should be grateful even if I struggled alot!

:w:
Reply

Banu_Hashim
04-14-2010, 07:35 PM
format_quote Originally Posted by Gossamer skye
^^ organic chemistry was the death of me.. I didn't even have internet when I was in under-grad.. I really think the number of hours I have vested studying would have been so much better utilized and elucidated if I had all that technology at my finger tips.. but al7mdlillah 3la kol 7al.. still there are plenty of folks with the latest technology who manage to do nothing at all with their lives as such I should be grateful even if I struggled alot!

:w:
Actually, I think struggle and hardships are what make a person ultimately successful. No hardships and ease all the time make a lazy student :X

Allahu 'Alam.
Reply

CosmicPathos
04-14-2010, 08:31 PM
hehe .. Orgo chem I and orgo chem II my life changers in college, allhamdulillah. Sort of a comebacks from miserable first year performance. Allhamdulillah. yet most folks dont like it and I can see why. You have to make those 2 courses your wives, at least for the whole semester and imagine that they are just nagging wives by nature. :)

Once you can relate pure orgo with the biochemical reactions and their mechanisms (the dreaded biochem courses) in your cells (purine/pryrimidine metabolism etc) you come to realize how each molecule is like a living being ... with its own life, its own ideals and its own destiny :p studying and "living" with these molecules for almost eternity make you start believing in them as each of them had a persona, a story to tell. There are universes within us. If you thought that this was crazy, we still dont know in detail how each atom and subatomic particles behave within these molecules ....as these chemicals dance to the tune of life.

*back to viro*
Reply

جوري
04-14-2010, 09:08 PM
I don't think they are akin to nagging wives for surely one would construe something from the nags.. they were elusive, problematic and mocking.. the worst type of partnership ..
al7mdlillah it is behind us...
Reply

جوري
06-10-2010, 07:15 PM
Statistics:


1- Giving everyone a test, wanting to find out how everyone did one ends up with loose individual scores.. they need to be put in a 'container' this container is called a distribution .. with that we don't have to remember individual numbers but a few summary numbers, and by those few summary numbers. we need to.. measure of central tendency (that lies in the middle of a bell shaped curve) and the other is the spread, how 'narrow' or 'broad' the distribution being used:


  • central tendency .. the way to measure CT is by the mean, median and mode
  • the mean being the average
  • median being the middle number
  • mode is the most common

in a normal distribution the mean=median=mode
now not all curves are normal.. some are skewed, some are skewed to the left some are skewed to the right.
positive skew the tail will point o the right, negative skew the tail will be to the left

on positive curve there will be three points 1, 2, 3
one is the mean, one is the median and one is the mode, which is which?
the mean will be 3 which is the last line to the tail of the positive curve, median of course is 2, and mode is 1.. go out to the tail and count in mean, median, mode in alphabetical order so now you can never forget this

  • the mean is most sensitive to extreme values because it is an average of them

now on the negatively skewed curve, same principal toward the tail end is mean, working in median, then mode .. remember that the mean is always toward the extreme values.
to find out if a distribution is skewed or not compare the mean (the average) to the median (middle number) if mean is greater than the median then we have a POSITIVE skew .. if mean is less than the median then we have a NEGATIVE distribution.
for normal distribution the measure of central tendency we have is the mean (the average) -- on a skewed distribution however, the mean (average) isn't the best measure of central tendency instead you use the median!

in addition to central tendency, another parameter of importance to get a sense of the distribution, which is the measure of variation or spread .. we need to know how narrow or wide is the distribution on a a bell shaped curve .
one of the most common is called the range the range is basically the difference between the highest and the lowest score on a bell shaped curve .. but it is a poor measure of the spread.. the range ignores all the data and worries only about two extreme scores. if given a test the extreme scores are likely to shift on another re-examination in a very likely direction and that direction is down closer toward the mean (average) the phrase to describe this phenomenon is called 'regression towards the mean' -- extreme values when re-measured will be less extreme!

let's say you are seeing a patient and s/he has very high blood pressure, you take measures to lower that pressure and ensure that they come back in a week for a re-measurement.. upon remeasurement their pressure is lower.. this isn't because of the diet/exercise/ medication, it is rather a 'regression toward the mean'
the range is thus a poor measure of score because it is concerned with the least stable scores we have ..

so if we don't use the range what should we use?
let's take every score and label it x and subtract from it
_

x called x bar which is how we represent the mean we put those results in quantities i.e parenthesis ( ) and let's sum them together

that is to see how close or far away from the mean every single score is, if it is close to the mean (average) we get a small value, if it is away from the mean we get a large value.. so we are getting the average deviation, but this formula doesn't work because the number it will always give us for any data is zero!
since for every number above the mean one below the mean, so statisticians squared these values, this way positives and negatives don't cancel each other out, but every number gets larger and larger .. now if we have a sample in the twenties and a sample in the two thousands the one in the two thousands would end up with a much larger number and that is because we are taking 2000 differences instead or 20 so to fix this problem we divide by the sample size which we label N (-1) the minus one is a degree of freedom, now since we 'squared' the numbers not to end up with zero we have to account for that by taking the square root of the entire formula and this is the formula for 'standard deviation' utterly broken down to its basic components!


so s=










now hopefully everyone understands what the standard deviation is and how the statisticians derived its variables. in a nutshell how close or how far every score is from the mean and taking its average.. that is what the standard deviation is!

so now if you are asked a simple question like
1- as the sample size increases the standard deviation increases? true or false? well if you've been paying very close attention to the above the answer will be false!
this is an average deviation, every time we added something up, we added something to n on the bottom so that the standard deviation doesn't change with sample size.
application here far outweighs being able to do the calculation.


here is a normal distribution curve:



now let's talk about percentages within 1~2~3 standard deviation of the mean (average) please have it memorized .. these figures are a constant!

numbers on top .. what numbers fall within one standard deviation from the mean is roughly ~68%
+/- two standard deviations is 95.5%
=/- 3 standard deviation is 99.7%

now let's look at just one particular area of the curve.
what percentage falls within the mean and one standard deviation (above) the mean
34% since that is half of the 68%
1~2 above is 13.5%
2~3 above 2.4%
what is left of the tail about 1.5%
the numbers ought to add to 100% but they don't since we have rounded numbers (easier to work with)

question 1- what percent of the cases that are normal distribution fall above one standard deviation below the mean (average) that is 34 + 50 (why plus 50) because the mean is also the median so it will be the 34 + 50= 84%
2- what percent of cases fall below two standard deviation below the mean roughly 2.5%

mean and standard deviation of the IQ distribution:
the mean of the IQ= 100
standard deviation is 15
what percentage of the population has an IQ below 70%
100-15= 85
what percentage is less than that.. (if you are having a difficult time please refer to the bell shaped curve) the percentage less than 70 on the curve will be 2.5%
if 100 i exactly in the measure of central tendency i.e lying right in the middle, and below the mean is going to the left of the curve.. (hope that makes it easier)

2- what percent of the population has an IQ over 130?
since it is a 'symmetric distribution) it is 2.5% on the other side as well!

USMLE has a mean of 200 and a standard deviation equal to 20 .
if you score 240 on the exam what percentile are you?
50+ 34+ 13.5= with a score averaging in the 97.5%
that percentile is how many people you beat, so you start at the bottom below the measure of central tendency that is 50% + add the other figures in and derive your score in percentage!

a schoolboy took two tests
test A and B for test A he got 60% on test B he got 50%
on which test did he do better? (no idea) since scores are only meaningful when related to distribution out of which they came
we measure distribution by central tendency .. the mean for test A was 40% mean for test B is 45%
still don't know on which test he has done better, since we also need standard deviation along with central tendency
standard deviation for test A = 20
standard deviation for test B = 5

so he did absolutely the same on both exams.

go to 40 and start adding standard deviations .. both tests he scored in the 84% percentile.

__________________________________________________ ________

now to use this for medical research

we have a new test, given to a random cross section of the population
95% got a score between 150-250

what is the 84% percentile on this test?
150 to 250 95% of the cases .. this is roughly +/- 2 standard deviations .. if this is a symmetric distribution our best guess about the mean is about 200
to get to the 84% you need mean + one standard deviation

+ 15
the mean is 200 and two standard deviation = 50 then one standard deviation = 25
therefore 200+25= 225

recap. 95% cases between 150-250
think normal distribution +/- two standard deviations
since it is a symmetric distribution the mean is right in the center thus 200 is a likely estimate of what the mean is .. now to get to the 84% percentile we need one standard deviation from the mean . if +/- 50 = two standard deviation then half of that is one standard deviation .. add the two and derive the answer.

based on this data the 16th percentile would be the mean minus one standard deviation and that is 200-25= 175
Reply

جوري
06-10-2010, 08:51 PM
Now we are going to cover 'descriptive statistics'

you take the data, summarize ..

Statistics are made better beyond 'descriptive when it is inferential..
taking information and summing from it what the whole world must be like.. that is how we do science and research:

you want to be able to take the data and apply it to the general population so that every patient isn't a unique event ..
a quick example..
you have taken a test, someone is in a hurry to find the results so they take random samples from the class.. take scores and calculate the mean .. is this mean likely to represent the class as a whole? it is unlikely, but it is likely to be close!
that is what inferential statistics.. we don't have all the data but we guess them and for that we pay a price.

we give up 'precision' for what we call a 'degree of confidence' a certainty about what the number probably is.. this is what we 'believe to be reality for the rest of the world'
a confidence interval!
this is how statistics is done in medicine and it is said that if you don't understand how confidence interval works, you don't understand the articles in the journals.
confidence interval is an intuitive concept.
class starts at 8, you stroll at 5 after 8, the single number isn't reality but around there..
meet a friend at 'sevenish' if you are ten mins late.. you are within the confidence interval.

calculate how much high and how much low.. to come up with the same solution.

confidence interval
confidence interval is the mean

+/- Z ( S/

N)

x bar is the mean +/- symmetrically (for z see bottom) (the standard error) this is a measure of error.. since error = bad you want this number to be as low as possible .. the standard error is a measure of the quality of your estimate .. based on data you have, if you have alot of error, you have a poor estimate, if you have little error, you have a good estimate . the standard error is made of two things (s) and (N) standard deviation and sample size which is designated (n)

now n in terms of sample size, if you have 200 participants of research subjects, it doesn't make for as good a picture as a sample size of 2000 . more people more reality to work .. so we put sample size on the bottom, since as sample size increases error should decrease .

s.. if everyone gets 80% on a test no exception, pull 5 people at random and compute the mean and now take that mean to represent the class as a whole, is that a good representation? Yes!

test, all score all over the map.. take 5 people at random use it to calculate the mean, we get a poor estimate. thus, the smaller the standard deviation the better my estimate, the larger my standard deviation the poorer my estimate .. thus S goes on top ..

as the standard deviation gets larger the standard error gets larger, as the sample size gets larger the standard error gets smaller!

larger standard deviation more error, the larger the sample size less error..

what you have control over a a researcher is the 'sample size' (N) the sample size is your control to reduce error ..

there is no reality in a small sample size.

standard error gives quality of estimate.

Z = standard score not standard error as in parenthesis
each standard means something very different.

if they give you a 'standard error' don't divide by square root of N, it has already happened -- don't treat it like a standard deviation ..

a standard score is a distribution from the curve with known properties ..
this curve was invented to have a mean = zero and a standard deviation = 1
this is born of the statisticians imagination because these numbers are easy to work with ..

that way if it is a positive number it is on one side of the mean, and if negative then on the other . Whether a number is positive or negative we know instantaneously whether it is to the right or the left of the mean. Standard deviation = 1 means score is in standard deviation units. concretely a z score of +1 you are one standard deviation over the mean .. if it is -1.6 you are 1.6 below the mean .
if we wanted to get 95% of the cases in a z score distribution .. if we go between +/-2 of the z score distribution that gives us 95.5% of the cases .. but if we wants 95% exactly we need to pull the numbers from the right and from the left so in order to get 95% we don't go +/- 2 it is +/- 1.6 giving us exactly 95% of the cases ..

now if we wanted to get 99% of the cases ..+/-3 using Z score we get 99.7%, need to tuck it in for a nice round number tuck it in 2.58.. so all we need to do to complete confidence interval calculation is .. we know how good our data is, now how 'confident' are we with the interval so we use either or those two numbers.

either will have a 95% confidence interval in which case we plug in 1.96 for Z or we are going to be 99% confident in which case we'll plug in 2.58 for Z
z in this case is simply a binary switch ..

how to decide which number to plug in depends on the confidence interval in the question.. but to make it easier for 1.96 use 2 and for 2.58 use 2.5 ..

now we can compute the confidence interval of the mean

let's do question
we have a mean of 60%
a standard deviation of 9%
and a sample size surveyed 9 from several hundreds

what is the 95% confidence interval for that estimate of the mean?

60 +/- 2 (9 / square root of 9) = 60+/- 6

so the data tells us that the truth lies somewhere between 66 and 54.. we are not 100% that the mean is within that range.. we are only 95% certain. we bought 95% certainty with out z score.. we don't know where the mean is in that.. it is a flat probability distribution.. any number inside the range is as good as the other.
we need to project from this data reality, and this range is our window to reality .. given precision up for confidence.

*error of measurement and confidence interval are terms that can be substituted for one another.

if there is an overlap in data means there is no difference.
try to use that for poll data for political campaigns when they tell you someone is ahead by but within the margin of error!

remember that 99% confidence interval gives you a broader not a narrower picture of reality
Reply

جوري
06-10-2010, 11:21 PM
confidence interval for 'relative risk'

if you look at a number, how can you tell statistically whether it is a relative risk or not? you can't tell, what you need is to look at the confidence interval .. confidence interval in research will be given as numbers in parenthesis an example for instance (1.12-2.5) from the low number as a possible low to the high number as a possible high .. the critical number if we are trying to determine if something is statistically significant is (1.0)
look inside the confidence interval and see if it contains the number 1!
if one is inside the interval then not statistically significant, if outside the interval then significant.

why is the number one a critical value? what do we do to compute a relative risk? we divide.. a number divided by itself give us 1 so if a risk in group A is the same as the risk in group B the output for the relative risk is one .. so one means the two groups are the same.. now how is it that you can prove that one group is different? you must prove that the output of the relative risk isn't one!
Exclude one and prove that you have a significant number ..

so using the above the lowest possible risk is 1.12 the highest possible value is 2.25 .. is that statistically significant? Yes, because one occurs outside the one.. we have excluded one as a possibility even in the worst case scenario therefore we do have statistical significance, and this indicated increased risk, why? because the relative risk is > 1

1.65 is that statistically significant relative risk? move your eyes to confidence interval and we have .. the CI has a low of 0.89 and a high of 2.34 .. not a statistically significant relative risk.. why? because on occurs directly inside the interval .. thus we haven't excluded one and therefore we can't assume we have statistical significance.
the second relative risk even though larger than the first isn't significant .

another sample relative risk of .76 a sample CI (.53-.93) excluded one therefore significant .. this one denotes 'decreased risk'
the difference between the 'pros' and the 'rookies' when reading these articles .. the pros are looking at the confidence interval, the rookies are staring at the relative risk .. the relative risk doesn't matter unless the confidence interval tells you it matters.

this is also true for odds ratio .. if you cross out 'relative risk' and write in its stead 'odds ratio' follows the same, look at the confidence interval one in means not significant, one out means it is significant .

if you read the abstract and understand the tables then that is it..
Reply

جوري
06-11-2010, 12:02 AM
probability and probability theory.. is the very foundation of statistics..

how to combine probabilities?

how do we combine independent events and mutually exclusive events?

now using a quarter ..
take the coin and flip it, it comes out heads, and again, heads, and again heads, and again heads .. four coin flips four heads in a row .. flipping the coin one more time .. you are not due for tails '' the gambler's fallacy'' .. the fallacy assumes that what came before has anything at all with what is about to happen and the response to that is that it doesn't .. the coin flips are 'independent events' the chance for a next flip being one or the other is always 50-50 as it has always been!


joint probability of independent events:

what is the chance of getting two heads in a row?
(.5%) x (.5%) = 25%
chance therefore of two heads in a row? 25%
3 heads in a row = 12.5%

_________________________
mutually exclusive events:

same coin flipped, comes up heads.. we know two facts. one it is heads and the second fact is it wasn't tails.
heads and tails in a signal trial event are mutually exclusive that is if one occurs, the other one can't occur.
the chance of getting a head or a tail is 100%
heads (.5) tails (.5) and the 'or' stands for add!

mutually exclusive means if you are A you can't be B and vice versa, there is no overlap!
nonmutual A and B with overlap .. when we have an overlap we have to account for the joined area which can't be A+B it is AxB
what is the chance of A, B or both?
chance of A or B but not both?
to get rid of the combined area subtract 2

now some real application
the chance of being obese is 30% the chance of having diabetes is 10% .. what is the chance of being obese, have diabetes but not both?
obesity and diabetes are non mutually exclusive
we start by adding A & B together so that is 10% + 30% = 40%
the overlap area is characterized by A x B = 3%
so our correction factor is 3%
we wanted to do A and B but not both
so it is A + B = 40 % - 2AB (2x3)= 6% (as a correction factor)
therefore the correction factor here is 34%
if obese/diabetes or both the answer is 37%
we'll not subtract two occurrences of AB but one occurrence
Reply

جوري
06-11-2010, 12:47 AM
the logic of statistics is you can never prove anything all you can do is disprove something!
if I want to prove a drug works, I can't, but what I can do is disprove that the drug doesn't work! Ahhhhhh the (double negative) that is the way to get to where you wanted!

we have a double blind randomized design to prove that a new wonder drug works, one half of the participants gets the wonder drug, the other half gets placebo
follow both groups and see when they will have relief from their symptoms,

1- the research design isn't flawed

now let's discuss the null hypothesis ..the 'null hypothesis is the opposite of what you want to find. group A fails to get over the sx. faster than group B.
with the null hypothesis we state it and then leave it alone .. we pass out pills and collect data next .. take the data feed it into the computer ..
what comes out T as in a t test, x^2, f etc etc.
we should only focus on the P value is key for making statistical decisions
we make decisions by putting a standard in place and comparing empirical evidence to it .. so that is what the P value is, the standard and the summary of the data .
this alpha criterion is something you decide before you make your research, you can set the value high or low, it is your discretion
people put the value of P at less than or equal to .05
a confidence interval of 95% means one is correct 95% of the time .. the other five percent is the time when one is wrong
a confidence interval of 95% corresponds exactly to a P value of </=0.05
95% chance of being right, 5 % chance of being wrong

outcomes for our study p=.02
.02 is under the bar which is very good because it means we get to 'reject the null hypothesis' because the null hypothesis is the opposite of what we are looking for .. if we reject the null hypothesis, the drug works!

is it possible that the drug works in the study but not out in the world?
yes possible though unlikely..

what this means is that we have made a type one error, or alpha error, this type of error basically states we rejected the null hypothesis but we shouldn't have . You'll never know for sure if you have made a type one error, all you know is the chance that you made a type one error that chance is found in the p value a .02 i.e a 2% chance .
p value is type one error
if the number for the p value gets too low, we'll take that chance

2nd outcome for the study, p=1.3 we are now above the bar, we can't reject the null hypothesis, we fail to reject the null hypothesis..
you never 'accept the null hypothesis'
same as 'jury logic' not that you are innocent, just that there isn't enough evidence to convict you . the chance for a type one error here? = 0 why? because to make a type one error you must first reject the null hypothesis.
however in this case we could have made a beta type error means, I didn't reject the null hypothesis but I should have .. in other words in the study the drug is crap, but out in the real world, it works well..
chance of making a type two error? we don't know.. can't look at P value because P value only tells us a type one error ONLY!

type one error is considered worse ..
which is worse looking at you and lying or simply forgetting to tell you something?
lying is worse, that is a type one error a 'sin of commission' because first do no harm is a physician's oath.

now you are giving this new drug because it has been approved and works great, the patient is now asking this drug works great in research, what is the chance it will work for me? best response is 'I don't know' this gives you statistical significance not clinical significance!

you can answer the patient by looking at the table the one that tells you, who got the drug, who didn't get the drug, got better, didn't get better
got drug got better 70%
got drug didn't get better 30%
no drug better 30%
not better no drug 70%

pt.s chance of getting better on drug
the answer here is 70% chance of getting better out of one hundred people that got the drug 70% of them got better!
Reply

جوري
06-11-2010, 01:06 AM
statistical power:
power is capacity to find a difference if there is a difference ...

like looking at a cell in small magnification can't tell what it is, so increase the power, to get more information :
1-power = beta
in other words one minus statistical power gives us a type 2 error
this is the only way to calculate a type 2 error .
the simplest thing to increase power is to increase sample size.

researcher conducted and got his research to equal to .15 set the study to have statistical power = 70%
what is the chance of a type 2 error? it should be equal to 30%.
the P value is irrelevant..

______________________________________________

which test should I run?

scaling, how to convert the world into numbers?

types of scales to focus on, nominal or categorical or interval
nominal or categorical
nominal is this or this or this like doing the laundry, whites, colored, knits, that is nominal data
what is the most common nominal variable? Gender! groups mutually exclusive
male and female, how many nominal variables? the answer is one. gender isn't a variable .. only the comparison of the two that gives us the variation .
so single nominal variable with two groups to it. Things in a classification.

interval data is within a scale, when you measure with a measuring device with equally graded intervals , lights years, weights, meters, mph. you can get means, and standard deviation. What is the mean of gender? for instance computing heights, computing sd of wights.

nominal.. groups, clusters. clumps.. how much.. what something is= nominal, how much it is= interval!

measuring rainfall, interval because there is a measurement

passing is nominal either pass or fail.

decide between nominal or interval you can tell which test to run.

statistical analyses

interval, nominal or combination.

all interval data two interval variables run Pearson correlation

all nominal data, all things in categories, run a chai square test

nominal/interval data combined together, run a t test.

heights of men and women? run a nominal, height interval

compare height to weight, interval/interval pearson correlation

give a drug to one not the other, see who is better or not, chai square. two nominal variables.
Reply

جوري
06-11-2010, 06:33 PM
Sensitivity:

about detection of disease.. perfect sensitivity means detecting 100% of diseased individuals.. diseased people are a little sensitive!
the rule for the formula of this one is trues on top and divide by everything ..
we only care about the left hand side of the table with this... left side is where the diseased individuals lie.. that is the secret on the two by two table.. just decide on the part of the table that matters..

true positives/ true positives + false negatives

detected disease divided by all the disease..
80% sensitivity is good.. for every ten finding 8, better than chance which gives us 50/50
sensitivity = detecting disease..


Specificity: detecting health.. concerned with right side of the table, only the healthy people..
specificity again, trues on top

true positive/true positives + false positives



everything in specificity is about a healthy person..
if we have a 60% specificity, out of every 10 healthy we were able to find 6, still better than chance but not very good...

a test with good sensitivity and specificity is difficulty, that is why we run two or more screening tests.

what are we trying to achieve with the test is most important thing. if trying to find disease, need a sensitive test even if specificity sucks..

or have a treatment that is horrible, but good, you'll rely more on specificity than sensitivity.

pick what matches problem presented.

you get a patient who wants to test for HIV, the test comes back positive, he is in denial he asks you, are you sure? you say let me find out for you..
finding out is called positive predictive value:
positive predictive value, is set so find out whether you should believe the test results or not!

we now just us the top row of the table :
__TP___ X 100 = Predictive Value of a Positive Result (%)
TP + FP

trues on top.. true positives divided by true positives + false positives .. all you need to answer the question is to know all the positives..
80/120= 67% is the PPV for this patients test...

so now proceed on this basis because there is a 1/3 chance the patient doesn't have this disease..

another scenario:
female patient wants to become pregnant, comes in for a pregnancy test..
results are negative..
patient wants to know if you are sure..
let's find out..

now we are talking about negative predictive value ..

now we are talking about the bottom row where all the negative values lies..
the rule is always trues on top in this case true negatives / all the negatives
60/80= 75%

__TN___ X 100 = Predictive Value Negative Result (%)
FN + TN


so one in every four is wrong..
test is not bad.. good sensitivity ..

accuracy .. again uses all the trues on top..

tp+tn/N (which stands for everything) 70% accuracy!

screening tests are about secondary intervention which is concerned with prevalence not incidence..


accuracy = sensitivity + specificity / 2
Reply

Hugo
06-13-2010, 05:16 PM
After discussion with τhε ṿαlε'ṡ lïlÿ I have decided to add some comments on research and statistics here. I will begin by setting out some thoughts.

1. Medical research is a difficult area because on the one hand it offer so much value and the other when things go wrong so much damage. One can point at huge advances made and the practical eradication of many diseases. However, things have gone wrong and one only has to consider thalidomide or say the use of cardiac anti-arrhythmic drugs which are known to have cost more American lives than the Vietnam war. (See "When Doctors Kill: why and how", by Joshua Perper and Stephen Cina)

2. It is also sadly true that in scientific research there have been many many cases of fraud or misconduct. In fact Professor David Goodstein from California Institute of Technology in a recent book shows that most scientific fraud or misconduct cases involving biological science with medical doctors disproportionately represented in these cases. (See "On Fact and Fraud" by David Goodstein, Princeton Press.

3. There are many reasons for what I have said in item 2 but for the purposes of this thread I will point out three and in subsequent posts elaborate on them.
Scientific Method - over centuries experimental methods and principles have been developed and these must be thoroughly learned and it takes a long to to learn and practice them. Indeed it is only when you do real work that its main points begin to sink in and getting to that point with humility is central to developing your research potential.

Ethics - in all science there is an ethical dimension and it has to be thought through with considerable care. In medical research it is paramount for obvious reasons. Indeed a large number of both fraud and misconduct cases can be traced to poor ethical standards.

Statistics - when one begins statistics it can seem quite easy but this is a false assumption and unless you really know what you are doing you can make horrendous blunders. These days we have SSPS and Excel so given a set of data one can generate a whole raft of statistics with zero effort. However, like any science, all statistics are hedged about with condition and limits so interpretation of what you have been given is likely to be very hard EVEN if you are expert. Statistics is ultimately based on probabilities and everyone have difficulty in that area.

Sadly, the literature is legion with cases of scientific blunders because researchers do not understand what they are doing. For example, there are many cased where a researchers confused correlation and regression, were selective in what data points they used, collected the wrong data and so on. So what we have to say here is NOT simple and if your are to get any benefit you will have to work hard. To give a simple example, suppose my risk of stroke is assessed as 12% and my doctor tells me that if I take a statin it will reduce my risk by 16% (we will not complicate it by adding in side effects). Almost no one outside of a numerate discipline can explain why your new level of risk if you take the statin is 10%.

It is also uncomfortably true that even the best researchers sometimes get over-confident, not to say arrogant, and try to go it alone and do not get advice from a competent statistician - that is unforgivable and may amount to misconduct
Reply

جوري
06-13-2010, 08:31 PM
^^Thanks for that..

under the heading of neurology, three syndromes amongst many seem to creep in when no one expects it.. here is a quick read from third part source, just know the artery affected and ipsilateral/contralateral symptoms:





:w:
Reply

Banu_Hashim
06-14-2010, 12:09 AM
Once I start my pharmaceutical chemistry and pharmacology modules next academic year, I'll be a lot more active in this thread inshaAllah :). I'll also me starting immunology - is it a tough subject?
Reply

جوري
06-14-2010, 12:21 AM
format_quote Originally Posted by Banu_Hashim
Once I start my pharmaceutical chemistry and pharmacology modules next academic year, I'll be a lot more active in this thread inshaAllah :). I'll also me starting immunology - is it a tough subject?
Great.. khyer insha'Allah.. (I think this is the most viewed thread on this forum?)

Immunology and microbiology are sister subjects they go hand in hand and they are light and fluffy.. part of the easier subjects in the sciences .. it is mostly memory not many concepts involved..

:w:
Reply

Hugo
06-14-2010, 03:37 PM
This is my second post on the theme of evidence based research and statistics and here I discuss two basic ideas which I shall call Research Style and Research Type. Please be aware I am INVENTING data here.

RESEARCH STYLE – at this stage you need to consider if your style is quantitative or qualitative. It is easy to confuse these two and simply think of them as describing data types but to do so means you are missing the whole point. In general, if you outcome is in some way intended to be predictive then your style is likely to be quantitative whereas if it is intended to be mostly descriptive then it is almost certainly qualitative. For example, suppose I decide to study infection rates after surgery.
Quantitative - here I would for simplicity chose two elements: the procedure and the infection rate. So over time and with a sample of patients I record relevant data. Now using this data I could process it statistically and predict say in knee surgery that the infection rate is likely to be 25% of patients.

Qualitative - knowing that 25% patients become infected after knee surgery is interesting but not of much use in deciding what to do about it. So my next research task could be to study surgical procedures used in knee surgery with a view to constructing a check-list which when used by the surgical team will lead to fewer infections. So here I am ending up with a description, as a series of questions (usually not more than 8), of what to do both at the start and end of an operation to minimise or prevent post-op infection.
Thus you can see that the terms Qualitative and Quantitative are to do with the kind of OUTCOME you want not primarily the data itself - it is VITAL that you understand this point.

RESEARCH TYPE – broadly speaking there are two types; the first is interventionist where you deliberately make a change in a situation and then study its consequences and the second is observational where you simply record what is currently going on. Using the same example as above.

Observational - as for the above example, I do nothing except record which surgical procedure was used and post-infection rates. I don't interfere in any way and make no changes.

Interventionist - as for the examples above, let us suppose I develop the necessary check-list to be used by the surgical teams - that is my intervention, that is the change I make. Now I start recording the data exactly as before about procedure and post-op infection rates. Therefore at the end of these two studies I CAN decide if my intervention made any difference at all.

You can also see in this example how studies can and in some cases must, as in this one, be linked.

1. Start by getting raw data on post-op infection rates
2. Next a study to develop a check-list
3. Finally, get the check-list into use and collect infection data again and see if there has been a reduction of significance (or of course an increase but we hope not). Here we might, indeed should, employ a statistical text of significance to be sure the change has made a real difference.
Reply

Hugo
06-16-2010, 02:06 PM
Measurements and Scales
The concept of measurement requires some scale along which different values can be placed. Four types of scale are possible.

Nominal - a scale used to represent unordered variables. For example we might collect statistics on colour preference. Clearly there is no sense in which a preference for BLUE is greater than RED so in this case any convenient ordering arrangement will do.

Ordinal - a scale used to represent an ordered series of positional relationships. That is where values only indicate position in a series. For example, in an examination we can say that one student got more marks than another. However, we cannot say that a student with 50% knows twice as much as one with 25% or that a student with 100% knows everything and one with 0% knows nothing.

Interval - On interval scales, one unit on the scale represents the same magnitude across the whole range of the scale. For example, if anxiety were measured on an interval scale between 1 and 10, then a difference between a score of 8 and a score of 9 would be assumed to be the same difference in anxiety as that between a score of 2 and 3 or 5 and 6. But interval scales do not have a true zero point; in this case we cannot say what zero anxiety means or that one person is twice as anxious as someone else.

Ratio - a scale where a particular interval is the same anywhere on the scale and it is meaningful to refer to zero or say that one value is a certain multiple of another. It is therefore meaningful on such a scale to say how many times higher one score is than another.

Ideally we want a ratio scale because it is mathematically tractable; we can talk about zero, we can say something is a certain multiple of another value, we can do exact comparisons. When we use statistics on numbers we want the data to be at least ordinal.
Reply

Hugo
06-17-2010, 10:52 AM
I will base my next few posts largely on an article which can be found at the following site. Please be aware though that the terms themselves are not absolute and you therefore will find variations in the literature and when you join a research team so focus on understanding the ideas. http://www.pharmj.com/pdf/cpd/pj_200...encebased1.pdf

Evidence-based research means extracting good evidence to make sound clinical decisions because valid evidence of clinical benefit and cost-effectiveness is what will influence purchasers of health care. There are two fundamental questions that one needs to answer: how do we assess the evidence itself and how do we know that the piece of research that generated it is good enough to be relied on? The point is that the evidence may be overwhelming BUT unless we know exactly how it was obtained we cannot rely on it.

Often we speak of an ‘evidence pyramid’ and what that means is we must look in detail at the data and the methods used to see that they were ethical, appropriate and well-executed. For example, if I trial is described as ‘randomised’ then we need to know exactly how the randomisation was carried out because if it was not done with the necessary scientific and mathematical rigour the results may be totally invalid. Indeed many studies have eventually been rejected because of poor randomisation processes – everyone can invent such a process and almost none of them will be valid.

The hierarchy of evidence beginning from the most trustworthy/reliable to the least is: systematic Reviews, Meta analysis, cohort studies, case control studies, cross sectional surveys, case series and case reports. In later posts I will explain each one in more detail as well as explain how methods can be evaluated. But here starting from the top.

Meta Analysis – before research can be readily accepted it must be reviewed by competent specialists both medical and statistical. One way of doing this is to gather the data from many trails together and pool them as results into one big spreadsheet (or whatever system you are using). This can be very useful if there have been many small trials but each one too small to be conclusive. So if you had 10 trails with a paltry 40 patients in each then do a Meta analysis and you have a 400 person trial to work with. In fact there have been several cases where a treatment previously believed to be ineffective turns out to be rather good but because each individual trial was too small to detect the benefits no one spotted it.

You might like to look into the case of New Zealand doctors having the idea of giving a short and cheap course of steroids to help improve premature babies between 1972 and 1981. Two trials showed benefits and five failed to detect any and the idea was dropped. Eight years later in 1989, a meta-analysis done by pooling the trials was carried out and when this was done a clear benefit emerged showing a 30-50% reduction in risk of babies dying from complications of immaturity. One must always remember the human cost of the abstract numbers: babies died unnecessarily because they were denied a life saving treatment for a decade.

By the same token a meta analysis can show that something that was thought to work does not in fact work and without going into details there have been several such cases.
Reply

جوري
06-20-2010, 08:26 PM
Here is a couple of questions for Hugo or anyone who has been reading notes in the statistics/epidemiology portion of this.. or who simply wants to offer a stab...



Reply

CosmicPathos
06-20-2010, 09:46 PM
format_quote Originally Posted by Banu_Hashim
Once I start my pharmaceutical chemistry and pharmacology modules next academic year, I'll be a lot more active in this thread inshaAllah :). I'll also me starting immunology - is it a tough subject?
I love microbiology. Microbiology is too broad of a discipline and includes bacteriology, virology, genetics and is also very closely intertwined with immunology because immunology studies those things which are related to "microbes" or their pathogenicity within mammals, more specifically humans, monkeys, rabbits, goats, mice etc.

I personally found virology to be interesting but quite heavy on memorizing. You are just supposed to know which virus causes which diseases. You'd be surprised to see that most viral infections show similar symptoms! Headache, fever, myalgia, dizziness and what not. That is because these symptoms are related to immunological reactions/weaknesses. Althought each virus has its own unique symptom. For example Hepatitis viruses have jaundice etc. Khayr, its a long long discussion and I can go on for days ... :D

Immunology by itself is not tough, thought it has tons of memorization. You will be required to memorize how T cells work and how they are activated. You will be required to memorize how leukocytes mature. You will look at immunology from a broad cell perspective and then go into nitty gritty molecular details as to which cellular receptors are involved and what chemicals are formed and on what enzymes they act, all within just one cell, so on and so forth. As I said previously, after you finish your studies, you'll be left in awe after realizing that we are walking talking universes in our own selves .... probably more intricate than the cosmos.

Pharmaceutical chemistry is also very interesting. It is related to medicinal chemistry as well. Its research focused and is concerned with drug design, specifically the chemical reactions, conformations of molecules etc required to produce a specific outcome. Very strong emphasis on organic chemistry but recently inornganic chemistry and transition metals and their complexes have shown to have strong anti-cancer activity as well so inorganic can be a part of it too.

Pharmacology is clinical aspect of drugs which are KNOWN. Crazy memorization. Not much innovation. Just memorizing facts for clinical setting and spilling them out when asked on the exam or in the clinic. Though its pretty important for medicine.
Reply

Hugo
06-20-2010, 10:57 PM
format_quote Originally Posted by τhε ṿαlε'ṡ lïlÿ
Here is a couple of questions for Hugo or anyone who has been reading notes in the statistics/epidemiology portion of this.. or who simply wants to offer a stab...


This is a fairly advanced sort of idea which calls on quite a lot of statistics and mathematics. The ROC was used initially to help radio/radar operators decide whether something was just noise, natural and usually small variations or whether there really was a signal there - one does not want to shoot off a missile just because there just might be something there. The idea was taken up by the medical profession because typically you take a test sample from patients and you have to decide when it is significant. There is a good tutorial on this at http://www.anaesthetist.com/mnm/stats/roc/Findex.htm

Take for example PSA, well it has a range of values and there is a cut off - above X there might be cancer and below this, healthy tissue. But of course human beings are biological animals so there is no exact figure for a given individual where one should start to worry, instead there is a range of values. Roughly speaking when the PSA test was mooted someone took a large number of measurement from a representative sample of men. When you do this you in effect get two samples: one of healthy men and one representing those with the condition and of course these two sample overlap.

The reason they overlap is that some of the men the test shows healthy have in fact the disease (false negative) and similarly some of those the test shows have the disease, in fact do not, a false positive. Therefore if we process the data and a with a knowledge of each patients history one can suggest a cut off value. Usually of course one has the test and if its in the cut off range one is sent to a specialist for further tests and that might end as a positive or negative overall result. Two simple statistical ideas are used:

Sensitivity - how good the test is at picking out patients with the condition, so sensitivity gives you the proportion of cases picked out by the test, relative to all cases who actually have the disease usually expressed normalised between 0 and 1.

Specificity - the ability of the test to pick out patients who do NOT have the disease again expressed as fraction between 0 and 1.

I can't draw a ROC curve here but it is constructed by plotting sensitivity on the y-axis and specificity on the x-axis and any good statistical package will do this for you - BUT you really do need to know what your doing so take advice from statistician.

Essentially, the closer the ROC curve is to a diagonal, the less useful the test is at discriminating between the two populations. The more steeply the curve moves up and then across, the better the test. A more precise way of characterising this "closeness to the diagonal" is simply to look at the AREA under the ROC curve. The closer the area is to 0.5, the more lousy the test, and the closer it is to 1.0, the better the test. Essentially, the diagonal is a line that means you might as well just toss a coin because it would be just as good as the medical test at predicting a particular condition.

If you find this hard to see just think of the ROC curve as residing in a square with sides 1, if you then draw a diagonal it is obvious the area under the diagonal line is 0.5 and the area of the whole square is 1. If you then think of the test as being very sensitive the the curve will shoot way up above the diagonal and gradually move across hence getting closer to the square whereas if its not all that sensitive it will rise only slowly and be nearer the shape of the diagonal. Obviously, confronted with a ROC curve it will not be easy for you by hand to find the area under the curve and that is why you MUST use a package.

Please be aware that what I am describing here is what one does to establish cut off points. In the example given in the previous post the curve looks as if it being used as a trauma diagnostic tool and in that setting it should ONLY be used by a competent physician and in concert with other patient data. Be aware that I am speaking mathematically here and have no competence as a physician.
Reply

جوري
06-20-2010, 11:05 PM
Greetings.. what is your answer for the two q's?

the choices should correlate to points on the graph or 'none of the above'
what is your response and rationale?

thanks
Reply

CosmicPathos
06-20-2010, 11:31 PM
format_quote Originally Posted by τhε ṿαlε'ṡ lïlÿ
Here is a couple of questions for Hugo or anyone who has been reading notes in the statistics/epidemiology portion of this.. or who simply wants to offer a stab...




Cant answer :(

statistics is not my forte, as of yet. Only took one course entitled "biometrics" in college and hated it to death.
Reply

جوري
06-20-2010, 11:36 PM
format_quote Originally Posted by mad_scientist
Cant answer :(

statistics is not my forte, as of yet. Only took one course entitled "biometrics" in college and hated it to death.
well now is a good time to get acquainted.. it won't go away unfortunately for even when you are done, you'll need to keep your CE by reading medical journals which requires understanding that crap... if you don't know what the numbers mean then you don't understand what the journals are about.. all you need are numbers and abstracts.. the extraneous details are just to wow other people..

:w:
Reply

جوري
06-20-2010, 11:54 PM
here is a good start:

http://www.amazon.com/High-Yield-Bio...lp_edpp_img_in

buy it used for seven bucks.. formulas don't change.. but people get greedier...

:w:
Reply

جوري
08-08-2010, 11:28 PM
Quick note on distinguishing between stress incontinence, urge incontinence and overflow incontinence..
obviously stress is easy to diagnose with the Q tip test and the patient will complain of loss of small amounts of urine, during coughing, sneezing, this is treated either with kiegel exercises or surgery depending on the severity by attaching the sphincter to the pubic symphysis.

Overflow incontinence also known as hypotonic incontinence results in involuntary loss of urine, typically due to ineffective detrussor contraction. the hypotonic bladder muscles allow for bladder pressure to exceed urethral pressure , resulting in intermittent emptying of urine in small amounts all throughout the day (the word small here is operative) Think of this etiology with diabetic neuropathy, MS or anticholinergic meds. These pts experience pelvic fullness due to the increased residual volume of urine. RX. is anticholinergic blockers to relax the bladder neck, intermittent cath maybe necessary in refractory cases.

similarly urge (hypertonic) incontinence also results in loss of urine day or night, but in much larger amounts and without warning due to involuntary rise in bladder pressure from excessive idopathic detrusor muscle contractions. these cases require treatment with anticholinergic meds like oxybutinin, note that this med would exacerbate (overflow) hypotonic incontinence as such it is important to work out the difference.

I know I have gone over this in the previous pages but thought I'd simplify it even more..
Reply

جوري
11-24-2010, 06:40 PM
Getting back on track with some disorders of the immune system.

1- Primary immune deficiency also known as (CVID) common variable immune deficiency
it is an abnormality in B cell function, presents from one year to adulthood, when the onset is in adulthood the patient may have an underlying lymphoid malignancy-- presentation with recurrent bacterial infections involving the sinuses, middle ears, when the lung is involved it means there is systemic disease, treatment is with lifelong IVIG

2- Hyper IgE syndrome also known as (job's syndrom)
recurrent skin, and visceral abscesses mostly due to staph with an elevated level of IgE

3- DiGeorge's syndrome -- secondary to deletion of chromosome 22q11, affecting the development of third and 4th pharyngeal pouch, causing anomalies of face, thymus, parathyroid and cardiac structure like tetraology of fallot. There is an absence of T cells secondary to absence of the thymus. there is also hypocalcemia secondary to hypoparathyroidism so they'll present with tetany and seizures rx is a BMT

4- Wiscott Aldrich syndrome:
mix of immunoglobulin defects as well a t cell deficiency, eczema in the early yrs resembling atopic dermatitis, thrombocytopenia, the immunoglobulin pattern is as such IgG normal IgM low, IgA and E very elevated. Infexns from encapsulated organisms. They have an increased incidence of Hodgkins. RX BMT
if BMT isn't feasible due to absence of HLA matched donor then a splenectomy is the RX of choice (so remember immunizations) and that is reserved for pts whose plts count is less than 50,000 IVIG every three to four weeks.
ABX prophylaxis with amoxil or tmp-smx daily.

5- SCID
absence of both cellular and humoral immunity, starts in the neonate, recurrent diarrhea, infexn and failure to thrive. chronic mucocutaneous candidaiasis is common early finding. attenuated live vaccines such as OPV can cause reversion and severe infxn.
blood transfusions can cause GVHD -- lab findings are lymphopenia absolute lymphocyte count less than 2000/mm cube, hypoglobulinemia, impaired specific antibody response, cutaneous anergy . RX BMT

6- ADA deficiency
profound lymphopenia absolute lymph count less than 500/mm cube
chondro-osseous dysplasia of constochondral junctions -- verteberal bodies reveal rachitic rosary rib cage

7- chediack Higashi syndrome
phagocytic disorder, neutrophis contain abnormal giant granules due to inappropriate fusion of lysosomes and endosomes. Recurrent pyogenic infections, partial oculocuteanous albinism, neurological abnormality photphobia, seizures, disorders of spinal tract and cerebellum.

8-
IgA deficiency
most common primary deficiency -- predominant Ig of nasal secretions. Most pts are asymptomatic but may develop recurrent sino-pulmonary infexns, recurrent gi infexns with giardia, anaphylaxis to transfusion rxn.

9- Ataxia Telangictasia:
progressive cebellar ataxia since the beginning of walking, slowly worsening by 10-12 become wheelchair bound
occular or facial telangiectasia, elevated alpha feto protein in more than 95%.. Immunodeficiency absent or low IgA and IgE level..
no effective rx
Reply

azzedinefaizan
05-13-2014, 07:21 AM
Very eloquent. I also find this explanation very profound: "Allah the Almighty, Who is the Maker of the universe and of all beings in it, created the universe for very significant purposes. Quran informs this as follows:

Not for (idle) sport did we create the heavens and the earth and all that is between. (Anbiyya Surah, 21:16)

Not without purpose did we create heaven and earth and all between (Sa Surah, 38:27)

All beings glorify and magnify the Exalted Creator with their languages. They fulfill the tasks with a pleasure and enthusiasm, which are assigned to them. For example, the sun, without falling behind even a second, moves around the orbit, which has been drawn for it. Rivers enthusiastically flow into seas. Animals, which have been given under the command of man, serve him with an absolute obedience.

Besides, if the universe had not been created, the never-ending perfection and beauty of the names and attributes of Allah Almighty would have never been known. This would have only been known by Allah (SWT). By manifesting the spiritual beauties of His names and attributes, Allah Almighty, besides beholding His own Beauty and Perfection on His own works, also wished to give a share to angels, man, and jinn from this honor and bestowal.

Regarding the question of whether to create the beings or not to, Allah (SWT) made His Divine preference for creation, and this preference has become an interminable mercy for all creatures. Or else it is unthinkable that Allahs, one of Whose names is Samed (everything is in need of Him but He is in need of nothing), creation of this universe stems from a necessity."

http://www.questionsonislam.com/arti...reate-universe
Reply

Hey there! Looks like you're enjoying the discussion, but you're not signed up for an account.

When you create an account, you can participate in the discussions and share your thoughts. You also get notifications, here and via email, whenever new posts are made. And you can like posts and make new friends.
Sign Up

Similar Threads

  1. Replies: 2
    Last Post: 05-26-2010, 02:35 PM
  2. Replies: 2
    Last Post: 11-13-2009, 03:51 PM
  3. Replies: 1
    Last Post: 08-28-2007, 11:08 PM
  4. Replies: 0
    Last Post: 04-21-2007, 01:10 PM
  5. Replies: 1
    Last Post: 02-19-2006, 10:50 PM
British Wholesales - Certified Wholesale Linen & Towels | Holiday in the Maldives

IslamicBoard

Experience a richer experience on our mobile app!